Exam 3 Questions and Answers, MCM Midterm Correlation Boxes, Pathways and Enzymes in MCM MED110 Course Exam 1, Biochemistry CAS #4, MTC Mod 3

Réussis tes devoirs et examens dès maintenant avec Quizwiz!

Fatty acid beta oxidation generates several products. Which of the following are direct products of beta oxidation? NADH FADH2 AcetylCoA Pyruvate beta-hydroxybutarate

1-3

Enzymes in the Carnitine Shuttle

1. Fatty Acyl CoA Synthetase 2. Carnitine palmitoyltransferase I (CPT-I) 3. Carnitine-acylcarnitine translocase (CACT) 4. Carnitine palmitoyltransferase II (CPT-II)

Protein turnover

100g of protein is ingested, while 400g are metabolized and 400g are created Ingested proteins are degraded by proteolytic enzymes AA metabolism leads to energy production or other important biomolecules Nitrogen created in metabolism is processed in the urea cycle in the liver and then excreted as urine or sweat

The answer is C: Methylmalonic acid. The patient is experiencing the symptoms of vitamin B12 deficiency. The macrocytic anemia is due to a lack of precursors for DNA synthesis in the red blood cell precursors due to the B12 deficiency. The numbness and tingling is due to hypomethylation in the nervous system, also due to the B12 deficiency. Vitamin B12 only participates in two reactions in humans. The fi rst is the conversion of homocysteine to methionine, requiring N5-methyl THF. The active form of B12 in that reaction is methyl-cobalamin. The second reaction is the conversion of L-methylmalonyl-CoA to succinyl-CoA. The active form of B12 in that reaction is adenosyl-cobalamin. In a B12 deficiency, neither of these reactions would proceed, so one would expect to see both homocysteine and methylmalonic acid accumulate in the circulation. B12 is not required for the metabolism of acetic acid, ketone bodies, propionic acid (although propionic acid is converted to methylmalonyl-CoA, so in a B12 deficiency there may be a slight rise in propionic acid levels as well), or succinate.

20 A 42-year-old woman presents with tiredness and lethargy. She has tingling in her hands and feet. Blood work shows a macrocytic anemia, along with elevated homocysteine levels. One would also expect to see elevated levels of which metabolite? (A) Acetic acid (B) Ketone bodies (C) Methylmalonic acid (D) Propionic acid (E) Succinate

The answer is E: Urate. Patients with Lesch-Nyhan syndrome develop severe gout as the free bases, guanine and hypoxanthine, can no longer be salvaged. As these bases accumulate, urate is produced in excess, leading to gout. This is frequently seen in infants with this disorder as an orange sand-like compound. Xanthine, hypoxanthine, guanine, and adenine are not accumulating in the urine, as these molecules are metabolized to produce urate.

A 1-year-old boy was brought to the pediatrician due to a developmental delay, biting of his lips and fingers, and the presence of orange crystals in his diapers. Enzymatic analysis shows loss of 99% of the activity of a particular enzyme. Considering the patient, the orange sand in the diapers was composed of which of the following? (A) Xanthine (B) Hypoxanthine (C) Guanine (D) Adenine (E) Urate

The answer is B: Guanine. The patient has Lesch-Nyhan syndrome, a defi ciency in HGPRT activity. HGPRT utilizes as substrates hypoxanthine, guanine, and PRPP, converting the free base to a nucleoside monophosphate (IMP and GMP). The enzyme does not utilize adenine, adenosine, guanosine, or GMP as a substrate. The reason for the aberrant behavior and developmental delay observed in this disorder has not yet been elucidated.

A 1-year-old boy was brought to the pediatrician due to a developmental delay, biting of his lips and fingers, and the presence of orange crystals in his diapers. Enzymatic analysis shows loss of 99% of the activity of a particular enzyme. The defective enzyme in this disorder would normally utilize which of the following as a substrate? (A) Adenine (B) Guanine (C) Adenosine (D) Guanosine (E) GMP

The answer is c. (Murray, pp 481-497. Scriver, pp 3897-3964.) Prolonged vitamin C deficiency (scurvy) usually occurs with severe malnutrition (famine, prisoners of war, alcoholism, extreme food fadism). Exclusive feeding of cow's milk, as may occur in areas of famine with poor supplies of maternal milk, can result in infantile scurvy with the symptoms described in the question. X-rays of the limbs are helpful in diagnosing scurvy, with a white line at the metaphysis and occasional subperiosteal hemorrhage. These radiologic features may be seen in copper deficiency associated with hyperalimentation, emphasizing the role of ascorbic acid (vitamin C) as a coenzyme for proline/lysine hydroxylases that modify collagen and also require copper. The causes of hemorrhagic disease of the newborn are desribed in the previous answer, and vitamin K deficiency is almost never seen after the newborn period because of wide dietary availability. Deficiencies of the fatsoluble vitamins A, E, and D can occur with intestinal malabsorption, but avid fetal uptake during pregnancy usually prevents infantile symptoms. Vitamin D deficiency (rickets) can also cause a series of rib lumps (rosary) and is more likely with darker skin pigmentation but has other symptoms. Hypervitaminosis A can cause liver toxicity but not bleeding, and deficiency of vitamin E can be associated with anemia in prematures but is unknown in older children and adults.

A 1-year-old child recently emigrated from Africa exhibits intermittent diarrhea, pallor (pale skin), extreme tenderness of the bones, "rosary" of lumps along the ribs, nose bleeds, bruising over the eyelids, and blood in the urine. Which of the following is the most likely cause? a. Deficiency of vitamin C due to a citrus-poor diet during pregnancy b. Hypervitaminosis A due to ingestion of beef liver during pregnancy c. Deficiency of vitamin C because of reliance on a milk only diet d. Deficiency of vitamin K because of neonatal deficiency and continued poor nutrition e. Deficiency of vitamin D due to darker skin pigmentation and poor sun exposure

Correct answer = E. The elevated excretion of orotic acid indicates that the patient has orotic aciduria, a rare genetic disorder affecting the de novo pyrimidine biosynthetic pathway. Deficiencies in the enzyme activities OMP decarboxylase and/or orotate phosphoribosyltransferase (both of which are domains of the enzyme UMP synthase) leave the patient unable to synthesize pyrimidines. Uridine, a pyrimidine nucleoside, is useful in treating this disorder because it bypasses the missing enzymes and can be salvaged to UMP, which can be converted to all the other pyrimidines. Although thymidine is a pyrimidine nucleoside, it cannot be converted to other pyrimidines. Hypo xanthine, guanine, and adenine are all purine bases that have no value in helping to replace the missing pyrimidines.

A 1-year-old female patient is lethargic, weak, and anemic. Her height and weight are both low for her age. Her urine contains an elevated level of orotic acid. The administration of which of the following compounds is most likely to alleviate her symptoms? A. Adenine. B. Guanine. C. Hypoxanthine. D. Thymidine. E. Uridine.

The answer is a. (Murray, pp 293-302. Scriver, pp 2537-2570.) Most forms of gout are probably X-linked recessive with deficiencies in phosphoribosyl pyrophosphate (PRPP) synthase, the first step of purine synthesis (e.g., 311850). Some patients may have a partial deficiency of hypoxanthineguanine phosphoribosyl transferase (HGPRTase), which salvages hypoxanthine and guanine by transferring the purine ribonucleotide of PRPP to the bases and forming inosinate and guanylate, respectively. In all of these patients, the hypoxanthine analogue allopurinol has two actions: (1) it inhibits xanthine oxidase, which catalyzes the oxidation of hypoxanthine to xanthine and then to uric acid stones and tissue deposits; and (2) it forms an inactive allopurinol ribonucleotide from PRPP in a reaction catalyzed by HGPRTase, thereby decreasing the rate of purine synthesis. In contrast, because of the total loss of HGPRTase activity in Lesch-Nyhan patients, the allopurinol ribonucleotide cannot be formed. Thus, PRPP levels are not decreased and de novo purine synthesis continues unabated. The gouty arthritis caused by urate crystal formation is relieved in LeschNyhan patients, but their neurological symptoms (mental deficiency, selfmutilation with compulsive chewing of fingers and lips) are not.

A 1-year-old male has a normal birth and infantile history except for delay in sitting up, crawling, and standing (delayed motor milestones). He begins the unusual habit of chewing on his fingers and lips, and in one instance bites through the lip and leaves a large wound. His physician documents an elevated serum uric acid and suspects Lesch-Nyhan syndrome (300322). In considering potential therapy, the physician reads that purines are overproduced in gout and Lesch-Nyhan syndrome, causing hyperuricemia, yet the hypoxanthine analogue allopurinol is only effective in gout. Allopurinol does not treat the neurologic symptoms of Lesch-Nyhan syndrome because it does not do which the following? a. Decrease de novo purine synthesis b. Decrease de novo pyrimidine synthesis c. Diminish urate synthesis d. Increase phosphoribosylpyrophosphate (PRPP) levels e. Inhibit xanthine oxidase

The answer is d. (Murray, pp 481-497. Scriver, pp 4293-4326.) The major role of vitamin K is in the synthesis of prothrombin and other clotting factors (e.g., VII, IX, and X). Vitamin K acts on the inactive precursor molecules of these proteins, allowing carboxylation of glutamic acid residues to γ-carboxyglutamate. Once carboxylated, the factors bind calcium through these groups and are able to attach to cell membranes as part of clot formation. A true vitamin K deficiency in adults is unusual because vitamin K is found in a variety of foods and can be produced by intestinal bacteria. Liver, egg yolk, spinach, cauliflower, and cabbage are some of the sources of vitamin K. Poor fat metabolism, decreased liver function (reduced clotting factor synthesis), sterile gut with reduced bacterial metabolism (to produce menadiones) and decreased bile excretion in newborns renders them susceptible to vitamin K deficiency and hemorrhagic disease of the newborn, prevented by routine vitamin K injection in industrialized countries. Disorders with increased coagulation are treated with analogues of vitamin K that inhibit its conversion from dietary phylloquinone to hydroquine, epoxide, and quinone. Such inhibitors include coumadin and warfarin, a substance used as rat poison. Heparin is a complex polysaccharide that potentiates antithrombin III and inhibits clotting without effects on vitamin K.

A 10-day-old child arriving in the United States from a refugee camp is found to have large areas of bruising on the skin (purpura) with oozing of blood from his umbilicus. Which of the following statements regarding his likely vitamin deficiency and its mechanism are true? a. The vitamin is broken down by intestinal bacteria and facilitates synthesis of clotting factors b. The vitamin is antagonized by heparin and faciliates glycosylation of clotting factors c. The vitamin is obtained by eating egg yolk and liver but not green vegetables d. The vitamin is antagonized by a rat poison, is active in glutamate carboxylation, and facilitates calcium chelation e. The vitamin was discovered by studying hemorrhagic disease of the newborn

Correct answer = C. A deficiency of uroporphyrinogen III synthase results in accumulation of hydroxymethylbilane and the spontaneous conversion of this substrate to porphyrins of the Type I series. A deficiency of ALA synthase or inhibition of ALA dehydratase by lead would not allow the synthesis of porphobilinogen, the first pyrrole product in the heme biosynthetic pathway, and thus would not result in uro- or coproporphyrin synthesis. Deficiency of the glucuronyltransferase would not present with the systems described, and lab studies would be remarkable for an elevation of unconjugated bilirubin. Down-regulation of tyrosinase would result in decreased pigmentation.

A 10-year-old boy is referred to a specialist because of skin that blisters easily, urine that darkens on standing, and stained teeth. Lab studies are remarkable for high levels of uroporphyrin I and coproporphyrin I in plasma, with uroporphyrin I being present in the urine. The most likely biochemical pathology in this case is: A. deficiency of ALA synthase. B. deficiency of bilirubin glucuronyltransferase. C. deficiency of uroporphyrinogen III synthase. D. down-regulation of tyrosinase. E. inhibition of ALA dehydratase by lead

The answer is d. (Murray, pp 205-218. Scriver, pp 2863-2914.) The shell of apoproteins coating blood transport lipoproteins is important in the physiologic function of the lipoproteins. Some of the apoproteins contain signals that target the movement of the lipoproteins in and out of specific tissues. B48 and E seem to be important in targeting chylomicron remnants to be taken up by liver. B100 is synthesized as the coat protein of VLDLs and marks their end product, LDLs, for uptake by peripheral tissues. Other apoproteins are important for the solubilization and movement of lipids and cholesterol in and out of the particles. CII is a lipoprotein lipase activator that VLDLs and chylomicrons receive from HDLs. The A apoproteins are found in HDLs and are involved in lecithin-cholesterol acyl transferase (LCAT) regulation. Familial hypercholesterolemia (144010) causes early heart attacks in heterozygotes, particularly in males, and childhood disease in rare homozygotes. The daughter's chest pain was likely angina due to coronary artery occlusion and her skin patches were fatty deposits known as xanthomata.

A 10-year-old female presents with chest pain and unusual skin patches over her elbows and knees. Her father died of a heart attack at age 35 and her mother is known to have high cholesterol. Her physician suspects familial hypercholesterolemia (144010) in the parents with homozygous severe disease in the daughter. This disease results from mutations in the receptor for low-density lipoprotein (LDL) or the ligand portion of its apoprotein coat, which is which of the following? a. AI b. B48 c. CII d. B100 e. E

The answer is E: Homocystine. The boy is exhibiting the symptoms of homocystinuria, usually caused by a defect in cystathionine β-synthase. Cystathionine β-synthase will condense homocysteine with serine to form cystathionine. An inability to catalyze this reaction will lead to an accumulation of homocysteine, which will oxidize to form homocystine. The elevated serine can be metabolized back into the glycolytic pathway. Methionine will not increase in blood as the homocysteine produced is converted into homocystine. Phenylpyruvate is a diagnostic marker for PKU, but it is not relevant for homocysteine production or degradation. Fibrillin is mutated in Marfan syndrome, but this disorder is not Marfan syndrome.

A 12-year-old boy is brought to the pediatrician because of behavioral problems noted by the parents. Upon examination, the physician notices brittle and coarse hair, red patches on the skin, long, thin arms and legs (reminiscent of Marfan syndrome patients), scoliosis, pectus excavatum, displaced lens, and muscular hypotonia. Blood work is likely to show an elevation of which of the following metabolites? (A) Methionine (B) Phenylpyruvate (C) Cysteine (D) Fibrillin fragments (E) Homocystine

The answer is D: B6. Cystathionine β-synthase is a B6 requiring enzyme (the reaction is a β-elimination of the serine hydroxyl group, followed by a β-addition of homocysteine to serine; both types of reactions require the participation of B6). In some mutations, the affinity of the cofactor for the enzyme has been reduced, so significantly increasing the concentration of the cofactor will allow the reaction to proceed. The enzyme does not require the assistance of B1, B2, B3 (niacin), or B12 to catalyze the reaction.

A 12-year-old boy is brought to the pediatrician because of behavioral problems noted by the parents. Upon examination, the physician notices brittle and coarse hair, red patches on the skin, long, thin arms and legs (reminiscent of Marfan syndrome patients), scoliosis, pectus excavatum, displaced lens, and muscular hypotonia. Considering the patient described, treatment with which of the following vitamins may be successful in controlling this disorder? (A) B1 (B) B2 (C) B3 (D) B6 (E) B12

The answer is D: Glyoxylate. The boy has primary oxaluria type I, an autosomal recessive trait, which is a defect in a transaminase that converts glyoxylate to glycine. If this transaminase is defective, glyoxylate will accumulate. The glyoxylate will then be oxidized to oxalate, which, in the presence of calcium, will precipitate and form stones in the kidney. The metabolic pathway for glycine being converted to glyoxylate is shown below, and the enzyme that catalyzes this reaction is the D-amino acid oxidase. Alanine, leucine, and lysine metabolism do not give rise to oxalate.

A 13-year-old boy is admitted to the hospital due to fl ank and urinary pain. Analysis demonstrates the presence of kidney stones. The stones were composed of calcium oxalate. Family history revealed that the boy's father and mother had had similar problems. Oxalate accumulation arises in this patient due to difficulty in metabolizing which of the following? (A) Alanine (B) Leucine (C) Lysine (D) Glyoxylate (E) Glycine

Patients with Tangier disease have mutations in the ABC1 gene that is responsible for producing an abnormal protein that cannot transport cholesterol out of the cell. Thus patients with this rare genetic disorder have an accumulation of cholesterol in their cells. Characteristics of this disease are orange tonsils due to the high levels of intracellular cholesterol and extremely low plasma levels of high density lipoprotein (HDL). Lipoproteins are macromolecular structures composed of an inner core of cholesterol esters and triglycerides and an outer core of apoproteins, phospholipids and unesterified free cholesterol. The major types of lipoproteins include chylomicrons synthesized in the intestinal enterocytes; very-low density lipoproteins ( VLDLs) synthesized in the liver; intermediate-density lipoproteins (IDLs) formed within the circulation from VLDL catabolism; low-density lipoproteins (LDLs) formed from VLDL catabolism in the circulation; and high-density lipoproteins (HDLs) also formed from VLDL catabolism within the circulation. High levels of HMGCoA reductase is incorrect. The presence of high cholesterol levels in cells leads to low levels of HMGCoA reductase. Elevated percentage of cholesterol in cell membrane is incorrect. Patients with Tangier disease have high intracellular levels of cholesterol that remain intracellular and not in the cell membrane. High apo-A-1 levels is incorrect. Apo-A-1 is incorrect. Apo-A-1 activates lecithin-cholesterol acyltransferase (LCAT) which catalyzes the esterification of cholesterol to form HDL. Since HDL levels are low, the major HDL lipoprotein, Apo-A-1 levels are low as well in Tangier disease patients. Very high levels of apoliprotein B-100 is incorrect. B-100 binds to LDL receptor and mediates VLDL secretion. LDL levels are ~40% below normal in Tangier patients so apo-B-100 levels are lower as well.

A 15-year-old boy presents with a sore throat, fatigue and malaise. Physical exam reveals yellow-orange tonsils, hepatomegaly and splenomegaly. Further tests show thrombocytopenia, anemia, corneal opacity and hypocholesterolemia indicating this patient has Tangier disease, a rare autosomal recessive disorder in which cholesterol is not transported out of the cell. Due to increased cellular cholesterol levels, what other lipoprotein abnormality would be expected in these patients? A. High levels of HMGCoA reductase B. Elevated percentage of cholesterol in cell membrane C. High apo-A1 levels D. Very high levels of apolipoprotein B-100 E. Extremely low levels of HDL

Under fasting conditions, the liver synthesizes glucose by the process of gluconeogenesis; this provides glucose for the organs and cells (e.g., the brain, red blood cells) that rely exclusively on glucose for energy (under normal conditions). The gluconeogenic enzyme phosphoenolpyruvate (PEP) carboxykinase can be induced under these conditions. In addition, 2 other gluconeogenic enzymes, fructose 1,6-bisphosphatase and glucose 6-phosphatase, can be induced during fasting/starvation conditions. At the same time, the liver glycolytic enzymes (i.e., glucokinase, phosphofructokinase, and pyruvate kinase) are repressed during fasting/starvation conditions; this preserves glucose, which is then released into the bloodstream from the liver. While gluconeogenesis uses several of the same enzymes as glycolysis, it also uses 4 unique enzymes to bypass the thermodynamically unfavorable reactions of glycolysis. In gluconeogenesis, pyruvate is converted to oxaloacetate by pyruvate carboxylase and then to phosphoenolpyruvate by phosphoenolpyruvate carboxykinase. The enzymes fructose 1,6-bisphosphatase and glucose 6phosphatase remove a phosphate-forming fructose 6-phosphate and glucose, respectively. This is a part of gluconeogenesis. The mechanism of the induction of these gluconeogenesis enzymes involves the increase in cAMP levels in response to glucagon, the hormone released in response to low blood glucose levels. This leads to kinase activation, which in turn activates coactivators of transcription factors required to upregulate the transcription of phosphoenolpyruvate carboxykinase, fructose 1,6-bisphosphatase, and glucose 6phosphatase. Triose phosphate isomerase is an enzyme used by both gluconeogenesis and glycolysis to interconvert glyceraldehyde 3-phosphate and dihydroxyacetone phosphate.

A 16-year-old girl presents with a 6-month history of not eating very much while attempting to lose weight. Her current weight is 90 pounds, she is 5' 2'', and she has not had her period in 6 months. As a consequence of her condition, levels of several metabolic enzymes are affected. What enzyme will be induced in the liver of this patient? A. Glucokinase B. Phosphoenolpyruvate carboxykinase C. Pyruvate kinase D. Phosphofructokinase E. Triose phosphate isomerase

The correct answer is vitamin D because it is a fat-soluble vitamin. The patient takes orlistat, a gastrointestinal and pancreatic lipase inhibitor that induces weight loss by inhibiting dietary fat absorption, and as a consequence fat-soluble vitamins are also eliminated. Vitamin B12, vitamin C,and folic acid are incorrect because these are all vitamins that are water-soluble; there is no influence of the drug on the absorption of these vitamins. Iron is an incorrect response. A lack of iron in the blood causes anemia, which can be also induced by a deficiency in vitamin K. Even though a decrease in vitamin K might be noticed, as it is also a fat-soluble vitamin, studies have shown that the decrease in vitamin K is only significant when orlistat is administered in combination with warfarin.

A 16-year-old obese girl has a BMI (Body Mass Index) of 32 kg/m² and a family history of diabetes; she has tried to lose weight before without success, so she is prescribed orlistat, which is taken after each meal. Within 2 weeks, she notices bowel movement changes, flatulence, oily discharge, and mild abdominal pain; she returns to the office. Because she lost some weight, she would like to continue the treatment if the side effects could be eliminated. In addition to telling her to reduce her fat consumption, you should prescribe what supplement? A. Vitamin B12 B. Vitamin C C. Vitamin D D. Folic acid E. Iron

The answer is a. (Murray, pp 481-497. Scriver, pp 5121-5138.) Vitamins A, D, E, and K are all fat-soluble. The physical characteristics of fat-soluble vitamins derive from the hydrophobic nature of the aliphatic chains composing them. The other vitamins listed are water-soluble, efficiently administered orally, and rapidly absorbed from the intestine. Fatsoluble vitamins must be administered intramuscularly or as oral emulsions (mixtures of oil and water). In intestinal disorders such as chronic diarrhea or malabsorption due to deficient digestive enzymes, fatsoluble vitamins are poorly absorbed and can become deficient. Supplementation of fat-soluble vitamins is thus routine in disorders like cystic fibrosis (219700), a cause of respiratory and intestinal disease that is the likely diagnosis in this child.

A 2-year-old child presents with neonatal meconium ileus, chronic cough and bronchitis, growth failure, and chronic diarrhea with lightcolored, foul-smelling stools. A deficiency of which of the following vitamins should be considered? a. Vitamin A b. Vitamin C c. Vitamin B1 d. Vitamin B2 e. Vitamin B6

The answer is a. (Murray, pp 122-129, 173-179. Scriver, pp 2297-2326.) Fatty acid oxidation is a major source of energy after glycogen is depleted during fasting. Fatty acids are first coupled with coenzyme A, transferred for mitochondrial import as acylcarnitines, and degraded in steps that remove two carbons. The fatty acyl CoA dehydrogenases, enoyl hydratases, hydroyacyl CoA dehydrogenases, and thiolases that carry out each oxidation step are present in three groups with specifities for very long/ long, medium, and short chain fatty acyl esters. As would be expected, deficiencies of long-chain oxidizing enzymes have more severe consequences than those for short chains since they impair many more cycles of two-carbon removal. Long chain deficiencies may be lethal in the newborn period, while medium or short chain deficiencies may be undetected until a child goes without food for a prolonged time and must resort to extensive fatty acid oxidation for energy. Medium chain coenzyme A dehydrogenase (MCAD) deficiency (201450) can be fatal if not recognized, and sometimes presents as Sudden Unexplained Death Syndrome (SUDS—usually at older ages than Sudden Infant Death Syndrome—SIDS—that is mostly from respiratory problems).The deficit of acetyl CoA from fatty acid oxidation impacts gluconeogenesis with hypoglycemia, and the energy deficit leads to heart, liver, and muscle disease that may be lethal. Unlike most causes of hypoglycemia, the impaired fatty acid oxidation does not produce ketones (nonketotic hypoglycemia). Carnitine is tied up as medium chain acylcarnitines and is secondarily deficient in fatty acid oxidation disorders. Rare primary carnitine deficiencies (as in answer option e) impair oxidation of all fatty acids because they cannot be imported into mitochondria.

A 2-year-old girl has been healthy until the past weekend when she contracted a viral illness at day care with vomiting, diarrhea, and progressive lethargy. She presents to the office on Monday with disorientation, a barely rousable sensorium, cracked lips, sunken eyes, lack of tears, flaccid skin with "tenting" on pinching, weak pulse with low blood pressure, and increased deep tendon reflexes. Laboratory tests show low blood glucose, normal electrolytes, elevated liver enzymes, and (on chest x-ray) a dilated heart. Urinalysis reveals no infection and no ketones. The child is hospitalized and stabilized with 10% glucose infusion, and certain admission laboratories come back 1 week later showing elevated medium chain fatty acyl carnitines in blood and 6-8 carbon dicarboxylic acids in the urine. The most likely disorder in this child involves which of the following? a. Defect of medium chain coenzyme A dehydrogenase b. Defect of medium chain fatty acyl synthetase c. Mitochondrial defect in the electron transport chain d. Mitochondrial defect in fatty acid transport e. Carnitine deficiency

B.

A 2-year-old girl, the daughter of an African immigrant, is admitted to the pediatric ward due to an increase in abdominal girth and failure to thrive. She recently arrived in the U.S. from her home country. She was breast-fed until one year of age, at which time her mother ran out of milk. She is apathetic and irritable and has been having frequent episodes of a diarrhea, dehydration, loss of muscle and weight loss. Labs:CBC: anemia; lymphopenia. Hypoalbuminemia. Lytes: hypokalemia, hypomagnesemia. Q:Which form of form of malnutrition caused by this condition : A.Selenium B.Marasmus C. Vitamin C D.Copper E.Fluoride

A.

A 21-year-old woman presents with severe abdominal pain and cramping. She denies nausea, vomiting, diarrhea, blood in stools, and jaundice. She states that she went on an alcoholic binge for the first time the night before. She is afebrile. Her past medical history is significant for recurrent bouts of abdominal pain of unexplained origin. Her temperature is 36.7°C (98°F), blood pressure is 110/65 mm Hg, pulse is 80 beats/min, and respirations are 18/min. Examination reveals no rebound tenderness, no hepatosplenomegaly, and negative Murphy's sign. The patient has an extensive workup. CT scan of the abdomen is negative. Question: Based on the suspected diagnosis, what additional finding would be most likely? The laboratory values are: Hemoglobin 7.2 g/dL MCV 72 fL Leukocyte count 9,000 cells/cmm Platelets 200,000/mL Serum sodium 142 mEq/L Serum potassium 5.0 mEq/L Serum calcium 9.8 mg/dL Serum phosphorus 2.5 mg/dL A. Red-tinted urine B. Elevated erythrocyte sedimentation rate C. Increased conjugated bilirubin D. Increased serum lead levels E. Increased urinary copper

A.

A 22-year-old female has a family history of phenylketonuria but does not have the metabolic disorder herself. The female is concerned that her unborn child may have phenylketonuria and asks her physician if the child's development could be affected. The physician reassures the female that her normal metabolism would supplement the needs of the developing fetus and that phenylketonuric symptoms only develop after birth. In an infant with phenylketonuria, treatment is possible by strictly monitoring the dietary intake of which amino acid? A. Phenylalanine B. Tyrosine C. Phenylpyruvate D. Tryptophan E. L-dopa

The answer is D. Creatine is synthesized from glycine, arginine, and S-adenosylmethionine (thus, intake of dietary creatine is not relevant). In muscle, creatine is converted to creatine phosphate, which is nonenzymatically cyclized to form creatinine (thus, C is incorrect). The amount of creatinine excreted by the kidneys each day depends on body muscle mass but is constant for each individual (therefore, if there is an increase in body muscle mass, there would be an increase in creatinine excretion). In kidney failure, the excretion of creatinine into the urine is low, and an elevation of serum creatinine would be observed.

A 24-hour urine collection showed that an individual's excretion of creatinine was much lower than normal. Decreased excretion of creatinine could be caused by which of the following? A. A decreased dietary intake of creatine B. A higher than normal muscle mass resulting from weight lifting C. A genetic defect in the enzyme that converts creatine phosphate to creatinine D. Kidney failure E. A vegetarian diet

A.

A 25-year-old woman comes to your office for a routine physical. She is in very good health and indicates that she and her husband would like to have a child. She tells you that she has a sister that had a child with spina bifida and is concerned about this. You advise her to start taking a vitamin which has been shown to greatly reduce the chances of having a baby born with spina bifida. This vitamin is A. Folic acid B. Vitamin D C. Vitamin C D. Niacin E. Vitamin B12

A.

A 27- year- old man is brought to the emergency room after a car accident. Toxicology screen shows a high level of ethanol in his blood. Oxidation of ethanol produces acetaldehyde and NADH. A high level of NADH/NAD+ ratio promotes the conversion of which of the following: a. Dihydroxyacetone phosphate to glycerol 3-P b. Citrate to isocitrate c. Pyruvate to acetyl CoA d. Malate to oxaloacetate e. Succinate to fumarate

The answer is C: Tyrosine. The boy has the inherited disorder tyrosinemia type I, which is a defect in fumarylacetoacetate hydrolase, the last step in the degradation pathway for tyrosine (see the figure below). In its acute form, this disorder will lead to liver failure and death within 1 year of life. The accumulation of intermediates in the tyrosine degradation pathway triggers apoptosis

A 3-month-old boy of French-Canadian ancestry is seen by the pediatrician for failure to thrive and poor appetite. Physical exam denotes hepatomegaly and a yellowing of the eyes. The boy had been vomiting and had diarrhea, and a distinct cabbagelike odor was apparent. This disorder is due to a defect in the metabolism of which of the following amino acids? (A) Alanine (B) Tryptophan (C) Tyrosine (D) Histidine (E) Lysine

The answer is b. (Murray, pp 481-497. Scriver, pp 2275-2296.)An elevation of pyruvate and a deficiency of acetyl-CoA suggest a deficiency of pyruvate dehydrogenase (PDH). This multisubunit enzyme assembly contains pyruvate dehydrogenase, dihydrolipoyl transacetylase, dihydrolipoyl dehydrogenase, and two enzymes involved in regulation of the overall enzymatic activity of the complex. PDH requires thiamine pyrophosphate as a coenzyme, dihydrolipoyl transacetylase requires lipoic acid and CoA, and dihydrolipoyl dehydrogenase has an FAD prosthetic group that is reoxidized by NAD+. Biotin, pyridoxine, and ascorbic acid are not coenzymes for PDH. An ATP-dependent protein kinase can phosphorylate PDH to decrease activity, and a phosphatase can activate PDH. Increases of ATP, acetyl-CoA, or NADH (increased energy charge) and of fatty acid oxidation increase phosphorylation of PDH and decrease its activity. PDH is less active during starvation, increasing pyruvate, decreasing glycolysis, and sparing carbohydrates. Free fatty acids decrease PDH activity and would not be appropriate therapy for PDH deficiency. PDH deficiency (246900, 312170) exhibits genetic heterogeneity, as would be expected from its multiple subunits, with autosomal and X-linked recessive forms. The infant also could be classified as having Leigh's disease (266150), a heterogenous group of disorders with hypotonia and lactic acidemia that can include PDH deficiency.

A 3-month-old boy presents with poor feeding and growth, low muscle tone (hypotonia), elevation of blood lactic acid (lactic acidemia), and mild acidosis (blood pH 7.3 to 7.35). The ratio of pyruvate to lactate in serum is elevated, and there is decreased conversion of pyruvate to acetyl coenzyme A in fibroblasts. Which of the following compounds should be considered for therapy? a. Pyridoxine b. Thiamine c. Free fatty acids d. Biotin e. Ascorbic acid

The answer is b. (Scriver, pp 2297-2326, 121-138, 287-320.) Untreated ethylene glycol of antifreeze can be converted to the kidney toxin oxalate crystals. This occurs by oxidation of ethylene glycol. The first committed step in this process is the oxidation of ethylene glycol to an aldehyde by alcohol dehydrogenase. This is normally the route for converting ethanol (drinking alcohol) to acetate. Patients who have ingested ethylene glycol or wood alcohol (methanol) are placed on a nearly intoxicating dose of ethanol by a nasogastric tube together with intravenous saline and sodium bicarbonate. This treatment is carried out intermittently along with hemodialysis until no traces of ethylene glycol are seen in the blood. Ethanol acts as a competitive inhibitor of alcohol dehydrogenase with respect to ethylene glycol or methanol metabolism.

A 3-year-old child is brought into the ER while you are on duty. She is cold and clammy and is breathing rapidly. She is obviously confused and lethargic. Her mother indicates she has accidentally ingested automobile antifreeze while playing in the garage. Following gastrointestinal lavage and activated charcoal administration, which of the following treatments should you immediately initiate? a. Intravenous infusion of oxalic acid b. Nasogastric tube for ethanol administration c. Flushing out the bladder via a catheter d. Intramuscular injection of epinephrine e. Simply waiting and measuring vital signs

B.

A 32-year-old, vegetarian woman in mid-pregnancy complains of lack of energy and says she becomes easily fatigued. Upon any strenuous movement, her heart pounds rapidly and she becomes short of breath. Question: Nutritional supplement of what mineral may alleviate the patient's symptoms? A. Cu2+ B. Fe2+ C. Mg2+ D. Zn2+ E. Ca2+

Pregnant females can experience iron deficiency anemia due to increased demands on their blood. Oxygen (O2) in the lungs binds to the iron ion, Fe2+ while complexed with the heme cofactor of hemoglobin in red blood cells. Iron supplements or foods with abundant iron, such as liver, lean meats, or vegetarian alternatives such as spinach, carrots, and raisins can alleviate anemic symptoms. The other minerals have biological roles but are not associated with anemic symptoms. Magnesium (Mg2+) coordinates with the negatively charged backbone of DNA and interacts with neurotransmitter receptors at excitatory synapses in the central nervous system. Mg2+ deficiency affects the nervous system, resulting in vasodilation, tremors, and depression. Calcium phosphate forms a hard material in bone and teeth. In addition, Ca2+ is a ubiquitous second messenger ion in cellular signaling coupled to G-protein signaling, hormone signaling, and ion channel activity. Ca2+ deficiency can give rise to muscle twitching or cramping and cardiac arrhythmias. Copper (Cu2+) participates in bone and blood formation and is an electron carrier in mitochondrial electron transfer proteins. Cu2+ deficiency is uncommon since the trace amounts needed are satisfied by most diets. Zinc (Zn2+) is a cofactor of many DNA and RNA binding proteins, including many transcription factors. Severe zinc deficiency can retard growth in children, can cause low sperm count in males, and can slow wound healing.

A 32-year-old, vegetarian woman in mid-pregnancy complains of lack of energy and says she becomes easily fatigued. Upon any strenuous movement, her heart pounds rapidly and she becomes short of breath. Nutritional supplement of what mineral may alleviate the patient's symptoms? A. Mg2+ B. Ca2+ C. Fe2+ D. Cu2+ E. Zn2+

The answer is b. (Murray, pp 237-241. Scriver, pp 2961-3062.) The porphyrias are a group of inborn errors that affect synthesis of porphyrins, the precursors of heme in hemoglobin. Defective synthesis of heme would not elevate heme breakdown products of the heme catabolic pathway, including bilirubin to conjugated bilirubin diglucuronide (in liver), bilirubin diglucuronide to urobilinogen and stercobilin (by bacteria in stool), and reabsorption of urobilinogen to be excreted in urine as urobilin. Delta-aminolevulinic acid (ALA) is synthesized from succinyl CoA and glycine followed by condensation of two ALA molecules to form porphobilinogen (PBG) with a 5-member pyrrole ring. Four molecules of PBG are converted to the four-ring uroporphyrin by hydroxymethylbilane synthase, the primary defect in acute intermittent porphyria (176000). Deficiencies in other enzymes of the pathway from ALA to heme cause symptoms varying from anemia to photosensitivity to the well-known but rarely encountered presentation with abdominal pain and neuropsychiatric symptoms.

A 35-year-old man presents to the emergency room with an acute abdomen (severe abdominal pain with tightness of muscles, decreased bowel sounds, and vomiting and/or diarrhea). He has been drinking, and a urine sample is unusual because it has a port-wine color. Past history indicates several prior evaluations for abdominal pain, including an appendectomy. The physician notes unusual neurologic symptoms with partial paralysis of his arms and legs. At first concerned about food poisons like botulism, the physician recalls that acute intermittent porphyria may cause these symptoms (176000) and consults a gastroenterologist. Elevation of which of the following urinary metabolites would support a diagnosis of porphyria? a. Urobilinogen and bilirubin b. Delta-aminolevulinic acid and porphobilinogen c. Biliverdin and stercobilin d. Urobilin and urobilinogen e. Delta-aminolevulinic acid and urobilinogen

The answer is B. The recommended total fat intake is less than 30% of total calories. His total caloric consumption was 4,110 kcal/day (thus, A is incorrect). His fat intake was 21% (855/4,110) of his total caloric intake. His alcohol intake was 7.7% (315/4,110) (thus, C is incorrect). His protein intake was well above the RDA of 0.8 g/kg body weight (thus, D is incorrect). His RMR is roughly 24 kcal/day/kg body weight or 2,880 kcal/day (it will actually be less because he is obese and has a greater proportion of metabolically less active tissue than the average 70-kg man). His daily energy expenditure is about 3,744 kcal/day (1.3 X 2,880) or less. Thus, his intake is greater than his expenditure, and he is in positive caloric balance and is gaining weight (thus, E is incorrect).

A 35-year-old sedentary male patient weighing 120 kg was experiencing angina (chest pain) and other signs of coronary artery disease. His physician, in consultation with a registered dietician, conducted a 3-day dietary recall. The patient consumed an average of 585 g of carbohydrate, 150 g of protein, and 95 g of fat each day. In addition, he drank 45 g of alcohol. The patient's diet is best described by which one of the following? A. He consumed between 2,500 and 3,000 kcal/day. B. He had a fat intake within the range recommended in current dietary guidelines (i.e., year 2010). C. He consumed 50% of his calories as alcohol. D. He was deficient in protein intake. E. He was in negative caloric balance.

C.

A 35-year-old woman comes to the hospital with severe trauma and hemorrhage after a car accident. Her prothrombin time is increased. Which of the following cells is likely to be damage? a. Kupffer cells b. Endothelial cells c. Hepatocytes d. Stellate cells

B.

A 35-year-old woman is evaluated for jaundice in an emergency department. For several days, the patient has had mild flu-Iike symptoms of anorexia, nausea and vomiting, fatigue, Iow-grade fever, and malaise. This morning, she noted that her urine was brown in color, and she has also today developed moderate, steady, pain of the right upper quadrant of her abdomen. She has not had any similar episodes in the past.On physical examination, the patient is noted to be jaundiced and to have an enlarged, tender liver. BIood chemistry studies are notable for alanine aminotransferase (ALT) of 15,000 mIU/L, aspartate aminotransferase (AST) of 11,000 mIU/L, and alkaline phosphatase of 100 U/L.Q:This patient's dark urine is due to the presence of which of the following? A. Bacteria B. Bilirubin C. Hemosiderin D. Ketone bodies E. Melanin

Correct answer = C. The patient's glucagon level will be elevated in response to the hypoglycemia. She is most likely experiencing alcohol-induced fasting hypoglycemia. Blood glucose is expected to be 40 mg/dl or less, insulin secretion depressed because of the low blood glucose, and liver glycogen levels low because of the fast. An insulinoma, an insulin-producing tumor of the pancreas, is unlikely.

A 39-year-old woman is brought to the emergency room complaining of dizziness. She recalls getting up early that morning to do as much shopping as possible and had skipped breakfast. She drank a cup of coffee for lunch and had nothing to eat during the day. She met with friends at 8 p.m. and had a drink at the bar. She soon became weak and dizzy and was transported to the hospital. Following examination, the patient was given orange juice and immediately felt better. Which one of the following best completes this sentence? "The patient has": A. blood glucose greater than 70 mg/dl. B. elevated insulin. C. elevated glucagon. D. elevated liver glycogen. E. presence of an insulinoma.

E.

A 4-year old girl presents in the clinic with megaloblastic anemia and failure to thrive. Blood chemistries reveal orotic aciduria. Enzyme measurements of white blood cells reveal deficiency of the pyrimidine biosynthesis enzyme orotate phosphoribosyltransferase and abnormally high activity of the enzyme aspartate transcarbamolayse. Which one of the following treatments will reverse all symptoms if carried out chronically? a. Blood transfusion b. White blood cell transfusion c. Dietary supplements of phosphoribosylpyrophosphate (PRPP) d. Oral thymidine e. Oral uridine

Correct answer = B. Alkaptonuria is a rare metabolic disease involving a deficiency in homogentisic acid oxidase, and the subsequent accumulation of homogentisic acid in the urine, which turns dark upon standing. The elevation of methylmalonate (due to methylmalonyl CoA mutase deficiency), phenylpyruvate (due to phenylalanine hydroxylase deficiency), αketoisovalerate (due to branched-chain α-keto acid dehydrogenase deficiency), and homocysteine (due to cystathionine β-synthase deficiency) are inconsistent with a healthy child with darkening of the urine.

A 4-year-old boy of a first-degree consanguineous couple was noted by the parents to have darkening of the urine to an almost black color when it was left standing. He has a normal sibling, and there are no other medical problems. Growth and development to date are normal. Which of the following is most likely to be elevated in this patient? A. Methylmalonate. B. Homogentisate. C. Phenylpyruvate. D. α-Ketoisovalerate. E. Homocysteine.

The answer is e. (Murray, pp 293-302. Scriver, pp 2663-2704.) Orotic aciduria (258900) is the buildup of orotic acid due to a deficiency in one or both of the enzymes that convert it to UMP. Either orotate phosphoribosyltransferase and orotidylate decarboxylase are both defective, or the decarboxylase alone is defective. UMP is the precursor of UTP, CTP, and TMP. All of these end products normally act in some way to feedbackinhibit the initial reactions of pyrimidine synthesis. Specifically, the lack of CTP inhibition allows aspartate transcarbamoylase to remain highly active and ultimately results in a buildup of orotic acid and the resultant orotic aciduria. The lack of CTP, TMP, and UTP leads to a decreased erythrocyte formation and megaloblastic anemia. Uridine treatment is effective because uridine can easily be converted to UMP by omnipresent tissue kinases, thus allowing UTP, CTP, and TMP to be synthesized and feedback-inhibit further orotic acid production.

A 4-year-old girl presents in the clinic with megaloblastic anemia and failure to thrive. Blood chemistries reveal orotic aciduria (258900). Enzyme measurements of white blood cells reveal a deficiency of the pyrimidine biosynthesis enzyme orotate phosphoribosyltransferase and abnormally high activity of the enzyme aspartate transcarbamoylase. Which of the following treatments will reverse all symptoms if carried out chronically? a. Blood transfusion b. White blood cell transfusion c. Dietary supplements of phosphoribosylpyrophosphate (PRPP) d. Oral thymidine e. Oral uridine

The answer is d. (Murray, pp 270-285. Scriver, pp 2961-3104.) Once bile is excreted into the gut, bilirubin diglucuronide is hydrolyzed and reduced by bacteria to form urobilinogen, which is colorless. Much of the urobilinogen of the stools is further oxidized by intestinal bacteria to stercobilin, which gives stools their characteristic brown color. Some urobilinogen is reabsorbed by the gut into the portal blood, transported to the kidney, and converted and excreted as urobilin, which gives urine its characteristic yellow color. The woman has usual risk factors for cholecystitis (inflammation of the gall bladder) remembered as fair, fat, and forty. The inflammation can block excretion of conjugated bilirubin into the intestine, reducing oxidation to stercobilin and producing white (acholic) stools.

A 40-year-old woman of fair complexion is admitted for evaluation of acute vomiting with abdominal pain. The episode began the night before after a fatty meal, and she has noted her stools are a peculiar grey white color. Abdominal examination is difficult because she is obese, but she exhibits acute tenderness in the right upper quadrant and has pain just below her left shoulder blade. Interference with which aspect of porphyrin metabolism best accounts for the white stools? a. Sterile gut syndrome with defective bilirubin oxidation b. Excess oxidation of bilirubin to urobilinogen c. Heme synthesis defect causing increased bilirubin clearance d. Bile duct excretion of bilirubin with oxidation to stercobilin e. Excess reabsorption of urobilinogen with excess Urobilin

Correct answer = E. The patient's pain is caused by gout, resulting from an inflammatory response to the crystallization of excess uric acid in his joints. Radiation therapy caused cell death, with degradation of nucleic acids and their constituent purines. Uric acid, the end product of purine degradation, is a relatively insoluble compound that can cause gout and kidney stones. Pyrimidine metabolism is not associated with uric acid production. Overproduction of purines can indirectly result in hyperuricemia. Purine salvage decreases uric acid production.

A 42-year-old male patient undergoing radiation therapy for prostate cancer develops severe pain in the metatarsal phalangeal joint of his right big toe. Monosodium urate crystals are detected by polarized light microscopy in fluid obtained from this joint by arthrocentesis. Uric acid crystals are present in his urine. This patient's pain is directly caused by the overproduction of the end product of which of the following metabolic pathways? A. De novo pyrimidine biosynthesis. B. Pyrimidine degradation. C. De novo purine biosynthesis. D. Purine salvage. E. Purine degradation.

D.

A 43-year-old man with chronic renal failure who requires hemodialysis complains of generalized bone pain. Physical examination shows bowing of the legs and a sallow complexion. When the man's blood pressure is taken, his thumb flexes into his palm. A radiograph of the bones shows generalized osteopenia. The patient most likely has a deficiency of which of the following vitamins? A. Niacin B. Thiamine C. Vitamin A D. Vitamin D E. Vitamin K

B.

A 44-year-old emaciated man is brought to the emergency room towards late morning by his friends. They report he was difficult to rouse. He has not eaten anything since a large meal the night before. His friends report a 25-year history of alcohol abuse. Examination shows severely restricted horizontal eye movements and ataxia of both lower limbs. What nutritional deficiency can cause this patient's symptoms? A. Vitamin B12 B. Vitamin B1 C. Folate D. Vitamin A E. Vitamin B4

The answer is b. (Murray, pp 205-230. Scriver, pp 2863-2914.) In the postabsorptional (postprandial) state, plasma contains all the lipoproteins: chylomicrons derived from dietary lipids packaged in the intestinal epithelial cells and their remnants; very-low-density lipoproteins (VLDLs), which contain endogenous lipids and cholesterol packaged in the liver; low-density lipoproteins (LDLs), which are end products of delipidation of VLDLs; and high-density lipoproteins (HDLs), which are synthesized in the liver. HDLs are in part catalytic, since transfer of their CII apolipoprotein to VLDLs or chylomicrons activates lipoprotein lipase. In normal patients, only LDLs and HDLs remain in plasma following a 12 hour fast, since both chylomicrons and VLDLs have been delipidated. Most of the cholesterol measured in blood plasma at this time is present in the cholesterol-rich LDLs. However, HDL cholesterol also contributes to the measurement. In addition to total plasma cholesterol, the ratio of HDL (good) to LDL (bad) cholesterol is also useful for predicting heart attack risks

A 45-year-old man is found to have an elevated serum cholesterol of 300 mg percent measured by standard conditions after a 12-hour fast. Which of the following lipoproteins would contribute to a measurement of plasma cholesterol in a normal person following a 12-hour fast? a. Very-low-density lipoproteins (VLDL) and low-density lipoproteins (LDL) b. High-density lipoproteins (HDL) and low-density lipoproteins (LDL) c. Chylomicrons and very-low-density lipoproteins (VLDL) d. Chylomicron remnants and very-low-density-lipoproteins (VLDL) e. Low-density lipoproteins (LDL) and adipocyte lipid droplets

A.

A 48-year-old man, who was born and raised in Mexico, complains of chronic diarrhea. His diet consists mainly of corn and corn-related products. He has a raised, hyper pigmented rash around his neck and on the back of both hands and forearms. Which of the following vitamins is most likely deficient in this patient? A. Niacin B. Pyridoxine C. Thiamine D. Vitamin A E. Vitamin C

The correct answer is vitamin B12. Deficiency of vitamin B12 leads to numbness and tingling of the hands and feet, weakness, fatigue, and anemia. It is one of the more common vitamin deficiencies to occur after gastric bypass surgery. Gastric bypass eliminates a substantial portion of the stomach where intrinsic factor is produced. Intrinsic factor is a carrier protein for vitamin B12 that helps it be adsorbed by the ileum. Vitamin B12 is required for 2 reactions in humans: the methylation of homocysteine to form methionine and the conversion of methylmalonyl CoA to succinyl CoA. The metabolism of vitamin B12 is tied to folic acid metabolism; therefore, a deficiency in vitamin B12 can lead to a metabolic deficiency of folic acid. This lack of usable folic acid leads to deficiencies in purine and dTMP synthesis. This, in turn, affects nucleotide synthesis needed for DNA synthesis. Rapidly dividing cells, such as blood cells, are affected, leading to pernicious anemia in cases of vitamin B12 deficiency. The absorption of biotin, thiamine, and vitamins K are usually less affected by the gastric bypass procedure. A deficiency in vitamin D can occur after gastric bypass but would not cause peripheral neuropathy. Deficiencies in vitamin D would manifest themselves in bone and calcium problems.

A 48-year-old woman underwent gastric bypass surgery. She followed the prescribed diet and lost approximately 100 pounds in 18 months. She presents with numbness in her feet, and blood tests reveal a vitamin deficiency. Based on these observations, this woman is most likely deficient in what vitamin? A. Biotin B. Thiamine C. Vitamin B12 D. Vitamin D E. Vitamin K

B.

A 5-year-old male is being evaluated for cognitive impairment, behavioral problems and mild anemia. He lives in an 80-year-old house. Which of the following enzymes would be most sensitive to environmental toxic exposure in this patient? A. Ö-Aminolevulinate synthase B. Ö-Aminolevulinate dehydratase C. Uroporphyrinogen I synthase D. Uroporphyrinogen decarboxylase E. Bilirubin glucuronyl transferase

Correct answer = A. The disease is associated with a deficiency in uroporphyrinogen decarboxylase, but clinical expression of the enzyme deficiency is influenced by hepatic injury caused by iron overload, chronic ethanol consumption, and the presence of hepatitis B or C and HIV infections. Exposure to sunlight can also be a precipitating factor. Clinical onset is typically during the fourth or fifth decade of life. Porphyrin accumulation leads to cutaneous symptoms and urine that is red to brown. Treatment of the patient's seizure disorder with phenytoin caused increased synthesis of ALA synthase, and, therefore, of uroporphyrinogen, the substrate of the deficient enzyme. The laboratory and clinical findings are inconsistent with other porphyrias.

A 50-year-old man presented with painful blisters on the backs of his hands. He was a golf instructor, and indicated that the blisters had erupted shortly after the golfing season began. He did not have recent exposure to poison ivy or sumac, new soaps or detergents, or new medications. He denied having previous episodes of blistering. He had partial complex seizure disorder that had begun about three years earlier after a head injury. The patient had been taking phenytoin—his only medication—since the onset of the seizure disorder. He admitted to an average weekly ethanol intake of about eighteen 12oz cans of beer. The patient's urine was reddish orange. Cultures obtained from skin lesions failed to grow organisms. A 24-hour urine collection showed elevated uroporphyrin (1,000 mg; normal, <27mg). The most likely presumptive diagnosis is: A. porphyria cutanea tarda. B. acute intermittent porphyria. C. hereditary coproporphyria. D. congenital erythropoietic porphyria. E. erythropoietic protoporphyria.

The answer is C. Platelet aggregation is often a determining factor in heart attacks. Thromboxane A2, produced by platelets, promotes platelet aggregation when clotting is required, and inhibition of thromboxane A2 synthesis by aspirin reduces the potential for inappropriate clot formation, and further heart attacks. Thromboxane A2 is produced from arachidonic acid by the action of cyclooxygenase, the enzyme covalently modified and irreversibly inhibited by aspirin. Leukotrienes are also synthesized from arachidonic acid, but utilize lipoxygenase in their synthesis, which is not inhibited by aspirin. Cholesterol, triglyceride, and cytokine synthesis do not require cyclooxygenase activity.

A 52-year-old man, after suffering a heart attack, was put on 81 mg of aspirin daily by his cardiologist. The purpose of this treatment is to reduce the levels of which one of the following? (A) Cytokines (B) Leukotrienes (C) Thromboxanes (D) Cholesterol (E) Triglycerides

The answer is C: dATP. Due to the lack of ADA activity, adenosine accumulates and is converted to AMP by adenosine kinase (and deoxyadenosine is converted to dAMP). The dAMP will eventually be converted to dATP, which accumulates within the cell. There is no accumulation of dUTP, dCTP, dGTP, and dTTP under these conditions. Adenosine and deoxyadenosine levels are also high in the blood, as all tissues of the body release these compounds when they can be no longer metabolized, due to ADA deficiency. This leads to accumulation of these toxic intermediates in the lymphocytes, which are the tissues that manifest the clinical aspects of the disease.

A 6-month-old boy was brought to the pediatrician due to frequent bacterial and viral infections. Blood work shows the complete absence of B and T cells. Radiographic analysis shows a greatly reduced thymic shadow. Treatment of the child with a stabilized protein reverses the deficiencies. Concerning the patient, which metabolite will accumulate in the blood cells? (A) dUTP (B) dCTP (C) dATP (D) dGTP (E) dTTP

The answer is E: Ribonucleotide reductase. The increase in dATP, which occurs when ADA is defective, leads to the binding of dATP to the allosteric activity site of ribonucleotide reductase, which leads to the inhibition of overall enzyme activity. Thus, deoxyribonucleotides cannot be produced for the synthesis of DNA, and cells are not capable of replication. Elevated levels of dATP do not have an inhibitory effect on purine nucleoside phosphorylase, hypoxanthine guanine phosphoribosyltransferase, APRT, or ADA.

A 6-month-old boy was brought to the pediatrician due to frequent bacterial and viral infections. Blood work shows the complete absence of B and T cells. Radiographic analysis shows a greatly reduced thymic shadow. Treatment of the child with a stabilized protein reverses the deficiencies. One possible reason for the lack of immune cells is inhibition of which of the following enzymes? (A) ADA (B) Purine nucleoside phosphorylase (C) Hypoxanthine guanine phosphoribosyltransferase (D) Adenine phosphoribosyltransferase (E) Ribonucleotide reductase

The answer is D: Converts adenosine to inosine. The patient has the symptoms of ADA defi ciency, which leads to severe combined immunodefi ciency syndrome. ADA catalyzes the conversion of adenosine to inosine. IMP dehydrogenase converts IMP to XMP. APRT converts adenine to AMP. HGPRT converts guanine to GMP, and there is no enzyme that can convert guanosine to inosine in one step (guanase can convert guanine to xanthine in one step, but does not work on nucleoside substrates).

A 6-month-old boy was brought to the pediatrician due to frequent bacterial and viral infections. Blood work shows the complete absence of B and T cells. Radiographic analysis shows a greatly reduced thymic shadow. Treatment of the child with a stabilized protein reverses the deficiencies. This protein has which of the following activities? (A) Converts IMP to XMP (B) Converts adenine to AMP (C) Converts guanine to GMP (D) Converts adenosine to inosine (E) Converts guanosine to inosine

A.

A 6-month-old infant is brought to your office. The mother indicates that the infant had several scratches that took a long time to stop bleeding. After ruling out various forms of hemophilia, you suspect that the infant has a vitamin deficiency that is causing this problem. The patient is deficient in what vitamin? A. Vitamin K B. Vitamin D C. Niacin D. Vitamin C E. Folic acid

The answer is E: Inhibition of carbamoyl phosphate synthetase II. Uridine bypasses the mutated step of the pathway, allowing UTP to be produced. UTP inhibits the rate-determining step of the pathway, carbamoyl phosphate synthetase II, which halts the production of orotic acid, thereby lowering the concentration of orotate in the urine. This is the mechanism whereby the crystals no longer form. Uridine is not inhibiting the enzyme that directly forms orotate, nor does it inhibit aspartate transcarbamoylase or CPS-I. While adding uridine does bypass the regulated step, it is the synthesis of UTP from the uridine that leads to the drop in orotate production. The pathway of orotate synthesis is shown in the answer to the previous question.

A 6-month-old infant is seen by the pediatrician for developmental delay. Blood work shows megaloblastic anemia, although measurements of B12 and folate are in the high normal range. Urinalysis demonstrates, upon standing, the formation of a crystalline substance. Supplementation of the child's diet with uridine reversed virtually all of the clinical problems. Considering this patient, after uridine treatment the crystals were no longer found in the urine. This is due to which of the following? (A) Inhibition of the enzyme producing the crystalline molecule (B) Bypassing the mutated step of the pathway (C) Inhibition of aspartate transcarbamoylase (D) Inhibition of nitrogen fixation by carbamoyl phosphate synthetase I (E) Inhibition of carbamoyl phosphate synthetase II

The answer is D: lack of thymidine for DNA synthesis. When UMP synthesis is inhibited, there are insufficient precursors for dTMP synthesis (which is derived from dUMP via the thymidylate synthase reaction). The lack of dTTP (which is derived from dTMP) leads to an inhibition of DNA synthesis in red blood cell precursors, leading to the megaloblastic anemia. The mutation in hereditary orotic aciduria does not affect folate or B12 metabolism. Since this is a mutation in a pyrimidine biosynthetic pathway, there is no effect on adenine synthesis. This mutation also does not affect the activity of ribonucleotide reductase.

A 6-month-old infant is seen by the pediatrician for developmental delay. Blood work shows megaloblastic anemia, although measurements of B12 and folate are in the high normal range. Urinalysis demonstrates, upon standing, the formation of a crystalline substance. Supplementation of the child's diet with uridine reversed virtually all of the clinical problems. Considering this patient, the observed megaloblastic anemia results from which of the following? (A) Interference with folate metabolism (B) Interference with B12 absorption (C) Inhibition of ribonucleotide reductase (D) Lack of thymidine for DNA synthesis (E) Lack of adenine for DNA synthesis

The answer is C: Orotate. The child has hereditary orotic aciduria, a mutation in the UMP synthase that leads to orotic acid accumulation in the urine (see the figure below). Treatment with uridine bypasses the block and allows UTP, CTP, and dTTP synthesis. Uridine treatment also has the benefi cial effect of blocking further orotate production, as UTP inhibits carbamoyl phosphate synthetase II, the rate-determining step of pyrimidine production. As CPS-II is inhibited, less orotate is produced. The megaloblastic anemia is the result of inadequate DNA synthesis in the red cell precursors due to the lack of dTTP and dCTP. The crystals are made of orotate, as that is the compound that is accumulating. Uracil, thymine, and cytosine would not be synthesized in a patient with this disorder. Aspartate is very soluble and would not form crystals if it were to accumulate.

A 6-month-old infant is seen by the pediatrician for developmental delay. Blood work shows megaloblastic anemia, although measurements of B12 and folate are in the high normal range. Urinalysis demonstrates, upon standing, the formation of a crystalline substance. Supplementation of the child's diet with uridine reversed virtually all of the clinical problems. The crystalline substance was most likely composed of which of the following? (A) Uracil (B) Thymine (C) Orotate (D) Aspartate (E) Cytosine

The answer is E: Heme synthesis. The boy has porphyria, a reduced ability to synthesize heme. The supersensitivity to the sun is due to the presence of heme precursors in skin cells that are easily converted to radical form by the energy in sunlight, and which severely damage the cell. The drug the boy is taking is metabolized via a cytochrome P450 system, which is induced when the drug first enters the circulation. Induction of P450 systems induces the synthesis of heme, leading to increased concentrations of the heme intermediates and an increased sensitivity to the effects of these intermediates as induced by sunlight. The anemia is due to reduced heme levels in the red blood cells. This disorder is not due to defects in DNA repair, glycogen metabolism, or fatty acid metabolism

A 6-year-old boy is slightly anemic and is very sensitive to the sun, to the point where his skin blisters instead of healing normally from sunburn. His condition worsened when he was taking rifampin for a Methicillin Resistant Staph Aureus. The boy most likely has a defect in which of the following biochemical pathways? (A) Glycogen synthesis (B) Fatty acid oxidation (C) DNA repair (D) Transcription-coupled DNA repair (E) Heme synthesis

The answer is A: 80% fat, 20% combined carbohydrate and protein, by weight. The patient has epilepsy, a disorder of the nervous system which triggers, at times, involuntary muscle movements (seizures). While there are drugs designed to reduce the electrical activity in the nervous system, there is an indication that a strict ketogenic diet can also help to alleviate the frequency of seizures in epileptic patients. The ketogenic diet should be 80%, by weight, fat, with the other 20% split between carbohydrates and protein. If one can use fat containing medium-chain triglycerides, the diet appears to be more effective. The reason for the ketogenic diet having this effect has not yet been elucidated. Of the diets listed as choices, the one with the highest amount of fat would be the most ketogenic.

A 6-year-old girl has a history of seizures, which are only marginally relieved by standard medications. Switching her diet to which of the following may help her condition? (A) 80% fat, 20% combined carbohydrate and protein, by weight (B) 50% fat, 50% combined carbohydrate and protein, by weight (C) 20% fat, 80% combined carbohydrate and protein, by weight (D) 75% protein, 25% combined carbohydrate and fat, by weight (E) 50% protein, 50% combined carbohydrate and fat, by weight

E.

A 70-year-old man comes to your office complaining of weakness in his arms and legs and disorientation. You suspect pernicious anemia. He tells you that he has been taking vitamin B12 capsules. You decide to give him injections of vitamin B12 to see if this alleviates the problems. He returns to your office a few weeks later and says that he no longer has the weakness or disorientation. This confirms your diagnosis of pernicious anemia. Which of the following biochemical reactions require vitamin B12 as a cofactor? A. removal of amino group from amino acids B. formation of methyl malonyl-CoA C. formation of N5,N10-methyl tetrahydrofolate D. formation of glycine from serine E. regeneration of tetrahydrofolate

B.

A 72-year-old man presents with swollen and bleeding gums. The examining doctor finds anemia, perifollicular hemorrhages, and loose teeth. A careful history reveals that the patient has been living alone since the death of his wife 18 months ago. His diet consists largely of cola and hot dogs or cheese sandwiches from a nearby deli. What is the most likely diagnosis? A. Osteomalacia B. Scurvy C. Pellagra D. Beriberi E. Iron deficiency

Correct answer: A. Removal of the stomach causes the loss of parietal cells that line the oxyntic glands of the gastric mucosa and are the source of gastric acid as well as intrinsic factor. Intrinsic factor is a protein required for the adequate absorption of vitamin B-12 in the ileum of the small intestine. Loss of intrinsic factor leads to a vitamin B-12 deficiency and the eventual development of pernicious anemia. Vitamin D deficiency is incorrect. Vitamin D is a fat soluble vitamin that depends on solubilization within bile salt micelles for intestinal absorption. Esters of Vitamin D are digested by cholesterol ester hydrolase from the pancreas before their solubilization in micelles. Thus, Vitamin D absorption in the intestine will not be affected by total gastrectomy. Hyperlipidemia is incorrect. Although fat assimilation begins in the stomach, there is no absorption of fat from the stomach. Lipolytic activity in the human stomach is due primarily to gastric lipase but it does not play a major role in fat digestion so the absence of gastric lipase after gastrectomy would not affect normal fat digestion in the intestine. Thus, in the absence of the stomach, blood lipid levels are unchanged and hyperlipidemia does not occur. Metabolic alkalosis is incorrect. Metabolic alkalosis is caused by an increased bicarbonate concentration in the blood and can result from the loss of fixed (H+) from the GI tract during vomiting. In this case, however, the entire stomach has been removed resulting in the absence of gastric acid secretion. Normal gastric acid secretion is associated with bicarbonate entering the blood in exchange for chloride increasing blood alkalinity after a meal. Since there is no gastric acid secretion or gain in bicarbonate due to the absence of gastric secretion, metabolic alkalosis does not normally develop after total gastrectomy. Jaundice is incorrect. Jaundice is a yellowish discoloration of the skin and sclera of the eyes due to the accumulation of either free or conjugated bilirubin. Regarding the GI tract, jaundice can occur with obstruction of bile ducts or liver disease causing increased levels of conjugated bilirubin in the circulation. Total gastrectomy does not affect bile duct or liver function and jaundice does not occur.

A 74-year-old female patient presents with a large gastric carcinoma and undergoes a total gastrectomy (removal of the stomach). Without supportive therapy, what is she most likely to develop? A. Pernicious anemia B. Vitamin D deficiency C. Hyperlipidemia D. Metabolic alkalosis E. Jaundice

The answer is A: Vitamin D. The child has rickets, which is due to a lack of vitamin D. Vitamin D is synthesized via a circuitous route, and due to the parents' (and child's) diet, there is insufficient vitamin D for the child to form healthy bones. UV light is required to form the active form of vitamin D, and the child is also lacking exposure to sunlight. While, due to the diet, the child may become deficient for vitamin B12, lack of B12 does not lead to these symptoms. The symptoms are also not consistent with a lack of vitamin K (which would lead to bleeding problems), folic acid (which would lead to anemia), or E (loss of protection against oxidative radicals).

A 9-month-old child of strict vegan parents is brought to the pediatrician due to perceived muscle weakness. Due to their strict dietary beliefs, the child has not been given vitamin supplements. An image of the anterior of the knee reveals cupped and widened metaphyses. As the child is very fair skinned, the parents always cover up the child when they go outside such that minimal skin is exposed to the sun. In order to correct these problems the physician prescribes treatment with which of the following? (A) Vitamin D (B) Vitamin K (C) Folic acid (D) Vitamin B12 (E) Vitamin E

The answer is D: Lack of large, neutral amino acids in the brain. The child has PKU. The elevated phenylalanine levels in the blood are saturating the large, neutral amino acid transport protein in the nervous system (L-system), preventing other substrates from entering the brain (such as tryptophan, tyrosine, lysine, and leucine). This alters the ability of the brain to synthesize proteins, and leads to neurological problems. Providing large amounts of these large, neutral amino acids prevents saturation of the system by phenylalanine, and can be used as a treatment, along with restricted phenylalanine diet, for children with this disorder. (See J Inherit Metab Dis. 2006 Dec;29(6):732-738.) The developmental delay does not appear to be due to acidosis, lack of tyrosine, an inhibition of hydroxylating enzymes, or inhibition of neuronal glycolysis.

A Russian child, 5 years old, was brought to the pediatrician for developmental delay. Blood analysis showed elevated levels of phenylalanine, phenyllactate, and phenylpyruvate. The developmental delay, in this condition, has been hypothesized to occur due to which of the following? (A) Acidosis due to elevated phenyllactate (B) Lack of tyrosine, now an essential amino acid (C) Inhibition of hydroxylating enzymes due to accumulation of phenylalanine (D) Lack of large, neutral amino acids in the brain (E) Inhibition of neuronal glycolysis by phenylpyruvate

E.

A cardiovascular researcher is conducting a study to evaluate the relationship between folate levels and cardiovascular disease. The results of the study show that persons with lower folate levels have twice the risk of cardiovascular disease mortality than do those with higher folate levels. Which of the following enzymes is most closely related to the role of folate in lowering the risk of cardiovascular disease? A.Dihydrofolate reductase B .Glycinamide ribonucleotide transformylase C.Homocysteine methyltransferase D.Ribonucleotide reductase E. Thymidylate synthase

The answer is b. (Murray, pp 237-241. Scriver, pp 1667-1724.) Phenylalanine is an essential amino acid that is converted to tyrosine by phenylalanine hydroxylase. Tyrosine is metabolized to various dopamine metabolites as well as melanin. Children with phenylketonuria (PKU-261600)have deficient phenylalanine hydroxylase and develop elevated phenylalanine with severe mental deficiency that is perhaps related to brain dopamine pathways. Dietary treatment of PKU is effective if begun before age 2-3 months (hence neonatal screening that may not be done in underdeveloped countries), but it must be monitored because phenylalanine cannot be completely excluded from the diet. Complete absence of phenylalanine will cause protein malnutrition and failure to grow. Tyrosine is a precursor to melanin, so deficient tyrosine synthesis in children with PKU often causes a lighter hair and skin color than usual for their family. Phenylalanine and tyrosine levels must be monitored every 3-4 months in children on low phenylalanine diets to ensure balance between accumulation and deficiency.

A child from Nigeria is evaluated for developmental delay. His coloring seems much lighter than that of his family background, and his physician orders a blood amino acid test that demonstrates elevated phenylalanine. A special low phenylalanine formula is begun (Lofenelac) as treatment for phenylketonuria (261600), but the parents refuse to come in for follow-up appointments. A public health evaluation reports that the child is failing to thrive despite apparent adherence to the diet by his parents. The symptoms of decreased skin pigment and later failure to thrive in this child are most likely related to which of the following? a. Deficiency of alanine b. Deficiency of tyrosine and melanin c. Deficiency of tryptophan and niacin d. Deficiency of leucine and isoleucine e. Deficiency of phenylalanine

The answer is g. (Murray, pp 249-263,490. Scriver, pp 2079-2108.) Hartnup disease (234500) is caused by defective neutral amino acid transport in the intestinal and/or kidney. Neutral aminoaciduria is observed as well as increased fecal excretion of indole derivations due to bacterial conversion of unabsorbed dietary tryptophan. The disorder is very rare in industrialized countries with good diets, since deficiency of tryptophan as a precursor for niacin synthesis causes the most severe symptoms. Niacin deficiency is called pellagra and produces skin rashes, psychiatric symptoms, and neurologic problems like ataxia (wobbly gait) or diplopia (double vision). Deficiencies of the other mentioned amino acids are not recognized as clinical syndromes, although excess of phenylalanine in phenylketonuria (PKU—261600) or of leucine or isovaline in maple syrup urine disease is associated with severe symptoms if untreated.

A child from a refugee camp presents with an unusual rash that suggests malnutrition. However, his parents relate that two sibs have had the same rash, and are affected with a disorder called Hartnup disease (234500). In this disorder, patients have a defect in neutral amino acid absorption from the intestine. The physician reads about Hartnup disease and decides the rashes are likely to be pellagra, caused by deficiency of niacin. Which of the following abnormalities in Hartnup disease would account for the niacine deficiency? a. High fecal levels of tryptophan and indole derivatives b. Deficiency of phenylalanine c. Deficiency of alanine d. Deficiency of leucine e. Deficiency of glycine f. Deficiency of isovaline g. Deficiency of tryptophan

The answer is b. (Murray, pp 481-497. Scriver, pp 3935-3964.) The vitamin biotin is the cofactor required by carboxylating enzymes such as acetyl-CoA, pyruvate, and propionyl-CoA carboxylases. The fixation of CO2 by these biotin-dependent enzymes occurs in two stages. In the first, bicarbonate ion reacts with adenosine triphosphate (ATP) and the biotin carrier protein moiety of the enzyme; in the second, the "active CO2" reacts with the substrate—e.g., acetyl-CoA.

A child presents with hair loss, skin, rashes, and exaggerated acidosis after infections. The child is found to have deficiency of biotinidase (253260), and improves dramatically with biotin therapy. Biotin is involved in which of the following types of reactions? a. Hydroxylations b. Carboxylations c. Decarboxylations d. Dehydrations e. Deaminations

The answer is e. (Murray, pp 38-39, 535-539. Scriver, pp 5241-5286.) Collagen has an unusual amino acid composition in that approximately one-third of collagen molecules are glycine. The amino acid proline is also present in a much greater amount than in other proteins. In addition, two somewhat unusual amino acids, 4-hydroxyproline and 5-hydroxylysine, are found in collagen. Hydroxyproline and hydroxylysine per se are not incorporated during the synthesis of collagen. Proline and lysine are hydroxylated by specific hydroxylases after collagen is synthesized. A reducing agent such as ascorbate (vitamin C) is needed for the hydroxylation reaction to occur. In its absence, the disease known as scurvy occurs. Only proline or lysine residues located on the amino side of glycine residues are hydroxylated. Because hydroxylation of proline and lysine occurs after collagen is synthesized, addition of labeled hydroxyproline to the tissue culture will not result in labeled collagen.

A child with a large head, multiple fractures, and blue sclerae (whites of the eyes) is evaluated for osteogenesis imperfecta (166200). One study involves labeling of collagen chains in tissue culture to assess their mobility by gel electrophoresis. Amino acids labeled with radioactive carbon 14 are added to the culture dishes in order to label the collagen. Which of the following amino acids would not result in labeled collagen? a. Serine b. Glycine c. Aspartate d. Glutamate e. Hydroxyproline

The answer is c. (Murray, pp 205-218. Scriver, pp 2705-2716.) Apo B is the major protein of chylomicrons, very-low-density lipoproteins (VLDL), and low-density lipoproteins (LDL); these serum lipids are reduced in the lipoprotein electrophoretic patterns of children with abetalipoproteinmia (200100). The disorder is benefited by administration of fat-soluble vitamins like E that are malabsorbed. Chylomicrons are one of four major groups of lipoproteins and are responsible for transport of lipids from digestion and absorption. The other groups of lipoproteins are very-low-density lipoproteins (VLDL), which are responsible for transport of triacylglycerol from the liver, low-density lipoproteins (LDL), which deliver cholesterol, and high-density lipoproteins (HDL), which remove cholesterol from tissues.

A child with chronic diarrhea and anemia is evaluated and found to have abetalipoproteinemia (200100), a disorder caused by defective transport and deficiency of the apoB protein. Which of the following classes of serum lipids would be expected to be deficient in abetalipoproteinemia? a. Lipoprotein (a) b. High density lipoproteins (HDL) c. Chylomicrons d. Triglycerides e. Low density lipoprotein receptor

C.

A chronic alcoholic is in treatment for alcohol abuse. The drug disulfiram is prescribed for the patient. This drug deters the consumption of alcohol by which of the following mechanism? a. Inhibiting the absorption of ethanol so that an individual cannot become intoxicated, regardless of how much he or she drinks b. Inhibiting the conversion of ethanol to acetaldehyde, which cause the excretion of unmetabolized ethanol. c. Blocking the conversion of acetaldehyde to acetate, which causes the accumulation of acetaldehyde d. Activating the excessive metabolism of ethanol to acetate, which causes inebriation with consumption of a small amount of alcohol e. Preventing the excretion of acetate, which causes nausea and vomiting

The answer is C. Disulfiram blocks the conversion of acetaldehyde to acetate. The accumulation of acetaldehyde is toxic and causes vomiting and nausea. Answers A and B are incorrect because disulfiram would not interfere with the absorption of ethanol or the first step of its metabolism. Answer D is incorrect because an acetaldehyde dehydrogenase inhibitor (such as disulfiram) would inhibit the conversion of ethanol to acetate, not increase the rate of the conversion. Answer E is incorrect because disulfiram does not interfere with the excretion of acetate, nor does acetate accumulation lead to nausea and vomiting.

A chronic alcoholic is in treatment for alcohol abuse. The drug disulfiram is prescribed for the patient. This drug deters the consumption of alcohol by which of the following mechanisms? A. Inhibiting the absorption of ethanol so that an individual cannot become intoxicated, regardless of how much he or she drinks B. Inhibiting the conversion of ethanol to acetaldehyde, which cause the excretion of unmetabolized ethanol C. Blocking the conversion of acetaldehyde to acetate, which causes the accumulation of acetaldehyde D. Activating the excessive metabolism of ethanol to acetate, which causes inebriation with consumption of a small amount of alcohol E. Preventing the excretion of acetate, which causes nausea and vomiting

The answer is D: B12. A B12 deficiency will block the methionine synthase reaction, in which homocysteine reacts with N5-methyltetrahydrofolate (THF) to regenerate methionine and free THF. In the absence of such an activity, the N5-methyl THF accumulates, and as that form is the most stable form, eventually all folate will be "trapped" as the N5-methyl derivative. N5-methyl THF cannot go back to N5, N10-methylene THF; once the N5-methyl form is synthesized, the folate is trapped in that form until the methionine synthase reaction occurs. Thus, folate is still available, but in the wrong form; the levels of N5, N10-methylene THF are too low to allow for thymidine synthesis, and the levels of N10-formyl THF are too low to allow for suffi cient purine synthesis. Thus, a functional folate deficiency occurs. Defi ciencies in the other vitamins listed (thiamine, niacin, riboflavin, and C) will not lead to a functional folate deficiency. The reaction involving folate and vitamin B12 is diagrammed below, along with other reactions involved in homocysteine metabolism.

A deficiency in which of the following vitamins will lead to a functional folate deficiency? (A) Thiamine (B) Niacin (C) Riboflavin (D) B12 (E) Vitamin C

Correct answer = B. With the exception of arginase, deficiencies of the enzymes of the urea cycle result in the failure to synthesize urea and lead to hyperammonemia in the first few weeks after birth. Glutamine will also be elevated because it acts as a nontoxic storage and transport form of ammonia. Thus, elevated glutamine always accompanies hyperammonemia. Asparagine does not serve this sequestering role. Urea would be decreased due to impaired activity of the urea cycle. Lysine and uric acid would not be elevated. Treatment of this patient includes limiting protein in the diet and administering compounds that bind covalently to amino acids, producing nitrogen-containing molecules that are excreted in the urine. For example, phenylbutyrate given orally is converted to phenylacetate. This compound condenses with glutamine to form phenylacetylglutamine, which is excreted.

A female neonate did well until approximately 24 hours of age when she became lethargic. A sepsis workup proved negative. At 56 hours, she started showing focal seizure activity. The plasma ammonia level was found to be 1,100 μmol/L (normal 5-35 μmol/L). Quantitative plasma amino acid levels revealed a marked elevation of argininosuccinate. Which one of the following would also be elevated in the blood of this patient? A. Asparagine. B. Glutamine. C. Lysine. D. Urea. E. Uric acid.

Correct answer = B. Genetic deficiencies of each of the five enzymes of the urea cycle, as well as deficiencies in N-acetyglutamate synthase, have been described. The accumulation of argininosuccinate in the plasma of this patient means that the enzymes required for its synthesis are functional, but the enzyme (argininosuccinate lyase or argininosuccinase) required for its cleavage to arginine plus fumarate is not.

A female neonate did well until approximately 24 hours of age when she became lethargic. A sepsis workup proved negative. At 56 hours, she started showing focal seizure activity. The plasma ammonia level was found to be 1,100 μmol/L (normal 5-35 μmol/L). Quantitative plasma amino acid levels revealed a marked elevation of argininosuccinate. Which one of the following enzymic activities is most likely to be deficient in this patient? A. Arginase. B. Argininosuccinate lyase. C. Argininosuccinate synthase. D. Carbamoyl phosphate synthetase I. E. Ornithine transcarbamoylase.

The answer is C. Folic acid is required for the conversion of serine to glycine and the serine is produced from 3-phosphoglycerate and alanine. Folic acid is not needed to synthesize aspartate (from oxaloacetate by transamination), glutamate (from alpha-ketoglutarate by transamination), proline (from glutamate by a series of steps that do not require folic acid), or serine (from 3-phosphoglycerate, with no one-carbon metabolism needed).

A folic acid deficiency would interfere with the synthesis of which of the following amino acids from the indicated precursors? A. Aspartate from oxaloacetate and glutamate B. Glutamate from glucose and ammonia C. Glycine from glucose and alanine D. Proline from glutamate E. Serine from glucose and alanine

This patient has vitamin D deficiency. Vitamin D is activated in the kidneys. The first step in the synthesis of Vitamin D is the conversion of 7-dehydrocholesterol to cholecalciferol in the skin by ultraviolet light. Several metabolic conversions must take place in order to obtain the active hormone. This prohormone is then further activated in the liver by the enzyme 25-hydroxylase to form 25-hydroxycholecalciferol. This is the form of vitamin D found in the serum. The 25-hydroxy-cholecalciferol is further converted to 1,25 dihydroxycholecalciferol in the kidney by the enzyme 1-alpha hydroxylase. This is the active form of the vitamin. No form of vitamin D activation occurs in the muscle. The softening of the bones in an adult is called osteomalacia. It is also known as adult rickets because it is caused by a lack of vitamin D and calcium in the diet. The bone structure remains normal, but the softening occurs due to defects in the normal calcification process. Vitamin D deficiency is seen in patients with inadequate sunlight exposure, such as elderly nursing home patients who also ingest inadequate amounts of vitamin D. Malabsorption of the vitamin can also cause a deficiency. Severe liver or kidney disease and genetic defects in the vitamin D receptor are other causes of this condition.

A friend asks you to check on his 85-year-old grandmother who resides in a nursing home. He tells you that she rarely gets outside, eats hardly any dairy products, and has aching bones. It is likely that his grandmother has a specific vitamin deficiency. Synthesis of the vitamin in question is dependent on ultraviolet radiation. The actual step in the activation of this vitamin occurs in what tissues? A. Liver B. Kidney C. Serum D. Skin E. Muscle

The answer is b. (Murray, pp 481-497. Scriver, pp 3897-3964, 121-138, 287-320.) Pantothenic acid combines with the amino acid cysteine to become the pentetheine sulfhydryl component of coenzyme A (CoA) and acyl carrier protein (important for fatty acid synthesis). AcetylCoA is the activated form of acetate employed in acetylation reactions, including the citric acid cycle and lipid metabolism. Loss of myelin in Hallavorden-Spatz disease correlates with a role for activated pantothenic acid as a cofactor for fatty acid synthesis and as a carrier of acyl chains (which must be added to glycerol to form triacylglycerols, alkylacylglycerols (ether lipids), and (by acyl addition to sphingosine) cerebrosides, sphingomyelin, and gangliosides. Mutations with severe impact on panthothenic acid kinase (mediating activation by its phosphorylation) present with neurologic signs in infancy (e.g., infantile neuroaxonal dystrophy—256600) while those with less impact present in the second or third decades with cognitive decline, dementia, and psychiatric symptoms (e.g., Hallavorden-Spatz disease, 234200). Nutritional deficiencies of pantothenic acid have not been described except in artificial studies, perhaps because they would limit CoA and have deadly consequences in mammals. However, because it is common in foodstuffs, there is little evidence of pantothenic acid deficiency in humans.

A group of neurodegenerative diseases with onset ranging from neonates to adults have been found to involve deficiency of pantothenic acid kinase (e.g., Hallavorden-Spatz disease, 234200). The correlation between defective pantothenic acid activation and loss of myelin to cause neurodegenerative disease relates to its role in which of the following? a. Decarboxylation reactions b. Acetylation and acyl group metabolism c. Dehydrogenation and redox reactions d. Phosphorylation reactions e. Methyl transfer reactions

The answer is C: Minimal insulin release. The biochemical basis of the Atkins diet is to minimize insulin release. Insulin will promote glycogen and fat synthesis, and by minimizing its release (by eating a low carbohydrate, high-protein diet), the ability of the liver to synthesize these energy storage molecules will be greatly reduced. While amino acids can stimulate insulin release, the amount released is about 10% that when glucose stimulates insulin release. A high-protein diet will not inhibit, allosterically, fatty acid synthesis. A high-protein diet does not increase glucagon release, nor does it reduce cortisol release. There will be increased urea production on such a diet, due to the increased level of protein degradation, but increasing urea production does not, by itself, lead to weight loss.

A high-protein low-carbohydrate diet has, as its biochemical basis, the potential to lead to weight loss due to which of the following? (A) Allosteric inhibition of fatty acid synthesis (B) Increased glucagon release (C) Minimal insulin release (D) Reduced cortisol release (E) Increased urea production

E.

A homeless man is seen in a downtown clinic. He appears to be very malnourished and tells you he has not eaten in several days. You do some routine blood analysis and find that although glucose levels are reduced somewhat, the concentration of another metabolite is relatively high. You know that under starvation conditions, the brain can use other compounds as sources of energy. Which of the following compounds would be high in his blood and is the main source of energy used by the brain under long term starvation conditions? A. fructose B. fatty acids C. protein D. acetate E. acetoacetate

The answer is d. (Murray, pp 481-497. Scriver, pp 3897-3964.) Chronic alcoholics are at risk for thiamine deficiency, which is thought to play a role in the incoordination (ataxia) and psychosis that can become chronic (WernickeKorsakoff syndrome). The thiamine deficiency produces relative deficiency of the pyruvate dehydrogenase complex. The administration of glucose without checking glucose levels can therefore be dangerous, since excess glucose is converted to pyruvate by glycolysis. The low rate of pyruvate dehydrogenase conversion of pyruvate to coenzyme A (and entry into the citric acid cycle) causes pyruvate to be converted to lactate (through lactate dehydrogenase). Lactic acidosis can be fatal. Chronic alcoholics can be deficient in the other vitamins mentioned, but thiamine is most likely to help the neurologic symptoms.

A homeless person is brought into the emergency room with psychotic imagery and alcohol on his breath. Which of the following compounds is most important to administer? a. Glucose b. Niacin c. Nicotinic acid d. Thiamine e. Riboflavin

The answer is A. Decreased serum albumin could have several causes, including hepatic disease that decreases the ability of the liver to synthesize serum proteins, protein malnutrition, marasmus, or diseases that affect the ability of the intestine to digest protein and absorb the amino acids. However, his BMI is in the healthy weight range (thus, B and D are incorrect). His normal CHI indicates that he has not lost muscle mass and is, therefore, not suffering from protein malnutrition (thus, B, C, D, and E are incorrect).

A hospitalized patient had low levels of serum albumin and high levels of blood ammonia. His CHI was 98%. His BMI was 20.5. BUN was not elevated, consistent with normal kidney function. The diagnosis most consistent with these finding is which one of the following? A. A loss of hepatic function (e.g., alcohol-induced cirrhosis) B. Anorexia nervosa C. Kwashiorkor (protein malnutrition) D. Marasmus (protein-calorie malnutrition) E. Decreased absorption of amino acids by intestinal epithelial cells (e.g., celiac disease)

The answer is A. The patient's BMI is in the obese range, with large abdominal fat deposits. He needs to decrease his intake of total calories because an excess of calories ingested as carbohydrate, fat, or protein results in deposition of triacylglycerols in adipose tissue. If he keeps his total caloric intake the same, substitution of one type of food for another will help very little with weight loss. (However, a decreased intake of fat may be advisable for other reasons). Limited food diets, such as the ice cream and sherry diet, or a high protein diet of shrimp, work if they decrease appetite and, therefore, ingestion of total calories.

A male patient exhibited a BMI of 33 kg/m2 and a waist circumference of 47 in. What dietary therapy would you consider most helpful? A. Decreased intake of total calories, because all fuels can be converted to adipose tissue triacylglycerols. B. The same amount of total calories, but substitution of carbohydrate calories for fat calories. C. The same amount of total calories, but substitution of protein calories for fat calories. D. A pure-fat diet, because only fatty acids synthesized by the liver can be deposited as adipose triacylglycerols. E. A limited food diet, such as the ice cream and sherry diet.

D.

A man comes into the emergency room and relates that he is a chronic alcoholic. He is clearly malnourished. Because of the possibility of hypoglycemia in alcoholics, you administer thiamine to protect the brain from ATP depletion and acidosis. You know that several enzymes require the thiamine derivative thiamine pyrophosphate as a cofactor including one that is part of the pentose phosphate pathway. What is the pentose phosphate pathway enzyme that requires thiamine pyrophosphate as a cofactor? A. Phosphopentose isomerase B. Transaldolase C. Glucose 6-phosphate dehydrogenase D. Transketolase E. Pyruvate dehydrogenase

The answer is B. The court-ordered medication is disulfiram. Disulfiram inhibits aldehyde dehydrogenase, which greatly reduces the amount of acetaldehyde that is converted to acetate. This causes an accumulation of acetaldehyde, which is the substance responsible for the symptoms of a "hangover," including nausea and vomiting. Alcohol dehydrogenase reduces ethanol to acetaldehyde. Acetyl-CoA synthetase converts acetate to acetyl-CoA.

A man has just received his fourth DUI citation. The judge orders an alcohol dependency program complete with a medication that makes him have nausea and vomiting if he drinks alcohol while taking the medication. The drug-induced illness is due to the buildup of which one of the following? (A) Ethanol (B) Acetaldehyde (C) Acetate (D) Acetyl-CoA (E) Acetyl phosphate

The answer is e. (Murray, pp 205-218.) Fatty liver is associated with a buildup of triacylglycerol due either to an inability to produce enough VLDL or an inability to produce plasma lipoproteins. Under normal conditions, VLDL is responsible for the transport of triacylglycerol from the liver to extrahepatic tissues. Alcoholism causes chronic injury to liver cells, producing increased fat retention in early stages and scarring due to cell death (cirrhosis) at later stages.

A man with chronic alcoholism is admitted with hematochezia (bright red blood in stools) and hematemesis (bloody vomitus). Transfusions and esophageal tube pressure fail to maintain his blood pressure, and he dies from shock and cardiac failure. Autopsy would expect to show which of the following? a. Normal liver with excess chylomicrons b. Cirrhotic liver with excess HDL c. Fatty liver with excess LDL d. Fatty liver with VLDL e. Cirrhotic and fatty liver with excess triacylglycerol

Which of the following best defines the role of a micelle?

A microdroplet that facilitates the transport of dietary fats out of the intestine during digestion

The answer is d. (Murray, pp 197-204. Scriver, pp 5875-5902.) Ceramide is an important signaling molecule for apoptosis and is a precursor for glycosphingolipids and gangliosides. Sphingolipids and phospholipids are structural lipids in membranes. Multiple sclerosis is a demyelinating disease in which both phospholipid and sphingolipid levels are decreased, unlike lipid storage diseases, which affect sphingolipid levels. The cause of multiple sclerosis is unknown, but it exhibits multifactorial determination with autoimmune characteristics.

A middle-aged man develops episodes of incoordination and slurred speech. His wife notes that he seems depressed and argumentative. His physician diagnoses multiple sclerosis, which is best described as which of the following? a. Demyelinating disease with loss of phospholipids and ceramide from brain and spinal cord b. Lipid storage disease with loss of sphingolipids and ceramide from brain and spinal cord c. Lipid storage disease with loss of sphingolipids and gangliosides from brain and spinal cord. d. Demyelinating disease with loss of phospholipids and sphingolipids from brain and spinal cord e. Lipid storage disease with accumulation of sphingolipids in brain

The answer is b. (Murray, pp 580-597. Scriver, pp 3127-3162.) Ferrous iron (Fe2+) is the form absorbed in the intestine by ferritin, transported in plasma by transferrin, and stored in the liver in combination with ferritin or as hemosiderin. There is no known excretory pathway for iron, either in the ferric or ferrous form. For this reason, excessive iron uptake over a period of many years may cause hemochromatosis (235200), the likely diagnosis for this man. This is a condition of extensive hemosiderin deposition in the liver, myocardium, pancreas, and adrenals. The resulting symptoms include liver cirrhosis, congestive heart failure, diabetes mellitus, and changes in skin pigmentation.

A middle-aged man presents with congestive heart failure with elevated liver enzymes. His skin has a grayish pigmentation. The levels of liver enzymes are higher than those usually seen in congestive heart failure, suggesting an inflammatory process (hepatitis) with scarring (cirrhosis) of the liver. A liver biopsy discloses a marked increase in iron storage. In humans, molecular iron (Fe) is which of the following? a. Stored primarily in the spleen b. Stored in combination with ferritin c. Excreted in the urine as Fe2+ d. Absorbed in the intestine by albumin e. Absorbed in the ferric (Fe3+) form

E.

A mother brings her 4-year-old son into your office. She tells you that he does not hear very well and has become unusually active. You notice that he is small for his age. The mother tells you that last year they moved into a 50-year-old house and have begun repainting and restoring the house. A blood test for lead levels shows a lead concentration in the child of 15 ug/dl, indicating lead poisoning. A person exhibiting lead poisoning would be expected to show decreases in the activity of which enzymes involved in porphyrin metabolism? A. δ-aminolevulinic synthase and uroporphyrinogen I synthase B. δ-aminolevulinic synthase and ferrochelatase C. Ferrochelatase and heme oxidase D. Coproporphyrinogen oxidase and uroporphyrinogen I decarboxylase E. δ-aminolevulinic dehydrase and ferrochelatase

D.

A mother comes to your office for dietary counseling concerning her 1-year-old daughter. She wants to know more about vitamin A and the essential amino acids and how to get them from foods. You tell her that large amounts of the precursor of vitamin A (b-carotene) are found in A. Corn B. Liver C. Potatoes D. Green vegetables E. Milk

The answer is C: Arginine and benzoate. Whenever there is a urea cycle defect, arginine becomes an essential amino acid (as its route of synthesis is the urea cycle). Benzoate, along with phenylbutyrate, is given to patients with urea cycle defects to conjugate with a nitrogen carrying molecule (benzoate conjugates with glycine while phenylbutyrate, after activation to phenylacetate, conjugates with glutamine), which is then excreted. The reactions of benzoate and phenylbutyrate with nitrogen containing amino acids are shown above. The excretion of glycyl-benzoate reduces the glycine levels of the body, forcing more glycine to be produced and providing an alternative pathway for nitrogen disposal in the absence of a functional urea cycle. Giving lysine or glutamine will not help to reduce ammonia levels in the patient.

A newborn becomes lethargic and drowsy 24 h after birth. Blood analysis shows hyperammonemia, coupled with orotic aciduria. Considering the patient discussed in the last two questions, a potential treatment for the patient is supplementation with which of the following? (A) Arginine and glutamine (B) Lysine and glutamine (C) Arginine and benzoate (D) Lysine and benzoate (E) Glutamine and phenylbutyrate

The answer is C: Bypassing carbamoyl phosphate synthetase II (CPS-II). The rate-limiting step for de novo pyrimidine synthesis is carbamoyl phosphate synthetase II (CPS-II), which produces carbamoyl phosphate in the cytoplasm (see the figure on page 132). In an OTC deficiency, the carbamoyl phosphate produced in the mitochondria leaks into the cytoplasm, leading to orotic acid synthesis as the regulated step of the pathway is being bypassed. The elevated ammonia is not a substrate of CPS-II, and while glutamine is also elevated, and is a substrate of CPS-II, higher glutamine concentrations will not overcome enzyme inhibition by its allosteric inhibitor, UTP. Aspartate transcarbamoylase is the regulated step of pyrimidine biosynthesis in many prokaryotic cells, but not in humans. This step is necessary for pyrimidines to be synthesized starting with carbamoyl phosphate. CPS-I is a mitochondrial enzyme not involved in pyrimidine production.

A newborn becomes lethargic and drowsy 24 h after birth. Blood analysis shows hyperammonemia, coupled with orotic aciduria. Orotic acid levels are high in this patient due to which of the following? (A) Elevated ammonia (B) Elevated glutamine (C) Bypassing carbamoyl phosphate synthetase II (CPS-II) (D) Bypassing aspartate transcarbamoylase (E) Inhibition of carbamoyl phosphate synthetase I (CPS-I)

The answer is C: Citrulline. The child has ornithine transcarbamoylase (OTC) deficiency, and cannot condense carbamoyl phosphate with ornithine to produce citrulline (see the figure on page 131). The excess carbamoyl phosphate produced leaks into the cytoplasm where it bypasses the regulated enzyme of de novo pyrimidine production, leading to excess orotic acid. Thus, in an OTC defect, carbamoyl phosphate can be produced, but citrulline cannot. Since citrulline cannot be produced, the later products of the urea cycle (argininosuccinate and arginine) are also produced at lower levels than normal, which is an indirect effect due to the inability to produce citrulline.

A newborn becomes lethargic and drowsy 24 h after birth. Blood analysis shows hyperammonemia, coupled with orotic aciduria. This individual has an enzyme deficiency that leads to an inability to directly produce which of the following? (A) Carbamoyl phosphate (B) Ornithine (C) Citrulline (D) Argininosuccinate (E) Arginine

B.

A newborn becomes progressively lethargic after feeding and increases his respiratory rate. He becomes virtually comatose, responding only to painful stimuli, and exhibits mild respiratory alkalosis. Suspicion of a urea cycle disorder is aroused and evaluation of serum amino acid levels is initiated. In the presence of hyperammonemia, production of which of the following amino acids is always increased? a. Glycine b. Arginine c. Proline d. Histidine e. Glutamine

The answer is e. (Murray, pp 242-248. Scriver, pp 1909-1964.) Deficiencies of urea cycle enzymes cause symptoms ranging from confusion and fatigue after a high-protein meal to neonatal lethargy, vomiting, coma, and death. Female carriers for ornithine transcarbamylase (OTC) deficiency (300461) and some mild urea cycle mutations have hyperammonemia with confusion and lethargy after a high protein meal (e.g., hamburgers) and develop aversions to such foods. On the other side of the spectrum, severe urea cycle blocks are incompatible with life. A major reason for the toxicity is the severe depletion of ATP levels caused by the siphoning off of α-ketoglutarate from the citric acid cycle in an attempt to consume ammonia. Glutamate dehydrogenase and glutamine synthetase, respectively, catalyze the following reaction: α-ketoglutarate + NH4+ → glutamate + NH4+ → glutamine As can be seen, this is the reverse order of steps whereby glutamine is successively deaminated first to glutamate and then to α-ketoglutarate by the enzymes glutaminase and glutamate dehydrogenase, respectively. It is thought that the high level of ammonia ions shifts the equilibrium of the dehydrogenase in favor of the formation of glutamate. Depending on the step in the urea cycle that is blocked, levels of arginine may be decreased.

A newborn becomes progressively lethargic after feeding and increases his respiratory rate. He becomes virtually comatose, responding only to painful stimuli, and exhibits mild respiratory alkalosis. Suspicion of a urea cycle disorder is aroused and evaluation of serum amino acid levels is initiated. In the presence of hyperammonemia, production of which of the following amino acids is always increased? a. Glycine b. Arginine c. Proline d. Histidine e. Glutamine

The answer is a. (Murray, pp 270-285. Scriver, pp 2961-3104.) Jaundice refers to the yellow color of the skin and eyes caused by increased levels of bilirubin in the blood. It has many causes, including increased production of bilirubin due to hemolytic anemia or malaria, blockage in the excretion of bilirubin due to liver damage, or obstruction of the bile duct. In newborns, jaundice is normal (physiologic) because of liver immaturity. Only excess jaundice is evaluated (bilirubin over 10-12 at age 3-4 days) based on the age of the infant. High levels of bilirubin in serum (indirect bilirubin) points toward hemolysis from maternofetal blood group incompatibility, whereas high levels of bilirubin diglucuronide (one of several conjugated bilirubins tested as direct bilirubin) suggest liver/gastrointestinal disease. Reticuloendothelial cells degrade red blood cells following approximately 120 days in the circulation. The steps in the degradation of heme include (1) formation of the green pigment biliverdin by the cleavage of the porphyrin ring of heme, (2) formation of the red-orange pigment bilirubin by the reduction of biliverdin, (3) uptake of bilirubin by the liver and the formation of bilirubin diglucuronide, and (4) active excretion of bilirubin into bile and eventually into the stool. The change in color of a bruise from bluish-green to reddish-orange reflects the heme degradation and the change in color of the bile pigments biliverdin and bilirubin. Bilirubin, which is quite insoluble, is transported to the liver attached to albumin. In the liver, bilirubin is conjugated to two glucuronic acid molecules to form bilirubin diglucuronide. Bilirubin diglucuronide is transported against a concentration gradient into the bile. If bilirubin is not conjugated, it is not excreted.

A newborn develops jaundice (yellow skin and yellow sclerae) that is greater than average at its usual peak at 3 days and requires laboratory evaluation. Which of the following porphyrin derivatives is conjugated, reacts directly, and is a major component of bile? a. Bilirubin diglucuronide b. Stercobilin c. Biliverdin d. Urobilinogen e. Heme

The answer is D: Tyrosinase. The child has albinism, a lack of pigment in the skin cells, which is produced by melanocytes. Melanocyte tyrosinase (a different isozyme than the neuronal tyrosinase that produces DOPA for catecholamine biosynthesis) is defective in albinism. The DOPA produced is then used for pigment production. A lack of phenylalanine hydroxylase leads to PKU. A lack of dihydrofolate reductase is most likely a lethal event as there are no reported cases of a lack of this enzyme. Tetrahydrofolate is not required for the conversion of tyrosine to DOPA in melanocytes. NADPH oxidase generates superoxide, which is not part of this pathway. Homogentisic acid is part of the phenylalanine and tyrosine degradation pathways, and is not involved in albinism.

A newborn has milky white skin, white hair, and red-appearing eye color (see the figure below). This disorder most often results from a defect in which of the following enzymes? (A) Phenylalanine hydroxylase (B) NADPH oxidase (C) Dihydrofolate reductase (D) Tyrosinase (E) Homogentisic acid oxidase

The answer is C. The classical form of PKU, a deficiency of phenylalanine hydroxylase, results in elevations of phenylalanine and phenylpyruvate. However, this enzyme is not a choice. In the nonclassical variant of PKU, there is a problem in either synthesizing or regenerating BH4. The enzyme that converts BH2 to BH4 is dihydropteridine reductase.

A newborn infant has elevated levels of phenylalanine and phenylpyruvate in her blood. Which of the following enzymes might be deficient in this baby? A. Phenylalanine dehydrogenase B. Phenylalanine oxidase C. Dihydropteridine reductase D. Tyrosine hydroxylase E. Tetrahydrofolate synthase

The answer is c. (Murray, pp 173-179. Scriver, pp 3181-3218.) Triacylglycerols are assembled from glycerol and saturated fatty acids that are synthesized from condensation of malonyl and acetyl CoA through the fatty acyl synthase complex. Plasmalogens and certain signaling agents like platelet activating factor are ether lipids, distinguished by an ether (C-O-C) bond at carbon 1 of glycerol. Ether lipid synthesis is initiated by placing an acyl group on carbon 1 of dihydroxyacetone phosphate (DHAP) using DHAP acyltransferase. The acyl side chain is then exchanged with an alcohol to form an ether linkage by an acylDHAP synthase—the acyltransferase and synthase plus other enzymes of ether lipid synthesis are localized in peroxisomes. Subsequent additions of phosphocholine yield ether/acyl glycerols analogous to lecithins (including platelet activating factor), and addition of a phosphoethanolamine to carbon 3 of ether (alkyl) glycerols forms plasmalogens. Acetyl and palmitoyl CoA can contribute to these ether lipid modifications after the core carbon 1 ether linkage has produced an alkylglycerol. Disruption of peroxisome structure by mutations in various peroxisomal membrane proteins ablates DHAP acyltransferase and other enzymes for ether lipid/plasmalogen synthesis, causing deficienty of brain lipids, severe neurologic disease, hypotonia, and liver failure—the most severe phenotype of which is Zellweger syndrome (214100).

A newborn male is evaluated because of inability to breast feed and found to have severe hypotonia (low muscle tone). The child lays in a frog leg posture with minimal spontaneous movements, and the head and legs dangle to the bed when suspended by his stomach. A large anterior fontanel is noted, and initial laboratory tests indicate elevated liver enzymes. The physician suspects Zellweger syndrome (214100), an end phenotype reflecting peroxisome dysfunction that may be caused by mutations in several different peroxisomal membrane protein genes. The diagnosis is confirmed by demonstrating elevated plasma levels of very long chain fatty acids and of erythrocyte plasmalogens. Which of the following compounds is the starting point of ether lipid and plasmalogen synthesis? a. Acetyl CoA b. Pyruvate c. Dihydroxyacetone phosphate d. Malonyl CoA e. Palmitoyl CoA

The answer is a. (Murray, pp 237-241. Scriver, pp 1667-2108.) Cysteine can be formed from the essential amino acid methionine, one of nine essential amino acids (histidine, isoleucine, luecine, lysine, methionine, phenylalanine, threonine, tryptophan, valine) and one semiessential amino acid (arginine) that is needed for growth. Tyrosine can be made from phenylalanine, proline or glutamine from glutamate, asparagines from aspartate. Other metabolites like 3-phosphoglycerate (serine), glyoxylate (glycine), pyruvate (alanine) can be converted to amino acids at reasonable rates, rendering them nonessential. Hydroxylysine, hydroxyproline, and selenocysteine are converted from parent amino acids after incorporation into protein. Hyperalimentation solutions required years of research to define essential nutrients, which include unsaturated fatty acids, vitamins, and trace elements. Though effective for healing and growth, hyperalimentation still has unexplained side effects like liver disease that can arise during or after therapy.

A newborn with meconium ileus (plugging of the small intestine with meconium or fetal stool) is found to have air in the bowel wall (pneumatosis intestinalis) and free air in the abdomen. Antibiotics are begun for suspected peritonitis, and emergency surgery is performed to remove the diseased intestinal segment and heal the intestinal perforation that led to air in the abdomen. Because the gut must be kept at rest for healing, meconium peritonitis was usually fatal until parenteral alimentation solutions were developed. Hyperalimentation consists of essential amino acids and other metabolites that provide a positive calorie balance while keeping the bowel at rest. The alimentation solution must be kept to a minimum of metabolites because of its high osmotic load that necessitates frequent changing of intravenous sites or catherization of a large vein. Which of the following amino acids could be excluded from the alimentation solution? a. Cysteine b. Phenylalanine c. Histidine d. Methionine e. Tryptophan

The correct answer is (A). 1. As the name implies, the reaction is a phosphorolysis. 2. The pyrimidine salvage is a two-step pathway. The next step is catalyzed by a kinase. 3. This is true for the degradation of both purine and pyrimidine nucleosides 4. This equilibration is catalyzed by nucleoside monophosphate kinases.

A nucleoside phosphorylase: 1. cleaves a nucleoside with the production of ribose 1-phosphate 2. is necessary for the major salvage pathway for pyrimidines 3. is used in the degradation of purine nucleotides 4. is responsible for the equilibration of nucleoside monophosphates and nucleoside diphosphate. a. 1, 2 and 3 b. 1 and 3 c. 2 and 4 d. 4 only e. All four

E.

A patient comes into your office for a nutrition consultation. He is interested in losing weight. You explain to him that calories can come from protein, carbohydrates, or fats. He asks which of these three groups have the highest amount of energy. You explain to him that the highest stores of energy (kcal/g) in the body are found in A. Protein B. Carbohydrate C. Protein, carbohydrate, and fat equally D. Protein and fat equally E. Fat

C.

A patient comes to your office complaining of a skin rash and diarrhea. After further discussion, he admits that he is an alcoholic. It is likely that he has the disease pellagra. Pellagra results from a dietary deficiency of A. Tryptophan only B. Niacin only C. Niacin and tryptophan D. Riboflavin E. Vitamin B6

B.

A patient comes to your office complaining of abdominal pain, nausea, and vomiting. You note several sores on the back of the patient's hands. He relates that his brother also has sores like these on his hands. Blood tests show increased levels of aminolevulinic acid and porphobilinogen in the urine. You suspect that he has hereditary coproporphyria. In addition to abnormalities in heme metabolism, many porphyrias are also associated with what result? A. Decreased immune function B. Neurological symptoms C. An enlarged liver D. An enlarged heart E. Abnormal mitochondrial gene expression

B.

A patient comes to your office complaining of abdominal pain, nausea, and vomiting. You note several sores on the back of the patient's hands. He relates that his brother also has sores like these on his hands. Blood tests show increased levels of aminolevulinic acid and porphobilinogen in the urine. You suspect that he has hereditary coproporphyria. This disease is caused by a decrease in what function? A. Activity of the enzyme aminolevulinic acid synthase B. Activity of the enzyme coproporphyrinogen oxidase C. Ability of porphyrinogens to absorb visible light D. Ability of porphyrins to bind iron E. Secretion of porphyrinogens

The answer is A: PRPP. The patient has von Gierke disease, a lack of glucose-6-phosphatase activity. When this individual tries to produce glucose for export in the liver, glucose-6-phosphate accumulates, which then goes through either glycolysis (generating lactate) or the HMP shunt pathway, producing excess ribose-5-phosphate. The excess ribose-5-phosphate is converted to PRPP, which then stimulates the amidophosphoribosyl transferase reaction (the rate-limiting step of purine production) to produce 5′-phosphoribosyl 1′-amine. This last reaction occurs because under normal cellular conditions, the concentrations of PRPP and glutamine are signifi cantly below the Km values for amidophosphoribosyl transferase. Any cellular perturbation that increases PRPP levels, then, will increase the rate of the reaction, producing purines that are not required by the cell. This leads to degradation of the excess purines, producing urate and leading to gout. The lactic acidosis associated with von Gierke disease also blocks the transport of urate from the blood into the urine, which contributes to the elevated uric acid levels seen in these patients. Von Gierke disease does not lead to elevated glutamine, ATP, NADH, or dTTP levels.

A patient exhibits fasting hypoglycemia and lactic acidosis under fasting conditions. Hepatomegaly is also evident. A glucagon challenge only releases about 10% of the expected level of glucose from the liver. The patient has also developed gout due to an increase in the levels of which of the following metabolites? (A) PRPP (B) Glutamine (C) ATP (D) NADH (E) dTTP

The answer is C: Elevated levels of sorbitol in the lens. Sorbitol synthesis from glucose in the polyol pathway occurs in the lens of the eye. Aldose reductase converts glucose to sorbitol which then accumulates in the lens. Sorbitol dehydrogenase can convert the sorbitol to fructose, which can also accumulate within the lens. In diabetes mellitus, fluctuating levels of glucose lead to fluctuating levels of sorbitol, which change the consistency of the lens and therefore the glasses prescription. Glucose and galactose by themselves do not directly affect the lens. Chronically, high glucose and sorbitol levels can increase cataract formation, but this patient is experiencing an acute problem. Hyperglycemia does not increase intraocular pressure; however, the conversion of glucose to sorbitol will.

A patient had new glasses prescribed by his optometrist. Less than a week later, his prescription was inadequate and he could not see well with his new glasses. His optometrist checked his vision twice more over the next week and the patient's prescription was different both times. His optometrist refers the patient to an ophthalmologist. What is the reason the patient is having such rapid changes in his glasses prescription? (A) Elevated levels of galactose in the lens (B) Elevated levels of glucose in the lens (C) Elevated levels of sorbitol in the lens (D) Cataract formation (E) Increased intraocular pressure from hyperglycemia

The answer is C: Tyrosine. This patient has Parkinson disease, which is a problem with dopamine synthesis in the substantia nigra. Dopamine is derived from tyrosine. Treatment with DOPA in the initial stages of the disease provides relief from the symptoms. DOPA cannot be synthesized from alanine, serine, tryptophan, or phenylalanine. The figure below indicates the biosynthetic pathway of DOPA and the catecholamines.

A patient has a "pill rolling" tremor, "cogwheel" rigidity, bradykinesis, speech difficulties, and a shuffling gait. The chemical that is lacking in this syndrome is a derivative of which of the following amino acids? (A) Alanine (B) Serine (C) Tyrosine (D) Tryptophan (E) Phenylalanine

The answer is D: Vitamin E. Abetalipoproteinemia is a disorder in which neither nascent chylomicrons nor nascent VLDL can be produced due to a defect in the microsomal triglyceride transfer protein. Fat-soluble vitamins are delivered to tissues via chylomicrons; in the absence of chylomicron formation, the fat soluble vitamins will remain in the intestinal epithelial cell, or not even be absorbed from the intestinal tract. Vitamin E is believed to be a major antioxidant factor in cells. All of the other vitamins listed (B1, B2, C, and niacin) are water-soluble vitamins, and do not require chylomicron formation for vitamin delivery.

A patient has been diagnosed with abetalipoproteinemia. A possible deficiency in which of the following vitamins could occur in this patient? (A) Vitamin B1 (B) Vitamin B2 (C) Vitamin C (D) Vitamin E (E) Niacin

The answer is B: Thymidylate synthase. 5-fl uorouracil is a thymine analog (thymine is 5-methyl uracil), which, after activation in the cells to F-dUMP, binds tightly to thymidylate synthase and blocks the enzyme from converting dUMP to dTMP (see the fi gure below). By blocking thymidine synthesis, cells can no longer synthesize DNA and will not replicate. 5-FU has no direct effect on dihydrofolate reductase, amidophosphoribosyl transferase, PRPP synthase, or UMP synthase. The figure also indicates the effect of methotrexate on dihydrofolate reductase.

A patient has been recently diagnosed with colorectal cancer. The physician treats the patient with a combination of chemotherapeutic drugs, one of which is 5-fluorouracil (5-FU). 5-FU is effective as an anticancer drug because it inhibits which one of the following enzymes? (A) Dihydrofolate reductase (B) Thymidylate synthase (C) Amidophosphoribosyl transferase (D) 5′-phosphoribosyl 1′-pyrophosphate (PRPP) synthetase (E) UMP synthase

The answer is E: formation of g-carboxyglutamate. Warfarin is a vitamin K antagonist, and blocks the regeneration of active vitamin K after it has participated in its reaction of creating a γ-carboxyglutamate residue (see below). In the absence of this side-chain modification, clotting proteins cannot bind to platelets, and the clotting cascade is inhibited. Warfarin does not interfere with platelet synthesis, nor does it alter phospholipid biosynthesis. Warfarin does not alter the transcription or translation of the clotting factors, and has no relationship with vitamin E, which protects against radical damage within cells and tissues. As seen in the figure below, warfarin blocks the activity of vitamin K epoxide reductase.

A patient has had a series of blood clots, and has been placed on warfarin to reduce such incidents. Warfarin exerts its effect by blocking which of the following? (A) Platelet biogenesis (B) Phospholipid synthesis (C) Clotting factor synthesis (D) Vitamin E activity (E) Formation of γ-carboxyglutamate

The answer is A. The accumulation of both phytanic acid and very long-chain fatty acids indicates a problem in peroxisomal fatty acid oxidation, which is where alpha-oxidation occurs. Lysosomal transport is, therefore, not required to metabolize these fatty acids (thus, C is incorrect). The finding that palmitate levels are low indicates that beta-oxidation is occurring; therefore, answer B is incorrect. The compounds that accumulate are not mucopolysaccharides nor is fatty acid elongation required in the metabolism of these compounds (thus, D and E are incorrect).

A patient presented with dysmorphia and cerebellar degeneration. Analysis of his blood indicated elevated levels of phytanic acid and very long-chain fatty acids, but no elevation of palmitate. His symptoms are consistent with a defect in an enzyme involved in which of the following? A. alpha-Oxidation B. Mitochondrial beta-oxidation C. Transport of enzymes into lysosomes D. Degradation of mucopolysaccharides E. Elongation of fatty acids

The answer is D: 5-hydroxyindoleacetic acid (5-HIAA). This patient has the classic presentation of a carcinoid tumor. This type of tumor secretes serotonin that causes these classic symptoms. The breakdown product of serotonin is 5-hydroxyindoleacetic acid (5-HIAA, see the figure below). Elevated levels of 5-HIAA in the urine confirms a high level of serotonin and the diagnosis of a carcinoid. VMA and/or catechols would be elevated if the patient had a pheochromocytoma producing epinephrine or norepinephrine (the VMAs are degradation products of these neurotransmitters, also seen in the figure below). The symptoms do not match a pheochromocytoma, particularly due to the lack of increase in heart rate or blood pressure. Dopamine is depleted in Parkinson disease, not in this condition. Cortisol levels would be high in Cushing syndrome, but not under these conditions.

A patient presents with episodes of flushing, diarrhea, abdominal cramping, and wheezing. His blood pressure and pulse rate are normal during these episodes. Physical exam is normal except for scattered telangiectasias. In order to diagnose this problem, a 24-h urine collection for which of the following would be most appropriate? (A) Vanillylmandelic acid (VMAs) (B) Catechols (C) Dopamine (D) 5-hydroxyindoleacetic acid (5-HIAA) (E) Cortisol

The answer is D: Tryptophan. This patient has the classic presentation of a carcinoid tumor. This type of tumor secretes serotonin which causes these classic symptoms. The breakdown product of serotonin is 5-hydroxyindoleacetic acid (5-HIAA, see the figure below). Elevated levels of 5-HIAA in the urine confirm a high level of serotonin and the diagnosis of a carcinoid. Serotonin is derived from tryptophan, and the patient has a carcinoid tumor secreting serotonin. Elevated levels of alanine, serine, tyrosine, or phenylalanine would not be observed in a patient with a carcinoid tumor.

A patient presents with episodes of flushing, diarrhea, abdominal cramping, and wheezing. His blood pressure and pulse rate are normal during these episodes. Physical exam is normal except for scattered telangiectasias. In order to diagnose this problem, a 24-h urine collection showed elevated levels of 5-HIAA (5-hydroxyindole acetic acid). The chemical responsible for this above syndrome is a derivative of which amino acid? (A) Alanine (B) Serine (C) Tyrosine (D) Tryptophan (E) Phenylalanine

B

A patient presents with swelling, redness, and severe pain of the great toe. On examination, there is inflammation of the first metatarsophalangeal joint. The serum uric acid level is 10 mg/dL (normal <8.5 mg/dL). The patient is prescribed indomethacin and is asked to come back after 15 days. At his next visit, the pain and inflammation had subsided, and the patient is prescribed allopurinol. What is the mechanism of action of this drug? A. It inhibits guanine deaminase B. It inhibits xanthine oxidase C. It inhibits adenosine deaminase D. It inhibits phosphoribosyl pyrophosphate (PRPP) synthetase E. It inhibits urate oxidase

The answer is B: Tyramine. Tyramine is a degradation product of tyrosine (decarboxylated tyrosine), which, when elevated, will lead to norepinephrine release. Tyramine is found in red wine and aged foods such as certain cheeses. When ingested, tyramine is degraded by monoamine oxidase to a harmless compound, and excessive norepinephrine release does not occur. However, if a patient is taking a monoamine oxidase inhibitor (MAOI), it is possible that tyramine does not get degraded appropriately. MAOIs which covalently modify (as opposed to being competitive inhibitors) the enzyme are very useful medications for atypical depression that is unresponsive to other modalities. Unfortunately, MAOIs have multiple interactions with many other medications and foods. A high tyramine level leads to a greatly elevated blood pressure due to the release of norepinephrine. Patients on MAOIs need to avoid foods high in tyramine, such as cheeses (aged and processed), red wine, caviar, brewer's yeast, miso soup, dried herring, and aged meats. MAOIs have no effect on glycoproteins or cholesterol.

A patient taking a drug for depression experienced a greatly increased heart rate and sweating after eating red wine and gourmet, aged cheese. These symptoms appeared due to an inability to degrade which of the following? (A) Tyrosine (B) Tyramine (C) Serotonin (D) Glycine (E) Glutamate

The answer is B: The baby's relative hyperinsulinemia. During pregnancy, the fetus is oversupplied with glucose from the mother causing the fetal pancreas to overproduce insulin. At delivery, the glucose supply from the mother is suddenly terminated and the relative hyperinsulinemia of the baby causes hypoglycemia until the baby's body can adjust to this new environment by decreasing insulin release and increasing glucose release. Hypoglycemia in the first few hours of the newborn's life is a common complication of gestational diabetes. Newborn hyperglycemia would not give a heel stick of 30 mg/dL of glucose. The placenta does not make insulin and the insulin molecule cannot cross the placenta, so the mother's relative hyperinsulinemia is not the cause of this problem. While the mother's hyperglycemia has led to the baby's relative hyperinsulinemia, the mother's blood glucose levels do not cause the drop in the baby's blood glucose levels after birth.

A patient who had gestational diabetes has just delivered a 10 lb baby. The baby appears "jittery" and a heel stick glucose is 30 mg/dL. Which of the following mechanisms is the explanation for the newborn's blood glucose reading? (A) The mother's relative hyperinsulinemia (B) The baby's relative hyperinsulinemia (C) The mother's hyperglycemia (D) The baby's hyperglycemia (E) Placental insulin production

The answer is B. The symptoms exhibited by the patient are caused by excessive release of epinephrine or norepinephrine. Vanillylmandelic acid is also the degradation product of norepinephrine; thus, these hormones are being overproduced. Acetylcholine degradation leads to the formation of acetic acid and choline, which are not observed (thus, A is incorrect). Although dopa degradation could lead to vanillylmandelic acid production, serotonin degradation does not (it leads to 5-hydroxyindoleacetic acid), and the symptoms exhibited by the patient are not consistent with dopa or serotonin overproduction (thus, C is incorrect). Histamine and melatonin also do not produce the symptoms exhibited by the patient (thus, D and E are incorrect).

A patient with a tumor of the adrenal medulla experienced palpitations, excessive sweating, and hypertensive headaches. His urine contained increased amounts of vanillylmandelic acid. His symptoms are probably caused by an overproduction of which of the following? A. Acetylcholine B. Norepinephrine and epinephrine C. Dopa and serotonin D. Histamine E. Melatonin

C.

A pediatrics' resident is explaining the condition to the mother of a child with an inborn error of metabolism. Among the advice he gives her, she is asked to avoid feeding the child food containing large amounts of the artificial sweetener aspartame. What condition is the child likely to have? A. Diabetes mellitus B. Maple syrup urine disease C. Phenylketonuria D. Hereditary fructose intolerance E. Lactose intolerance

The answer is C: Sulfonamides inhibit a metabolic pathway not present in eukaryotic cells. Sulfonamides inhibit the synthesis of THF, a compound that eukaryotic cells cannot synthesize (which is why folic acid is a required vitamin in the human diet). Via inhibition of THF synthesis, the target prokaryotic cells can no longer synthesize dTMP and purines and are unable to grow and replicate. Sulfonamides do not affect DNA polymerases directly, nor do they alter mismatch repair. Sulfonamides also have no effect on ribonucleotide reductase.

A penicillin-allergic child was given a sulfonamide for otitis media. Human cells are resistant to sulfonamides due to which of the following? (A) Sulfonamides are specific for prokaryotic DNA polymerases (B) Sulfonamides are specific for prokaryotic RNA polymerases (C) Sulfonamides inhibit a metabolic pathway not present in eukaryotic cells (D) Sulfonamides inhibit bacterial ribonucleotide reductase, but not eukaryotic ribonucleotide reductase (E) Sulfonamides inhibit prokaryotic mismatch repair, but not eukaryotic mismatch repair

The answer is B: The anabolic effects of insulin. In pregnancy, the placenta preferentially shunts glucose to the developing fetus. This, along with placental hormones, causes a functional "insulin resistance" in the mother. Because of the higher glucose level in the fetus, the fetal pancreas produces more insulin. Insulin is the major anabolic hormone of the body stimulating glucose uptake into the cells and stimulating extra growth. Glucagon, growth hormone, epinephrine, corticosteroids, and thyroid hormone are all catabolic hormones that counter insulin and stimulate glucose release from the cells to increase blood glucose. Glucose itself is neither anabolic nor catabolic. Large for gestational age babies, dehydrated babies (from the osmotic diuresis of hyperglycemia), and a fivefold increase in stillborn rates are all complications of uncontrolled gestational diabetes.

A pregnant patient has developed gestational diabetes. One of the consequences of gestational diabetes is fetal macrosomia. Which of the following is the mechanism that causes these large for gestational age babies? (A) The anabolic effects of glucose (B) The anabolic effects of insulin (C) The anabolic effects of glucagon (D) The anabolic effects of growth hormone (E) The anabolic effects of thyroid hormone

The answer is a. (Murray, pp 122-129, 173-179. Scriver, pp 2297-2326.) Fatty acids are bound to coenzyme A as thiol esters for synthesis or degradation, each proceeding in two-carbon steps. The serially repeated steps in fatty acid oxidation involve (1) removal of two hydrogens to form a double bond between the carbons adjacent to the acid group (acyl-CoA dehydrogenase), (2) addition of water to the double bond so that a hydroxyl is on the second carbon (enoyl-CoA hydratase), oxidation of the hydroxyl group to a ketone (3-hydroxyacylCoA dehydrogenase), and removal of acetyl CoA (thiolase) to leave a fatty acyl CoA that is two carbons shorter. At least three groups of these sequentially acting enzymes are present in the mitochondrion, specific for very long or long chain, medium chain, or short chain fatty acids. Children with very long or long chain oxidation enzyme deficiencies, e.g. very long chain fatty acyl CoA dehydrogenase deficiency (VLCAD, 201475) accumulate fat in their heart and liver and have energy deficits in heart and muscle due to inadequate fat oxidation. Severely affected children often die of cardiac failure in the newborn period, and their enzyme deficiency combined with the maternal heterozygote state may cause HELLP syndrome in the last trimester of pregnancy, a variant of toxemia or preeclampsia that can be fatal. Premature delivery may be necessary for maternal health, and therapy with low-fat diets, frequent feeding (to minimize need for fat oxidation), and carnitine (to maximize transport of fatty acyl CoAs into mitochondria) may be attempted with the affected child.

A pregnant woman is found to have elevated blood pressure on her check-up at 8 months gestation, and testing by her obstetrician demonstrates anemia with Hemolysis, Elevated Liver enzymes, and Low Platelets that are characteristic of disease represented by the acronym HELLP syndrome. The woman is hospitalized and fetal maturity tests are performed that allow elective premature delivery. The woman quickly recovers but the premature newborn has a dilated heart and elevated liver enzymes that are characteristic of a defect in long chain fatty acid oxidation. The potential enzyme deficiencies are those responsible for sequential oxidation of fatty acids, which include which of the following? a. Dehydrogenase, hydratase, dehydrogenase, thiolase b. Transacylase, synthase, reductase c. Hydratase, reductase, thiesterasel d. Thioesterase, dehydrogenase, thiolase e. Dehydrogenase, thiolase, thioesterae

The answer is E: Aspartate, glycine, and glutamine. As shown in the figure below, the nitrogen in a purine ring is directly derived from glycine, glutamine, and aspartic acid. Glutamate, N5-formimino tetrahydrofolate, and asparagine do not directly donate nitrogen to the ring.

A researcher wants to develop a method of labeling purines with 15N for use in future spectroscopic studies. Purine synthesis will be done in a test tube using only the enzymes necessary to synthesize purines via the de novo pathway. Which starting materials should be labeled with the heavy nitrogen in order to maximize 15N incorporation into purines? (A) Aspartate, glycine, and glutamate (B) Aspartate, glycine, and N5-formimino tetrahydrofolate (C) Asparagine, glycine, and glutamine (D) Asparagine, glutamate, and glutamine (E) Aspartate, glycine, and glutamine

Correct answer = B. The intake of saturated fat most strongly influences plasma cholesterol in this diet. The patient is consuming a high-calorie, high-fat diet with 40% of the fat as saturated fat. The most important dietary recommendations are: lower total caloric intake, substitute monounsaturated and polyunsaturated fats for saturated fats, and increase dietary fiber. A decrease in dietary cholesterol would be helpful, but is not a primary objective.

A sedentary 50-year-old man, weighing 80 kg (176 pounds), requests a physical examination. He denies any health problems. Routine blood analysis is unremarkable except for plasma cholesterol of 280 mg/dl. The man refuses drug therapy for his hypercholesterolemia. Analysis of a 1-day dietary recall showed the following: Kilocalories 3,475 kcal Cholesterol 822 mg Protein 102 g Saturated fat 69 g Carbohydrate 383 g Total Fat 165 g Fiber crude 6 g Changes in which one of the following dietary components would have the greatest effect in lowering plasma cholesterol? A. Cholesterol. B. Saturated fat. C. Polyunsaturated fat. D. Monounsaturated fat. E. Carbohydrate.

D.

A severe thiamin deficiency with symptoms of eye problems, difficulty walking and deranged mental function caused by excessive alcohol consumption is called: a. Tetany b. Scurvy c. Polyneuropathy d. Wernicke-Korsakoff Syndrome

C.

A term infant is born at home and does well with breast-feeding. Two days later, the mother calls frantically because the baby is bleeding from the umbilical cord and nostrils.The most likely cause is a. Deficiency of vitamin C due to a citrus-poor diet during pregnancy b. Hypervitaminosis A due to ingestion of beef liver during pregnancy c. Deficiency of vitamin K because infant intestines are sterile d. Deficiency of vitamin K because of disseminated intravascular coagulation (disseminated clotting due to infantile sepsis) e. Deficiency of vitamin E due to maternal malabsorption during pregnancy

The answer is B: Enhance fatty acid oxidation. Metformin, through its activation of the AMP-activated protein kinase, will stimulate glucose entry into the muscle (thus, answer choice A is incorrect) and also increase fatty acid oxidation. The AMP-activated protein kinase will phosphorylate and inhibit acetyl-CoA carboxylase and will phosphorylate and activate malonyl-CoA decarboxylase. Thus, malonyl-CoA levels drop, leading to enhanced entry of fatty acids into the mitochondria, and an increase in fatty acid oxidation. This occurs as the malonyl-CoA inhibition of carnitine palmityl transferase 1 is now lifted due to the reduction of malonyl-CoA levels (see the figure below). Metformin does not stimulate glucose release from the muscle, as the muscle lacks glucose-6-phosphatase activity. Metformin also reduces gluconeogenesis in the liver at a transcriptional level.

A type 2 diabetic has been taking metformin to help regulate blood glucose levels. What effect will metformin also exert within the muscle? (A) Reduce glucose uptake from the circulation (B) Enhance fatty acid oxidation (C) Reduce fatty acid oxidation (D) Stimulate glucose release (E) Enhance gluconeogenesis

The answer is e. (Murray, pp 242-248. Scriver, pp 2513-2570.) Although defects in any of these enzymes will result in a buildup of ammonia in the bloodstream and ammonia intoxication, blocks at carbamoyl phosphate synthase I and ornithine transcarbamylase are usually more severe. Ornithine transcarbamylase deficiency (300461) is an X-linked recessive disorder, allowing for mild manifestations in female carriers; deficiencies in the other four enzymes are autosomal recessive traits. Gene therapy approaches to treatment are being tested, but resulted in the death of one patient due to suspected reaction to the adenovirus vector.

A woman consults her physician because of occasional periods of confusion and lethargy, usually after a meal. The physician suspects a deficiency in an enzyme of the urea cycle, which in this case would most likely be which of the following? a. Arginase b. Argininosuccinase c. Argininosuccinic acid synthase d. Carbamoyl phosphate synthase I e. Ornithine transcarbamylase deficiency

The answer is A: N5, N10-methylenetetrahydrofolate reductase. A thermolabile (temperature-sensitive) N5, N10-methylene-THF reductase would reduce the amount of N5-methyl THF which can be formed, and therefore reduce the amount of homocysteine converted to methionine. This would lead to a reduction of S-adenosylmethionine levels, and hypomethylation in the nervous system, which may lead to altered gene expression and a neural tube defect. Elevated homocysteine would also be evident. This is a common mutation in the general population, and can be overcome by taking folic acid. An inactivating mutation in serine-hydroxymethyl transferase would reduce the major entry point of carbons into the THF pool, but there are other means to do this, so a loss of this enzyme would not result in reduced S-adenosylmethionine levels in the nervous system. A defect in ornithine transcarbamoylase (a urea cycle enzyme) would not affect homocysteine and methionine metabolism. Similarly, mutations in phenylalanine hydroxylase, or tyrosine aminotransferase (enzymes involved in phenylalanine and tyrosine metabolism, respectively) would not affect homocysteine and methionine metabolism.

A woman, who eats a standard meat-containing diet, has had one child born with a neural tube defect, and is considering becoming pregnant again. Blood work showed normal levels of B12 and total folic acid (specific type of folic acid not specified). One possible explanation for the woman's difficulties in her first pregnancy is a thermolabile variant of which of the following enzymes? (A) N5, N10-methylenetetrahydrofolate reductase (B) Serine hydroxymethyl transferase (C) Ornithine transcarbamoylase (D) Phenylalanine hydroxylase (E) Tyrosine aminotransferase

A.

A. A 20-year-old woman ingests 100 tablets (500 mg per tablet) of acetaminophen in a suicide attempt; she does not lose consciousness. She seems healthy for the next 2 days and then develops massive hepatic failure.Q;Which of the following is the primary mechanism responsible for the hepatic failure? A.Depletion of intracellular reduced glutathione B.Formation of excessive superoxide in the hepatocytes C. Inhibition of adenylyl cyclase D. Inhibition of ATPase-dependent transport E. Stimulation of cytochrome oxidase

A 25-year-old man complains of dyspnea. He has a family history of lung disease developing at an early age. Physical examination shows inspiratory crackles and diminished breath sounds in the lower lobes of both lungs. A chest radiograph reveals enlargement of the air spaces in both lower lobes. A serum protein electrophoresis most likely shows which of the following? A. Absent α1-globulin peak B. Absent α2-globulin peak C. Absent β-globulin peak D. Flat γ-globulin peak E. Monoclonal spike

A. Absent α1-globulin peak The patient has α1-antitrypsin (AAT) deficiency, which is an autosomal recessive disorder. Serum protein electrophoresis shows absence of the α1-globulin peak, which is normally occupied by AAT, an antiprotease that destroys elastases produced by neutrophils. In the lungs, decreased production of AAT causes neutrophilic damage to the elastic tissue structure of the entire respiratory unit (respiratory bronchioles, alveolar ducts, and alveoli) in the lower lobes, which is known as panacinar emphysema. Air is trapped in the respiratory unit, causing permanent enlargement of the distal airways.

All of the following statements regarding the actions and inactivation of acetylcholine are true EXCEPT which one? A. Acetylcholine is inactivated by a reuptake mechanism operating in the presynaptic terminus. B. Acetylcholine is the neurotransmitter for the preganglionic neurons of the autonomic nervous system. C. Postganglionic acetylcholine receptors are classified pharmacologically as either nicotinic or muscarinic. D. Acetylcholine is inactivated by hydrolysis by acetylcholinesterase. E. Following synthesis in the presynaptic terminal, approximately one-half of the acetylcholine remains free in the cytoplasm.

A. Acetylcholine is inactivated by a reuptake mechanism operating in the presynaptic terminus. Ach is inactivated by Acetylcholinesterase.

A 65-year-old woman with acute myelogenous leukemia is going to be placed on remission chemotherapy. Which of the following drugs should be used to prevent urate nephropathy? A. Allopurinol B. Aspirin C. Probenecid D. Thiazides E. Sulfinpyrazone

A. Allopurinol The correct answer is A, allopurinol. Allopurinol is a xanthine oxidase inhibitor. In cancer patients this drug prevents excessive production of uric acid released from dying cancer cells. Excessive uric acid in the kidney may result in acute renal failure due to blockage of the renal tubules by urate crystals. Aspirin at low doses decreases uric acid in uric acid, and in high doses is uricosuric. Probenecid is a uricosuric drug that is used in treating underexcretors with gout. Thiazides can produce hyperuricemia by increasing proximal tubule reabsorption of uric acid and decreasing secretion of uric acid. Sulfinpyrazone is a uricosuric drug used to treat underexcretors with gout.

A compensatory mechanism to allow adequate oxygen delivery to the tissues at high altitudes, where oxygen concentrations are low, would be which of the following? A. An increase in 2,3-bisphosphoglycerate synthesis by the red cell B. A decrease in 2,3-bisphosphoglycerate synthesis by the red cell C. An increase in hemoglobin synthesis by the red cell D. A decrease in hemoglobin synthesis by the red cell E. Decreasing the blood pH

A. An increase in 2,3-bisphosphoglycerate synthesis by the red cell 2,3-bisphosphoglycerate favors the deoxy conformation of Hb. Therefore, increased concentrations of 2,3-BPG would allow more oxygen to be released in the tissues. See Marks' 4th edition, Chap 44, ques 1, page 965

Which of the following drugs leads to the covalent modification and inactivation of both COX1 and COX2? A. Aspirin B. Tylenol C. Advil D. Celebrex E. Vioxx

A. Aspirin Only Aspirin irreversibly (covalently) binds to both COX-1 and COX-2. Other NSAIDS, like Advil, reversibly (non-covalently) bind to COX-1 and COX-2. Why is this important? In preparation for a major surgery, NSAIDs must be discontinued prior to surgery due to the risk of bleeding. Aspirin exerts a longer-lasting effect and must be discontinued earlier than other NSAIDS, like Advil. Recall that Tylenol is not considered an NSAID, though its MOA is not fully understood. The selective COX-2 inhibitors (Celebrex and Vioxx) must be used with care, because they can increase the risk of clot formation in some patients due to the selective nature of their MOA.

All of the statements concerning vitamin B12 are CORRECT EXCEPT: A. B12 is primarily transported in the blood by binding to albumin. B. The Schilling test helps confirm a B12 deficiency. C. Absorption of B12 occurs in the terminal ileum. D. In B12 deficiency, urinary methyl malonate excretion increases. E. Removal of the stomach could lead to a deficiency of B12.

A. B12 is primarily transported in the blood by binding to albumin. The correct answer is A. Dietary B12 binds to R-binders (haptocorrins) in the stomach and then travels to the intestine, where R-binders are released and destroyed by pancreatic proteases. Free B12 binds to intrinsic factor (IF) (synthesized by parietal cells in the stomach) in the intestine. B12-IF complex travels to the terminal ileum and carried by proteins called transcobalamins (TC) to the liver, where B12 is stored.

What is the major type of cell that responds to cytokines? A. Blood-forming B. Fibroblasts C. Hepatic D. Peripheral nervous system E. Smooth muscle

A. Blood-forming Cytokines have far reaching effects, and they are especially important in the immune system. Cytokines regulate the maturation, growth, and responses of many cell populations, including components of both the cell-mediated and humoral immune systems.

A 28-year-old woman has just finished eating a high-protein dinner. Which of the following enzymes currently is most active in this patient? A. Carbamoyl phosphate synthase I B. Carnitine acyltransferase I C. Fructose-1,6-bisphosphatase D. Hormone-sensitive lipase E. Liver phosphorylase

A. Carbamoyl phosphate synthase I Carbamoyl phosphate synthase I is the rate-limiting enzyme in the urea cycle, which increases in the well-fed state to dispose of excess nitrogen after a high-protein meal. The enzyme is located in the mitochondrion and catalyzes the formation of carbamoyl phosphate from ammonia, carbon dioxide, and ATP.

A 30-year-old woman has just finished eating dinner. Which of the following rate-limiting enzymes is currently operative? A. Carbamoyl phosphate synthetase II B. Carnitine acyltransferase I C. Fructose 1,6-bisphosphatase D. Hormone-sensitive lipase E. Liver phosphorylase

A. Carbamoyl phosphate synthetase II The patient is in the fed state. Carbamoyl phosphate synthetase (CPS II) is the rate-limiting enzyme in pyrimidine synthesis, which occurs in the fed state. The enzyme is located in the cytosol and catalyzes the formation of carbamoyl phosphate from glutamine, CO2, and ATP.

What type of proteases are linked to apoptosis? A. Caspases B. Collagenases C. Elastases D. Metalloprotease E. Serine proteases

A. Caspases In both intrinsic and extrinsic pathways, activated caspases (cytosolic proteases) cause cellular breakdown via chromatin condensation, membrane blebbing, and formation of apoptotic bodies to be phagocytosed.

Which catalyzes the conversion of hydrogen peroxide to water? A. Catalase B. Superoxide dismutase C. Dioxygenase D. Oxidase E. Monooxygenase

A. Catalase The correct answer is A, catalase. Superoxide dismutase converts superoxide anion to hydrogen peroxide. Dioxygenase use two oxygen atoms into substrate. Oxidase produces hydrogen peroxide, and monooxygenase produces one water and incorporates other oxygen into substrate.

Shortly after his normal birth at term, a breast-fed male infant becomes irritable and is found to have hypoglycemia. Testing the urine reveals the presence of a reducing substance. What is the most likely diagnosis? A. Classic galactosemia B. Galactokinase deficiency C. Glycogen storage disease type 1 D. Hereditary fructose intolerance E. Islet cell hyperplasia

A. Classic galactosemia Classic galactosemia, caused by deficiency of galactose 1-phosphate uridyltransferase, typically causes hypoglycemia when milk feeding is started; galactose excreted in the urine causes a positive test for reducing substances. Galactokinase deficiency causes galactosuria but not hypoglycemia. GSD type 1 is a cause of neonatal hypoglycemia, but is not associated with the presence of a urinary reducing sugar. Hereditary fructose intolerance causes hypoglycemia when fructose or sucrose is ingested; neither is present in breast milk. Islet cell hyperplasia can cause neonatal hypoglycemia, but would not account for the presence of a urinary reducing substance.

A 66-year-old man has severe chest pain that radiates down the inside of his left arm. His medical history suggests chronic alcoholism, and the physical examination shows no neurological abnormalities, but laboratory results show macrocytic anemia with hyper-segmented neutrophils and mild pancytopenia. His plasma homocysteine concentration is increased, and an electrocardiography shows evidence of a prior acute anterior myocardial infarction. Decrease in which of the following best explains the increase in plasma homocysteine in this patient? A. Cystathionine synthase B. Thymidylate synthase C. Serum folate D. Dihydrofolate reductase E. Serum vitamin B12

A. Cystathionine synthase Alcoholism is the most common cause of folate deficiency due to poor diet that decreases liver stores of folate and decreases reabsorption of monoglutamate form of folate in the jejunum. Vitamin B12 removes the methyl group from N5 methyltetrahydrofolate (circulating form of folate) to produce methyl-vitamin B12 and tetrahydrofolate. The methyl group is transferred to homocysteine to produce methionine, and thus decrease in either N5-methyltetrahydrofolate or vitamin B12 results in increased plasma homocysteine concentration. An increase in plasma homocysteine leads to endothelial cell damage and potential vessel thrombosis that can have a role in the patient's acute myocardial infarction. Vitamin B12 deficiency produces neurologic abnormalities in the spinal cord and central nervous system. This patient has no neurologic symptoms. Decreased activity of cystathionine synthase is present in homocystinuria, a rare autosomal recessive disease. Cystathionine synthase catalyzes conversion of homocysteine to cystathionine. Dihydrofolate reductase converts oxidized dihydrofolate to tetrahydrofolate. Thymidylate synthase requires folate and converts deoxyribose uridine monophosphate to deoxyribose thymidine monophosphate.

A 40-year-old man with chronic bronchitis due to cigarette smoking is being treated with theophylline. He also has peptic ulcer disease and is taking cimetidine to control gastric acidity. A therapeutic drug level for theophylline is in the toxic range. Assuming that the patient is taking the appropriate dose of theophylline, what is the most likely cause for the toxic levels of the drug? A. Decreased liver metabolism of the drug B. Decreased liver uptake of the drug C. Increased gastrointestinal reabsorption of the drug D. Increased renal excretion of the drug

A. Decreased liver metabolism of the drug The correct answer is A. Cimetidine, an imidazole compound that blocks histamine receptors, also inhibits cytochrome enzymes in the liver microsomal mixed function oxidase system. This system is located in the smooth endoplasmic reticulum (SER) and is responsible for drug and chemical metabolism. Inhibition of the system produces drug toxicity.

A mother with an increase in serum α-fetoprotein gives birth to a child with a meningomyelocele. Which of the following supplement regimes could have prevented this malformation? A. Folic acid prior to pregnancy B. Prenatal vitamins throughout pregnancy C. Vitamin B6 (pyridoxine) prior to birth D. Vitamin B12 throughout pregnancy E. Vitamin C throughout pregnancy

A. Folic acid prior to pregnancy The correct answer is A. A meningomyelocele is an example of a neural tube defect and is characterized by a cystic mass that contains the spinal meninges and spinal cord. Neural tube defects are caused by failure of fusion of the lateral folds of the neural plate or rupture of a previously closed neural tube. In either case, the maternal α-fetoprotein is increased in serum and amniotic fluid. Dietary folic acid deficiency is associated with an increased risk of developing neural tube defects. Levels of folic acid must be adequate prior to pregnancy to prevent the defects.

A 22-year-old African American man with a bacterial pneumonia develops a hemolytic anemia characterized by the presence of Heinz bodies in the red blood cells. Which of the following enzymes is most likely deficient? A. Glucose 6-phosphate dehydrogenase B. NADPH oxidase C. Myeloperoxidase D. Pyruvate dehydrogenase E. Pyruvate kinase

A. Glucose 6-phosphate dehydrogenase The patient has glucose 6-phosphate dehydrogenase (G6PD) deficiency, which is an X-linked recessive disorder. G6PD is the rate-limiting enzyme of the pentose phosphate pathway, which is the major source of NADPH for reductive biosynthesis. It is also the major source of glutathione (GSH), which is an antioxidant that neutralizes the oxidant activity of hydrogen peroxide by converting it to water. Deficiency of G6PD causes a hemolytic anemia characterized by oxidant damage of the red blood cell membrane and damage to hemoglobin (Hb) producing Heinz bodies (clumps of denatured Hb). Infection is the most common initiating event for hemolysis in G6PD deficiency. This is due to the lack of NADPH, which is the cofactor for NADPH oxidase in the cell membrane of neutrophils and monocytes. In normal circumstances, this enzyme converts molecular O2 to superoxide free radicals. Superoxide dismutase then converts superoxide free radicals to hydrogen peroxide. Myeloperoxidase (MPO) combines hydrogen peroxide with chloride anions to produce hypochlorous acid (component of bleach), which kills the phagocytosed bacteria. Because of the lack of NADPH in G6PD deficiency, this system is dysfunctional, causing increased susceptibility to bacterial infections. Oxidant drugs (e.g., primaquine, dapsone) also precipitate hemolysis in G6PD deficiency.

cAMP regulates the transcription of many genes. What is the major mechanism for this action? A. It induces the phosphorylation of transcription factors. B. It binds directly to cAMP response elements in promoters and enhancers. C. It mediates this effect by increasing the calcium concentration in the nucleus. D. It binds directly to nuclear transcription factors. E. It activates release of cortisol from adrenal medulla, which activates glucocorticoid receptor.

A. It induces the phosphorylation of transcription factors. The correct answer is A, it induces the phosphorylation of transcription factors by activating PKA.

A key cellular characteristic that permits the local expansion and evolution of the pre-tumor cell into a tumor cell is which of the following? A. Limitless replicative potential B. Tissue invasion C. Angiogenesis D. Anti-cancer drug resistance

A. Limitless replicative potential While all the choices are crucial for tumor growth, the first step that happens to a pre-tumor cell becoming a tumor cell is the idea of replicating non-stop. Tissue invasion, angiogenesis, and drug resistance happens once the tumor has been established.

The cytoplasmic chemical most responsible for controlling the concentration of methemoglobin in the red blood cell is: A. NADH. B. NADPH. C. ATP. D. 2,3-BPG. E. Ca2+.

A. NADH. NADH is used to reduce methemoglobin to hemoglobin. So, decreased concentrations of NADH would lead to increased concentrations of methemoglobin.

A group of scientists found that a nuclear hormone receptor CAR (Constitutive Androstane Receptor) directly activates cytochrome P450 enzyme gene Cyp2b10 (human CYP2B6) in mouse liver. Phenobarbital, a drug for seizure treatment, injection kills the mice deficient in CAR but not wild type mice. Based on these scientific observations, individuals with loss of function mutations in the CAR gene should be categorized as which of the following in terms of phenobarbital treatment? A. Poor metabolizers B. Intermediate metabolizers C. Extensive metabolizers D. Ultrarapid metabolizers E. Supernatural metabolizers

A. Poor metabolizers

The choline of acetylcholine, a neurotransmitter, is obtained from which amino acid? A. Serine B. Alanine C. Tyrosine D. Tryptophan E. Lysine

A. Serine

A 59-year-old man is brought into the emergency room complaining of chest pain. He has been taking a drug for angina and coronary artery disease, which his physician told him helps open his coronary blood vessels. Which of the following best explains the biochemical mechanism of action of the angina medication? A. Simulation of guanylyl cyclase activity B. Activation of NADPH oxidase C. Inhibition of cAMP phosphodiesterase D. Stimulation of adenyl cyclase activity

A. Simulation of guanylyl cyclase activity Stimulation of guanylyl cyclase activity could increase production of Nitrous Oxide (NO), which induces vasodilation.

Which of the following is involved in formation of deoxyribonucleotides from ribonucleotides? A. Thioredoxin B. Dihydrofolate reductase C. Hypoxanthine phosphoribosyl transferase D. 5phosphoribosyl1pyrophosphate E. Glutamine

A. Thioredoxin The conversion of ribonucleotide diphosphates to deoxyribonucleotide diphosphate is performed by ribonucleotide reductase (RR), which requires reduced thioredoxin to transfer NADPH derived hydrogen to 2' position of the ribose sugar.

Zinc finger domains are characteristic of hormone receptors that: A. bind steroids. B. have seven transmembrane domains. C. interact with heterotrimeric guanine nucleotide-binding proteins. D. possess intrinsic tyrosine kinase activity. E. regulate the flux of Ca2+ into the cytoplasm.

A. bind steroids.

The identification of zinc finger domains in the structure of a hormone receptor would predict: A. binding of the receptor to double-stranded DNA. B. interaction of the receptor with a heterotrimeric G protein. C. migration of the receptor from vesicles to the plasma membrane. D. regulation of the receptor by protein kinase C. E. transduction of a signal by intrinsic tyrosine phosphorylation.

A. binding of the receptor to double-stranded DNA.

Which of the following conversions involves a cytochrome P450-catalyzed reaction? A. cholesterol to estradiol B. glucose to glycogen C. glucose 6-phosphate to ribose 5-phosphate D. pyruvate to acetyl-CoA E. propionyl-CoA to succinate

A. cholesterol to estradiol P450 is necessary for the formation of all hormones of the adrenal cortex, including aldosterone, cortisol, and androgens.

Hormones affect the rates of metabolic pathways by all of the following EXCEPT: A. forming covalent bonds with intermediates in metabolic pathways. B. stimulating uptake of metabolites by cells. C. stimulating the genome to produce specific transcripts. D. activating protein kinases. E. binding to specific receptors.

A. forming covalent bonds with intermediates in metabolic pathways.

The viscosity of the RBC membrane is decreased by: A. lowering the cholesterol content. B. increasing the percentage of saturated fatty acids in the membrane phospholipids. C. decreasing the temperature. D. increasing the chain length of the constituent fatty acids. E. replacing cis-unsaturated fatty acids with trans-unsaturated fatty acids.

A. lowering the cholesterol content. By lowering cholesterol content, the viscosity will decrease. All the other options would increase the viscosity.

When a major coronary artery is blocked, ischemia (lower oxygen supply) results leading to inhibition of mitochondrial electron transport and oxidative phosphorylation. Reperfusion (introducing oxygen) causes more damage as oxygen radicals are formed. All of the following statements are correct EXCEPT: A. reactive oxygen species are produced when there is an addition of 4 electrons to O2. B. superoxide and hydroxyl radical are two forms of reactive oxygen. C. reactive oxygen damages lipids, proteins and nucleic acids. D. glutathione peroxidase protects against hydrogen peroxide by reducing it to water.

A. reactive oxygen species are produced when there is an addition of 4 electrons to O2. Only 1 electron is added to make a ROS.

Insulin: A. stimulates the phosphodiesterase enzyme which breaks down cAMP. B. decreases the concentration of fructose-2,6-bisphosphate in liver. C. inhibits hepatic glycolysis. D. stimulates glucose transport into the liver. E. has the same effect on glycolysis in the liver as glucagon.

A. stimulates the phosphodiesterase enzyme which breaks down cAMP. Glucagon works via cAMP signaling, so this mechanism allows insulin to inhibit the effect of glucagon when glucose levels rise.

The insulin-receptor has in common with other growth-factor receptors, such as epidermal growth factor, A. the ability to phosphorylate tyrosine residues on certain proteins. B. the ability to phosphorylate serine residues on certain proteins. C. their location in the cytosol. D. the ability to bind steroid hormones. E. the presence of a bound GTP.

A. the ability to phosphorylate tyrosine residues on certain proteins.

Some vitamins have antioxidant properties and therefore are considered promising for the prevention of cancer, atherosclerosis, and age-related degenerative diseases. All of the following have been shown to be part of the body's primary antioxidant defense system EXCEPT: A. thiamine (vitamin B1). B. tocopherol (vitamin E). C. ascorbic acid (vitamin C). D. retinol (vitamin A).

A. thiamine (vitamin B1).

A non-essential amino acid which serves as the biological precursor of the catecholamine neurotransmitters: A. tyrosine B. arginine C. leucine D. methionine E. tryptophan

A. tyrosine

A cell line with an inactive hormone receptor has been constructed through site-directed mutagenesis. When a specific hormone is added to both the mutant and the control, the mutant line shows no formation of inositol triphosphate, and the control shows normal increases in this intracellular signal. Which receptor is most likely the one that was inactivated? A. α1-Adrenergic receptor B. β-Adrenergic receptor C. Cholinergic nicotinic receptor D. GABA receptor

A. α1-Adrenergic receptor The α1-adrenergic receptor (α1- adrenoceptor) is a G protein-linked receptor. G alpha-q protein-linked receptors act indirectly to regulate the activity of a separate plasma membrane-bound target protein. The α1-adrenergic receptors are coupled to phospholipase C via G proteins. Phospholipase C cleaves phosphatidylinositol 4,5-bisphosphate (PIP2) to inositol triphosphate (IP3) and diacylglycerol (DAG). The β-adrenergic receptor is a G protein-linked receptor that is coupled to the activation of adenylyl cyclase. Cholinergic nicotinic receptors and GABA receptors are ligand-gated ion channels. The insulin receptor has an inherent enzymatic activity as part of its structure. Its cytosolic domain has tyrosine kinase activity.

Which of the following inhibitors is correctly paired with the glycolytic or gluconeogenic enzyme that it inhibits?

AMP: fructose-1,6-bisphosphatase

Enzymes that require Mg as Cofactor

ATPases Adenylate cyclase Kinases

Aspirin overdose

Acetyl salicylate is used as an antipyretic drug However, overdose leads to hyperthermia At high levels this uncouples ox phos by disrupting the H+ gradient across the inner membrane and causes energy dissipation as heat Stimulates the brain to cause hyperventilation Tx is gastric lavage and hemodialysis Dinitrophenol uncouples and causes hyperthermia as well(classical)

Following a meal, the ratio of insulin to glucagon will be > 1. Which of the following enzymes will be dephosphorylated and active under these conditions?

AcetylCoA Carboxylase

Enterokinase

Activates trypsinogen

Coenzyme A

Activator of acyl groups, synthesized from ATP and vitamin B5 Forms thioester linkage with acyl groups (i.e. acetate or FA) This activation facilitates transacylation, condensation and red-ox rxn

Acetyl CoA Carboxylase (ACC)

Adds a CO₂ to acetyl CoA Uses ATP (for energy) and Biotin as co-factor Exists in dimeric (inactive) or polymeric (active) forms

C

Adenosine deaminase deficiency: A. Causes dysfunction of cellular immunity not effecting the humoral immune response B. Causes elevation of DNA synthesis in T lymphocytes C. Causes "pyrimidine starvation" D. Is inherited as an X-linked recessive trait E. Can be effectively cured by enzyme replacement therapy

D.

Aflatoxin B1 is directly involved in hepatocarcinogenesis in humans primarily due to: a. Increased glycolysis b. Increased the activity of the Ito cells c. Increased apoptosis in hepatocytes d. Increased mitosis in hepatocytes e. Decreased senescence in hepatocytes

D.

After a 2-week alcoholic binge, Hugh Heffner ingested some Tylenol to help him with a severe headache. He took three times the suggested dose because of the severity of the pain. Within 24 hours, Hugh became very lethargic, vomited frequently, and developed severe abdominal pain. The symptoms Hugh is experiencing are attributable to a reaction to Tylenol because of which of the following? a. The hypoglycemia experienced by the patient b. Ethanol-induced inhibition of Tylenol metabolism c. The hyperglycemia experienced by the patient d. Ethanol-induced acceleration of Tylenol metabolism e. Acetaminophen inhibition of VLDL secretion by the liver

The answer is D. Ethanol induces the CYP2E1 system, which converts acetaminophen to NAPQI, a toxic intermediate. Under normal conditions (noninduced levels of CYP2E1), the conversion of acetaminophen to NAPQI results in low levels of NAPQI being produced, which can easily be detoxified. However, when CYP2E1 is induced, the excessive levels of NAPQI being produced when Tylenol is taken in greater than recommended amounts cannot be readily detoxified, and NAPQI binds to proteins and inactivates them, leading to hepatocyte death. The toxicity is neither related to blood glucose levels (thus, A and C are incorrect), nor to secretion of VLDL (thus, E is incorrect). Ethanol does not inhibit the detoxification of Tylenol per se, but rather accelerates one of its potential metabolic fates (thus, B is also incorrect).

After a 2-week alcoholic binge, Jean Ann Tonich ingested some Tylenol to help her with a severe headache. She took three times the suggested dose because of the severity of the pain. Within 24 hours, Jean Ann became very lethargic, vomited frequently, and developed severe abdominal pain. The symptoms Jean Ann is experiencing are attributable to a reaction to Tylenol because of which of the following? A. The hypoglycemia experienced by the patient B. Ethanol-induced inhibition of Tylenol metabolism C. The hyperglycemia experienced by the patient D. Ethanol-induced acceleration of Tylenol metabolism E. Acetaminophen inhibition of VLDL secretion by the liver

The answer is C. After a high-carbohydrate meal, glucose is the major fuel for most tissues, including skeletal muscle, adipose tissue, and liver. The increase in blood glucose levels stimulates the release of insulin, not g lucagon. Insulin stimulates the transport of glucose in skeletal muscle and adipose tissue, not brain. Liver, not skeletal muscle, converts glucose to fatty acids. Although the red blood cell uses glucose as its only fuel at all times, it generates ATP from conversion of glucose to lactate, not CO2.

After digestion of a high-carbohydrate meal, which one of the following is most likely to occur? A. Glucagon is released from the pancreas. B. Insulin stimulates the transport of glucose into the brain. C. Liver and skeletal muscle use glucose as their major fuel. D. Skeletal muscles convert glucose to fatty acids. E. Red blood cells oxidize glucose to CO2

The answer is b. (Murray, pp 190-196.) Linoleic and α-linolenic acid are nutritionally essential in that they cannot be made by most animals and are instead made only by plants. Oleic acid and palmitoleic acid can be produced by introduction of a double bond at the ∆9 position of a saturated fatty acid. These essential unsaturated fatty acids are required for synthesis of prostaglandin, thromboxane, leukotriene, and lipoxin.

After finding that infants, particularly those with prematurity, are vulnerable to fatty acid deficiencies, major manufacturers began supplementing their infant formulas with these compounds. Which of the following is a nutritionally essential fatty acid along with its usual dietary source? a. Eicosapentaenoic acid-plants b. Linoleic acid-plants c. Oleic acid-animals d. Palmitoleic acid-animals e. Linolenic acid-animals

D.

Alcohol is most damaging to: a. the brain cells since alcohol can be used as an energy source even before glucose b. the stomach cells since this is the site where ADH metabolizes alcohol c. the cells of the gastrointestinal tract since they are in direct contact with ingested alcohol d. the liver cells where alcohol is oxidized

The jaundice experienced by some neonates can be attributable to which of the following? (select multiple correct answers) A. Low corticosteroid concentrations in immature adrenals of the newborns B. Decreased Y-protein in neonates C. Decreased glucuronyl transferase in neonates D. Decreased Z-protein in neonates E. Increased free bilirubin in neonates caused by competition between albumin and fatty acid in mother's milk for bilirubin binding sites

All answers are correct! :D

The answer is B. Allopurinol inhibits the conversion of both hypoxanthine to xanthine and xanthine to uric acid. This occurs because both of those reactions are catalyzed by xanthine oxidase, the target of allopurinol. Answer A is incorrect because AMP is not converted directly to XMP (AMP, when degraded, is deaminated to form IMP, which loses its phosphate to become inosine, which undergoes phosphorolysis to generate hypoxanthine and ribose 1-phosphate). Answer C is incorrect because the inosine-to-hypoxanthine conversion, catalyzed by nucleoside phosphorylase, is not inhibited by allopurinol. Answer D is incorrect because the conversion of hypoxanthine to xanthine occurs at the free base level, not at the nucleotide level. Answer E is incorrect because GMP is converted first to guanosine (loss of phosphate), the guanosine is converted to guanine and ribose 1-phosphate, and the guanine is then converted to xanthine by guanase.

Allopurinol can be used to treat gout because of its ability to inhibit which one of the following reactions? A. AMP to XMP B. Xanthine to uric acid C. Inosine to hypoxanthine D. IMP to XMP E. XMP to GMP

The correct answer is (D). A. As long as HG-PRT is available, the increased xanthine and hypoxanthine will be salvaged to GMP and IMP, both of which inhibit the rate limiting step of the synthesis.. B. Allopurinol directly inhibits the production of urate. C. Hypoxanthine is a substrate for the inhibited enzyme and, therefore, increases. D. As long as HG-PRT is available, the increasing xanthine and hypoxanthine will consume PRPP. Contrast this to the situation with the Lesch-Nyhan syndrome. E. Xanthine is a substrate for the inhibited enzyme.

Allopurinol is an inhibitor of xanthine oxidase. Administration of allopurinol to a patient with gout and normal HGPRT levels would be expected to lead to all of the following EXCEPT: a. decreased de novo synthesis of IMP b. decreased urate in the urine c. an increase in hypoxanthine in the blood d. increased levels of PRPP e. increased xanthine in the blood

D.

Ammonia,one of the principal products of nitrogen metabolism, is normally converted to urea, in the liver, by a series of enzymatic reactions. Liver disease and congenital or acquired defects in the urea cycle may cause elevations in the blood ammonia concentration .Ammonia is highly neurotoxic and, along with other factors, undoubtedly contributes to the development of encephalopathy and coma that is often a terminal event in patients with severe liver disease .Because the concentration of ammonia in hepatic portal vein blood is normally 5-10 times greater than in mixed venous blood, the gastrointestinal tract is presumed to be the site of most ammonia production . Much of the ammonia in the hepatic portal system is metabolized by the liver,.Q:Which enzymes are responsible for producing the direct donors of nitrogen into the pathway producing urea? A. Arginase and argininosuccinate lyase B. Xanthine oxidase and guanine deaminase C. Glutamate dehydrogenase and glutaminase D. Argininosuccinate synthetase and ornithinc transcarbamoylase E. Aspartate aminotransferase and carbamoyl phosphate synthetase

The answer is d. (Murray, pp 49-59. Scriver, pp 3897-3964.) A coenzyme is a nonprotein organic molecule that binds to an enzyme to aid in its catalytic function. Usually it is involved in the transfer of a specific functional group. A coenzyme usually binds loosely and can be separated from the enzyme. When a coenzyme binds tightly to an enzyme, it is spoken of as a prosthetic group of the enzyme. Coenzymes can be viewed as a second substrate for the enzyme, often undergoing chemical changes that counterbalance those of the substrate. Lipoic acid is a short-chain fatty acid with two sulfhydryl groups that is a coenzyme for the pyruvate dehydrogenase reaction that commits pyruvate to the citric acid cycle by converting it to acetyl CoA rather than lactate. Mutations affecting the pyruvate dehydrogenase enzyme complex cause Leigh syndrome of lactic acidosis and neurologic disease (256000), those affecting glucose-6-phosphate dehydrogenase produce anemia (305900), those affecting glucose-1-phosphate incorporation into glucose and glycogen cause type I glycogen storage disease (232200), those affecting ornithine conversion to citrulline in the urea cycle (ornithine transcarbamylase deficiency—311250) cause hyperammonemia and coma, and those affecting conversion of UDP-galactose to UDP-glucose cause the severe form of galactosemia (230400).

Among the following compounds that may be affected by an inborn error of metabolism, which is a coenzyme? a. Glucose-6-phosphate b. Glucose-1-phosphate c. Ornithine d. Lipoic acid e. UDP-galactose

The answer is D: dGTP. With a purine nucleoside phosphorylase defi ciency, guanosine will accumulate (see the fi gure in the answer to the previous question), which will inhibit the conversion of GMP to guanosine via the actions of 5′-nucleotidase (this is also true for dGMP). As dGMP accumulates, it will be phosphorylated to form dGTP. Concurrently, inosine will accumulate, blocking the conversion of adenosine to inosine and also leading to an increase in dATP levels. The combination of dATP and dGTP leads to inhibition of ribonucleotide reductase in the thymocytes, leading to T-cell depletion. It has also been reported that the accumulation of deoxyguanosine triggers apoptosis in T cells, providing another mechanism for T-cell depletion. This does not affect the B cells in this disorder. None of the other nucleotides listed (dCTP, dTTP, dIMP, and dUTP) will accumulate in this disorder.

An 18-month-old infant has had a history of recurrent bacterial and viral infections. The child has failure to thrive, developmental delay, and tremors. Physical exam shows a lack of peripheral lymphoid tissue. Blood work shows lymphopenia, but normal levels of B-cells and circulating immunoglobulins. Considering the child in the previous question, which one of the following metabolites would you expect to accumulate in the thymocytes? (A) dCTP (B) dTTP (C) dIMP (D) dGTP (E) dUTP

The answer is E: Purine nucleoside phosphorylase. The child has purine nucleoside phosphorylase deficiency, which, for reasons not yet fully elucidated, specifically reduces T-cell counts but not B-cells. Purine nucleoside phosphorylase is one of the salvage enzymes that converts guanosine or inosine to the free base plus ribose-1-phosphate (adenosine is not a substrate for this enzyme). HGPRT defi ciency leads to Lesch-Nyhan syndrome, whose symptoms are quite different (there is no immune defi ciency with an HGPRT defect). APRT defi ciency leads to a buildup of an insoluble metabolite (2, 8-dihydroxyadenine) that precipitates in the kidney and will lead to renal failure. ADA deficiency will lead to an immune disorder, but in ADA deficiency, both B and T cells are defi cient. An adenosine kinase deficiency has not been reported in humans. An overview of the purine salvage pathway is shown below.

An 18-month-old infant has had a history of recurrent bacterial and viral infections. The child has failure to thrive, developmental delay, and tremors. Physical exam shows a lack of peripheral lymphoid tissue. Blood work shows lymphopenia, but normal levels of B-cells and circulating immunoglobulins. This child most likely has a defect in which of the following enzymes? (A) Hypoxanthine guanine phosphoribosyltransferase (HGPRT) (B) Adenine phosphoribosyltransferase (APRT) (C) Adenosine deaminase (ADA) (D) Adenosine kinase (E) Purine nucleoside phosphorylase

The answer is B: Restriction of dietary methionine. The boy has cystinuria, elevated levels of cystine in the urine, due to a defect in a kidney transporter that removes cystine from the urine and sends it back into the blood. Due to this, the concentration of cystine in the urine is higher than normal and reaches levels close to its solubility limit. Cysteine is derived from methionine, so a reduction in methionine levels will reduce cysteine levels, which then leads to a reduction in cystine levels. Increasing ethanol content will lead to dehydration, which will increase the concentration of cystine in the urine, leading to increased precipitation. This would also be the case if the urine were acidified (acidification also reduces the solubility of the cystine stones). Restricting glycine is not effective, as glycine is not a precursor of cysteine biosynthesis. Prescribing diuretics would force the boy to urinate more frequently, and would raise a risk for dehydration, which would lead to possible elevation of cystine concentrations.

An 18-year-old boy was brought to the hospital by his mother due to a sudden onset of flank pain in his left side, radiating toward his pubic area. His urine was reddish-brown in color, and a urinalysis showed the presence of many red blood cells. When his urine was acidified with acetic acid, clusters of flat, hexagonal transparent crystals were noted. A radiograph of the abdomen showed radio-opaque stones in both kidneys. The boy eventually passed a stone whose major component was identified as cystine. A suggestion for treatment is which of the following? (A) Increased ethanol consumption (B) Restriction of dietary methionine (C) Utilize drugs that acidify the urine (D) Restrict dietary glycine (E) Prescribe diuretics

The answer is d. (Murray, pp 481-497. Scriver, pp 4223-4240.) People with bowed legs and other bone malformations were quite common in the northeastern United States following the industrial revolution. This was caused by childhood diets lacking foods with vitamin D and by minimal exposure to sunlight due to the dawn-to-dusk working conditions of the textile mills. Vitamin D is essential for the metabolism of calcium and phosphorus. Soft and malformed bones result from its absence. Liver, fish oil, and egg yolks contain vitamin D, and milk is supplemented with vitamin D by law. In adults, lack of sunlight and a diet poor in vitamin D lead to osteomalacia (soft bones). Dark-skinned peoples are more susceptible to vitamin D deficiency. Biotin deficiency can be caused by diets with excess egg white, leading to dehydration and acidosis from accumulation of carboxylic and lactic acids. Retinoic acid is a vitamin A derivative that can be helpful in treating acne but not vitamin D deficiency. Leafy vegetables are a source of B vitamins such as niacin and cobalamin.

An African American infant presents with prominent forehead, bowing of the limbs, broad and tender wrists, swellings at the costochondral junctions of the ribs, and irritability. The head is deformable, able to be depressed like a ping-pong ball, while palpation of the joints is very painful. Which of the following treatments is recommended? a. Lotions containing retinoic acid b. Diet of baby food containing leafy vegetables c. Diet of baby food containing liver and ground beef d. Milk and sunlight exposure e. Removal of eggs from diet

The answer is e. (Murray, pp 249-269. Scriver, pp 2007-2056.)In the synthesis of cysteine, the following sequence of steps occurs, where SAM is S-adenosylmethionine, cys is cysteine, and α-KG is α-ketoglutarate: methionine → SAM → homocysteine + adenosine homocysteine + serine → cystathionine → cys +α-KG + NH4+ Cystathionine synthetase, a pyridoxal phosphate (vitamin B6) enzyme, catalyzes the condensation of serine and homocysteine to form cystathionine. A deficiency of this enzyme leads to a buildup of homocysteine, which oxidizes to form homocysteine. This may result in mental retardation, but sometimes causes dislocated lenses and a tall, asthenic build reminiscent of Marfan syndrome. Patients with homocystinuria also have a clotting diathesis, requiring care to avoid dehydration during anesthesia. Their cysteine deficiency must be made up from dietary sources. In some cases, dietary intake of vitamin B6 (pyrixodal phosphate) may alleviate symptoms because of its requirement by the crucial enzymes.

An adolescent female develops hemiballismus (repetitive throwing motions of the arms) after anesthesia for a routine operation. She is tall and lanky, and it is noted that she and her sister both had previous operations for dislocated lenses of the eyes. The symptoms are suspicious for the disease homocystinuria (236300). Which of the following statements is descriptive of this disease? a. Patients may be treated with dietary supplements of vitamin B12 b. Patients may be treated with dietary supplements of vitamin C c. There is deficient excretion of homocysteine d. There is increased excretion of cysteine e. There is a defect in the ability to form cystathionine from homocysteine and serine

The answer is E. Ethanol metabolism (which produces high NADH levels) does not prevent glycogen degradation. In fact, glycogen stores would be rapidly depleted under these conditions because of decreased gluconeogenesis. Lactate is converted to pyruvate during gluconeogenesis. The pyruvate-lactate equilibrium greatly favors lactate when NADH is high. Thus, alanine and lactate are prevented from producing glucose. Lactate levels are elevated, and a lactic acidosis can result. Glycerol normally enters gluconeogenesis by forming glycerol3-phosphate, which is oxidized to DHAP. High NADH levels prevent this oxidation. Aspartic acid is converted to oxaloacetate (via transamination), as do other amino acid degradation products that enter the TCA cycle (α-ketoglutarate, succinyl-CoA, fumarate). However, the high NADH levels favor malate formation from oxaloacetate, reducing the amount of oxaloacetate available for gluconeogenesis (through the phosphoenolpyruvate carboxykinase reaction). Thus, the three major gluconeogenic precursors (alanine, glycerol, and lactate) do not form glucose because of the high NADH levels, and as glycogen stores are depleted, hypoglycemia results.

An alcoholic who went on a weekend binge without eating any food was found to have severe hypoglycemia. Hypoglycemia occurred because the metabolism of ethanol prevented the production of blood glucose from which one of the following? Choose the one best answer. (A) Glycogen (B) Lactate (C) Glycerol (D) Oxaloacetate (E) Lactate, glycerol, and oxaloacetate

The answer is C: Substrate-induced activation of hormonesensitive lipase. Individuals with metabolic syndrome have a high concentration of triglyceride in their adipocytes. Hormone-sensitive lipase has a low basal activity in the absence of activation by phosphorylation via protein kinase A. If you combine the high substrate level (triglyceride) with the low level of activity of hormone-sensitive lipase, one sees a release of fatty acids from the adipocyte which is greater than normal (when triglyceride levels are low). The increase in free fatty acid levels then aids in promoting insulin resistance in both the adipocytes and muscle cells. This occurs because the muscle and fat cells will use fatty acids as an energy source instead of glucose, thereby contributing to high blood glucose levels. The increased serum free fatty acid levels do not affect the activity of lipoprotein lipase (which removes fatty acids from VLDL and chylomicrons), nor pancreatic lipase (which digests dietary triglycerides in the intestinal lumen). The fatty acids do not affect the transcription rate of colipase, and while the levels of fatty acids within the liver may alter the activity of microsomal triglyceride transfer protein (MTTP), an alteration in MTTP activity does not lead to the increase in free fatty acid levels in the blood.

An increase in serum free fatty acid levels, as evident in individuals exhibiting metabolic syndrome, occurs due to which of the following? (A) Increased activity of lipoprotein lipase (B) Increased activity of pancreatic lipase (C) Substrate-induced activation of hormone-sensitive lipase (D) Increased transcription of colipase (E) Activation of microsomal triglyceride transfer protein

The answer is C. The glucokinase-regulatory protein regulates glucokinase expression at a posttranscriptional level. A lack of regulatory protein activity results in less glucokinase being present in the cell and a reduced overall rate of glucose phosphorylation by the liver. This results in less circulating glucose being removed by the liver, and a longer clearance time for glucose levels to return to fasting levels. A decrease in the glucokinase Km, or an increase in the Vmax for glucokinase, would lead to the opposite effect, enhanced glucose phosphorylation by the liver, and an accelerated clearance from the circulation (thus, A and B are incorrect). The liver does not express hexokinase, so D and E are also incorrect.

An individual displays impaired glucose tolerance; blood glucose levels remain elevated after a meal for a longer time than is normal, although they do eventually go down to fasting levels. The patient has a normal release of insulin from the pancreas in response to elevated blood glucose levels. Hepatocytes obtained from the patient display normal levels of insulin binding to its receptor, and normal activation of the intrinsic tyrosine kinase activity associated with the insulin receptor. Analysis of glucose 6-phosphate formation within the hepatocytes, however, indicates a much slower rate of formation than in hepatocytes obtained from a normal control. A possible mutation that could lead to these results is which of the following? A. A decrease in the Km of glucokinase B. An increase in the Vmax of glucokinase C. A nonfunctional glucokinase-regulatory protein D. An increase in hexokinase activity E. A decrease in hexokinase activity

The answer is C. Leucine can be transported by several different amino acid systems. Leucine is an essential amino acid, so the body cannot synthesize it (thus, A is incorrect). If the intestine cannot absorb leucine, then the kidneys do not have a chance to reabsorb it, so B is incorrect. Leucine and isoleucine have different structures and cannot substitute for each other in all positions within a protein, thus D is incorrect. Leucine is an important component of proteins and is required for protein synthesis; hence, E is incorrect.

An individual has been shown to have a deficiency in an intestinal epithelial cell amino acid transport system for leucine. However, the individual shows no symptoms of amino acid deficiency. This could be due to which of the following? A. The body synthesizes leucine to compensate for the transport defect. B. The kidney reabsorbs leucine and sends it to other tissues. C. There are multiple transport systems for leucine. D. Isoleucine takes the place of leucine in proteins. E. Leucine is not necessary for bulk protein synthesis.

The answer is B: Easy bruising. The patient, due to the pancreatitis, is not able to adequately digest triglycerides in the intestinal lumen, which is what leads to the steatorrhea. Fat-soluble vitamin absorption is dependent upon triglyceride digestion and absorption, so under these conditions, the patient can become deficient in fat-soluble vitamins (A, D, E, and K). The loss of night vision is an early warning for lack of vitamin A. If the patient is becoming deficient for vitamin K, bruising would become a problem, due to ineffective clotting with slight internal damage. Nystagmus is a symptom of vitamin B1 deficiency, a water-soluble vitamin. Dermatitis is a symptom of niacin deficiency (vitamin B3). Loss of teeth can occur with a vitamin C deficiency, and orange tonsils is not due to a vitamin deficiency, but rather to a lack of ABC-1 activity, and is indicative of Tangier disease.

An individual has developed pancreatitis, and with it, steatorrhea. The patient also reports problems with his night vision, although visual acuity appears normal. Another expected finding in this patient would be which of the following? (A) Nystagmus (B) Easy bruising (C) Dermatitis (D) Loss of teeth (E) Orange tonsils

The answer is B. An individual in sepsis will be catabolic; protein degradation exceeds protein intake (leading to negative nitrogen balance; thus, A, C, and E are incorrect). The liver is synthesizing glucose from amino acid precursors to raise blood glucose levels for the immune cells and the nervous system (thus, gluconeogenesis is active, and E and F are incorrect). Fatty acid release and oxidation has also been stimulated to provide an energy source for the liver and skeletal muscle (indicating that C, D, and F are incorrect).

An individual in sepsis will display which of the following metabolic patterns?

The answer is A: malic enzyme. The transcription of five key enzymes for fatty acid synthesis are regulated by diet; malic enzyme, acetyl-CoA carboxylase, citrate lyase, fatty acid synthase, and glucose-6-phosphate dehydrogenase. The other four enzymes listed in the answer (carnitine acyltransferase I and II, medium-chain acyl-CoA dehydrogenase, and fatty acyl-CoA synthetase) are all involved in fatty acid oxidation, a process which would be increased during fasting, and whose enzyme levels would not be decreased under these conditions.

An individual who has been on a long-term diet will have reduced the transcription of which of the following enzymes? (A) Malic enzyme (B) Carnitine acyltransferase 1 (C) Carnitine acyltransferase 2 (D) Medium chain acyl-CoA dehydrogenase (E) Fatty acyl-CoA synthetase

The answer is C. Trypsinogen, which is secreted by the intestine, is activated by enteropeptidase, a protein found in the intestine (thus, D is backward and incorrect). Once trypsin is formed, it activates all of the other zymogens secreted by the pancreas. Trypsin does not activate pepsinogen (thus, B is incorrect) because pepsinogen is found in the stomach and autocatalyzes its own activation when the pH drops as a result of acid secretion. Trypsin has no effect on intestinal motility (hence, E is incorrect) and also does not have a much broader base of substrates than any other protease (trypsin cleaves on the carboxy side of basic side chains, lysine, and arginine; thus, A is incorrect).

An individual with a deficiency in the conversion of trypsinogen to trypsin would be expected to experience a more detrimental effect on protein digestion than an individual who was defective in any of the other digestive proteases. This is a result of which of the following? A. Trypsin has a greater and wider range of substrates on which to act. B. Trypsin activates pepsinogen, so digestion can begin in the stomach. C. Trypsin activates the other zymogens that are secreted by the pancreas. D. Trypsin activates enteropeptidase, which is needed to activate the other pancreatic zymogens. E. Trypsin inhibits intestinal motility, so the substrates can be hydrolyzed for longer periods.

Correct answer = B. Insulin resistance is the decreased ability of target tissues, such as liver, adipose, and muscle, to respond properly to normal circulating concentrations of insulin. Obesity is the most common cause of insulin resistance. Most of the people with obesity and insulin resistance do not become diabetic. In the absence of a defect in β-cell function, nondiabetic, obese individuals can compensate for insulin resistance with elevated levels of insulin. The elevated insulin levels normalize fasting blood glucose levels. Insulin resistance without overt diabetes requires no pharmacologic treatment.

An individual with insulin resistance and normal βcell function: A. usually shows elevated fasting glucose levels. B. usually shows elevated fasting insulin levels. C. will eventually become diabetic. D. is rarely obese. E. is treated by injection of insulin.

Correct answer = C. Eighty percent of type 2 diabetics are obese, and almost all show some improvement in blood glucose with weight reduction. Symptoms usually develop gradually. These patients have elevated insulin levels, and usually do not require insulin (certainly not 6 hours after a meal). Glucagon levels are typically normal.

An obese individual with type 2 diabetes: A. usually shows a sudden onset of symptoms. B. usually has a lower plasma level of insulin than a normal individual. C. usually shows significant improvement in glucose tolerance if body weight is reduced to normal. D. usually benefits from receiving insulin about 6 hours after a meal. E. usually has lower plasma levels of glucagon than a normal individual.

The answer is A: a-ketoglutarate dehydrogenase. The child has a mutation in the shared E3 subunit of pyruvate dehydrogenase, α-ketoglutarate dehydrogenase, and the branched-chain α-ketoacid dehydrogenase. All three reactions are oxidative decarboxylation reactions and utilize a three-component enzyme complex, designated as E1, E2, and E3 (see the fi gure below). The E1 subunit binds thiamine pyrophosphate and catalyzes the decarboxylation reaction. The E2 subunit is a transacylase and is involved in the oxidation-reduction part of the reaction. The E3 component (dihydrolipoyl dehydrogenase) is shared among all three enzymes, and a mutation in this subunit will affect the activity of all three enzymes. This subunit reduces NAD+, using electrons obtained from reduced lipoic acid. The key to solving the problem is the recognition that lactic acidosis occurs, which would happen when pyruvate dehydrogenase was defective. None of the other dehydrogenases listed (isocitrate dehydrogenase, malate dehydrogenase, and succinate dehydrogenase) require the E3 subunit for their activity, nor do they catalyze oxidative decarboxylation reactions. Acetyl-CoA carboxylase catalyzes a carboxylation reaction, and does not share subunits with the enzymes that catalyze oxidative decarboxylations.

Another routine newborn screening identified a child with elevated levels of the branched-chain amino acids and their α-ketoacid derivatives. In addition, the child also exhibited lactic acidosis. Which enzyme listed below would you expect to be negatively affected (reduced activity) by this disorder? (A) α-ketoglutarate dehydrogenase (B) Isocitrate dehydrogenase (C) Malate dehydrogenase (D) Succinate dehydrogenase (E) Acetyl-CoA carboxylase

Drugs are being developed that will induce the transcription of globin genes, which are normally silent in patients affected with sickle cell disease. A good target gene for such therapy in this disease would be which of the following? A. The a1-gene B. The a2-gene C. The g-gene D. The β-gene E. The x-gene

Answer Key: C Feedback: Incorrect. The correct answer is C. By turning on the g-gene (normally only present in fetal Hb), the defective beta-gene can be bypassed. See Marks' 4th edition, Chap 44, ques 3, page 965 for explanation.

"Sulfa" drugs

Antibacterial agents in the sulfonanilamide family act as inhibitors of the enzyme that incorporates PABA into folate This disrupts DNA replication in bacteria, humans get folate from the diet and are not affected

Carnitine-acylcarnitine translocase (CACT)

Antiporter: FA-carnitine (in) ↔ carnitine (out) Moves FA-carnitine into intermembrane space

Which of the following apoproteins is present on an LDL particle and required for uptake by peripheral tissues?

ApoB 100

Which of the following is the full complement of apoproteins present on VLDLs?

ApoB100; ApoCII; ApoE

Which of the following apoproteins is required for activation of lipoprotein lipase?

ApoCII

The answer is c. (Murray, pp 130-135, 249-263 Scriver, pp 1909-1964.) Arginase catalyzes the conversion of arginine to ornithine, which is subsequently converted to α-ketoglutarate. The guanido group of arginine is converted to urea. Both proline and histidine can also be converted to α-ketoglutarate.

Arginine can be converted to which citric acid cycle intermediate? a. Citrate b. Fumarate c. α-ketoglutarate d. Malate e. Oxaloacetate

Oncogenes, even if they are not enzymes, can still be connected to cancer cell metabolic reprogramming by which of the following? A. A transcription factor that helps to increase glycolysis enzyme levels B. They can convert ATP into energy for biosynthesis. C. They affect mitochondria such that the organelle becomes corrupted and cannot function. D. They shut down the TCA cycle by repressing enzymes essential for proper cycle function.

As noted in table 18.1 from the Marks' textbook, oncogenes can code for many different proteins such as growth factors, receptors, and transcription factors. An oncogene transcription factor may function to elevate glycolysis enzymes that would assist the Warburg effect and they would not be sensitive to regulatory signals to shut the pathway down. Thus, mutant oncogenes may help to reprogram the tumor cell so that sustained growth and limitless replication potential hallmarks will be accomplished by the tumor cell.

Vitamin C disorders

Ascorbic acid which forms collagen(oxidize proline to hydroxyproline) and carnitine(FA oxidation) Deficiency causes scurvy with petechiae(purple hemorrhagic spots on skin), ecchymoses(bruising), spongy/bleeding gums and poor wound healing Citrus fruits are a great source of vitamin C

C.

Asians and Native Americans may flush, vomiting and feel ill after drinking small amounts of ethanol. This reaction is due to genetic variation in an enzyme that metabolizes the liver metabolite of ethanol: a. Acetyl CoA b. Acetate c. Acetaldehyde d. Acetone

The correct answer is (A). A. UTP to CTP does involve an amination but the amino group used is the amide of glutamine. B. Aspartate provides nitrogen 1 of the purine ring. C. The whole aspartate molecule (at least the carbons and nitrogen) becomes part of the pyrimidine ring. D. Maintenance of the adenine nucleotide pool involves salvaging hypoxanthine and then converting IMP to AMP using aspartate. E. Most, probably all, proteins include aspartate.

Aspartate plays a role in all of the following EXCEPT: a. conversion of UTP to CTP b. de novo synthesis of AMP c. de novo synthesis of orotic acid d. maintenances of the adenine nucleotide pool by a salvage mechanism e. the synthesis of most proteins

A 55-year-old man is taking aspirin as a prophylaxis to prevent myocardial infarction and stroke. Which of the following compounds is decreased by aspirin? A. Arachidonic acid B. Bradykinin C. Histamine D. Leukotrienes E. Thromboxane A2 (TXA2)

Aspirin interferes with the production of prostaglandins by irreversibly inhibiting cyclooxygenase, which converts arachidonic acid to prostaglandin H2. In platelets, thromboxane synthase converts prostaglandin H2 to TXA2. TXA2 is a vasoconstrictor and also promotes platelet aggregation to form a platelet thrombus. Since myocardial infarctions and stroke are most often caused by development of a platelet thrombus overlying an atherosclerotic plaque, aspirin reduces the risk for these vascular disorders.

Acetyl CoA Acetyltransferase (β-ketothiolase)

Attaches Sulfur of CoA to ketone formed from cleavage of acetyl CoA from fatty acyl chain which is shortened by 2C

Von Gierke Disease(GSD1)

Autosomal recessive deficiency of G6Phosphatase activity which releases glucose into the blood Marked fasting hypoglycemia due to inability to replenish BS levels through glycogenolysis and gluconeogenesis Present with lactic acidosis and hepatomegaly due to glycogen build up and hyperlipidemia Lactic academia in those who present with retarded growth Different subunit mutations of G6Phophatase cause the different sub divisions of GSD1(catalytic-a and transporter-b,c)

A baby boy, born at term by normal vaginal delivery and initially well, develops tachypnea on the third day of life. Arterial blood gas results are: [H+] 50 nmol/L[34-46nmol/L, pH 7.36-7.44] (pH 7.30) [, pCO2 3.3 kPa [4.5 6.0kPa]. Urine is negative for reducing substances. He becomes progressively more acidotic, and the next day his plasma ammonia concentration is found to be very high. Which diagnosis is most likely? A. A urea cycle defect B. An organic acidemia C. Congenital hypothyroidism D. Galactosemia E. Respiratory distress syndrome (RDS)

B. An organic acidemia Organic acidemias can cause hyperammonemia as well as acidosis. Although hyperammonemia is characteristic of urea cycle defects, it is not unique to them, and patients with these conditions are not usually acidotic. There is nothing in the information provided to suggest the diagnosis of urea cycle deficit. Galactosemia can cause hepatic dysfunction and hence hyperammonemia, but a positive test for urinary reducing substances would be expected. Any severe systemic illness can cause hyperammonemia in the newborn, but RDS typically causes a respiratory acidosis as a result of carbon dioxide retention, and this acidosis is non-respiratory.

What is the most common enzyme deficiency in adrenal hyperplasia? A. CYP11A1 (scc) B. CYP21A2 C. CYP24A1 D. CYP3A4 E. CYP7A1

B. CYP21A2 CYP21A2 is necessary for the synthesis of both aldosterone and cortisol. In CYP21A2 adrenal hyperplasia, no aldosterone nor cortisol will be synthesized, and all processes will shift to overproduction of androgens.

A 72-year-old male presents with a complaint of recent onset of difficulty in performing activities of daily living. Upon questioning, his spouse reveals that he "hasn't been able to get around the way he used to." Physical examination reveals a well-nourished man who walks with an exaggerated kyphosis. His gait appears to be quite slow and wide-based. He also appears to have a resting tremor. The appropriate management of his case would target which of the following? A. Amino acid degradation B. Catecholamine synthesis C. Ganglioside degradation D. Prostaglandin synthesis E. Sphingolipid degradation

B. Catecholamine synthesis The above case describes a patient with Parkinson's disease, which is caused by degeneration of the substantia nigra. This leads to dopamine deficiency in the brain and results in resting tremors, bradykinesia, cog-wheeling of the hand joints, and rigidity of musculature. Dopamine is one of the catecholamines synthesized in a common pathway with norepinephrine and epinephrine. The diseases involving amino acid degradation (choice A), ganglioside degradation (choice C), and sphingolipid degradation (choice E) do not match the presentation seen in the case.

Many herbal remedies are not prescription drugs, and patients often fail to inform their physicians that they are using such products. A patient is taking St. John's wort to improve his mood, which also induces CYP3A4. His statins lower his cholesterol. What adverse drug-xenobiotic interaction is caused by the St. John's wort in this patient? A. Increase in the effectiveness of statin to lower cholesterol B. Decreased effective therapeutic dose of statin C. Increased mood disorder D. Decreased synthesis of coenzyme Q

B. Decreased effective therapeutic dose of statin St. John's wort will increase metabolism of this statin drug, so the effective dose will decrease. Statins are metabolized by CYP3A4 and statin since they inhibit cholesterol synthesis at HMG CoA reductase, they also decrease the synthesis of Coenzyme Q, which is required for electron transport in the mitochondria.

A 35-year-old woman with systemic lupus erythematosus is being treated with aspirin and corticosteroids. Which of the following anti-inflammatory effects is shared by both drugs? A. Decreased synthesis of leukotrienes B. Decreased synthesis of prostaglandins C. Inhibition of cyclooxygenase D. Inhibition of lipoxygenase E. Inhibition of phospholipase A2

B. Decreased synthesis of prostaglandins Corticosteroids inhibit phospholipase A2, which prevents the release of arachidonic acid from the cell membrane. Arachidonic acid is the precursor for prostaglandins and leukotrienes. Aspirin inhibits cyclooxygenase, which decreases production of prostaglandins derived from prostaglandin H2.

A 5-year-old boy has recurrent infections caused by Staphylococcus aureus. The peripheral blood leukocytes are normal, but a leukocyte nitroblue tetrazolium (NBT) test is abnormal. Which of the following best describes the pathogenesis of the recurrent infections? A. Defect in phagolysosome formation B. Defect in producing hydrogen peroxide C. Deficiency of complement C3 D. Deficiency of immunoglobulins E. Deficiency of myeloperoxidase

B. Defect in producing hydrogen peroxide The patient has chronic granulomatous disease, which is an X-linked recessive disease caused by a deficiency of NADPH oxidase in the cell membrane of neutrophils and monocytes. In normal individuals, phagocytosis of bacteria activates NADPH oxidase in the cell membrane. The enzyme oxidizes reduced NADPH, which converts molecular O2 to superoxide free radicals (O2•). The energy released during this reaction is called the respiratory (oxidative) burst. Superoxide is converted to hydrogen peroxide by superoxide dismutase in the phagolysosomes of neutrophils and monocytes. Myeloperoxidase (MPO), a lysosomal enzyme, catalyzes a reaction that combines hydrogen peroxide with chloride anions to form hypochlorous acid free radicals that kill the phagocytosed bacteria. The leukocyte nitroblue tetrazolium test (NBT) evaluates whether the conversion of molecular O2 to superoxide free radicals is intact. A colorless dye is added to a test tube containing neutrophils. Normal neutrophils phagocytose the dye and produce superoxide free radicals that convert the dye to a blue color. In chronic granulomatous disease, the respiratory burst is absent (abnormal leukocyte nitroblue tetrazolium test) and superoxide free radicals and hydrogen peroxide are not produced, resulting in a defect in microbicidal activity (cannot kill Staphylococcus aureus).

A 20-year-old woman complains of chest pain. Physical examination reveals a systolic click followed by a murmur. Which of the following glycosaminoglycans is associated with the valvular lesion that is producing the click and murmur? A. Chondroitin sulfate B. Dermatan sulfate C. Heparan sulfate D. Hyaluronic acid E. Keratan sulfate

B. Dermatan sulfate Glycosaminoglycans (GAGs) are the major components of ground substance in the interstitial tissue. They are complexes of branched, negatively charged polysaccharide chains containing amino sugars (e.g., glucosamine or galactosamine) and acid sugars (iduronic acid or glucuronic acid). The patient has mitral valve prolapse (MVP). MVP is characterized by redundancy of the mitral valve leaflets, which causes them to prolapse into the left atrium during systole. Sudden cessation of valve movement produces the click and this is followed by a short systolic murmur caused by mitral regurgitation. Dermatan sulfate is the glycosaminoglycan (GAG) that is present in excess in the mitral valve tissue. Sections through the mitral valve in MVP show myxomatous degeneration.

A patient with adenocarcinoma is undergoing chemotherapy with 5-Fluorouracil and methotrexate at a standard dose. After two rounds of therapy, a CAT scan revealed that the tumor had grown, and the patient experienced no toxic effects of the chemotherapy. What enzyme is overexpressed in this patient's tumor and normal tissues, which may account for the ineffectiveness of chemotherapy? A. CTP synthetase B. Dihydropyrimidine dehydrogenase C. Thymidylate synthase D. Deoxycytidylate deaminase E. DNA polymerase

B. Dihydropyrimidine dehydrogenase The correct answer is B, dihydropyrimidine dehydrogenase. Overexpression of this enzyme in both the tumor and normal intestinal tissue would account for the ineffectiveness of 5-fluorouracil in inhibiting thymidylate synthase. Overexpression of CTP synthetase would produce more CTP that would inhibit CPS II and de novo pyrimidine synthesis, and thus this would inhibit tumor DNA synthesis and replication of normal intestinal epithelial cells. Increased synthesis of thymidylate synthase in tumor can account for tumor resistance to 5-FU, however this enzyme is rarely overexpressed in normal colon tissue, therefore the toxic effects of 5-FU would result in intestinal problems. Overexpression of deoxycytidylate deaminase produce dUMP, which would compete with 5-FU for conversion to dTMP. Overexpression of DNA polymerase can account for tumor growth, however overexpression in colon would present as a hyperproliferative intestinal disorder.

In the pathway for bilirubin breakdown, the deficiency of what enzyme leads to Crigler-Najjar syndrome, type I? A. Alkaline phosphatase B. Glucuronyl transferase C. Heme oxygenase D. Biliverdin reductase E. UDPG dehydrogenase

B. Glucuronyl transferase

Which finding most suggests that fluid depletion is due to predominant water loss rather than isotonic fluid loss? A. Elevated plasma urea concentration B. Highly concentrated urine C. Increased hematocrit D. Normal plasma sodium concentration E. Tachycardia

B. Highly concentrated urine Predominant water loss tends to raise plasma osmolality, triggering the secretion of vasopressin (antidiuretic hormone). This maximizes the reabsorption of water from the tubular fluid so that the urine becomes highly concentrated. Plasma urea concentration tends to rise with fluid loss of either type. A decrease in the glomerular filtration rate (GFR) is the main cause with isotonic loss and increased reabsorption of urea from the highly concentrated urine with predominant water loss. Hypernatremia suggests that water has been lost without a concomitant loss of sodium. Loss of isotonic fluid is born primarily by the extracellular compartment and (unless the fluid lost is blood) causes an early increase in hematocrit. Predominant water loss is borne by the whole body water compartment and has a lesser effect on the extracellular fluid volume.

A one-year-old male child is presented to the pediatrician because his parents have noticed that he is mutilating himself by chewing on his lips and hitting his head. A routine examination revealed that the child has signs of gouty arthritis and exhibits signs of mental retardation. Which of the following enzymes is most likely deficient in this child? A. Xanthine oxidase B. Hypoxanthine-guanine phosphoribosyltransferase C. Adenosine deaminase D. Phosphoribosylpyrophosphate aminotransferase

B. Hypoxanthine-guanine phosphoribosyltransferase

Consecutive repeat measurements of blood glucose concentration (all values in mmol/L) in a single blood sample, using the same point of care analyzer, are: 5.1, 4.9, 5.0, 5.0, 5.1, 5.0, 5.2, 5.1, 5.0, 5.1. On the basis of these data, which statement about the analyzer/method is most applicable? A. It has high accuracy. B. It has high precision. C. It is specific for glucose. D. It has high sensitivity. E. It has high specificity.

B. It has high precision. It is suitable for use by patients with diabetes to monitor their blood glucose concentrations. It should be capable of detecting hypoglycemia in patients even if they are asymptomatic. B is correct answer: Precision is a measure of the reproducibility of a measurement; these results fall within a very small range, indicating good precision.

The resting potential of a typical nerve cell membrane is determined primarily by a transmembrane equilibrium dominated by its permeability to: A. Na+. B. K+. C. Ca2+. D. both Na+ and K+. E. all three ions.

B. K+.

Which eicosanoids cause neutrophil and macrophage chemotaxis, respectively? A. PGE2, TXA2 B. LTB4, LXA4 C. LTC4, LTB4 D. PGI2, LXA4 E. 5-HETE, LTB4

B. LTB4, LXA4 LTB4 attracts neutrophils while LXA4 attracts macrophages. Other neutrophil attractors include: interleukin-8 (IL-8), interferon gamma (IFN-γ), and C3a and C5a (part of complement system). Other macrophage attractors include: interferon gamma (IFN-γ) and bacterial endotoxins, such as lipopolysaccharide (LPS).

Aspirin induced asthma is a severe reaction to non-steroidal anti-inflammatory drugs (NSAIDS) characterized by bronchoconstriction after ingestion. It is seen in 20% of the adult population. Which of the following statements best explains the symptoms seen in patients with aspirin induced asthma? A. NSAIDS inhibit the activity of the CFTR protein, resulting in thickened secretions that block the airway. B. NSAIDS inhibit COX but not lipoxygenase resulting in the flow of arachidonic acid to leukotriene synthesis. C. NSAIDS activate phospholipase, leading to decreased amounts of dipalmitoylphosphatidylcholine and alveolar collapse. D. NSAIDS activate COX activity of PGH synthase, resulting in increased synthesis of prostaglandins that promote vasodilation.

B. NSAIDS inhibit COX but not lipoxygenase resulting in the flow of arachidonic acid to leukotriene synthesis. NSAIDS inhibit COX but not lipoxygenase, so arachidonic acid is shunted toward synthesis of leukotrienes, which cause bronchoconstriction. NSAIDS have no effect on the CFTR, the chloride conductance channel, which is defective in cystic fibrosis. Steroids, not NSAIDS, inhibit phospholipase A2, therefore aspirin has no effect on phospholipases.

The angina medication most likely causes vasodilatation of coronary blood vessels by producing which of the following? A. Hydrogen peroxide B. Nitric oxide C. Superoxide D. Lipid peroxides

B. Nitric oxide

A 60-year-old man who has smoked cigarettes for the past 35 years develops a cough and weight loss. Physical examination reveals coarse inspiratory crackles in both lungs. A chest radiograph shows a centrally located mass in the left hilum. Laboratory studies show a serum sodium of 115 mEq/L (136-145 mEq/L). A sputum cytology reveals small, hyperchromatic cells and necrotic debris. Which of the following laboratory results is most likely present? Note: ADH, antidiuretic hormone; POsm, plasma osmolality. A. POsm decreased, serum ADH decreased B. POsm decreased, serum ADH increased C. POsm increased, serum ADH decreased D. POsm increased, serum ADH increased

B. POsm decreased, serum ADH increased A decrease in POsm and an increase in serum ADH occur in the syndrome of inappropriate ADH (SIADH). It is most often caused by ectopic secretion of ADH by a primary small cell carcinoma of the lung (left hilar mass; small, hyperchromatic cells in the sputum) in a patient who smokes cigarettes. Excess ADH causes increased reabsorption of free water, a dilutional hyponatremia (decreased POsm), and a concentrated urine (increased UOsm). The coarse inspiratory crackles are caused by increased mucus in the segmental bronchi and terminal bronchioles (chronic bronchitis).

Arachidonic acid is an w-6 unsaturated fatty acid while eicosapentaenoic acid (EPA) is an w-3 fatty acid, commonly obtained from fish oil. Both the PUFA, EPA and docosahexaenoic acid (DHA) have been shown to suppress cardiac arrhythmias, reduce serum triacylglycerols, decrease the tendency for thrombosis, lower blood pressure and substantially reduce risk of cardiovascular mortality. What is the most likely reason that these w-3 fish oils decrease thrombosis and the incidence of cardiovascular disease? A. Activate COX2 B. Produce less potent prostaglandins and leukotrienes C. Increase the transcellular synthesis of lipoxins D. Inhibit phospholipase A2 activity E. Increase the synthesis of vasodilatory prostacyclin PGI2

B. Produce less potent prostaglandins and leukotrienes The activation of phospholipase A2 releases eicosapentaenoic acid from the sn-2 position of phospholipids, which are metabolized to less potent prostaglandins PGE3 and leukotriene B5. The synthesis of the potent vasoconstrictive TXA2 is reduced and replaced by TXA3.

In one form of severe combined immunodeficiency disease (SCID) an increase in one purine metabolite leads to inhibition of DNA synthesis by reducing the activity of which enzyme? A. PRPP synthase B. Ribonucleotide reductase C. Thymidine kinase D. Thymidylate synthetase

B. Ribonucleotide reductase A defect in adenosine deaminase leads to an accumulation of dATP which inhibits ribonucleotide reductase—inhibiting dNTP synthesis.

Two vitamins which can be considered to function in part as hormones are: A. Vitamin E and niacin. B. Vitamin D and Vitamin A. C. Vitamin K and Vitamin E. D. Vitamin A and Vitamin E. E. none of the above.

B. Vitamin D and Vitamin A.

In the de novo synthesis of pyrimidine nucleotides: A. reactions take place exclusively in the cytosol. B. a free base is formed as an intermediate. C. PRPP is required in the rate-limiting step. D. UMP and CMP are formed from a common intermediate. E. UMP inhibition of OMP-decarboxylase is the major control point.

B. a free base is formed as an intermediate. The answer is B, which is in contrast with purine synthesis where the sugar is produced first and base built on sugar. Part of this process takes place in mitochondria. PRPP is required to convert orotate to OMP and is not rate limiting. OMP to UMP to CTP is a sequential process. UMP does inhibit OMP decarboxylase, but this is not the rate limiting step.

Insulin normally does all of the following EXCEPT: A. recruit glucose transporters 4 from intracellular sites to plasma membrane. B. activate protein kinase A and AMP protein kinase (AMPK). C. activate phosphoprotein phosphatase. D. signal an increase in sterol response element binding proteins SREBP-1. E. inhibits ketogenesis at levels lower than required to inhibit gluconeogenesis.

B. activate protein kinase A and AMP protein kinase (AMPK). These enzymes are phosphorylated in fasting state and insulin promotes dephosphorylation. Insulin causes Glut 4 in muscle and adipose to take in glucose during hyperglycemia. Increased lipogenic enzyme synthesis is mediated by SREBP-1 transcription factor. This accounts in part for the lack of ketoacidosis in type 2 diabetics.

Vasopressin (ADH): A. acts on uterine smooth muscle. B. acts on the distal part of the nephron. C. is secreted in response to elevated blood pressure. D. is secreted in response to decreased plasma osmolality.

B. acts on the distal part of the nephron.

Phenobarbital is a potent inducer of CYP2C9. Warfarin, an anticoagulant is metabolized by CYP2C9. If phenobarbital is added to the therapeutic regimen of a patient with no change in the dosage of warfarin, the expected consequences would be: A. no change in the clinical results. B. an increased possibility of clot formation. C. an increased possibility of hemorrhaging. D. to make the patient sleepy

B. an increased possibility of clot formation. The correct answer is B, increased possibility of clot formation. A potent inducer of CYP2CP will increase the function of CYP2CP, which in the case of warfarin, metabolizes the drug. If a drug is metabolized more quickly, it will be less effective. The function of warfarin is to inhibit clot formation. Therefore, co-administration of phenobarbital and warfarin will increase the risk of clot formation.

Thyroid hormone receptors: A. are histone proteins localized in the chromatin of target cells. B. are intrinsic non-histone chromosomal proteins. C. initially bind triiodothyronine in the cytoplasm and, after undergoing a conformational change, translocate to the nucleus. D. are enzymes that deiodinate thyroxine to triiodothyronine. E. have higher affinity for thyroxine than triiodothyronine.

B. are intrinsic non-histone chromosomal proteins.

Serum isoenzyme measurements are useful in diagnosis because: A. the composition of the serum changes specifically with specific diseases. B. different tissues may contain different isoenzymes, and any damage to a particular tissue releases only its characteristic isoenzymes into serum. C. different tissues may have the same isoenzyme composition, and a given disease specifically releases only certain disease-specific isoenzymes into serum. D. the disease process causes the damaged tissue to synthesize and release specific isoenzymes as a defense mechanism specific for a given disease. E. isoenzyme measurements are expensive and are a good way for hospitals to increase their profits.

B. different tissues may contain different isoenzymes, and any damage to a particular tissue releases only its characteristic isoenzymes into serum.

The neurotransmitter acetylcholine is: A. synthesized by the enzyme choline esterase. B. hydrolyzed by the enzyme acetylcholine esterase at the synapse. C. synthesized in the adrenal medulla. D. the primary neurotransmitter in the brain. E. competitively inhibited by diisopropylfluorophosphate.

B. hydrolyzed by the enzyme acetylcholine esterase at the synapse.

Conversion of hepatic enzymes from non-phosphorylated to their phosphorylated form: A. always activates the enzyme. B. is more likely to occur in the fasted than well-fed state. C. is signaled by insulin. D. is always catalyzed by cAMP dependent protein kinase.

B. is more likely to occur in the fasted than well-fed state. In the well-fed state, insulin:glucagon is high and cAMP levels are low. Some enzymes are active when phosphorylated and the reverse for others. Glucagon signals the phosphorylation of hepatic enzymes by elevating cAMP, which is a common mechanism of phosphorylation, but not the only.

Treatment of phosphatidylcholine with which enzyme would be most likely to liberate arachidonic acid? A. phospholipase A1 B. phospholipase A2 C. phospholipase C D. phospholipase D

B. phospholipase A2 Phospholipase A2 is breaks down cell membrane phospholipids into arachidonic acid.

In muscle, the most important kinase in transducing the insulin binding event to metabolic changes in the target cell is: A. protein kinase A. B. protein kinase B. C. protein kinase C. D. protein kinase D. E. protein kinase G.

B. protein kinase B.PKB (aka Akt) is important in the signaling pathway that ends with GLUT4 insertion into the membrane.

As coproporphyrinogen III moves from the liver cytosol to the liver mitochondria and becomes protoporphyrinogen IX, A. protoporphyrinogen IX becomes more soluble in water because of oxidation of coproporphyrinogen III. B. protoporphyrinogen IX becomes less soluble in water because of oxidation of coproporphyrinogen III. C. protoporphyrinogen IX becomes more soluble in water because of reduction of coproporphyrinogen III. D. protoporphyrinogen IX becomes more soluble in lipids because of oxidation of coproporphyrinogen III. E. protoporphyrinogen IX becomes less soluble in lipids because of reduction of coproporphyrinogen III.

B. protoporphyrinogen IX becomes less soluble in water because of oxidation of coproporphyrinogen III.

In ketoacidosis, there is increased b-oxidation in the liver. A metabolite formed in this process contributes to increased flux through gluconeogenesis by inhibiting: A. acetyl CoA carboxylase. B. pyruvate dehydrogenase. C. pyruvate carboxylase. D. PEPCK. E. pyruvate kinase.

B. pyruvate dehydrogenase. Pyruvate dehydrogenase is inhibited by NADH and Acetyl CoA.

Enzyme abnormalities that can lead to hyperuricemia and gout include: A. reduced activity of PRPP synthetase. B. reduced activity of xanthine oxidase. C. increased activity of hypoxanthine-guanine phosphoribosyltransferase. D. reduced activity of PRPP amidotransferase. E. reduced activity of IMP dehydrogenase.

B. reduced activity of xanthine oxidase. The correct answer is B, reduced activity of xanthine oxidase. This is why allopurinol is only useful prophylactically and can make an active attack of gout worse.

The formation of deoxyribonucleotides involves which of the following processes? A. reduction of ribose in monophosphates B. reduction of ribose in diphosphates C. reduction of ribose in triphosphates D. reduction of ribose in 5phosphoribosyl pyrophosphate (PRPP) and subsequent reaction with a purine or pyrimidine E. oxidation of ribose in diphosphates

B. reduction of ribose in diphosphates All ribonucleotides must be diphosphate nucleotides to be converted to deoxyribonucleotides

The distribution of inosine monophosphate (IMP) conversion to either guanosine monophosphate (GMP) or adenosine monophosphate (AMP) is regulated by: A. feedback inhibition of phosphoribosylamine formation by GMP and/or AMP. B. the requirement of GTP for AMP formation, and ATP for GMP formation. C. availability of phosphoribosyl pyrophosphate (PRPP). D. feedback inhibition of GMP on AMP formation and AMP on GMP formation.

B. the requirement of GTP for AMP formation, and ATP for GMP formation.

Transfer of the methyl group from 5methyl tetrahydrofolate to homocysteine to form methionine requires: A. NADPH. B. vitamin B12. C. FAD. D. pyridoxal phosphate. E. none of the above

B. vitamin B12. Recall, Vitamin B12 (also called cobalamin) is involved in two reactions in the body: 1, the transfer of a methyl group from N5-methyl-FH4 to homocysteine to form methionine (above), and 2, the rearrangement of L-methylmalonyl-CoA to form succinyl CoA,

Allopurinol is used in the treatment of gout by reducing the production of uric acid. In Lesch-Nyhan syndrome the decrease in uric acid is balanced by an increase in xanthine and hypoxanthine in the blood. However, in other forms of gout the decrease in uric acid is greater than the increase in xanthine and hypoxanthine in the blood. The explanation for this difference in the two forms of gout is: A. PRPP levels are reduced in Lesch-Nyhan. B. xanthine is salvaged to IMP which inhibits PRPP amidotransferase. C. in Lesch-Nyhan xanthine oxidase activity is reduced. D. in non-Lesch-Nyhan gout PRPP levels are reduced.

B. xanthine is salvaged to IMP which inhibits PRPP amidotransferase. A is incorrect because PRPP levels are increased in Lesch Nyhan due to decreased activity of HGPRTase. C is incorrect because in LeschNyhan, HGPRT activity is reduced. D is incorrect because in other forms of gout, PRPP levels are also increased.

B

Bacteriostatic action of sulfa drugs is due to the inhibition of: A. Oxidative phosphorylation in mitochondria B. Folate synthesis C. Protein synthesis D. Transcription of DNA E. NAD synthesis

D.

Before the advent of AIDS, this disease was considered the main cause of permanent immunodeficiency. A. β-Aminoisobutyric aciduria B. Lesch-Nyhan syndrome C. Jaundice D. Adenosine deaminase deficiency E. Ornithine transcarbamoylase deficiency

Which of the following compounds can inhibit the actions of pancreatic lipase?

Bile Salts

Enzymes and medical Dx

Bone disease- alkaline phosphatase Obstructive liver disease- sorbitol DH, LDH Prostatic cancer- acid phosphatase Acute pancreatitis- amylase Muscular dystrophy- aldolase and AST Liver disorder- ALT, CK-MM(creatine kinase isoform)

In the metabolic pathway of heme biosynthesis, a deficiency in the enzyme ferrochelatase in a patient will result in which of the following? (Select 2 answers) A. An increase in urogen III cosynthase greater than normal in the patient B. A decrease in d-aminolevulinic acid greater than normal in the patient C. An increase in d-aminolevulinic acid synthase greater than normal in the patient D. An increase in d-aminolevulinic acid greater than normal in the patient E. Increased excretion of coproporphyrin I in the patient

Both C and D are correct. The lack of ferrochelatase will cause the heme synthesis pathway to back up, resulting in a buildup of d-aminolevulinic acid. Additionally, the lack of negative feedback from heme on ALA-synthase will result in greater than normal levels in this patient. Also, the excess precursors to heme (protoporphyrin) are not water-soluble and are not excreted, which leads to erythropoietic protoporphyrin's unique manifestations of photosensitivity. See this article for more information: http://www.ncbi.nlm.nih.gov/pmc/articles/PMC3096108/

The answer is c. (Murray, pp 173-179. Scriver, pp 2297-2356.) The almost universal carrier of acyl groups is coenzyme A (CoA). However, acyl carrier protein (ACP) also functions as a carrier of acyl groups. In fatty acid synthesis, ACP carries the acyl intermediates. The reactive prosthetic group of both ACP and CoA is a phosphopantetheine sulfhydryl. In ACP, the phosphopantetheine group is attached to the 77-residue polypeptide chain via a serine hydroxyl. In CoA, the phosphopantetheine is linked to the 5′-phosphate of adenosine that is phosphorylated in its 3′-hydroxyl.

Both acyl carrier protein (ACP) of fatty acid synthetase and coenzyme A (CoA): a. Contain reactive phosphorylated tyrosine groups b. Contain thymidine c. Contain phosphopantetheine-reactive groups d. Contain cystine-reactive groups e. Carry folate groups

Glycolysis

Breakdown glucose, forms two pyruvate molecules, 2 ATP net gain(anaerobic) Pyruvate is further oxidized to form 36 molecules of ATP(aerobic) RBCs use glycolysis as only energy means(lack mito) Pasteur effect causes glycolysis to be inhibited in aerobic conditions(using citrate and ATP) Cancers exhibit high rate of of glycolysis due to O2(Warburg effect) PET scans use this phenomenon to tag glucose which is more readily taken up by cancer cells

The answer is A. By 24 hours after a meal, hepatic (liver) gluconeogenesis is the major source of blood glucose because hepatic glycogen stores have been nearly depleted. Muscle and other tissues lack an enzyme necessary to convert glycogen or amino acids to glucose (thus, B is incorrect). The liver is the only significant source of blood glucose. Glucose is synthesized in the liver from amino acids (provided by protein degradation), from gly cerol (provided by hydrolysis of triacylglycerols in adipose tissue), and from lactate (provided by anaerobic glycolysis in red blood cells and other tissues). Glucose cannot be synthesized from fatty acids or ketone bodies (thus, D and E are incorrect).

By 24 hours after a meal, which one of the following is most likely to occur? A. Gluconeogenesis in the liver will be the major source of blood glucose. B. Muscle glycogenolysis provides glucose to the blood. C. Muscles convert amino acids to blood glucose. D. Fatty acids released from adipose tissue provide carbon for synthesis of glucose. E. Ketone bodies provide carbon for gluconeogenesis.

HFCS and obesity

Byspasses rate limiting PFK step in glycolysis thus is more readily turned into fat Fructose consumption may play a role in obesity and insulin resistance but also could purely be related to the overall calorie intake increase not just HFCS

Which of the following is not a necessity for dividing cells? A. Rapid ATP generation to maintain energy status B. Increased biosynthesis of macromolecules C. Increased rate of protein degradation D. Maintenance of appropriate cellular redox status

C.

A 32-year-old alcoholic has fever, jaundice, and tender hepatomegaly. A liver biopsy exhibits fatty change, Mallory's bodies, and a neutrophilic infiltrate. UBG urine urobilinogen, % CB is conjugated bilirubin. What are the most likely values you would see in this patient? A. % CB <20, Urine Bilirubin negative, Urine UBG ↑↑, AST ↑, ALT normal, ALP normal, GGT normal B. % CB >50, Urine Bilirubin ↑↑, Urine UBG negative, AST ↑, ALT ↑, ALP ↑↑↑, GGT ↑↑↑ C. % CB 20-50, Urine Bilirubin ↑, Urine UBG ↑↑, AST ↑↑, ALT ↑, ALP ↑↑, GGT ↑↑↑ D. % CB 20-50, Urine Bilirubin ↑, Urine UBG ↑↑, AST ↑↑, ALT ↑↑↑, ALP ↑, GGT ↑ E. % CB <20, Urine Bilirubin negative, Urine UBG normal, AST normal, ALT normal, ALP normal, GGT normal

C. % CB 20-50, Urine Bilirubin ↑, Urine UBG ↑↑, AST ↑↑, ALT ↑, ALP ↑↑, GGT ↑↑↑ The patient has alcoholic hepatitis, which produces a mixed jaundice (CB 20-50%). Urine UBG is increased, because UBG that is normally recycled back to the liver is redirected into the urine. Serum transaminases are increased with serum AST (mitochondrial enzyme) greater than ALT (cytosol enzyme). Because AST is normally located in the mitochondria of hepatocytes, alcohol, a mitochondrial toxin, causes preferential release of AST into the blood. Serum ALP is slightly increased; however, serum GGT is markedly increased, because alcohol induces enzyme synthesis in the hepatocyte cytochrome P-450 system in the smooth endoplasmic reticulum. GGT is also located in the SER and is increased as well.

An infant has a history of recurrent bacterial and fungal infections. Serum protein analysis showed no gamma globulin and further tests showed an increase in the white cell concentration of dATP. Which of the following is the most likely diagnosis? A. Orotic aciduria B. AMP deaminase deficiency C. Adenosine deaminase deficiency D. Lesch-Nyhan syndrome

C. Adenosine deaminase deficiency Adenosine deaminase deficiency leads to severe combined immunodeficiency syndrome (SCIDS) characterized by the inability of T and B cells to proliferate during infection and death due to dATP accumulation.

The deoxyribose of dCDP is made most directly from: A. ribose 5-phosphate. B. ribulose 5-phosphate. C. CDP. D. PRPP. E. none of the above.

C. CDP.

A young woman undergoes investigation for Cushing's syndrome having presented with acne and oligomenorrhea. Early morning plasma cortisol concentration is 824 nmol/L [140-690 nmol/L]; plasma adrenocorticotropic hormone (ACTH) concentration is at the upper limit of the normal range. Early morning plasma cortisol concentration after taking dexamethasone 1 mg at midnight is 766 nmol/L. After taking dexamethasone 2.0 mg 6-hourly for 48 hours, plasma cortisol concentration is 40 nmol/L. What is the most likely diagnosis? A. Adrenal adenoma B. Adrenal carcinoma C. Cushing's disease (pituitary tumor secreting ACTH) D. Depression E. Ectopic secretion of ACTH by small cell carcinoma of bronchus

C. Cushing's disease (pituitary tumor secreting ACTH) Suppression of ACTH (and thus cortisol) secretion after a high dose of dexamethasone (2 mg 6-hourly for 48 hours) is characteristic of Cushing's disease, though can sometimes occur with ectopic secretion of ACTH, particularly by carcinoid tumors. The high-end normal early morning ACTH is also typical of Cushing's disease. ACTH secretion would be expected to be suppressed by the excess cortisol secreted by an adrenal adenoma; because they are not ACTH-dependent, cortisol secretion by adenomas is not usually suppressed by dexamethasone. Adrenal carcinoma is a rare cause of Cushing's syndrome, although, in women, virilisation can be a prominent clinical feature. However, ACTH secretion would be expected to be suppressed by the excess cortisol secreted by an adrenal carcinoma, and cortisol secretion is not suppressed by dexamethasone. Depression can cause an increase in cortisol secretion that is not suppressed by a single 1 mg dose of dexamethasone; suppression is, however, usual (though not invariable) following a 2-day high dose (2.0 mg) dexamethasone suppression test. ACTH (and thus cortisol) secretion is typically resistant to suppression by dexamethasone in ectopic ACTH secretion by carcinomas. This is, anyway, a very rare cause of Cushing's in younger people and less likely to cause virilisation of females.

What hormone is excreted from the chromaffin cells of the adrenal medulla? A. ACTH B. Cortisol C. Epinephrine D. Oxytocin E. Thyroid hormone

C. Epinephrine Chromaffin cells produce catecholamines.

Thrombin regulates two events that interrupt the normal clotting process. These are its interactions with which of the following? A. Blood platelets and lymphocytes B. Factors V and VII C. Factors Va and VIIIa D. Fibrinogen and tissue factor E. Factors IX and X

C. Factors Va and VIIIa Thrombin binds the thrombomodulin receptor and activates protein C. Protein C and S then destroy factors Va and VIIIa.

Regarding the endocrine system and hormone feedback, which of the following statements is FALSE? A. The feedback of thyroid hormone is an example of a short-loop feedback system. B. The feedback of insulin is an example of a long-loop feedback system. C. For the most part, the hormones of the endocrine system utilize positive feedback. D. Tight control of hormone levels and their actions is important in normal human homeostasis. E. Many hormones exert their feedback either directly on the gland that produces them, on a gland that controls their synthesis/secretion or some combination of these.

C. For the most part, the hormones of the endocrine system utilize positive feedback. The large majority of hormones utilize negative feedback.

The second step of prostaglandin synthase converts PGG2 to PGH2. What is the cofactor for this reaction? A. Biopterin B. FADH2 C. Glutathione D. NADH E. Pyridoxal phosphate

C. Glutathione

A febrile 54-year old alcoholic comes to the emergency room complaining of an inability to walk because his foot hurts. The night before he consumed a rather sumptuous meal and drank a large amount of alcohol. Physical examination is notable for a tender erythematous area with heat over the metatarsophalangeal joint. His breath has a fruity sweet odor, and he displays deep shallow breathing. He has been taking medication for swelling of the joints. His clinical values reveal: Which of the following enzymes involved in nucleotide metabolism is most likely the target of his current medication? A. Hypoxanthine-guanine phosphoribosyltransferase (HGPRTase) B. Orotate phosphoribosyltransferase (OPRTase) C. Xanthine oxidase D. Thymidine kinase E. Phosphoribosylpyrophosphate synthase (PRPP synthase)

C. Gout or hyperuricemia is treated with allopurinol.

42-year-old African American man is seen in the emergency room for a severe headache. His blood pressure is 180/110 mm Hg, and he has evidence of retinal hemorrhage. An infusion of nitroprusside (which metabolized to produce nitric oxide) is given. Which of the following enzymes is affected most directly by the active metabolite of this drug? A. Phospholipase A2 B. Cyclic AMP phosphodiesterase C. Guanylate cyclase D. Cyclic GMP phosphodiesterase E. Phospholipase C

C. Guanylate cyclase The correct answer is C. Nitroprusside is metabolized to produce nitric oxide. NO, normally produced by the vascular endothelium, stimulates the Guanylate cyclase in vascular smooth muscle to increase cGMP, activate protein kinase G, and cause vessel dilation.

A 45-year-old woman with rheumatoid arthritis has an increased serum total protein. The hyperproteinemia is most likely due to an increase in which of the following protein fractions? A. Albumin B. Immunoglobulin A C. Immunoglobulin G D. Immunoglobulin M

C. Immunoglobulin G An increased serum total protein is most often due to an increase in immunoglobulins (Igs). Igs are primarily increased in chronic inflammation and malignant plasma cell disorders (e.g., multiple myeloma). Because the patient has rheumatoid arthritis, a chronic inflammatory disease, the immunoglobulins are increased. The primary Ig to increase in chronic inflammation is IgG. A serum protein electrophoresis would show a polyclonal gammopathy (diffuse increase in the γ-globulin curve).

A 9-month-old infant is evaluated because of a 2-day history of incessant coughing with some vomiting after coughing bouts. Medical history shows no immunizations. On physical examination, the infant is in acute distress with a continual cough and an inspiratory whoop after each coughing episode. Radiography shows no evidence of epiglottitis. Which of the following best describes the biochemical basis for the clinical findings? A. Activation of a Gq protein B. Activation of a Gs protein C. Inactivation of a Gi protein D. Inhibition of tyrosine phosphorylation E. Stimulation of guanylyl cyclase activity

C. Inactivation of a Gi protein This infant has classic symptoms of infection with Bordetella pertussis, the etiologic agent in whooping cough. One of the toxins secreted by B. pertussis covalently modifies the Gi protein in respiratory phagocytes, leading to impaired function of the Gi protein and increased activity of cyclic adenosine monophosphate (cAMP). This action leads to ineffective phagocytosis and chemotaxis, and it inhibits lysosomal degradation of the bacteria. Pertussis is caused by inactivation of a Gi protein that produces a reduction in cAMP. By comparison, Gq proteins activate phospholipase C, producing the second messengers, diacylglycerol and inositol triphosphate. Pertussis is caused by inactivation of a Gi protein leading to a reduction of cAMP. Gs proteins, when activated, increase the concentration of cAMP. Pertussis involves the cAMP signaling pathway, while tyrosine phosphorylation, not affected in a whooping cough infection, is found in the insulin signaling pathway. Guanylyl cyclase, which produces cGMP from GTP, is not affected by the pertussis toxin.

Which of the following ideas has the best potential for new cancer therapies today? A. A single anti-cancer drug that will kill all tumor cells in the body B. Gene therapy for germ line cancers C. Inactivation or interference with an oncogene or oncogenes that are found to be responsible for maintenance of an individual's tumor D. Radiation

C. Inactivation or interference with an oncogene or oncogenes that are found to be responsible for maintenance of an individual's tumor

A 55-year-old man with a history of chronic alcohol use and chronic bronchitis secondary to cigarette smoking, is taking theophylline to improve his breathing. His wife states that he is taking the medicine as prescribed. A therapeutic drug level of serum theophylline is reported to be in a suboptimal range. Which of the following best explains this test result? A. Decreased gastrointestinal reabsorption of the drug B. Decreased renal excretion of the drug C. Increased liver metabolism of the drug D. Patient noncompliance

C. Increased liver metabolism of the drug Alcohol induces the liver microsomal mixed function oxidase system, which is located in the smooth endoplasmic reticulum (SER). This enzyme system is responsible for drug and chemical metabolism. Drugs like alcohol and chemicals like polycyclic aromatic hydrocarbons in cigarette smoke "induce" this system by enhancing the rate of synthesis of NADPH-cytochrome P450 reductase and cytochrome P450 oxidase, which are the two key enzymes in the system. Induction of these enzymes increases the metabolism of drugs - in this case theophylline - leading to suboptimal concentration of the drug.

A 55-year-old farmer has a raised crateriform lesion on the inner aspect of the bridge of his nose and numerous gray-white hyperkeratotic lesions on the face, hands, back of the neck, and forearms. Which of the following best characterizes the pathogenesis of these skin lesions? A. Cleavage of DNA strands B. Deficiency of DNA repair enzymes C. Increased production of pyrimidine dimers D. Point mutations with production of stop codons

C. Increased production of pyrimidine dimers Ultraviolet B (UVB) light (nonionizing radiation) is responsible for sunburns and the rapid increase of skin cancers in the United States. In this patient, the crateriform lesion on the nose is a basal cell carcinoma, the most common cancer caused by excessive UVB light exposure. The gray-white hyperkeratotic lesions in sun-exposed areas are actinic (solar) keratoses, which are precursors for squamous cell carcinoma of the skin. UVB light excites adjacent pyrimidine bases on DNA strands, causing them to form thymine-thymine dimers.

Why might inhibition of CYP19A1 be therapeutic for breast tumors? A. Increase in testosterone synthesis B. Inhibition of estrogen metabolism C. Inhibition of synthesis of estrogens D. Production of estrogen antagonists E. Reduction of binding to p53

C. Inhibition of synthesis of estrogens Aromatase, also called estrogen synthetase or estrogen synthase, synthesizes estrogens, and it is CYP19A1. Many breast cancers are formed by overproduction of estrogen; therefore, inhibition of CYP19A1/aromatase can be therapeutic in the treatment of breast cancer.

The toxic form of benzo[a]pyrene binds covalently to DNA. What mechanism does benzo[a]pyrene use to potentiate its toxicity? A. It binds covalently to cytochrome P450. B. It increases the amount of AhR. C. It induces cytochrome P450 and its own metabolism. D. It inhibits glucuronidation. E. It inhibits its own further metabolism.

C. It induces cytochrome P450 and its own metabolism.

What is cell-cell signaling that requires the cells to be in contact called? A. Autocrine B. Endocrine C. Juxtacrine D. Paracrine E. Synaptic

C. Juxtacrine Juxtacrine requires cell-to-cell contact. Endocrine describes cell signaling over long distances; paracrine signaling over short distances; autocrine is self-stimulation; and synaptic specifically describes neuronal communication via neurotransmitters.

Isoforms of Z-bilirubin may be generated through photosensitizing the compound. Such isoforms are critical in the prevention of which pathology in bilirubin metabolism? A. Gilbert's syndrome B. Crigler-Najjar syndrome, type II C. Kernicterus D. Hepatitis A E. Dubin-Johnson disease

C. Kernicterus Kernicterus happens when too much bilirubin is produced and is absorbed in the brain tissue. All the other listed choices can produce too much bilirubin, but the deposition of bilirubin can be prevented if the bilirubin stays soluble and can be excreted. Photosensitizing the bilirubin can keep it soluble.

A 10-year-old girl is brought to the pediatrician because of shortness of breath and itchy eyes whenever she plays with her cat and dog. Physical examination reveals expiratory wheezing at both lung bases. Which of the following chemical mediators is most likely responsible for these respiratory findings? A. Bradykinin B. Complement C3a and C5a C. Leukotrienes C4, D4, and E4 D. Prostaglandin I2 E. Thromboxane A2

C. Leukotrienes C4, D4, and E4 Hydroxylation of arachidonic acid by lipoxygenase yields the leukotrienes C4, D4, and E4, which are known as the slow-reacting substances of anaphylaxis. These leukotrienes are vasoconstrictors and bronchoconstrictors. This child has extrinsic asthma, which is a type I hypersensitivity reaction induced by exposure to an extrinsic allergen, in this case, dog and cat dander. LTC4-D4-E4 contribute to luminal narrowing in the segmental bronchi and terminal bronchioles by contracting smooth muscle cells and increasing vessel permeability, which causes expiratory wheezing and trapping of air behind the obstructed bronchioles.

Lipid peroxidation of the cellular membranes is initiated by reactive oxygen and propagated as lipid peroxides during reperfusion injury. What blood constituent would be indicative of cellular damage to lipids? A. Ascorbic acid B. Uric acid C. Malondialdehyde D. Glutathione

C. Malondialdehyde The lipid hydroperoxide is susceptible to degradation into malondialdehyde and 4- hydroxynonenal (HNE).

What enzyme(s) is/are mainly responsible for catabolism of serotonin? A. L-aromatic amino acid decarboxylase B. Catechol-O-methyl transferase (COMT) C. Monoamine oxidase D. Both A and B E. Both B and C

C. Monoamine oxidase

Which one of the following statements relating to the thymidylate (dTMP) synthase reaction is FALSE? A. dUMP is a substrate for this enzymatic reaction. B. A 1-carbon THF is a substrate for this enzymatic reaction. C. NADPH is a substrate for this enzymatic reaction. D. F-dUMP directly inhibits this enzymatic reaction. E. Dihydrofolate (DHF) is produced by this enzymatic reaction.

C. NADPH is a substrate for this enzymatic reaction.

Upon reperfusion after opening a coronary artery, neutrophils and macrophages infiltrate the site of tissue damage. What enzyme is involved in the respiratory burst of phagocytes? A. Glutathione peroxidase B. Cytochrome P450 C. NADPH oxidase D. Xanthine oxidase

C. NADPH oxidase NADPH oxidase generates superoxides and is involved in the respiratory burst. Glutathione peroxidase protects the body from oxidative damage. Xanthine oxidase produces superoxides, but it is more important in the uric acid formation pathway.

What kind of enzyme is a monooxygenase? A. One that catalyzes formation of an epoxide B. One that incorporates O2 into a substrate C. One that inserts one atom of oxygen into a substrate D. One that produces superoxide E. One that removes hydrogens from a substrate

C. One that inserts one atom of oxygen into a substrate

Concerning the anterior pituitary hormones, which one of the following statements is FALSE? A. The anterior pituitary makes six hormones: thyrotropin, follitropin, lutropin, somatotropin, prolactin and adrenocorticotropin. B. Production of the anterior pituitary hormones are all stimulated by releasing factors made by the hypothalamus and which are carried from the hypothalamus to the anterior pituitary via the blood. C. Overproduction of lutropin/follitropin is seen in Kallman's Syndrome which is characterized by failure to sexually mature and grow. D. Thyrotropin stimulates the thyroid gland and is made up of an alpha subunit and a beta subunit, but the beta subunit is the only one of the two subunits that is specific for thyrotropin. E. Somatotropin's main function of growth promotion is indirect via raising the levels of somatomedin c (insulin-like Growth Factor I).

C. Overproduction of lutropin/follitropin is seen in Kallman's Syndrome which is characterized by failure to sexually mature and grow.

Photosensitivity is a symptom of which porphyria(s)? A. PCT and CEP B. CEP C. PCT, CEP, EPP D. VP and EPP E. AIP

C. PCT, CEP, EPP

Certain prostaglandins after binding to their receptors induce an increase in intracellular calcium levels. The signal that leads to the elevation of intracellular calcium is initiated by which of the following enzymes? A. Protein Kinase A B. Phospholipase A2 C. Phospholipase C D. Protein Kinase B E. Cyclooxygenase

C. Phospholipase C Phospholipase C is activated by Gq protein coupled to EP prostaglandin receptor. Activation of PLC leads to hydrolysis of phosphatidylinositol bisphosphate (P1P2) to produce diacylglycerol and inositol triphosphate (IP3). IP3 leads to an increase in calcium levels.

All of the following regarding eicosanoids are true EXCEPT: A. Omega-6 fatty acids are essential for the production of eicosanoids. B. Prostaglandins and thromboxanes are derived from arachidonic by cyclooxygenase. C. Prostaglandins and prostacyclin are characterized by long half-lives. D. Leukotrienes are derived from arachidonic acid by lipoxygenase.

C. Prostaglandins and prostacyclin are characterized by long half-lives. Prostaglandins are potent but have a short half-life. They are quickly inactivated. Due to this short half-life, they send only paracrine (locally active) or autocrine (acting on the same cell) signals.

Under hypoglycemic conditions, WHICH ONE of the following enzymes in the liver has LITTLE activity? A. Fructose 2,6-bisphosphatase B. Fructose 1,6-bisphosphatase C. Pyruvate kinase D. Pyruvate carboxylase E. PEPCK

C. Pyruvate kinase In hypoglycemic conditions, glucagon would be elevated. Pyruvate kinase is repressed by glucagon.

Which of the following best describes the mechanism by which steroid hormones increase the relative abundance of specific proteins? A. Steroids increase the half-life of specific proteins. B. Steroids increase the half-life of the mRNAs that encode specific proteins. C. Steroids increase the rate of synthesis of specific mRNAs. D. Steroids increase the translatability of specific mRNAs. E. Steroids increase RNA polymerase II activity.

C. Steroids increase the rate of synthesis of specific mRNAs. Most steroid hormones bind to their receptors upregulating the transcription of certain genes.

All the following are true about hemosiderin except? A. Hemosiderin is from iron that exceeds ferritin's storage capacity. B. Hemosiderin is composed of ferritin, denatured ferritin, and other substances. C. The iron within hemosiderin is readily available. D. Macrophages hold hemosiderin. E. Hemosiderin is often associated with hemorrhage.

C. The iron within hemosiderin is readily available.

Which of the following characteristics best describes prostaglandins? A. They are derived from fatty acids with 22 carbons. B. They do not contain keto or hydroxyl groups. C. They are synthesized from polyunsaturated fatty acids. D. They are linear compounds with no ring structures. E. They inhibit cell response through EP receptors.

C. They are synthesized from polyunsaturated fatty acids. Prostaglandins are synthesized from 20 carbon polyunsaturated fatty acids with three, four, and five double bonds. A cyclooxygenase converts the fatty acid to a compound that contains a 5-membered ring. In subsequent reactions, a series of prostaglandins is produced that contains various keto and hydroxyl groups.

Which enzyme functions in both the salvage of pyrimidine bases and in the de novo synthesis of pyrimidines? A. Carbamoyl phosphate synthetase II B. Dihydroorotase C. Orotate phosphoribosyltransferase D. Phosphoribosylamidotransferase E. Dihydrofolate reductase

C. This is the only enzyme that salvages pyrimidine bases and is involved in de novo synthesis of pyrimidines.

Of the following characteristics found for tumor cells, which one is most commonly considered of greatest clinical significance? A. Limitless replicative potential B. Evasion of apoptosis C. Tissue invasion and metastasis D. Insensitivity to anti-growth signals E. Angiogenesis

C. Tissue invasion and metastasis This is what makes cancers the most deadly. While all other answers are important for cancer, their ability to invade and metastasize makes them deadly.

Conversion of fibrinogen to fibrin by thrombin in the coagulation cascade results in the production of: A. Factor X. B. Factor XIII. C. a soluble gel-like clot. D. excess Ca++. E. prothrombin.

C. a soluble gel-like clot. Fibrin forms a gel-like clot, factor XIII hardens the clot.

The characteristic odor of the breath of a patient with diabetic ketoacidosis is due to: A. carbon dioxide. B. b-hydroxybutyric acid. C. acetone. D. acetoacetate. E. nitric oxide.

C. acetone. The breakdown of beta-hydroxybutyrate and acetoacetate (ketone bodies) into acetone is what cases the fruity odor.

Therefore, allopurinol is a more effective treatment to reduce hyperuricemia in the non-Lesch-Nyhan patient because: A. allopurinol inhibits xanthine oxidase. B. allopurinol inhibits PRPP synthetase. C. allopurinol is converted to allopurinol ribose. D. allopurinol is converted to alloxanthine.

C. allopurinol is converted to allopurinol ribose. Since a non-Lesch-Nyhan patient still has use of HGPRT, allopurinol is converted by HGPRT to allopurinol ribose which can inhibit PRPP aminotransferase and halt purine de novo synthesis.

Hydroperoxyeicosatetraenoic acids (HPETEs): A. are derived from arachidonic acid by a peroxidase reaction. B. are mediators of hypersensitivity reactions. C. are intermediates in formation of leukotrienes. D. are relatively stable compounds persisting for as long as 4 hours. E. are inactivated by cytochrome P450s of two families.

C. are intermediates in formation of leukotrienes. These are formed by addition of oxygen to arachidonic acid. The enzyme is lipoxygenase. Lipoxygenase catalyzes the monodioxygenation of unsaturated fatty acids to allylic endoperoxides. HPETEs themselves are not hormones but highly unstable intermediates that are converted to either HETEs, mediators of hypersensitivity or leukotrienes. Members of the CYP2 family produce HPETEs.

Elevated homocysteinemia affects vascular structure by altering the structure of: A. hemoglobin. B. albumin. C. collagen. D. blood clotting zymogens. E. chymotrypsin.

C. collagen. High homocysteine will alter the structure or collagen, which will affect vascular structure.

Prostaglandin synthase is a bifunctional enzyme that: A. is feedback inhibited by prostaglandins. B. is inhibited by anti-inflammatory steroids. C. contains both cyclooxygenase and peroxidase component. D. produces PGG2 as the end product. E. uses as a substrate the pool of free arachidonic acid in the cell.

C. contains both cyclooxygenase and peroxidase component. Cyclooxygenase oxidizes arachidonic acid and peroxidase converts PGG2 to PGH2. The release of precursor fatty acids by phospholipase A2 is the rate limiting step and the one inhibited by anti-inflammatory steroids. The peroxidase component converts the PGG2 to PGH2. Arachidonic acid is not free in the cell but is part of the membrane phospholipids or sometimes a cholesterol ester.

The liver microsomal monooxygenase system containing cytochrome P450 can modify drugs by: A. formation of a sulfate group. B. addition of an acyl residue. C. formation of a hydroxyl group. D. addition of a glucuronic acid residue. E. formation of a disulfide bond.

C. formation of a hydroxyl group. Formation of hydroxyl group is the primary mechanism to P450, which makes them more water soluble, and easily excreted. Recall that CYP21A2, which is the most common deficiency in congenital adrenal hyperplasia, is also called 21-hydroxylase.

The elimination of non-polar xenobiotics from the body requires that the xenobiotic be oxidized by phase I enzymes and then be acted upon by phase II enzymes. All of the following enzymes function as phase II enzymes except: A. UDP glucuronyltransferase. B. glutathione S-transferase. C. gamma glutamyl transferase. D. acetyltransferase.

C. gamma glutamyl transferase. Gamma glutamyl transferase is a phase III enzyme. Phase II are primarily transferase reactions. Phase I enzymes include oxidation, reduction, hydrolysis, cyclization, decyclization, and addition of oxygen or removal of hydrogen.

A 54-year-old man is diagnosed with prostate cancer and elects to have surgery and radiation therapy. Radiation works in part by: A. inhibiting NADH dehydrogenase. B. inhibiting the cytochrome b-c1 complex. C. generating reactive oxygen species. D. stimulating production of glutathione. E. producing single strand breaks in proteins.

C. generating reactive oxygen species. The correct answer is C, produces ROS. NADH is inhibited by rotenone, and cytochrome b-c complex is inhibited by antimycin. Glutathione protects against ROS.

After removing the coronary blockage, the administration of the following agents may have a beneficial effect in preventing reperfusion injury, EXCEPT: A. tocopherol B. ascorbic acid C. glutathione D. arginine

C. glutathione Tocopherol or vitamin E would possibly prevent lipid peroxidation and ascorbic acid would reduce oxidized Vit E, thereby serving as a two electron donor antioxidant. However, giving one vitamin without the other would be of minimal benefit. Glutathione would be the best choice since it would provide a substrate for glutathione peroxidase/reductase system thereby reducing reactive oxygen species. An additional benefit would be the reduction of cysteine in proteins to prevent mis-folding and formation of cysteine.

After a two week alcohol binge, an alcoholic takes four times the prescribed daily dose of acetaminophen for a severe headache. Within 24 hours he becomes lethargic, vomits frequently, and has severe abdominal pain. The symptoms experienced are due to an adverse reaction with acetaminophen most likely caused by: A. hypoglycemia experience by the alcohol. B. inhibition of acetaminophen metabolism. C. increased acetaminophen metabolism by CYP2E1. D. inhibition of CYP3A4 by alcohol.

C. increased acetaminophen metabolism by CYP2E1. CYP2E1 is induced by alcohol, and it metabolizes acetaminophen into the toxic intermediate, NAPQI.

Human growth hormone (HGH), in vivo, in specific tissues: A. causes an increase in the size of cells. B. causes a decrease in the size of cells. C. increases the rate of cell proliferation. D. decreases the rate of cell proliferation. E. can be replaced therapeutically by other mammalian growth hormones.

C. increases the rate of cell proliferation.

The catecholamines, epinephrine and norepinephrine, adjust metabolic activity throughout the body to satisfy the energy demands of the working muscles. All of the following catecholamine effects are important during physical activity EXCEPT: A. stimulation of glycogenolysis in liver. B. stimulation of glycogenolysis in skeletal muscle. C. inhibition of glycolysis in skeletal muscle. D. inhibition of glycolysis in liver. E. stimulation of lipolysis in adipose tissue.

C. inhibition of glycolysis in skeletal muscle. The correct answer is C, inhibition of glycolysis in skeletal muscle. A, B, & D increase the body's available glucose. E increases the availability of ketone bodies for energy.

Methotrexate is normally given with 5-fluorouracil in chemotherapy, and is also given to patients with severe inflammation as seen in gout and arthritis. What effect does methotrexate have on nucleotide metabolism? A. inhibits purine catabolism B. inhibits pyrimidine catabolism C. inhibits purine de novo synthesis D. inhibits purine salvage E. inhibits pyrimidine de novo synthesis

C. inhibits purine de novo synthesis

Down-regulation of insulin receptors is due to: A. irreversible binding of insulin to its receptor. B. negative cooperativity of insulin binding. C. insulin-stimulated pinocytosis of the insulin-receptor complex. D. insulin-stimulated proteolysis of its receptor. E. insulin-stimulated release of inhibitory prostaglandins.

C. insulin-stimulated pinocytosis of the insulin-receptor complex.

A pharmacological dose of a non-steroidal anti-inflammatory agent would not inhibit the synthesis of: A. thromboxanes. B. prostacyclins. C. leukotrienes. D. E-series prostaglandins. E. F-series prostaglandins.

C. leukotrienes. NSAIDs block COX-1 and COX-2, blocking prostaglandin production, but have no effect on 5-lipoxygenase, which produces leukotrienes.

All of the following are compounds that are products of tyrosine metabolism EXCEPT which one? A. melanin B. norepinephrine C. melatonin D. thyroxine E. acetoacetic acid

C. melatonin

What enzyme functions to convert N5,N10methyleneFH4 to N5 methylFH4, which serves as a methyl donor of Vitamin B12 cofactor in conversion of homocysteine to methionine by methionine synthase? A. serine hydroxymethyltransferase B. methylene synthase C. methylene FH4 reductase D. glycine cleavage enzyme E. dihydrofolate reductase

C. methylene FH4 reductase

The signaling mechanism whereby glucagon results in an increased breakdown of triacylglycerol in adipose tissue involves all of the following EXCEPT: A. protein kinase A. B. adenylate cyclase. C. phospholipase A2. D. a transmembrane receptor. E. a trimeric Gs protein.

C. phospholipase A2. Glucagon binds a transmember G-protein coupled receptor that leads to activation of adenylate cyclase and PKA.

Phospholipases A1 and A2 are best described as: A. having no role in phospholipid synthesis. B. responsible for the interconversion of phosphatidylethanolamine and phosphatidylserine. C. removing a fatty acid in an sn-1 and sn-2 position so it can be replaced by another phospholipid synthesis. D. hydrolyzing phosphatidic acid to an acylglycerol. E. responsible for the initial insertion of saturated fatty acids in the sn-1 and sn-2 position during synthesis.

C. removing a fatty acid in an sn-1 and sn-2 position so it can be replaced by another phospholipid synthesis. Phospholipases A1 and A2, as their names suggest, hydrolyze a fatty acid from a phospholipid and so are part of phospholipid degradation. They are also important in synthesis by ensuring the asymmetric distribution of fatty acids that occurs in phospholipids.

The index which indicates that an anemia is caused by impaired production or RBCs rather than excessive destruction is: A. total bilirubin. B. hematocrit. C. reticulocyte count. D. blood hemoglobin. E. RBC count.

C. reticulocyte count. The amount of immature RBC will allow you to determine if an adequate number of RBC are being produced. If the correct number of immature RBC are being produced, you know that RBC are being degraded prematurely.

In the conversion of cholesterol to steroid hormones in the adrenal gland: A. all cytochrome P450 oxidations occur in the endoplasmic reticulum. B. all cytochrome P450 oxidations occur in the mitochondria. C. side-chain cleavage of cholesterol to pregnenolone by P450 requires adrenodoxin reductase for electron transfer. D. P450 is necessary for formation of aldosterone and cortisol but not for formation of androgens and estrogen. E. aromatization of first ring of steroid does not use P450 because it involves removal of a methyl group and not hydroxylation.

C. side-chain cleavage of cholesterol to pregnenolone by P450 requires adrenodoxin reductase for electron transfer. A, B: Hormone synthesis involves a series of reactions in mitochondria and endoplasmic reticulum. D: P450 is necessary for the formation of all hormones of the adrenal cortex, including aldosterone, cortisol, and androgens. E: Removal of side chains frequently begins with oxidation.

Hypothalamic hormones which regulate the synthesis and secretion of anterior pituitary hormones include all of the following EXCEPT: A. TRH (thyrotropin releasing hormone). B. GnRH (gonadotropin releasing hormone). C. somatomedin c. D. somatostatin. E. CRH (corticotrophin releasing hormone).

C. somatomedin c. Somatomedin C (IGF-1) is produced by the liver in response to GH stimulation.

A peptide hormone which is located both in the hypothalamus and in the pancreas is: A. cholecystokinin. B. proinsulin. C. somatostatin. D. neurophysin. E. none of the above.

C. somatostatin.

Thyroid hormone is produced in the thyroid gland: A. in a process that involves the iodination of single tyrosine molecules prior to incorporation into thyroglobulin. B. after which T4 and T3 are stored as active hormones in the colloid of the thyroid follicles. C. stored in the follicle, and released into the blood stream after proteolysis of thyroglobulin in phagolysosomes of the thyroid cell. D. with a ratio of T3 to T4 of approximately 151. E. C and D.

C. stored in the follicle, and released into the blood stream after proteolysis of thyroglobulin in phagolysosomes of the thyroid cell. Iodination occurs to tyrosine residues of thyroglobulin. T4/T3 are stored as inactive compounds bound to thyroglobulin. There is more T4 than T3.

Which of the following is the rate-limiting enzyme in catecholamine biosynthesis? A. phenylalanine hydroxylase B. phenylethanolamine N-methyltransferase C. tyrosine hydroxylase D. DOPA decarboxylase E. catechol-o-methyl transferase

C. tyrosine hydroxylase

Patients with nephrotic syndrome have deficiencies of most plasma proteins, but one plasma protein fraction actually is increased. This fraction is: A. albumin. B. α1-Globulin. C. α2-Globulin. D. β-Globulin. E. γ-Globulin.

C. α2-Globulin. The presence or proteinuria, hypoproteinemia and edema constitute the nephrotic syndrome. Edema is in part due to redistribution of ECF between the vascular and interstitial compartments and secondary aldosteronism, with evidence of potassium depletion. Protein loss of albumin and other plasma proteins, such as hormone binding proteins, transferrin, and antithrombin II, are reduced with other large molecular weight protein increase in concentration, α2-macroglobulin, apoliproteins and coagulation factors (fibrinogen, Factor VIII, etc).

Defective Cl transport leads to CF

CF-autosomal recessive, mutation in CFTR protein(chloride channel mediating active Cl transport using ATP Defective CFTR causes buildup of Cl- and salt in epithelial cells, causing viscous mucus lining these cells (i.e. Lungs) which can harbor bacteria

Cyanide poisoning

CN binds to oxidized form of iron(Fe3+) in the heme of cytochrome-a3 of complex IV preventing O2 reduction Mitochondrial respiration and ATP formation cease leading to cell death Asphyxia, especially in CNS If caught early nitrites can convert Fe2+ into Fe3+ resulting in methemoglobin which competes with CN for binding to complex IV Subsequent administration of thiosulfate will neutralize CN Azide(N3-) also binds complex IV Fe3+

Cyanide versus carbon monoxide action

CO and NO compete with O2 for binding to reduced heme-a3 in complex IV where as CN, N3- and H2S bind to the oxidized form CO is competitive thus raising Km and CN is noncompetitive thus reducing Vmax, NO is more complicated

Galactosemia

Calssic-Galactose in the blood/urine, occurs when G1P conversion is prevented due to galactose 1-uridyltransferase deficiency Sx are failure to thrive, liver failure, sepsis and bleeding Duarte variant has 5-25% acitivity of GALT in RBCs Nonclassical occurs due to defect in galactokinase Galacticol accumulates in the eyes(cataract), urine, blood some is preventable by galactose restriction early

Alcohol calories

Can act as energy due to higher energy content than carbs and proteins

Aminophospholipid translocases

Cannot move from one leaf to the other without a special enzyme(flippase, floppase, scramblase)

Urea Cycle

Carbamoyl phosphate synthetase (rate-limiting)

De Novo Synthesis of Pyrimidine

Carbamoyl phosphate synthetase II Aspartate transcarbamoylase (rate-limiting step) Dihydroorotase Dihydroorotate dehydrogenase (rxn in mitochondria) UMP synthase UMP synthase Ribonucleotide reductase and Thymidylate synthase CTP synthase and Ribonucleotide reductase

Pyruvate Carboxylase

Carboxylates pyruvate into OAA. Needs Biotin.

Caridotonic drugs

Cardiac glycosides such as ouabain and digoxin act as cardiotonic(contraction inducing) drugs Inhibit the Na+/K+ ATPase on cardiac myocytes Leads to increase IC Na+ and increase in Ca2+ due to slowing of NCX Increased sarcoplasmic Ca2+ results in stronger excitation contraction of heart muscles with each AP

Oxidation levels of purines

Catabolism involves addition of O2 to make a molecule more polar Adenine lacks O2, guanine and hypoxanthine have one molecule of O2, xanthine has two, and uric acid has three Uric acid. Is the end point of catabolism and therefore is most oxidized Its insolubility plays a role in gout

Citrate Synthase

Catalyze the condensation of Acetyl CoA with oxaloacetate (OAA) to form citrate.

Citrate Lyase (ATP Citrate Lyase)

Catalyze the reaction to convert citrate back to Acetyl CoA and OAA.

Pyruvate carboxylase deficiency

Causes more pyruvate to be formed into lactic acid than OAA, lactic acid accumulates in the blood Sx shortly after birth include SZ, hypotonia, ataxia Auto recessive

Sugar alcohols and cataracts in diabetics

Cellular sugar alcohol elvels are reduced by sorbitol DH (sorbitol into fructose) Eye lens cells do not express this enzyme-susceptible to sorbitol which cannot cross the membrane so it builds up in the eye and causes swelling then rupture Preventable and presents as cataracts, retinopathy, and peripheral neuropathy

The answer is d. (Murray, pp 242-248. Scriver, pp 1909-1964.) The steps of the urea cycle are divided between the mitochondrial matrix and cytosol of liver cells in mammals. The formation of ammonia, its reaction with carbon dioxide to produce carbamoyl phosphate, and the conversion to citrulline occur in the matrix of mitochondria. Citrulline diffuses out of the mitochondria, and the next three steps of the cycle, which result in the formation of urea, all take place in the cytosol. Fatty acid oxidation and some reactions of organic acid degradation also occur in the mitochondria. Peroxisomes have single membranes, in contrast to the double membranes of mitochondria. They house catalase and enzymes for medium- to long-chain fatty acid oxidation.

Certain organic acidemias or fatty acid oxidation disorders will also involve elevated ammonia. This relates to the location of reactions of the urea cycle, occurring in which of the following? a. In the cytosol b. In lysosomes c. In the mitochondrial matrix d. In the mitochondrial matrix and the cytosol e. In peroxisomes

The answer is c. (Murray, pp 481-497. Scriver, pp 3897-3964.) The retinal pigment rhodopsin is composed of the 11-cis-retinal form of vitamin A coupled to opsin. Light isomerizes 11-cis-retinal to all-trans-retinal, which is hydrolyzed to free all-trans-retinal and opsin. In order for regeneration of rhodopsin to occur, 11-cis-retinal must be regenerated. This dark reaction involves the isomerization of all-trans-retinal to 11-cis-retinal, which combines with opsin to reform rhodopsin. A deficiency of vitamin A, which is often derived from the β-carotene of plants, results in night blindness. Excess of vitamin A (hypervitaminosis A) is toxic, causing cerebral edema and other problems.

Children with autism and other disorders with mental disability are often put on megavitamin supplements despite no scientific evidence of benefit. Although most vitamins are harmless in excess, merely being excreted in urine, vitamin A can be toxic. Which of the following statements regarding vitamin A is true? a. It is not an essential vitamin b. It is related to tocopherol c. It is a component of rhodopsin d. It is derived from ethanol e. It is also known as opsin

The answer is E. Because of the lack of protein in the diet, protein synthesis in the liver is impaired (lack of essential amino acids). The liver can still synthesize fatty acids from carbohydrate or fat sources, but very lowdensity lipoprotein (VLDL) particles cannot be assembled because of the shortage of apoprotein B-100. Thus, the fatty acids remain in the liver, leading to a fatty liver. None of the other answers explains this finding.

Children with kwashiorkor usually have a fatty liver. This is due to which of the following? A. The high-fat content of their diet B. The high-carbohydrate content of their diet C. The high-protein content of their diet D. The lack of substrates for gluconeogenesis in the liver E. The lack of substrates for protein synthesis in the liver F. The lack of substrates for glycogen synthesis in the liver

he answer is e. (Murray, pp 242-248. Scriver, pp 1909-1964.)In the liver, the urea cycle converts excess NH4+ to a form amenable to excretion by the kidneys. Free NH4+ condenses with CO2 to form carbamoyl phosphate in a reaction catalyzed by carbamoyl phosphate synthetase. This is an energy-expensive, essentially irreversible reaction requiring two molecules of ATP. Carbamoyl phosphate (compound E in the urea cycle diagram) then combines with ornithine to produce citrulline in the first step of the urea cycle proper. The second nitrogen of urea is derived from the amino acid aspartate, which condenses with citrulline to form arginosuccinase in the second step of the cycle. This step is catalyzed by arginosuccinase synthetase and also requires a molecule of ATP.

Children with urea cycle disorders have low tolerance for ingested protein because they cannot excrete nitrogen efficiently. In the urea cycle diagrammed below, which compound is derived from a condensation of CO2 and NH4+? a. Compound A b. Compound B c. Compound C d. Compound D e. Compound E

The answer is b. (Murray, pp 242-248. Scriver, pp 1909-1964.) In humans and other land mammals, excess NH4+ is converted into urea (compound C in the urea cycle diagram of Fig. 35) in the liver for excretion by the kidneys. Malfunctions of the urea cycle can lead to hyperammonemia and result in brain damage. Compound C is arginine, the next to last step in the cycle and available as an intravenous solution. Adding arginine bypasses the most severe urea cycle defects—carbamoyl phosphate synthetase deficiency (formation of compound E, 237300), ornithine transcarbamylase deficiency (compound E to A, 311250), citrullinemia (A to argininosuccinate, 215700), argininosuccinic aciduria (argininosuccinate to arginine (compound B-209700)—and increases the substrate concentration for the last enzyme defect, argininemia (208700). Note that nitrogen is excreted in the form of urea with its two nitrogen and one carbonyl group. The first nitrogen is derived from free NH4+ (condensed with CO2 to form carbamoyl phosphate—compound E) and the second nitrogen from the amino group of aspartate.

Children with urea cycle disorders usually present in the newborn period with lethargy progressing to coma as they are exposed to protein in breast milk or infant formula.The diagnosis is confirmed by finding elevated serum ammonia, often in the thousands with the upper limit of normal of 30-50 mmol/L, depending on the laboratory. Initial therapy consists of giving glucose instead of protein, and "priming" the urea cycle with a compound from the diagram in the above figure. Specific therapy will depend on which step of the cycle is blocked, diagnosed by blood amino acids (e.g., elevated citrulline versus ornithine) and enzyme assays. Which of the compounds in the figure would be most helpful in priming the cycle? a. Compound A b. Compound B c. Compound C d. Compound D e. Compound E

The answer is d. (Murray, pp 205-218.) Triacylglycerol is the major fuel reserve in the body and is mainly stored and hydrolyzed in adipose tissue. Triglyceride and fatty acid metabolism become active during fasting after glycogen stores are depleted in 3-4 hours. However, stores of triglycerides in heart and muscle are needed for energy even with adequate feeding. Longer chain fatty acids will contain many more two-carbon units for oxidation than shorter chains, accounting for disproportionate energy depletion and severe cardiac and skeletal disease in children with longer chain enzyme deficiencies. Glucose is important as a brain nutrient but will not be depleted by fatty acid oxidation. Glycogen (in animals) and starch (in plants) are glucose storage molecules. Glycogen is depleted with fasting but is not the major fuel reserve in mammals.

Children with very long or long chain fatty acid oxidation disorders are severely affected from birth, while those with short or medium chain oxidation defects may be asymptomatic until they have an intercurrent illness that causes prolonged fasting. The severe symptoms of longer chain diseases are best explained by which of the following statements? a. Longer chain fatty acids inhibit gluconeogenesis and deplete serum glucose needed for brain metabolism b. Glycogen is the main fuel reserve of the body but is quickly depleted with fasting c. Starch is an important source of glucose and is inhibited by high fatty acid concentrations d. Triacylglycerols are the main fuel reserve of the body and are needed for energy production in actively metabolizing tissues e. Longer chain fatty acids from micelles and block synapses

Which of the following compounds is the primary component of an HDL?

Cholesterol Ester

The answer is a. (Murray, pp 481-497. Scriver, pp 3897-3964.) Thiamine (vitamin B1) activated as its pyrophosphate is a cofactor for pyruvate dehydrogenase, α-ketoglutarate dehydrogenase of the citric acid cycle, branched chain keto-acid dehydrogenase that metabolizes leucine/ isoleucine/valine, and transketolase of the pentose phosphate pathway. Deficiency of thiamine causes beri beri and exacerbates encephalopathy in alcoholics, having impact on the nervous system in both diseases. Since pyruvate dehydrogenase commits pyruvate from glycolysis to acetyl CoA in the citric acid cycle, its impairment will increase lactate (lactic acidosis), deplete energy (by impacting the citric acid cycle and the first steps of oxidative phosphorylation), and impair glucose metabolism—all key to neural function. Impairment of transketolase and the pentose phosphate shunt would reduce NADPH production, key to glutathione maintenance and reduction of oxidants in brain. Certain mutations in transketolase may thus increase susceptibility to Wernicke-Korsakoff syndrome (277730), a nice example of a Mendelian enzyme alteration brought out by environment (alcohol dependency) to cause a multifactorial disease (encephalopathy). Lipoamide is also a cofactor in pyruvate dehydrogenase, transferring the acetyl group in pyruvate to coenzyme A. Lipoamide becomes acetyllipoamide and then dihydro-lipoamide as it first accepts and then transfers an acyl group. This reaction and the regeneration of lipoamide are catalyzed by different parts of the dehydrogenase enzyme complex. ATP transfers phosphoryl groups, thiamine pyrophosphate transfers aldehyde groups, NADH and FADH transfer protons. Mutations in the multipeptide pyruvate dehydrogenase complex occur in Leigh disease (256000), an end phenotype of many mutations that simulate the lactic acidosis and encephalopathy accompanying acute forms of thiamine deficiency (beri beri).

Chronic alcoholics are at risk to develop lactic acidosis and neurologic symptoms, one example of which is the Wernicke-Korsakoff syndrome (277730). This complex of symptoms includes nerve problems like nystagmus (oscillating eyes), ophthalmoplegia (deviated or weak eye), peripheral numbness/tingling and cerebral problems like confusion, delirium, coma, and memory loss in survivors. Explanation of why certain alcoholics get Wernicke-Korsakoff encephalopathy was suggested when altered transketolase (an important enzyme in the pentose phosphate pathway) was found in these individuals. Which of the following is most likely to be important in the development of lactic acidosis and/or Wernicke-Korsakoff susceptibility in alcoholics? a. Thiamine pyrophosphate b. Lipoamide c. ATP d. NADH e. FADH

The answer is D: Formation of acetaldehyde adducts with cytoplasmic proteins. As shown on page 185, acetaldehyde, a product of ethanol metabolism, forms covalent adducts with tubulin, which interferes with the movement and secretion of VLDL and other proteins normally secreted from the liver. It is not ethanol that is interfering with microtubule formation, but its metabolite that with interferes with microtubule function. Ethanol does not directly inhibit gene transcription (it actually leads to the transcription of MEOS, the microsomal ethanol oxidizing system). The levels of acetate formed are not suffi cient to lower the pH of the liver cytosol. The ethanol carbons can be used for ketone body formation, but the formation of ketones is not related to the impaired secretion of VLDL.

Chronic alcoholics often develop fatty and leaky livers, as the primary site of alcohol detoxification is the liver. Protein and VLDL secretion can be impaired in such patients due to which of the following? (A) Ethanol inhibition of microtubule formation (B) Ethanol inhibition of gene transcription (C) Acetate reduction of intracytoplasmic pH (D) Formation of acetaldehyde adducts with cytoplasmic proteins (E) Increased ketone body levels within the cytoplasm

Immediately after eating a cheeseburger and fries you would expect to see an elevation in which of the following plasma lipoproteins?

Chylomicrons

Which of the following lipoproteins is responsible for the transport of dietary triacylglycerols in the blood?

Chylomicrons

Adenosine Deaminase (ADA)

Clinically important Plays an important role in adenosine homeostasis Breakdown of adenosine from food Turnover of nucleic acids in tissues Overproduction of ADA (an erythrocyte form): - Causes hemolytic anemia (rare) →increased degradation of adenosine, depletes adenine nucleotide pool, triggers premature destruction of RBCs Underproduction of ADA: - Associated with the second most common form of severe combined immunodeficiency (SCID)

The answer is A: B6, biotin, and niacin. Biotin is required for pyruvate carboxylase (the enzyme that converts pyruvate to oxaloacetate), an enzyme necessary for gluconeogenesis from any TCA cycle precursor (and pyruvate). The B6 is required for glycogen phosphorylase (the ability to produce glucose from glycogen) and for transamination reactions, which are necessary for amino acids (such as alanine, aspartic acid, and glutamic acid) to provide carbons for gluconeogenesis. The niacin is required to produce NADH, which is needed to reverse the glyceraldehyde-3-phosphate dehydrogenase step during gluconeogenesis. It is also needed for lactate to be converted to pyruvate, and lactate (provided by the red blood cell) is a major gluconeogenic precursor. The lipid-soluble vitamins (D and K) are not required for glucose production and release (vitamin D is needed for calcium metabolism, and vitamin K is required for the carboxylation of glutamic acid side chains of proteins involved in blood clotting). Vitamin B12 is not required for glucose production directly, and its absence does not lead to hypoglycemia.

Consider a 40-year-old man who has just initiated a 24 h fast. Which of the following cofactors are necessary so that his blood glucose levels can be kept constant? (A) B6, biotin, and niacin (B) B6, niacin, and vitamin D (C) B6, biotin, and vitamin D (D) Biotin, niacin, and B12 (E) Biotin, niacin, and vitamin K

Nucleotidases

Converts NMPs/dNMPs into nucleosides and deoxynucleosides

Phosphodiesterases

Converts short oligomers to NMPs or dNMPs

The answer is b. (Murray, pp 219-230. Scriver, pp 2863-2914.) Mevastatin, an analogue of mevalonic acid, acts as a feedback inhibitor of 3′-hydroxy-3′-methylglutaryl CoA (HMG-CoA) reductase, the regulated enzyme of cholesterol synthesis. Effective treatment with mevastatin, along with a low-fat diet, decreases levels of blood cholesterol. The lowering of cholesterol also lowers the amounts of the lipoprotein that transports cholesterol to the peripheral tissues, low-density lipoprotein (LDL). Because lipids like cholesterol and triglycerides are insoluble in water, they must be associated with lipoproteins for transport and salvage between their major site of synthesis (liver) and the peripheral tissues. Those lipoproteins associated with more insoluble lipids thus have lower density during centrifugation (see the table in answer 258), a technique that separates the lowest-density chylomicrons from very-low-density lipoproteins (VLDLs with pre-β-lipoproteins), low-density lipoproteins (LDLs with β-lipoproteins), intermediate-density lipoproteins (IDLs), and high-density lipoproteins (HDLs with α-lipoproteins). Each type of lipoprotein has typical apolipoproteins such as the apo B100 and apo B48 (translated from the same messenger RNA) in LDL. LDL is involved in transporting cholesterol from the liver to peripheral tissues, whereas HDL is a scavenger of cholesterol. The ratio of HDL to LDL is thus a predictor of cholesterol deposition in blood vessels, the cause of myocardial infarctions (heart attacks). The higher the HDL/LDL ratio, the lower the rate of heart attacks.

Coronary artery disease is a multifactorial disorder involving occlusion of the coronary artery with atherosclerotic plaques. Several Mendelian disorders affecting lipid metabolism increase susceptibility for heart attacks, while environmental factors include smoking and high-fat diets. A 45-year-old man has a mild heart attack and is placed on diet and mevastatin therapy. Which of the following will be the most likely result of this therapy? a. Low blood glucose b. Low blood LDLs c. High blood cholesterol d. High blood glucose e. Low oxidation of fatty acids

Cortisol in fasting state

Cortisol has a proteolytic effect, during starvation glycogen quickly Gluconeogenesis is initiated using AA in the presence of cortisol but in its absence death ensues(minimal elevation of cortisol) Corticosteroids prime pathways to respond to other hormones such as epinepherine on glycogen synthesis and degradation(potentiated) Cortisol enhances lipolysis of FA into glucose

Defective AA transporter leads to cystinuria

Cystinuria- autosomal recessive, defect in AA transporter of Cystine as well as Arg, Lys, and Ornithine Results in formation of Cystine stones in kidneys and causes patients to present with renal cholic(pain) which comes in waves and is linked to kidney stones

Other electron transport system

Cytochrome P-450 (CYP) superfamily located primarily in the ER Active in metabolism of hydrophobic compounds such as steroid hormones, eicosanoids, and vitamin D, xenobiotics(drugs and toxins) Electron transfer from NADPH to the heme in mitochondrial CYP involved flavoprotein and Fe-S

Enzymes that require Cu as Cofactor

Cytochrome c oxidase Ferroxidase Superoxide dismutase (antioxidant) Lysyl oxidase

An obese 19-year-old man has a free health screen arranged by his employer. Apart from fairly frequent headaches, he is well. The results of liver function tests include a serum aspartate aminotransferase (AST) activity of 72 U/L [0-35U/L]; total bilirubin concentration and alkaline phosphatase and γ-glutamyl transferase activities are within normal limits. His fasting blood glucose concentration is 6.9 mmol/L [3-6mmol/L]. What is the most likely cause of the elevated AST? A. Early viral hepatitis B. Excessive alcohol ingestion C. Gallstones D. Non-alcoholic steatohepatitis E. Therapeutic use of aspirin

D. This is the most common cause of an isolated increase in serum aminotransferase activity, particularly in overweight and obese subjects. Although γ-glutamyl transferase activity is also usually elevated, it is not always so, and impaired glucose tolerance is a recognized consequence of obesity. Viral hepatitis would not present with normal bilirubin concentration, alkaline phosphatase and gammaglutamyl transferase. Excessive alcohol consumption would not explain the elevated glucose and normal values of other liver enzymes. Gallstones do not explain the elevated AST level. Aspirin does not explain the elevated blood glucose.

Increased catabolism in liver of which metabolite (A-E) contributes MOSTLY to the elevated BUN in ketoacidosis? A. Triglycerides B. Glycogen C. Creatinine D. Alanine E. Glycerol

D. Alanine

Plasma proteins released from damaged tissues provide a diagnostic tool in not only identifying the tissue that is damaged, but also as a marker of the severity of tissue damage. Of the plasma proteins released from the damaged liver, an increase in which protein's enzymatic activity would be the best indicator of the most cell destruction? A. Alanine aminotransferase (ALT) B. Υ-glutamyl transferase (GGT) C. Alkaline phosphatase (ALP) D. Aspartate aminotransferase (AST) E. 5' nucleotidase (5'NT)

D. Aspartate aminotransferase (AST) Both alanine and aspartate aminotransferase are found primarily in liver and muscle, while GGT is found in liver and kidney. ALP is found in bone and liver bile duct epithelial cells, while 5'nucleotidase is found on most cells that uptake nucleoside from blood for nucleotide synthesis. Elevated levels of AST would indicate the most cell damage since this enzyme is found not only in the cytosol but also the mitochondria of liver and muscle cells.

Receptors for the inhibitory neurotransmitters GABA and glycine: A. are ligand-gated anion channels. B. are ligand-gated cation channels. C. permit hyperpolarization of neuronal cell membranes. D. Both A and C are correct. E. Both B and C are correct.

D. Both A and C are correct.

A high school student football player received a scholarship to play at a Mac school. His coach said he needs to put on weight to become a starting offensive line man. He begins a regime of weight lifting and consumption of creatine and whey protein. After 4-months, he has put on 30 lbs of muscle and becomes the starting tackle for the Mac team. However two months into the season he is having trouble urinating and the team physician after examination notices hepatomegaly and elevated creatinine. He asks the boy whether he has taken steroids, and the athlete confesses he has taken anabolic steroids to put on weight. What cytochrome P450 is induced by steroids in this boy? A. CYP1A2 B. CYP2C9 C. CYP2E1 D. CYP3A4 E. CYP4A1

D. CYP3A4 The correct answer is D, CYP3A4, which is induced by steroids and metabolizes over 50% of prescribed drugs including statins, nifedipine, and a calcium channel blocker to prevent vessel contractions. CYP3A4 is inhibited by grapefruit.

A 50-year-old man who has a 10-year history of alcohol abuse complains of chest pain with radiation of the pain down the inner aspect of his left arm. An electrocardiogram shows an acute anterior myocardial infarction. A complete blood cell count reveals a macrocytic anemia with hypersegmented neutrophils and mild pancytopenia. No neurologic abnormalities are present on physical examination. The plasma homocysteine is increased. Which of the following best explains the mechanism for the increase in plasma homocysteine? A. Decreased activity of cystathionine synthase B. Decreased activity of dihydrofolate reductase C. Decreased activity of thymidylate synthetase D. Decreased serum folic acid E. Decreased serum vitamin B12

D. Decreased serum folic acid The most common cause of folic acid deficiency is alcoholism. Mechanisms for the deficiency include a poor diet leading to decreased liver stores of the vitamin and decreased reabsorption of the monoglutamate form of folate in the jejunum. Vitamin B12 (cobalamin) removes the methyl group from N5-methyltetrahydrofolate (circulating form of folic acid) to produce methyl-B12 and tetrahydrofolate. The methyl group is then transferred to homocysteine to produce methionine. Therefore a decrease in either N5-methyltetrahydrofolate or vitamin B12 causes an increase in the plasma homocysteine levels. Since there are no neurologic findings present, the most likely diagnosis is folic acid deficiency rather than vitamin B12 deficiency, which is associated with neurologic abnormalities in the spinal cord and central nervous system. An increase in plasma homocysteine damages endothelial cells and is a risk factor for coronary artery thrombosis.

A 70-year-old man is in hypovolemic shock resulting from a ruptured abdominal aortic aneurysm. Which of the following is a reversible cellular event that is most likely to occur in the straight portion of the proximal tubular cells of the kidneys? A. Activation of phospholipase in the cell membrane B. Diffusion of calcium into the cytosol C. Diffusion of cytochrome c out of the mitochondria D. Diffusion of sodium and water into the cytosol E. Increase in intracellular pH

D. Diffusion of sodium and water into the cytosol Blood loss due to a ruptured abdominal aortic aneurysm causes tissue hypoxia (inadequate oxygenation of tissue). The straight portion of the proximal tubule is the most sensitive portion of the nephron to hypoxia. The lack of oxygen in blood in hypovolemic shock causes a decrease in ATP synthesis in the mitochondria, which adversely affects the Na+/K+-ATPase pump by allowing Na+ and H2O to enter the cytosol. This produces swelling of the cell, which is reversible if oxygen is restored to the cell.

A patient was given a standard dose of 5-fluorouracil with no reduction in tumor size. The reduced level of 5-fluorouracil in the serum and increased levels of fluoro-alanine in the urine suggest that this patient has increased activity of which enzyme? A. Ribonucleotide reductase B. Dihydrofolate reductase C. Thymidylate synthase D. Dihydropyrimidine dehydrogenase E. OMP decarboxylase

D. Dihydropyrimidine dehydrogenase This enzyme is polymorphically expressed in the human population. In individuals undergoing treatment for cancer with the uracil analogue 5' Fluorouracil, the efficacy of treatment is largely determined by the activity of this enzyme. High metabolizers do not benefit from 5-fluorouracil chemotherapy while individuals with low activity of DPD can experience increased toxicity.

Which of the following best describes glutathione (GSH)? A. It is known as GSH because it is tripeptide of glutamate, serine, and histidine. B. GSH can be reduced and bind cysteine through disulfide bonds. C. Protein disulfide isomerase can make GSH dimers. D. GSH is involved in amino acid transport through the Υ-glutamyl cycle. E. GSH is composed of glutamate, cysteine and histidine.

D. GSH is involved in amino acid transport through the Υ-glutamyl cycle. The correct answer is D, involved in the transport of amino acids across the cell membrane, especially cysteine. It is also found in the smooth endoplasmic reticulum and induced by drug to supply glutathione for phase II conjugation reactions. GSH is glutamate, cysteine, and glycine. GSH can dimerize by glutathione reductase and can rearrange protein disulfide bonds.

Efficacy of drugs varies between individuals. Thus, significant cautions are required to treat patients with certain drugs. What factor affects this individual variation the most? A. Physical activities B. Age C. Physical appearance (weight, height, etc.) D. Genetic polymorphism E. Diet

D. Genetic polymorphism

Which enzyme functions to make a chemical more polar for excretion from the body? A. Epoxide hydratase B. Alanine aminotransferase C. Ferrochelatase D. D. Glutathione s-transferase E. Multi-drug resistance protein

D. Glutathione s-transferase Glutathione S-transferase attaches glutathione to the hydroxyl groups of chemicals metabolized by cytochrome P450 making them more polar and thus easier for excretion from the body.

Allelic variants of cytochrome P450 can best be described as having what amino acid identity with the wild type P450? A. Between 40 and 55% B. Less than 40% C. Between 55 and 95% D. Greater than 95% E. Less than 95%

D. Greater than 95%

A 35-year-old woman complains of palpitations that keep her awake at night. Physical examination reveals exophthalmos, thyromegaly, sinus tachycardia, and pretibial myxedema. Which of the following glycosaminoglycans is responsible for the pretibial myxedema and exophthalmos? A. Dermatan sulfate B. Heparan sulfate C. Heparin D. Hyaluronic acid

D. Hyaluronic acid Glycosaminoglycans (GAGs) are the major components of ground substance in the interstitial tissue. They are complexes of branched, negatively charged polysaccharide chains containing amino sugars (e.g., glucosamine or galactosamine) and acid sugars (iduronic acid or glucuronic acid). The patient has Graves' disease, which is due to an IgG antibody directed against the thyroid-stimulating hormone (TSH) receptor. The patient has exophthalmos and pretibial myxedema, which is due to fibroblasts secreting hydrophilic GAGS, like hyaluronic acid and chondroitin sulfate into the interstitium. Because of the gel nature of the excess fluid (myxedema), it is mainly immobile, and the edema is nonpitting.

The reaction of hypoxanthine and PRPP (5'-phosphoribosyl pyrophosphate) leads directly to the formation of: A. XMP. B. AMP. C. GMP. D. IMP. E. XDP.

D. IMP.

WHICH ONE of the following is an action of insulin? A. Stimulation of glycogenolysis B. Stimulation of gluconeogenesis C. Inhibition of fatty acid synthesis D. Increased activity of lipoprotein lipase E. Stimulation of protein catabolism

D. Increased activity of lipoprotein lipase LPL liberates fatty acids from chylomicrons and VLDLs for uptake into adipocytes.

Which statement is true of familial hypercholesterolemia? A. It is associated with decreased plasma concentrations of high density lipoprotein (HDL) cholesterol. B. It is caused by an abnormality in apolipoprotein B-100. C. It is caused by increased hepatic secretion of very low density lipoprotein (VLDL). D. It is characterized by decreased clearance of low density lipoprotein (LDL) from the plasma. E. It is inherited as an autosomal recessive trait.

D. It is characterized by decreased clearance of low density lipoprotein (LDL) from the plasma. Familial hypercholesterolemia is caused by any one of many mutations that reduce the expression or activity of the LDL receptor, thus decreasing cellular uptake of LDL from the plasma. Familial hypercholesterolemia is not associated with a change in serum HDL levels and is autosomal dominant. Lipoprotein B100 abnormalities are more associated with abetalipoproteinemia. Type IV hypertriglyceridemia is associated with increased hepatic secretion of VLDL.

The mechanism of action of many of these foods acts as electrophiles and converts reduced sulfhydryl bonds to oxidized disulfides. Similarly, ROS depletes the body of reduced glutathione, which maintains protein cysteine sulfhydryl groups in the reduced state. Which protein's cysteine sulfhydryl groups, when oxidized, release Nuclear factor erythroid related factor 2 (Nrf2) that functions as a transcription factor to increase the body's expression of antioxidant genes? A. Aryl hydrocarbon receptor (Ahr) B. Lipofuscin C. Glutathione peroxidase D. Kelch-like ECH-associated protein 1 (Keap1) E. Peroxisome proliferator activated receptor gamma (PPAR)

D. Kelch-like ECH-associated protein 1 (Keap1) The correct answer is D, Keap1, which binds NRF2 and prevents NRF2 activation of antioxidant genes. Consumption of foods that act as electrophiles or ROS oxidizes Keap's cysteine SH groups to disulfides release NRF2 to stimulate gene transcription of antioxidant genes.

A 58-year-old patient has a medical history of multiple infections involving the liver, lung, and kidney as well as excessive inflammation with granulomas, gingivitis, and dermatitis. This patient most likely has a deficiency of which of the following enzymes? A. Transketolase B. Xanthine oxidase C. Myeloperoxidase D. NADPH oxidase E. Cytosine deaminase

D. NADPH oxidase The patient shows signs and symptoms of chronic granulomatous disease due to the inability of phagocytic cells to produce superoxides to kill invading microbes. Myeloperoxidase deficiency does not present these clinical abnormalities except in patients with diabetes. However these patients have an increased frequency of fungal infections.

The resting membrane potential (membrane polarization) of the neuron can be attributed, in part, to the relatively low permeability of the membrane to which cation? A. Ca2+ B. K+ C. Mg++ D. Na+ E. SO4-

D. Na+

Oncogenes are best described by which of the statements? A. Oncogenes will contribute to tumor formation only if both alleles are mutated. B. Oncogenes are often part of structural components of the cell architecture. C. Oncogenes are always mutant versions of proto-oncogenes. D. Oncogenes are unregulated or mutated versions of proto-oncogenes.

D. Oncogenes are unregulated or mutated versions of proto-oncogenes.

What type of signaling involves molecules called "local mediators"? A. Autocrine B. Endocrine C. Juxtacrine D. Paracrine E. Synaptic

D. Paracrine Paracrine signaling is over short distances. Juxtacrine requires cell-to-cell contact; endocrine describes cell signaling over long distances; autocrine is self-stimulation; and synaptic specifically describes neuronal communication via neurotransmitters.

A mature blood cell that lacks a nucleus is which of the following? A. Lymphocyte B. Basophil C. Eosinophil D. Platelet E. Neutrophil

D. Platelet

Acute intermittent porphyria results from lack of which enzyme? A. Uroporphyrinogen decarboxylase B. Protoporphyrinogen oxidase C. ALA synthase D. Porphobilinogen deaminase E. Ferrochelatase

D. Porphobilinogen deaminase

The conjugation of albumin to bilirubin is made through bonds between lysine of albumin and what chemical group of bilirubin? A. Vinyl B. Methyl C. Methene D. Propionyl E. Iron

D. Propionyl

Cancer has been described as a genetic disease and which of the following statements directly best supports this idea? A. Smoking causes lung cancer. B. H. pylori causes stomach cancer. C. Proto-oncogenes are inherited. D. The conversion of a proto-oncogene into an oncogene that will be inherited by all tumor cells within a tumor. E. Tumor suppressor genes are inherited.

D. Proto-oncogenes can be obtained from the environment. Tumor suppressor genes are a part of every person's genetic make-up.

Which chemical mediator is an anti-inflammatory? A. Prostaglandin B. Cytokine C. Leukotriene D. Resolvin E. Chemokine

D. Resolvin Resolvin is the only specific mediatory listed that is involved in calming the inflammatory process. And, it is aptly named.

S-Adenosyl methionine is required for which conversion? A. tyrosine ® dihydroxyphenylalanine (L-DOPA) B. L-DOPA ® dopamine C. tryptophan ® serotonin D. norepinephrine ® epinephrine E. dopamine ® norepinephrine

D. SAM is needed as a methyl donor in this reaction.

Which class of hormones is synthesized by a series of cytochrome P450s? A. Amino acid derivatives B. Cytokines C. Peptides D. Steroid E. Thyroid

D. Steroid P450 is necessary for the formation of all hormones of the adrenal cortex, including aldosterone, cortisol, and androgens. All of these hormones are derived from cholesterol, and therefore are steroid hormones.

Which one of the following eicosanoids causes platelet aggregation and vasoconstriction? A. PGE2 B. LTC4 C. LXA4 D. TXA2 E. PGI2

D. TXA2 TXA2 is a potent vasoconstrictor and critical in platelet aggregation. TXA2 is an important mediator of clot formation in primary hemostasis. The coagulation cascade is an important component of secondary hemostasis.

What is the rate-limiting factor in the synthesis of acetylcholine? A. The activity of choline acetyltransferase B. The availability of acetyl-CoA C. The activity of acetylcholinesterase D. The availability of choline E. The activity of butyrylcholinesterase

D. The availability of choline

married couple has been charged by the prosecuting attorney of Portage County with child abuse in the apparent bruising and beating of their 2-year-old male child, now placed in surrogate care away from the family home. The evidence for the allegation is that the child was brought to the local hospital by the couple and found to have multiple hematomas and severe pain. On physical examination by the attending physician, the child appeared to be of short stature for his age group and had sclerae tinged slightly blue and had teeth that were defective in size and shape for a 2-year-old male. Precautionary x-rays revealed fractures of the left tibia and left humerus and the locations of the fractures coincided with the predominant sites of skin bruising. As an expert witness and the emergency room attending physician who examined the child on his admission to the hospital, you are asked to offer an opinion as to the cause of the problem in this case. You suggest correctly which one of the following? A. The child has the symptoms of mucopolysaccharide storage disorder that results in the observed changes in the eye coloring, skin bruising, and cartilage, bone and tooth fractures and abnormalities. B. The child has a skeletal dysplasia that is manifested by problems in growth plate development and changes in eye coloring and the features of the teeth. C. The child has the bone development disorder, osteogenesis imperfecta. D. The cause of the problem cannot be conclusively identified until collagen genotyping of the child and his parents is completed. E. The parents appear to have severely mistreated their son.

D. The cause of the problem cannot be conclusively identified until collagen genotyping of the child and his parents is completed. Conclusive diagnosis of osteogenesis imperfecta cannot be confirmed until collagen genotyping, however, blue sclera is highly suggestive of the diagnosis and not consistent with child physical abuse.

If this child is diagnosed with osteogenesis imperfecta and has blue sclerae, which of the following is most likely true regarding his future life? A. The child will likely not survive beyond age 5. B. The child will likely survive for no more than 10 years. C. The child will likely present with disorders affecting his lungs and heart in addition to those of the skeleton and dentition. D. The child will not reach full, normal height and weight for his age and will likely present with osteoporosis, dentinogenesis imperfecta and premature otosclerosis. E. On reaching puberty, the child will completely recover from the disorder.

D. The child will not reach full, normal height and weight for his age and will likely present with osteoporosis, dentinogenesis imperfecta and premature otosclerosis. Lifespan varies among types of osteogenesis imperfecta, and is not provided in the question stem. For example, in Type I, lifespan is reduced. In Type II, children most likely will not live past year 1. In Type III, lifespans may be normal. What is true of all subtypes is that normal height will not be reached.

A family has two children, one with a mild case of thalassemia, and a second with a severe case of thalassemia, requiring frequent blood transfusions as part of the treatment plan. One parent is of Mediterranean descent, the other is of Asian descent. Neither parent exhibits clinical signs of thalassemia. Both children express 20% of the expected level of β-globin; the more severely affected child expresses normal levels of a-globin, whereas the less severely affected child only expresses 50% of the normal levels of a-globin. Why is the child who has a deficiency in a-globin expression less severely affected? A. Thalassemia is caused by a mutation in the a-gene, and the more severely affected child expresses more if it. B. The less severely affected child must be synthesizing the x-gene to make up for the deficiency in a-chain synthesis. C. The more severely affected child also has HPFH. D. The more severely affected child produces more inactive globin tetramers than the less severely affected child. E. Thalassemia is caused by an iron deficiency, and when the child is synthesizing normal levels of a-globin, there is insufficient iron to populate all of the heme molecules synthesized.

D. The more severely affected child produces more inactive globin tetramers than the less severely affected child.

A 2-year-old female infant is weak, lethargic and anemic, and her height and weight are low for her age. Her urine contains elevated levels of orotic acid, and the activity of uridine monophosphate synthase is low. The administration of which of the following will most likely alleviate her symptoms? A. Adenine B. Hypoxanthine C. Thymidine D. Uridine E. Guanine

D. Uridine The correct answer is D, Uridine, because the increased excretion of orotic acid and low activity or UMP synthase suggest that the patient has orotic aciduria. Deficiencies in one or both catalytic domains of UMP synthase result in the patient being unable to synthesize pyrimidines. Uridine bypasses the missing activity and can be salvaged to UMP that can also be converted to other pyrimidine nucleosides. Thymidine cannot be demethylated and converted to other pyrimidines. Hypoxanthine, adenine and guanine are all purine bases and, therefore, cannot be converted to pyrimidines

Which one of the following sets of three vitamins would be the MOST EFFECTIVE in reducing homocysteine levels in blood? A. Vitamin B6, biotin, thiamine B. Vitamin B12, lipoamide, pantothenic acid C. Folic acid, vitamin E, niacin D. Vitamin B6, vitamin B12, folic acid E. Riboflavin, vitamin K, vitamin B

D. Vitamin B6, vitamin B12, folic acid A high level of homocysteine in the blood raises the risk for heart disease. Two mechanisms of reducing homocysteine levels via three vitamins include the following: Homocysteine can be re-methylated to methionine: Homocysteine + methyl-cobalamin (where methyl group comes from folic acid) → methionine, catalyzed by methionine synthase. Homocysteine can be condensed with serine to form cystathionine: Homocysteine + B6 → cystathionine (catalyzed by cystathionine ß-synthase); cystathionine + B6 → cysteine.

In the normal blood clotting cascade, many of the various coagulation factors must be carboxylated in order for them to bind to phospholipid surfaces and participate in subsequent reactions leading to fibrin formation. The action of which specific vitamin is required for these carboxylation events? A. Vitamin C B. Vitamin B C. Vitamin D D. Vitamin K E. Vitamin E

D. Vitamin K Vitamin K is needed for γ‐carboxylation of factors II, VII, IX, and X.

Hereditary orotic aciduria is characterized by severe anemia, growth retardation, and high levels of orotic acid excretion. The defect may be either orotate phosphoribosyl transferase, orotidine decarboxylase, or both. The preferred treatment is dietary uridine, which reverses anemia and decreases the formation of orotic acid. Elements involved in the effectiveness of the dietary treatment include: A. conversion of uridine to UMP by uridine phosphotransferase. B. UTP from exogenous uridine provides substrate for synthesis of CTP. C. inhibition of carbamoyl phosphate synthetase II by UTP. D. all of the above. E. none of the above.

D. all of the above. Exogenous uridine requires conversion to monophosphate and ultimately UTP. The cell is deficient in UTP and CTP because the conversion of orotic acid is blocked, however the blocked and bypassed by uridine, which is converted to UTP that inhibits CPSII.

GABA (gamma-aminobutyrate), a neurotransmitter of inhibitory neurons, is synthesized by: A. transamination of alpha-ketobutyrate. B. carboxylation of alanine. C. deamination of glutamine. D. decarboxylation of glutamate. E. decarboxylation of aspartate.

D. decarboxylation of glutamate.

Some mucopolysaccharidoses cause only connective tissue problems. In others, however, the patients have mental deficiency. Mental deficiency is most likely to occur in diseases with impaired breakdown of: A. hyaluronic acid. B. chondroitin sulfate. C. dermatan sulfate. D. heparan sulfate. E. keratan sulfate.

D. heparan sulfate. Mucopolysaccharidoses are characterized by mental retardation present in the severe forms of Hurler, Hunter, and Sanfilippo diseases. High values of urine heparin sulfate are found in these patients.

Gout is a disease characterized by hyperuricemia from overproduction of purine nucleotides. All of the following reactions or conditions can lead to hyperuricemia, EXCEPT: A. increased activity of the pentose monophosphate pathway. B. decreased activity of HGPRTase. C. increased consumption of protein. D. increased allosteric inhibition of PRPP synthetase.

D. increased allosteric inhibition of PRPP synthetase. If PRPPs synthetase is inhibited, no new purine nucleotides will be made. A will increase purine nucleotide synthesis. B will cause hyperuricemia. C will cause hyperuricemia by increasing purine nucleotide synthesis.

Diabetes mellitus is a disease in which glucose metabolism is impaired. Type 2 occurs primarily in middle-aged to older obese individuals, while type 1 usually appears in childhood or teens. A third type of diabetes occurs in older individuals and has the characteristics of both type 2 and type 1. Insulin's ability to control blood glucose levels is affected either because of lack of insulin or inadequate function of insulin. In type 2 diabetes: A. hypertriglyceridemia does not occur. B. ketoacidosis in the untreated state is always present. C. Β-cells of the pancreas are no longer able to produce insulin. D. may be accompanied by high levels of insulin in the blood. E. severe weight loss always occurs.

D. may be accompanied by high levels of insulin in the blood. The major problem is insulin resistance and not the failure to produce insulin. Hypertriglyceridemia is a characteristic of disease, and ketoacidosis is very uncommon. Most patients are obese and remain so.

The catabolism and excretion of steroid hormones: A. occur primarily as the bile salts. B. usually involve transmethylation reactions. C. require reduction and cleavage of the steroid nucleus. D. may involve conjugation of a metabolite with sulfate or glucuronate. E. produce carbon dioxide.

D. may involve conjugation of a metabolite with sulfate or glucuronate. Conjugation turns the steroids into hydrophilic molecules that can be excreted in the urine.

Biological conversion of testosterone: A. never results in the formation of estrogens. B. leads only to the production of 17-ketosteroids. C. does not take place. D. may lead to even more active androgens being formed. E. does not occur in androgen target tissues.

D. may lead to even more active androgens being formed. Testosterone is converted to dihydrotestosterone (DHT) which is a more potent form.

During a routine checkup of an asymptomatic middle-aged woman, her level of haptoglobin is found to be extremely low. Other blood values are normal. This finding could indicate: A. chronic damage to skeletal muscle. B. either acute liver damage or biliary obstruction. C. an acute-phase response. D. mild chronic hemolysis. E. multiple myeloma.

D. mild chronic hemolysis. Haptoglobin is an α2-globulin and functions to bind free hemoglobin released into the plasma during intravascular hemolysis. The haptoglobin-Hb complex is removed by the reticuloendothelial system, thereby conserving iron, and the concentration of haptoglobin falls correspondingly. Low concentrations are also due to chronic liver disease, metastatic disease and severe sepsis. Haptoglobin is an acute phase protein increased by cytokines, tumor necrosis factor and vasoactive substances.

Following the binding of insulin to its receptor, the next step in this signal transduction pathway is: A. activation of a phospholipase C isoform. B. decreased synthesis of cAMP (3', 5'-AMP). C. hydrolysis of cGMP (3', 5'-GMP). D. phosphorylation of one or more proteins. E. release of Ca2+ from the endoplasmic reticulum.

D. phosphorylation of one or more proteins. The insulin receptor is a tyrosine kinase receptor that leads to the phosphorylation of other proteins/enzymes.

The glucocorticoid hormones act: A. at the cell surface by binding to specific membrane-bound receptors. B. by directly activating a protein kinase. C. by blocking expression from target genes. D. through an intracellular receptor that binds to DNA. E. by directly binding to glucocorticoid responsive elements (GREs) near the end of target genes.

D. through an intracellular receptor that binds to DNA. The correct answer is D. Glucocorticoid (e.g. cortisol) is a steroid hormone.

Tissue edema immediately following an injury is the result of increased: A. capillary blood flow. B. capillary oncotic pressure. C. tissue hydrostatic pressure. D. tissue oncotic pressure. E. venous stasis.

D. tissue oncotic pressure. After histamine stimulation, contractile elements within the endothelial cell of the postcapillary venule pull the interdigitating membranes apart. Spaces develop between the tight junctions that allow plasma and small plasma proteins to escape into the interstitium. Water is easily pushed out through the larger spaces and leaking protein will pull out more water. The excess water and protein establish a new equilibrium, called edema, where there is more fluid in the nterstitium. Histamine and PGD2 cause smooth muscle cells in the wall of the arterioles to vasodilate with a localized increase in blood, including leukocytes and plasma proteins flowing through the capillary network. The increased blood to this area is known as rubor. Simply put: edema is acclamation of fluid in the interstitium. While increased capillary blood flow occurs in inflammation, called rubor, it is not the mechanism that causes edema. Edema is caused by increased vascular permeability. Increased proteins in interstitium pull fluid from the capillaries into the interstitium; this phenomenon is due to increased tissue oncotic pressure.

The correct biosynthetic pathway from tyrosine to epinephrine is: A. tyrosine ® dopamine ® dopa ® norepinephrine ® epinephrine. B. tyrosine ® dopamine ® dopa ® epinephrine. C. tyrosine ® dopamine ® norepinephrine ® epinephrine. D. tyrosine ® dopa ® dopamine ® norepinephrine ® epinephrine. E. tyrosine ® dopa ® dopamine ® epinephrine.

D. tyrosine ® dopa ® dopamine ® norepinephrine ® epinephrine.

Aspartate Transcarbamoylase

De Novo Synthesis of pyrimidine First step (committed step) of the synthesis of the pyrimidine ring Carbamoyl Phosphate + Aspartate → N-Carbamoylaspartate

Carbamoyl Phosphate Synthetase II

De Novo Synthesis of pyrimidine Gln + HCO₃⁻ + 2 ATP → Carbamoyl phosphate + Glu + 2 ADP + Pi (+) PRPP (-) UTP

Glucagon stimulates gluconeogenesis through which of the following mechanisms?

Decreasing levels of fructose 2,6 bisphosphate

Pyruvate DH deficiency(neonatal lactic acidosis

Defect in PDH specifically E1 Infants present with high serum pyruvate and lactate although the ratio is normal Tx is vitamin B1(thiamine PP), lipoic acid and biotin.....(cofactor for E1, stimulator of PDH overall, cofactor for pyruvate carboxylase, respectively), also dichloroacetate which inhibits PDK and activates PDC As well as ketogenic diet to reduce pyruvate production and generating Acetyl CoA bypassing PDC

Lesch-Nyhan Syndrome

Defective HGPRT enzyme in purine salvage pathway causing a rare form of hyperuricemia The elevated circulating levels of hypoxanthine and guanine lead to increased uric acid formation(hyperuricemia) The PRPP leftover from not being used in purine salvage is shunted to purine synthesis leading to even more purines(A,G) which can become uric acid

Vitamin D disorders

Deficiencies can be caused by dietary intake, disruption of absorption of lipids, poor functioning of liver/kidneys, hypoparathyroidism, lack of sunlight Present as brittle bones observed as rickets in children(growth deficiencies and deformities), osteomalacia in adults(marked fractures), and hypocalcemic tetany(involuntary muscle contractions). Excess vitamin D leads to hypercalcemia(blood) and hypecalciuria(urine), patients appear dazed, loss of appetite and with sarcoidosis(inflammation causing immune cell clusters in lungs, skin and lymph nodes)

Infant I has a defect in carbamoyl phosphate synthetase I (CPSI; answer A), and infant IV has a defect in ornithine transcarbamoylase (OTC; answer B). Infants with high ammonia, low arginine, and low citrulline levels must have a defect in a urea-cycle enzyme before the step that produces citrulline, that is, CPSI or OTC. If CPSI is functional and carbamoyl phosphate is produced but cannot be further metabolized, more than the normal amount is diverted to the pathway for pyrimidine synthesis and the intermediate orotate appears in the urine. Therefore, infant I has a defect in CPSI (citrulline is low and less than the normal amount of orotate is in the urine). Infant IV has an OTC defect; carbamoyl phosphate is produced, but it cannot be converted to citrulline, so citrulline is low and orotate is present in the urine. Infants II and V have high levels of citrulline but low levels of arginine. Therefore, they cannot produce arginine from citrulline. Argininosuccinate synthetase or argininosuccinate lyase is defective. The very elevated citrulline levels in infant II suggest that the block is in argininosuccinate synthetase (answer C). In infant V, citrulline levels are more moderately elevated, suggesting that citrulline can be converted to argininosuccinate and that the defect is in argininosuccinate lyase (answer D). Thus, the accumulated intermediates of the urea cycle are distributed between argininosuccinate and citrulline (both of which can be excreted in the urine). The high levels of arginine and more moderate hyperammonemia in infant III suggest that, in this case, the defect is in arginase (answer E).

Deficiency diseases have been described that involve each of the five enzymes of the urea cycle. Clinical manifestations may appear in the neonatal period. Infants with defects in the first four enzymes usually appear normal at birth, but after 24 hours, progressively develop lethargy, hypothermia, and apnea. They have high blood ammonia levels, and the brain becomes swollen. One possible explanation for the swelling is the osmotic effect of the accumulation of glutamine in the brain produced by the reactions of ammonia with alpha-ketoglutarate and glutamate. Arginase deficiency is not as severe as deficiencies of the other urea-cycle enzymes. Given the following information about five newborn infants (identified as I through V) who appeared normal at birth but developed hyperammonemia after 24 hours, determine which urea-cycle enzyme might be defective in each case (for each infant, choose from the same five answers, lettered A through E). All infants had low levels of blood urea nitrogen (BUN). (Normal citrulline levels are 10 to 20 microM.) A. Carbamoyl phosphate synthetase I B. Ornithine transcarbamoylase C. Argininosuccinate synthetase D. Argininosuccinate lyase E. Arginase

Neimann-Pick disease

Deficiency in Acid Sphingmyelinase(A-SMase), which breaks down SM Leads to accumulation of SM in liver, spleen and CNS lysosomes, causing hepatomegalya, splenomegaly and neurological damage

GSD III/Cori disease

Deficiency in debranching enzyme(alpha-1,6-glucosidase) Patients have glycogen chains with many short branches, present with hepatomegaly and light hypoglycemia

GSD 0

Deficiency in glycogen synthase Glucose cannot be stored as glycogen and patients become dependent on dietary glucose Suffer from fasting hypoglycemia and muscle cramps due to lack of muscle glycogen

GSD VI/Hers disease

Deficiency in liver glycogen phosphorylase Prevents glycogen breakdown leading to hepatomegaly and low blood glucose levels

GSD II/Pompe dsease

Deficiency in lysosomal acid maltase(alpha-glucosidase) Autosomal recessive, only GSD that that results in lysosomal accumulation of glycogen Accumulation of glycogen disrupts normal function of liver and muscle Muscle weakness(myopathy) in cardiac and skeletal Children tend to die in infancy

GSD V/McArdle disease

Deficiency in muscle glycogen phosphorylase, which is the rate-limiting enzyme of glycogen degradation, tasked with cleaving the glucose from glycogen releasing G1P Without the ability to provide glucose to muscles these patients suffer from fatigue, muscle cramps and muscle breakdown(myoglobinuria) marked by myoglobin in the urine during intense exercise Treated with reduced strenuous activity

GSD IV/Andersen disease

Deficiency of glucosyl (4:6) transferase, the enzyme which creates branches(alpha-1,6 bonds) Presents with long chains with fewer than normal branches Manifests with hepatosplenomegaly, liver cirrhosis and death within 5 years of birth

Thiamine; vitamin C; niacin; vitamin A, vitamin D; vitamin B12 (due primarily to a deficiency of intrinsic factor), both B12 and folate; vitamin K.

Deficiency of which vitamin results in: beri-beri; scurvy; pellagra; night blindness; rickets or osteomalacia; pernicious anemia; megaloblastic anemia; bleeding?

The correct answer is (E). 1. Nitrogens 3 and 9 2. Carbons 2 and 8 3. Nitrogen 1 4. Atoms 4, 5, and 7

Direct sources of purine ring atoms in the de novo synthesis of IMP include: 1. Glutamine 2. A component of the tetrahydrofolate one-carbon pool 3. Aspartate 4. Glycine A. 1, 2 and 3 B. 1 and 3 C. 2 and 4 D. 4 only E. All four

C.

Drinking grapefruit juice while taking statins can lead to potentially devastating side effects. This is caused by a component of grapefruit juice doing which of the following? a. Interfering with hepatic uptake of statins b. Accelerating the conversion of the statin to a more toxic form c. Inhibiting the inactivation of statins d. Upregulating the HMG-CoA reductase e. Downregulating the HMG-CoA reductase

The answer is C. Grapefruit juice contains a component that blocks CYP3A4 activity, which is the cytochrome P450 isozyme that converts statins to an inactive form. If the degradative enzyme is inhibited, statin levels rise above normal, accelerating their damage of muscle cells. Grapefruit juice does not affect hepatic uptake of the drug (thus, A is incorrect), nor is it accelerating statin metabolism (thus, B is also incorrect). Although HMG-CoA reductase is the target of the statins, grapefruit juice neither upregulates nor downregulates the amount of enzyme present in the cell (thus, D and E are incorrect).

Drinking grapefruit juice while taking statins can lead to potentially devastating side effects. This is caused by a component of grapefruit juice doing which of the following? A. Interfering with hepatic uptake of statins B. Accelerating the conversion of the statin to a more toxic form C. Inhibiting the inactivation of statins D. Upregulating the HMG-CoA reductase E. Downregulating the HMG-CoA reductase

The answer is E. During digestion of a mixed meal, starch and other carbohydrates, proteins, and dietary triacylglycerols are broken into their monomeric units (carbohydrates into simple monosaccharides, protein into amino acids, triacylglycerols into fatty acids and glycerol). Glucose is the principal sugar in dietary carbohydrates, and thus it increases in the blood. Amino acids and monosaccharides enter the portal vein and go to the liver first. After digestion of fats and absorption of the fatty acids, most fatty acids are converted back into triacylglycerols and subsequently into chylomicrons by intestinal cells. Chylomicrons go through lymphatic vessels and then blood, principally to adipose tissue.

During digestion of a mixed meal, which one of the following is most likely to occur? A. Starch and other polysaccharides are transported to the liver. B. Proteins are converted to dipeptides, which enter the blood. C. Dietary triacylglycerols are transported in the portal vein to the liver. D. Monosaccharides are transported to adipose tissue via the lymphatic system. E. Glucose levels increase in the blood.

A.

During periods of strenuous exercise there is a build-up of lactic acid in the muscle. In order to relieve the stress of lactic acid buildup, a pathway known as the Cori Cycle exists. Lactate dehydrogenase is a part of this cycle in the muscle and liver. The relationship between the lactate dehydrogenase in the two tissues is that they A. Have different catalytic properties B. Can only catalyze the formation of pyruvate C. Have identical catalytic properties D. Can only utilize NADH E. Are the products of the same gene

C.

During periods of strenuous exercise there is a build-up of lactic acid in the muscle. In order to relieve the stress of lactic acid buildup, a pathway known as the Cori Cycle exists. The metabolic function of the Cori Cycle is to: A. Recycle lactate in the muscle only B. Carry lactate from liver to muscle C. Carry lactate from muscle to liver D. Regenerate NAD+ in the liver E. Generate a proton gradient

Which of the following is the major excretory by-product of the catabolism of norepinephrine? A. Normetanephrine B. 3,4-Dihydroxymandelic acid C. 3-Methoxy, 4-hydroxyphenylacetic acid D. 3-Methoxy, 4-hydroxyphenylglycol E. 3-Methoxy, 4-hydroxymandelic acid

E. 3-Methoxy, 4-hydroxymandelic acid The correct answer is E. NE is degraded by COMT or MAO. Methylation and oxidation may occur in any order. Methylated and oxidized derivatives of norepinephrine and epinephrine are produced, and 3-methoxy-4-hydroxymandelic acid is the final product. These compounds are excreted in the urine.

Which of the following statements is/are true? A. Parathyroid hormone is produced by the parathyroid glands located within the thyroid gland, and calcitonin is produced by the clear cells within the thyroid gland. B. The main targets for parathyroid hormone are the kidney and bone, while the main target for calcitonin is the bone osteoclasts. C. The main actions of parathyroid hormone are to raise plasma calcium and lower plasma phosphate levels, while the main action of calcitonin is to lower the plasma calcium level. D. A and B only. E. A, B and C.

E. A, B and C. A. Parathyroid hormone is produced by the parathyroid glands located within the thyroid gland, and calcitonin is produced by the clear cells within the thyroid gland. B. The main targets for parathyroid hormone are the kidney and bone, while the main target for calcitonin is the bone osteoclasts. C. The main actions of parathyroid hormone are to raise plasma calcium and lower plasma phosphate levels, while the main action of calcitonin is to lower the plasma calcium level.

Carbamoyl phosphate is a key substance in: A. purine biosynthesis. B. pyrimidine biosynthesis. C. urea biosynthesis. D. A and B. E. B and C.

E. B and C.

The hydrophilic character of which of the following in bilirubin catabolism enables it to be water soluble, cross the canalicular membrane and enter bile? A. Z-bilirubin B. Urobilinogen C. Photobilirubin-EZ D. Urobilin E. Bilirubin diglucuronide

E. Bilirubin diglucuronide

Of the following enzymes involved in nucleotide metabolism, which is correctly paired with its pharmacological inhibitor? A. Xanthine oxidase-probenecid B. Thymidylate synthase-hydroxyurea C. Ribonucleotide reductase-5-fluorouracil D. IMP dehydrogenase-allopurinol E. Dihydrofolate reductase-methotrexate

E. Dihydrofolate reductase-methotrexate Methotrexate interferes with folate metabolism by acting as a competitive inhibitor of dihydrofolate reductase, which starves cells of tetrahydrofolate and thus is unable to synthesize purine and dTMP. Ribonucleotide reductase is inhibited by hydroxyurea, while thymidylate synthase is inhibited by 5-fluorouracil. Probenecid increases renal excretion of urate and does not inhibit production. Xanthine oxidase is inhibited by allopurinol.

The interaction between fibrinogen and thrombin produces an unstable network of fibrin as an initial blood clotting product. What coagulation factor will induce fibrin crosslinking to promote clot stability? A. Factor V B. Factor Va C. Factor VIII D. Factor XIII E. Factor XIIIa

E. Factor XIIIa Activated Factor XIIIa induces fibrin crosslinking.

The drug methotrexate is used as a therapy for rheumatoid arthritis, and has the side effects related to a competitive inhibition of dihydrofolate reductase. These side effects can be decreased by giving leucovorin (folinic acid), which is an active form of folic acid, 5-formyl THF that does not require DHF reductase. However, what would happen if this patient treated with methotrexate is given sufficient folate? A. DHF reductase will have same Km for dihydrofolate as it would in the absence of methotrexate. B. DHF reductase will have high Vmax than it would in the absence of methotrexate. C. DHF reductase will have a lower Km for dihydrofolate. D. DHF reductase will increase its affinity for methotrexate. E. DHF reductase will have the same Vmax as it would in the absence of methotrexate. F. DHF reductase will have a higher Km for dihydrofolate.

E. Folate (as 7,8 dihydrofolate) is converted to tetrahydrofolate by DHF reductase. Methotrexate is a competitive inhibitor of DHF reductase, therefore providing sufficiently high concentrations of folate will allow enzyme to reach same Vmax as obtained in the absence of the inhibitor.

Many food components, including curcumin, cinnamon, green tea, red grapes, garlic and broccoli, induce the body's antioxidant defense system by increasing the synthesis antioxidant genes. Which enzyme is not involved in the antioxidant defense system of the body? A. Superoxide dismutase B. Heme oxidase C. Glutathione S-transferase D. Catalase E. Glutamine synthetase F. Glutathione synthase

E. Glutamine synthetase Glutamine synthetase captures ammonia and to glutamine from glutamate.

What is required for STAT (signal transducers and activators of transcription) to respond to a cytokine signal? A. Binding of cytokine to intracellular receptor B. Dissociation of the receptor subunits C. Dissociation of the STAT subunits D. Formation of cAMP E. Phosphorylation of the cytokine receptor

E. Phosphorylation of the cytokine receptor STAT is part of the JAK-STAT pathway. A tyrosine kinase receptor has intracellular tyrosine kinase activity. Upon activation by a cytokine, the kinase induces phosphorylation which begins downstream signaling pathways. A describes a lipophilic signaling molecule, such as a steroid, which binds its receptor intracellularly. Options B and D describe Gs protein reception.

Which of these coagulation factors DOES NOT have serine protease activity? A. Factor II (prothrombin) B. Factor VII C. Factor X D. Factor XII E. Protein S

E. Protein S

Why does a polymorphism in CYP2D6 decrease the efficacy of codeine? A. Alters the number of AhR B. Decreases uptake of codeine by the brain C. Increases excretion of codeine D. Increases Phase II metabolism E. Slows formation of morphine

E. Slows formation of morphine The hepatic CYP2D6 enzyme metabolizes a quarter of all prescribed drugs, including codeine. CYP2D6 converts codeine in to its active metabolite, morphine, which provides its analgesic effect. However, pain relief may be inadequate in individuals who carry two inactive copies of CYP2D6 ("poor metabolizers"), because of reduced morphine levels. In contrast, individuals who carry more than two functional copies of the CYP2D6 gene ("ultrarapid metabolizers") are able to metabolize codeine to morphine more rapidly and more completely. As a result, even with normal doses of codeine, these individuals may experience the symptoms of morphine overdose, which include extreme sleepiness, confusion, and shallow breathing." Reference: Dean L. Codeine Therapy and CYP2D6 Genotype. 2012 Sep 20 [Updated 2016 Mar 8]. In: Medical Genetics Summaries [Internet]. Bethesda (MD): National Center for Biotechnology Information (US); 2012-. Available from: https://www.ncbi.nlm.nih.gov/books/NBK100662/ The question here reflects what happens in poor metabolizers.

A 30-year-old woman has a macrocytic anemia with hypersegmented neutrophils. Additional history is not available. Physical examination reveals decreased vibratory sensation in the lower extremities. Which of the following is a potential cause of these findings? A. Alcohol abuse B. Oral contraceptives C. Phenytoin D. Pregnancy E. Terminal ileal disease

E. Terminal ileal disease The correct answer is E. Vitamin B12 and folic acid are both involved in DNA synthesis. Deficiency of either vitamin produces a macrocytic anemia with hypersegmented neutrophils. However, vitamin B12 is also involved in the conversion of methylmalonyl CoA to succinyl CoA in odd chain fatty acid metabolism. Deficiency of vitamin B12 causes a buildup of methylmalonyl CoA and its precursor, propionyl CoA, leading to demyelination in the posterior column and lateral corticospinal tract of the spinal cord. Decreased vibratory sensation occurs in posterior column disease, while muscle spasticity is seen in lateral corticospinal tract disease. In vitamin B12 metabolism, intrinsic factor produced by parietal cells in the stomach complexes with vitamin B12 in the duodenum. The complex is then reabsorbed in the terminal ileum. Therefore terminal ileal disease (e.g., Crohn's disease) is a cause of vitamin B12 deficiency.

A 40-year-old man with a history of myocardial infarction is instructed by his physician to take aspirin prophylactically. What eicosanoid is most likely to be reduced in his blood that may cause ischemia? A. Leukotriene A4 B. Lipoxin B4 C. Prostaglandin E2 D. Prostaglandin F2alpha E. Thromboxane A2

E. Thromboxane A2 Thromboxane A2, which stimulates thrombocyte aggregation and causes vasoconstriction, thus reducing blood flow to the site of a clot and may cause ischemia in the coronary circulation. LTA4 synthesis is not inhibited by aspirin nor is Lipoxin B4 production. PGE2 is not involved directly in the pathogenesis of myocardial infarction because it causes vasodilation and smooth muscle relaxation and is used to induce pregnancy labor. PGF2alpha is not involved in myocardial infarction but stimulates uterine contractions.

Serum obtained from a blood specimen from a 52-year-old man with pancreatitis is noticed to be uniformly turbid. After standing in a refrigerator overnight, the appearance is unchanged. Which lipoprotein is most likely to be present in excess? A. Chylomicrons B. High density lipoproteins (HDL) C. Intermediate density lipoproteins (IDL) D. Low density lipoproteins (LDL) E. Very low density lipoproteins (VLDL)

E. Very low density lipoproteins (VLDL) VLDL have the same density as plasma and remain in suspension. An increase in plasma VLDL is a frequent finding in acute pancreatitis. Chylomicrons are less dense than plasma and float to the surface. HDL particles are too small to scatter light and do not cause the plasma to appear turbid. IDL are only slightly denser than plasma and can remain in suspension but are an uncommon cause of plasma turbidity. LDL particles are too small to scatter light and do not cause the plasma to appear turbid.

This enzyme first recognizes allopurinol as a substrate and is then inhibited by the product of enzyme catalysis. A. PRPP synthase B. HGPRT (hypoxanthine-guanine phosphoribosyl transferase) C. Adenosine deaminase D. PRPP amidotransferase E. Xanthine oxidase

E. Xanthine oxidase

Genetic polymorphism in genes for cytochrome P450: A. could cause an individual to poorly metabolize certain drugs. B. could cause an individual to more rapidly metabolize certain drugs. C. could cause a specific ethnic group to experience more effects with certain drugs. D. could lead to greater adverse drug reactions. E. all of the above.

E. all of the above. A. This occurs if the allelic variants code for inactive or less active protein, B. If there is a gene duplication, an individual may have larger amounts of active protein, C. There may be greater expression of specific allelic variants in specific ethnic populations, D. Dependent on the drug and a person polymorphic profile of P450 adverse interactions with specific drugs can occur.

Pathogens produce fever by: A. decreasing the production of interleukin-1 (IL-1). B. decreasing the set-point temperature in the hypothalamus. C. inducing non-shivering response D. causing vasodilation of blood vessels in the skin. E. causing shivering.

E. causing shivering. Pathogenic organisms cause release of IL-1 from phagocytic cells that increase the production of prostaglandins, which raise the temperature set point in the anterior hypothalmus. The hypothalmus thinks the body temperature is too low because the core temperature is lower than the new set point and thus initiates a mechanism to generate heat such as shivering, vasoconstriction and shunting of blood from venous plexus near skin surfaces.

In diabetic nephritis, kidney failure is MOST LIKELY the result of the reaction of glucose with: A. hemoglobin A. B. apolipoprotein C. C. actin. D. Na+/K+ ATPase E. collagen IV.

E. collagen IV. High amounts of sugar bind to collagen IV which damage the capillaries in the glomeruli.

A diet consisting almost exclusively of protein will initiate insulin secretion of β-cells of the pancreas by activating: A. enolase. B. adenylate cyclase. C. glucokinase. D. phospholipase C. E. glutamate dehydrogenase.

E. glutamate dehydrogenase. When the ATP:ADP ratio rises, the beta cells secrete insulin. Amino acids from the protein meal are converted to alpha-ketoglutarate, this raises the ATP:ADP ratio and insulin is released.

All of the following effects can be produced by elevated levels of glucocorticoids EXCEPT: A. a decrease of muscle protein. B. elevated tyrosine transaminase. C. elevated transport of amino acids into the liver. D. increased activities of enzymes of the urea cycle. E. increased net conversion of glucose to amino acids.

E. increased net conversion of glucose to amino acids. The opposite occurs. Glucocorticoids cause protein degradation and conversion of AA to glucose.

Inhibitors of lipoxygenase can be used for the treatment of asthma because they prevent the formation of: A. prostaglandins. B. HETE. C. thromboxanes. D. prostacyclin. E. leukotrienes.

E. leukotrienes. leukotrienes, which are involved in bronchoconstriction, specifically leukotriene D4.

Cytochrome P450 cannot or does not: A. cause carcinogen activation. B. detoxify some toxins. C. facilitate excretion. D. be induced. E. make xenobiotics more lipid soluble.

E. make xenobiotics more lipid soluble. Drug metabolism, including Phase I reactions involving P450, often converts lipophilic compounds into hydrophilic products that are more readily excreted.

The induction of cytochrome P450: A. occurs only by exogenous compounds. B. occurs only at the transcriptional level. C. results from increased transcription of the appropriate mRNA. D. needs the formation of inducer-receptor protein complex. E. may occur by a post-transcriptional process.

E. may occur by a post-transcriptional process. There may be stabilization of mRNA or decrease in the degradation of protein. A. Both endogenous and exogenous substances can induce P450, C. Transcriptional induction is only one mechanism to increase P450. D. This has been shown with induction by some chemicals but not all.

A common intermediate in the biosynthesis of both purine and pyrimidine nucleotides is: A. hypoxanthine. B. phosphoribosylamine. C. inosinic acid. D. orotidine 5-phosphate. E. phosphoribosyl pyrophosphate (PRPP).

E. phosphoribosyl pyrophosphate (PRPP).

Thyroxine: A. is synthesized by iodination of free tyrosine molecules in the thyroid gland. B. is the only hormone synthesized by the thyroid gland. C. binds to plasma membrane receptors and activates adenylate or guanylate cyclase. D. is transported to its target tissue free in solution in the blood. E. synthesis requires iodination of a cellular protein in the thyroid.

E. synthesis requires iodination of a cellular protein in the thyroid. Thyroxine synthesis requires iodination of thyroglobulin. Calcitonin is also made by the thyroid. Thyroxine is transported to tissues bound to proteins.

The thyroid hormone found in highest concentration in blood is: A. thyroglobulin. B. monoiodotyrosine. C. diiodotyrosine. D. triiodothyronine. E. thyroxine.

E. thyroxine. Thyroxine = T4. T4 is more abundant in the blood than T3.

Answer is B. Chylomicrons are the lipoprotein particles formed in intestinal epithelial cells from dietary fats, and they contain principally triacylglycerols formed from components of dietary triacylglycerols. A decreased intake of calories in general would include a decreased consumption of fat, carbohydrate, and protein, which might not lower chylomicron levels. Dietary cholesterol, although found in chylomicrons, is not their principal component.

Elevated levels of chylomicrons were measured in the blood of a patient. A dietary therapy, which decreased which one of the following answer choices would be most helpful in lowering chylomicron levels? A. Overall calories B. Fat C. Cholesterol D. Starch E. Sugar

Gibbs free energy

Energy available to do useful work, difference in G between products and reactants Rate of reaction is based on catalyst enzymes not the deltaG

When cortisol binds to its receptor, which of the following events occur to stimulate gluconeogesis?

Enhanced transcription of regulatory enzymes

Sweeteners erythritol, xylitol, mannitol, sorbitol

Erythritol- 4C sugar alcohol, artificial sweetener, little is metabolized thus little energy, less tooth decay, less flatulence due to small intestine absorption Xylitol- 5C sugar alcohol, natural sweetener, low glycemic index(BS) and lower energy content than sucrose, same sweetness as sucrose Mannitol- sugar alcohol used for energy storage by plants, treats head trauma, kidney failure, heart-lung machine primer, and to make BBB permeable to drugs, can be inhaled to osmotically liquify mucus in CF patients, low glycemic index, tooth-friendly, half as sweet as sucrose and in high doses causes flatulence Sorbitol- sweetener in chewing gum, toothpaste and mouthwash, also as a laxative

C.

Ethanol itself is toxic in high quantities. Mixed with phenobarbital, creates an extremely lethal combination. Why is this combination toxic? a. When the two mix a deadly carcinogen is formed b. Phenobarbital make the drinker so sleepy they don't realize how much they are drinking and will over-consume more ethanol than they normally would c. Ethanol is an inhibitor of phenobarbital-oxidizing P450 system. If large amounts of ethanol and phenobarbital are consumed simultaneously, toxic levels of the barbiturate accumulate e. More CO2 is produced in the metabolism of alcohol than the lungs can handle

E.

Ethanol metabolism can potentially lead to hypoglycemia as a consequence of which of the following: a. The conversion of ethanol to acetaldehyde increase the NADH/NAD+ ratio b. The reduction of pyruvate to lactate is favored in the liver c. Impaired gluconeogenesis d. Malnourished e. All of the above

Ammonia toxicity

Excessive ammonia due to urea cycle disorder or liver failure can have toxic effects on brain and CNS NH3 is toxic due to its permeability to membranes pH imbalance, causes astrocyte swelling leading to cerebral edema and intracranial HTN Glutamate DH catalyzes deamination of glutamate alpha-ketoglutarate causing inhibition of the TCA cycle thus no ATP Postsynaptic excitatory proteins are inhibited depressing CNS Depletion of glutamate results in disruption of its neurotransmitter activity(GABA) Ammonia causes mitochondrial dysfunction

α-Glucosidase inhibitors prevent glucose production from those products of carbohydrate digestion in which glucose is attached through an α-glycosidic linkage, thus reducing the postprandial rise in blood glucose. Note that the digestion of lactose is unaffected because it has a β-linkage.

Explain why drugs that inhibit the α-glucosidase activity of the intestinal saccharidases aid in glycemic control in patients with diabetes.

What would happen to DHF reductase if this patient on methotrexate was given folinic acid? A. DHF reductase will have same Km for dihydrofolate as it would in the absence of methotrexate. B. DHF reductase will have high Vmax than it would in the absence of methotrexate. C. DHF reductase will have a lower Km for dihydrofolate. D. DHF reductase will increase its affinity for methotrexate. E. DHF reductase will have the same Vmax as it would in the absence of methotrexate. F. DHF reductase will have a higher Km for dihydrofolate.

F. DHF reductase will have a higher Km for dihydrofolate. Folinic acid bypasses DHF reductase supplying THF, therefore the methotrexate would compete with endogenous folate for catalytic site of DHF reductase thus increasing enzyme Km.

The answer is D. Hepatocellular disease reduces protein synthesis in the liver, which leads to reduced levels of both LCAT and hepatic triglyceride lipase being produced (thus, A and C are incorrect). Because LCAT activity is reduced, cholesterol ester formation in circulating particles is reduced (thus, B is incorrect). Because a diseased liver has trouble synthesizing glucose, fatty acid release from adipocytes is increased to provide energy (thus, E is incorrect). Serum triacylglycerol levels are increased as a result of the reduced hepatic triglyceride lipase activity; LPL activity is also reduced in liver disease.

Fairly predictable changes occur in the various metabolic pathways of lipid metabolism in patients with moderately advanced hepatocellular disease. Which one of the following changes would you expect to see under these conditions? A. The activity of plasma lecithin cholesterol acyltransferase (LCAT) is increased. B. Serum cholesterol esters are increased. C. Hepatic triglyceride lipase (HTGL) activity is increased. D. Serum triacylglycerol levels are increased. E. Serum nonesterified fatty acid levels are decreased.

SCID

Fatal genetic disorder affecting both B and T cells (adaptive) X-linked(males) ADA deficiency thus adenosine build up in these cells, leading to ADP and AMP build up, then dADP then dATP This rising level lowers the production of other dNDPs as well as the dNTPs impairing DNA synthesis in lymphocytes

Energy from glucose vs FA

Fats are more energy dense than protein and carbs Glucose-36 or 38 ATP FA(16C)-129 ATP

Carb metabolism fed vs. fasting

Fed=abundant glucose thus energy storage Fasting= low glucose thus storage depletion Initiated by insulin/glucagon levels

C.

Feedback inhibition of pyrimidine nucleotide synthesis can occur by which of the following means? a. Increased activity of carbamoyl phosphate synthetase b. Increased activity of aspartate transcarbamoylase c. CTP allosteric effects d. UMP competitive inhibition e. TTP allosteric effects

Fuel supplies during fasting and starvation

First few hours of fasting- nergy is form glucose then glycogen storages 1 day fasting- TAGs are energy which undergo beta-oxidation 3 day fasting- ketone bodies made in the liver and proteins are broken down, glycerol from TAGs and glucogenic AA from proteins enter gluconeogenesis and supply energy to brain and RBCs 1-2 weeks of starvation- ketone bodies are main energy source 2-3 months of starvation- TAGs depleted, proteins main source Coma and death

Rat poison inhibits the TCA cycle

Fluoroacetate reacts with CoA for torm fluoroacetyl CoA which condenses with citrate and is a competitive inhibitor of aconitase which catalyzes citrate to isocitrate leading to an increase in citrate concentration and inhibiting the TCA cycle Fluoroacetyl CoA also inhibits PFK-1 thus shutting down glycolysis

The answer is e. (Murray, pp 130-135, 249-263. Scriver, pp 3897-3944.) Of the indicated amino acids and catabolic intermediates—the serine to glycine, glycine to carbon dioxide and ammonia, and formaminoglutamate (figlu—from histidine) to glutamate reactions—all require tetrahydrofolate as a cofactor. In folate deficiency, the figlu accumulates abnormally and is an unusual metabolite that serves as a diagnostic marker. Methyltetrahydrofolate and vitamin B12 (cobalamin) are required for conversion of homocysteine to methionine (with accompanying conversion of methyl- to tetrahydrofolate, so vitamin B12 deficiency can "trap" folate reserves as methyltetrahydrofolate and cause secondary folate deficiency. Mutations in the converting enzyme methionine synthase can cause one form of homocystinuria (236200) with connective tissue disease resembling Marfan syndrome. Certain of these mutations can be ameliorated by folate and B12 supplementation to augment residual enzyme activity.

Folic acid deficiency may present with megaloblastic anemia or be diagnosed by its effect on certain metabolic pathways. Which of the following amino acids and their catabolic products require folate along with an intermediate that, when elevated in serum, indicates folate deficiency? a. Serine, glycine, carbon dioxide b. Cystine, cysteine, mercaptopyruvate c. Glycine, alanine, pyruvate d. Threonine, acetaldehyde, acetate e. Histidine, formaminoglutamate, glutamate

States of tissue hypoxia lead to high lactate levels. The level of oxygen delivery to tissues is proportional to the product of three quantities: the hemoglobin level, the level of oxygen saturation of the hemoglobin, and the level of blood flow or cardiac output. In the case above, the most obvious cause of hypoxia is low blood flow due to hypovolemia. The hypoxic cell turns to anaerobic glycolysis for its source of energy. Because there is little blood flow to the cell, the lactate and hydrogen ions produced during anaerobic glycolysis cannot exit the cell through the symport gate. Because of the low amount of oxygen, the level of ATP is low as well. A high hydrogen ion concentration is inhibitory and a low ATP level is stimulatory to the central enzyme of glycolysis, 6-phosphofructo-1-kinase. The ATP signal is stronger than the hydrogen ion signal so that the cell continues to carry out glycolysis under anaerobic conditions and hence continues to produce more lactate and hydrogen ions to the eventual death of the cell. So initially when the blood flow is low, the blood lactate levels remain low, when the blood flow is restored then lactate levels rise as the increased tissue perfusion can carry away the lactate produced. The lactate levels remain high after the second episode of hypotension because there is no to little blood flow to wash away the lactate already produced.

For laboratory rotation you are assigned to ER to collect blood lactate levels from patients arriving in ER in likely hypovolumic shock. The lactate levels are to be obtained at clinically significant times. A young male is brought to ER by ambulance. Emergency Medicine Technicians found him face up in a pool of blood on the sidewalk in front of a bar. They scoop and run. He has been down approximately 20 minutes. Unresponsive to deep pain his pupils are equal and reactive to light; breathing is agonal; carotid pulse is palpable but neither radial pulse is. Central lines are started above and below the chest; the patient is intubated and ventilated manually by Ambu bag at an inspired oxygen concentration of 100%. A urinary Foley catheter is inserted. Visual inspection of the anterior aspect of the body reveals a large wound in the right femoral triangle oozing dark blood. Manual pressure is applied here. Upon insertion of the central lines you obtain the 1st lactate blood level (specimen A). 3 liters of warm lactated ringers solution have been infused in the last 10 minutes. The patient is restless and responsive to deep pain with retraction of the limbs. Pupils are equal round and responsive to light. Pulses are palpable throughout; blood pressure is 120 mm Hg systolic. Urine output is clear in color with 40 mL in the last 10 minutes. The femoral wound is spurting bright red blood. You draw the 2nd blood lactate level (specimen B). Suddenly the patient's blood pressure becomes non-palpable. The femoral wound stops spurting bright red blood. The technician who is manually ventilating with the Ambu bag shouts that he is difficult to ventilate. The central line above the chest is reading 20 cm H2O of pressure (high). The patient is turned on his side revealing a gunshot entrance wound in the left posterior chest wall. You collect the 3rd lactate blood specimen (specimen C) as ER staff prepares the left anterior chest wall for open cardiac massage. OR is re-notified. Given the patient's clinical course, you predict that the lactate levels in blood samples A, B, and C will be A. High, normal, high B. High, normal, normal C. Low, high, high D. Low, high, normal E. Normal, normal, normal

The answer is b. (Murray, pp 130-135, 153-162. Scriver, pp 3897-3964.) Pyruvate carboxylase catalyzes the conversion of pyruvate to oxaloacetate in gluconeogenesis: pyruvate + HCO3− + ATP → oxaloacetate + ADP + Pi In order for pyruvate carboxylase to be ready to function, it requires biotin, Mg2+, and Mn2+. It is allosterically activated by acetyl-CoA. The biotin is not carboxylated until acetyl-CoA binds the enzyme. By this means, high levels of acetyl-CoA signal the need for more oxaloacetate. When ATP levels are high, the oxaloacetate is consumed in gluconeogenesis. When ATP levels are low, the oxaloacetate enters the citric acid cycle. Gluconeogenesis only occurs in the liver and kidneys.

Fully activated pyruvate carboxylase depends on the presence of which of the following substances? a. Malate and niacin b. Acetyl-CoA and biotin c. Acetyl-CoA and thiamine pyrophosphate d. Oxaloacetate and biotin e. Oxaloacetate and niacin

Cytochrome C and apoptosis

Functions as a mobile electron carrier between complex III and IV Cytochrome is released from the mitochondria which initiates caspases Cyt C is marker for apoptosis, using centrifugation and western blot detection

Pentose Phosphate Pathway (PPP)

G6P dehydrogenase

Gallstones

GB is pear shaped and stores bile to emulsify lipids, cholesterol uses bile salts to stay in solution With too much cholesterol and too little bile salt, the cholesterol hardens into pebbles called gallstones Continued disruption can lead to absorption syndromes such as steatorrhea and deficiency in fat-soluble vitamins Oral chendeoxycholic acid can replace bile salts and dissolve gallstones

The answer is a. (Murray, pp 415-433. Scriver, pp 2297-2326.) Plasma membranes are unique as compared to intracellular membranes in that their composition contains cholesterol, glycoproteins, and glycolipids known as gangliosides. Plasma membranes of the cells of different tissues are distinguished from each other because of the properties that make them unique. Hormone receptors allow each cell type to respond to systemic stimulation appropriately. All chronic hormone receptors are localized to plasma membranes and upon stimulation release a second messenger into the interior of the cell. Glucagon, like epinephrine and norepinephrine, stimulates adenylate cyclase to produce cyclic AMP. Glucagon is found on the plasma membranes of liver and adipose tissue cells.

Gangliosides and receptors for hormones such as glucagon can be found in which of the following structures? a. Plasma membrane b. Mitochondria c. Lysosomes d. Endoplasmic reticulum e. Ribosomes

Correct answer = D. The total energy intake is (275 g carbohydrate × 4 kcal/g) + (75 g protein × 4 kcal/g) + (65 g lipid × 9 kcal/g) = 1100 + 300 + 585 = 1,985 total kcal/day. The percentage calories from carbohydrate is 1,100/1,985 = 55; percentage calories from protein is 300/1,985 = 15; and percentage calories derived from lipid is 585/1,985 = 30. These are very close to current recommendations. The amount of fiber or nitrogen balance cannot be deduced from the data presented. If the protein is of low biologic value, a negative nitrogen balance is possible.

Given the information that a 70-kg man is consuming a daily average of 275 g of carbohydrate, 75 g of protein, and 65 g of lipid, one can draw which of the following conclusions? A. Total energy intake per day is approximately 3,000 kcal. B. About 20% of the calories are derived from lipids. C. The diet does not contain a sufficient amount of dietary fiber. D. The proportions of carbohydrate, protein, and lipid in the diet conform to the recommendations of academic groups and government agencies. E. The individual is in nitrogen balance.

Polyol Pathway

Glucose --(Aldose reductase)--> Sorbitol Sorbitol --(Sorbitol dehydrogenase)--> Frucose

Brain and glucose

Glucose is the only fuel molecule that can cross the BBB During starvation the liver makes glucose for the brain Severe starvation, the brain can use ketone bodies

Enzymes that require Se as Cofactor

Glutahione peroxidase (antioxidant)

Glycogenolysis

Glycogen phosphorylase (GP) (rate-limiting enzyme) with pyridoxal phosphate (vitamin B₆) as cofactor Debranching enzyme transferase (4:4)

List of Pathways

Glycolysis Citric Acid Cycle (TCA Cycle) Oxidative Phosphorylation Gluconeogenesis Cori Cycle Polyol Pathway Fructose Metabolism Galactose Metabolism Pentose Phosphate Pathway Glycogenesis Glycogenolysis Fatty Acid Synthesis β-Oxidation Urea Cycle

Glycolysis versus gluconeogenesis

Glycolysis, G6P from glucose by glucokinase/hexokinase, the reverse is done by G6-phosphatase in gluconeogenesis Glycolysis, F6P to F16BP by PFK1, the reverse is done by F16Bphosphatase Glycolysis, PEP to pyruvate by pyruvate kinase, reverse is pyruvate carboxylase and PEP CK

The answer is B. A lack of glucose 6-phosphatase activity (von Gierke disease) leads to an accumulation of glucose 6-phosphate, which leads to an increase in ribose 5-phosphate levels, and then an increase in PRPP levels. As PRPP levels rise, purine synthesis is stimulated, leading to excessive levels of purines in the blood. The degradation of the extra purines leads to uric acid production and gout. A loss of either PRPP synthetase activity or glutamine phosphoribosyl aminotransferase activity would lead to reduced purine synthesis and hypouricemia (thus, A and D are incorrect). A lack of glucose-6-phosphate dehydrogenase would hinder ribose 5-phosphate production and thus would not lead to excessive purine synthesis. A lack of purine nucleoside phosphorylase would hinder the salvage pathway, leading to an accumulation of nucleosides. Purine nucleoside phosphorylase activity is required to synthesize uric acid, so in the absence of this enzyme, less uric acid would be produced (thus, E is incorrect).

Gout can result from a reduction in activity of which one of the following enzymes? A. Glutamine phosphoribosyl amidotransferase B. Glucose 6-phosphatase C. Glucose-6-phosphate dehydrogenase D. PRPP synthetase E. Purine nucleoside phosphorylase

The answer is e. (Murray, pp 293-302.) PRPP synthetase catalyzes the first reaction in purine biosynthesis from ribose-5-phosphate. This enzyme is normally subject to feedback inhibition. Defects in this enzyme or its regulation can result in overproduction and overexcretion of purine catabolites, leading to gout.

Gout is an inflammatory reaction due to uric acid crystallization in soft tissues and joints. One defect responsible for this is in which enzyme of purine biosynthesis? a. Adenylosuccinase b. Formyltransferase c. IMP cyclohydrolase d. PRPP glutamyl amidotransferase e. PRPP synthetase

Gastric proton pump inhibitors

H+/K+ ATPase, located in parietal cells of gastric lumen pumps H+ into lumen where it forms HCl with Cl-(gastric acid) When overproduced drugs are prescribed to inhibit this pump Omeprazole, lansoprazole, esomeprazole Reduction of pumps leads to hypochlorhydria which leads to a decrease in in absorption of nutrients and increase sensitivity to food poisoning and reduced efficacy of gastric enzymes

Several familial disorders are characteristic of defects in the LDL-receptor that inhibit cellular uptake of LDLs and reduce intracellular cholesterol. In these patients all of the following are true except:

HMGCoA reductase is phosphorylated

Defective AA transporter leads to Hartnup disease

Hartnup disease-autosomal recessive Defect in nonpolar/neutral AA transporter(Ala, Val, Thr, Leu, Try) Primarily found in kidneys and intestine Tryptophan deficiency leads to serotonin deficiency Clinical Fx- cerebral ataxia(lack of muscle coordination), photodermatitis, and photosensitivity

Enzymes that require Fe as Cofactor

Heme proteins (hemoglobin and myoglobin) Cytochromes Catalases and peroxidases

Spur cell anemia

Hemolytic anemia Elevated levels of cholesterol in RBC, decreases fluidity RBCs lyse as they pass through capillaries

Aldolase B deficiency

Hereditary fructose intolerance Autosomal recessive Aldolase B in the liver cleaves F1P into DHAP and glyceraldehyde Without fructose and F1P accumulate, this sequesters phosphorus thus phosphorylation cannot occur Leads to low levels of blood phosphorus and glucose levels as well as low tolerance to fructose and sucrose

Glycogenesis

Hexokinase/Glucokinase Phosphoglucomutase UDP-glucose pyrophosphorylase Glycogen synthase (rate-limiting enzyme) Glucosyl (4:6) transferase

Glycolysis

Hexokinase/Glucokinase Phosphoglucose isomerase Phosphofructokinase-1 Aldolase A TPI (TIM): Triose phosphate isomerase Gly-3-P dehydrogenase Phosphoglycerate kinase Mutase Enolase Pyruvate kinase

ATP:ADP and NADH:NAD+ ratios

High ATP ratio is an energy rich state the opposite goes for low ratios NADH ratio is inverse to that of ATP, such that at low ATP ratio NADH ratio is high to provide electrons for the ETC High ATP ratios inhibit TCA cycle and ETC

Differential effects of citrate on rate-limiting enzyme

High citrate is indicative of high ATP levels, consistent with citrate being an allosteric inhibitor of PFK-1(RL) Citrate is also an allosteric activator of ACC the rate limiting step in FA synthesis Citrate helps maintain energy charge at stable level in cells

Gout

High levels of uric acid in the blood, overproduction(Lesch-Nyahan; primary hyperuricemia) or under excrection(secondary hyperuricemia) Deposits of sodium urate in joints cause extreme pain and can damage the kidneys caused by diets rich in purines along with meats and alcohol Acidemia stimulates reabsorption of uric acid in kidney via urate transporter URAT1 Tx involves drugs such as colchicine which decrease granulocyte movement to the area and allopurinol to inhibit XO and increase levels of soluble purines hypoxanthine and guanine(also urate oxidase which converts uric acid into allantoin) old pathway

The answer is c. (Murray, pp 293-302. Scriver, pp 2537-2570.) Purine salvage reactions convert purines, purine ribonucleosides, and purine deoxyribonucleoside to mononucleotides. Such salvage reactions require much less energy than de novo synthesis. The liver is the major site of purine nucleotide biosynthesis and provides excess purines for other tissues that cannot synthesize purines. A defect in hypoxanthine-guanine phosphoribosyl transferase, one of the enzymes of purine salvage, is responsible for purine overproduction and subsequent hyperuricemia observed in Lesch-Nyhan syndrome.

Hyperuricemia in Lesch-Nyhan syndrome is due to a defect in which of the following pathways? a. Purine biosynthesis b. Pyrimidine biosynthesis c. Purine salvage d. Pyrimidine salvage

Two Salvage Purine Pathway Enzymes

Hypoxanthine guanine phosphoribosyl transferase (HGPRT) Adenine phosphoribosyl transferase (APRT)

Hypoxia and ATP preservation

Hypoxic conditions following MI or stroke cause decrease in activity of respiratory chain and proton motive force O2 deprivation causes a boost in glycolysis leading to lactic acidosis Lowered pH in mito matrix leads to dimerization of IF3 that inhibits ATP synthase and prevents it from reversing thus hydrolyzing ATP. So ATP is conserved

The correct answer is (A). 1. The synergistic effect of both AMP and GMP is needed for complete inhibition. 2. This assures that the limited amount of IMP formed will be channeled to the production of the guanine nucleotides. 3. ATP provides the energy for this branch. 4. The formation of dATP is not applicable in this situation.

If a cell has an adequate supply of adenine nucleotides but requires more guanine nucleotides for protein synthesis: 1. Glutamine-PRPP amidotransferase will not be fully inhibited 2. AMP will be a feedback inhibitor of the condensation of IMP with aspartate 3. ATP will stimulate the production of GMP from IMP 4. ATP will inhibit nucleoside diphosphate reductase a. 1, 2 and 3 b. 1 and 3 c. 2 and 4 d. 4 only e. All four

The answer is A. Tyrosine is derived from phenylalanine, which requires BH4 but not vitamin B6. Vitamin B6 is required in the synthesis of serine (transamination), alanine (another transamination), cysteine (beta-elimination, beta-addition, beta-elimination), and aspartate ( transamination).

If an individual has a vitamin B6 deficiency, which of the following amino acids could still be synthesized and be considered nonessential? A. Tyrosine B. Serine C. Alanine D. Cysteine E. Aspartate

PDC in tissue

Important in skeletal, cardiac muscle and adipose tissue Ca2+ released during muscle contraction stimulates PDC by binding PDP(activates PDC) and inhibiting PDK PDP is Mg2+ dependent thus insulin in adipose tissue lower Km of Mg2+ leading to activation of PDP(mediators unknown) In cardiac muscle, PDC is sitmulated by catecholamines such as epi which is mediated by Ca2+

The answer is D: Reduced activity of liver glycogen phosphorylase. Glycogen phosphorylase requires pyridoxal phosphate (derived from vitamin B6) as an essential cofactor. The role of B6 in the mechanism of the phosphorolysis reaction is still controversial, but it may play a role in general acid-base catalysis. B6 is not required for the PEP carboxykinase reaction, the pyruvate carboxylase reaction (which requires biotin), or the glucose-6-phosphatase reaction. B6 is also not involved in regulating insulin secretion.

In a laboratory study, volunteers were made experimentally vitamin B6 deficient, and much to the investigator's surprise, a mild hypoglycemia, with ketosis, was noted. The hypoglycemia is a result of which of the following? (A) Inhibition of phosphoenolpyruvate carboxykinase (B) Inhibition of pyruvate carboxylase (C) Reduced activity of glucose-6-phosphatase (D) Reduced activity of liver glycogen phosphorylase (E) Reduced insulin secretion

The answer is C. The major change during prolonged fasting is that as muscles decrease their use of ketone bodies, ketone bodies increase enormously in the blood and are used by brain as a fuel. However, even during starvation, glucose is still required by the brain, which cannot oxidize fatty acids to an appreciable extent (thus, D is incorrect). Red blood cells can use only glucose as a fuel (thus, B is incorrect). Because the brain, red blood cells, and certain other tissues are glucose dependent, the liver continues to synthesize glucose, and blood glucose levels are maintained at only slightly less than fasting levels (thus, A is incorrect). Adipose tissue stores (approximately 135,000 kcal) are not depleted in a well-nourished individual after 1 week of fasting (thus, E is incorrect).

In a well-nourished individual, as the length of fasting increases from overnight to 1 week, which one of the following is most likely to occur? A. Blood glucose levels decrease by approximately 50%. B. Red blood cells switch to using ketone bodies. C. Muscles decrease their use of ketone bodies, which increase in the blood. D. The brain begins to use fatty acids as a major fuel. E. Adipose tissue triacylglycerols are nearly depleted.

B.

In addition to being incorporated into proteins, amino acids serve as precursors for many different types of biologically active molecules. Among these are the neurotransmitters dopamine, noradrenaline, and histamine. The neurotransmitter histamine is derived from which of the following amino acids? A. Phenylalanine B. Histidine C. Glutamine D. Tyrosine E. Tryptophan

C.

In addition to being incorporated into proteins, amino acids serve as precursors for many different types of biologically active molecules. Among these are the neurotransmitters dopamine, noradrenaline, and serotonin. The neurotransmitter serotonin is derived from which of the following amino acids? A. Lysine B. Tyrosine C. Tryptophan D. Glutamine E. Phenylalanine

The answer is b. (Murray, pp 481-497. Scriver, pp 3897-3964.) Osteomalacia is the name given to the disease of bone seen in adults with vitamin D deficiency. It is analogous to rickets, which is seen in children with the same deficiency. Both disorders are manifestations of defective bone formation. The osteogenesis imperfectas are a group of genetic bone disorders caused by collagen gene mutations. Osteopetrosis is a hardening of the bones that occurs in certain hereditary conditions. Night blindness is associated with vitamin A deficiency.

In adults, a severe deficiency of vitamin D causes which of the following disorders? a. Night blindness b. Osteomalacia c. Rickets d. Osteogenesis imperfecta e. Osteopetrosis

Methotrexate

In humans, antineoplastic agents target dihydrofolate reductase which converts folate into tetrahydrofolate in the liver This inhibits rapidly dividing cancer cells, efficacy depends on cancer cell uptake versus normal cells

A.

In most patients with gout as well as those with Lesch-Nyhan syndrome, purines are overproduced and overexcreted. Yet the hypoxanthine analogue allopurinol, which is effectively treats gout, has no effect on the severe neurological symptoms of Lesch-Nyhan patients because it does not: a. Decrease de novo purine synthesis b. Decrease de novo pyrimidine synthesis c. Diminish urate synthesis d. Increase PRPP levels e. Inhibit xanthine oxidase

C

In nucleotide metabolism, the common precursor of both AMP and GMP is: A. Adenosine triphosphate B. Adenosine monophosphate C. Inosine monophosphate D. Xanthine monophosphate E. Inosine triphosphate

D.

In parts of the world where rice is the main component of the diet, a disease called "beriberi" is common. The addition of a vitamin to the diet can reverse the disease. Which of the following vitamins should be added to the diet of someone with beriberi? A. Vitamin C B. Vitamin B12 C. Riboflavin D. Thiamin E. Folic acid

The answer is C. A decrease in the concentration of ATP (which occurs as muscle contracts) stimulates processes that generate ATP. The proton gradient across the inner mitochondrial membrane decreases as protons enter the matrix via the ATPase to synthesize ATP; NADH oxidation by the electron-transport chain increases to reestablish the proton gradient; fuel use also increases to generate more ATP. Glycogen synthesis is inhibited because of phosphorylation of glycogen synthase as induced by epinephrine. Palmitate is oxidized and glycolysis increases because of the activation of PFK-1 by AMP. As ATP decreases, AMP rises as a result of the myokinase reaction (2ADP → ATP + AMP).

In skeletal muscle, increased hydrolysis of ATP during muscular contraction leads to which of the following? A. A decrease in the rate of palmitate oxidation to acetyl-CoA B. A decrease in the rate of NADH oxidation by the electron-transport chain C. Activation of PFK-1 D. An increase in the proton gradient across the inner mitochondrial membrane E. Activation of glycogen synthase

The answer is c. (Murray, pp 481-497. Scriver, pp 3897-3964.) In the Far East, rice is a staple of the diet. When rice is unsupplemented, beriberi can be manifest, because rice is low in vitamin B1 (thiamine). Thiamine pyrophosphate is the necessary prosthetic group of enzymes that transfers activated aldehyde units. Such enzymes include transketolase, pyruvate dehydrogenase, and α-ketoglutarate dehydrogenase. Beriberi is a wasting disease whose symptoms include pain in the limbs induced by peripheral neuropathy, weak musculature, and heart enlargement. Yeast products, whole grains, nuts, and pork are rich in thiamine. Choline, ethanolamine, and serine are polar head groups of phospholipids. Glycine is a common amino acid.

In the Far East, beriberi is a serious health problem. It is characterized by neurologic and cardiac symptoms. Beriberi is caused by a deficiency of which of the following vitamins? a. Choline b. Ethanolamine c. Thiamine d. Serine e. Glycine

The answer is E. Glycine is required for the synthesis of heme (combining with succinyl-CoA in the initial step), for the synthesis of purine rings (the entire glycine m olecule is incorporated into adenine and guanine), and for creatine, where glycine reacts with arginine in the first step.

In the biosynthetic pathways for the synthesis of heme, creatine, and guanine, which one of the following amino acids directly provides carbon atoms that appear in the final product? A. Serine B. Aspartate C. Cysteine D. Glutamate E. Glycine

The answer is B. In the process of respiration, O 2 is consumed and fuels are oxidized to CO2 and H2O. The energy from the oxidation reactions is used to generate ATP from ADP and Pi. However, a small amount of energy is also released as heat (thus, C is incorrect). Although fuels can be stored as triacylglycerols, this is not part of respiration (thus, A is incorrect). Respiration is a catabolic pathway (fuels are degraded), as opposed to an anabolic pathway (compounds combine to make larger molecules) (thus, E is incorrect).

In the process of respiration, fuels most often undergo which one of the following fates? A. They are stored as triacylglycerols. B. They are oxidized to generate ATP. C. They release energy principally as heat. D. They combine with CO2 and H2O and are stored. E. They combine with other dietary components in anabolic pathways.

B

In the synthesis of the deoxyribonucleotides, the starting molecule is: A. The ribonucleotide monophosphate B. The ribonucleotide diphosphate C. The ribonucleotide triphosphate D. Deoxyribose E. Dependent on the base which makes up the ribonucleotide

Correct answer = C. Transaminase reactions always have an amino acid and an α-keto acid as substrates. The products of the reaction are also an amino acid (corresponding to the α-keto substrate) and an α-keto acid (corresponding to the amino acid substrate). Three amino acid αketo acid pairs commonly encountered in meta bolism are: alanine/pyruvate aspartate/oxaloacetate glutamate/α-ketoglutarate In this question, glutamate is deaminated to form α-ketoglutarate, and oxaloacetate is aminated to form aspartate.

In the transamination reaction shown below, which of the following are the products, X and Y? A. Alanine, α-ketoglutarate. B. Glutamate, α-ketoglutarate. C. Asparate, α-ketoglutarate. D. Pyruvate, aspartate. E. Pyruvate, alanine.

A. A-PRT is not considered to be the major enzyme because we produce very little free adenine. B. Phosphorylated compounds generally don't cross the plasma membrane. C. This enzyme is primarily for the salvage of pyrimidines. D. Adenylate kinase equilibrates ATP, AMP and ADP E. Correct. The IMP formed is converted to AMP by amination with aspartate.

In tissues that do not carry out active de novo synthesis, maintenance of an adequate supply of adenine nucleotides: a. occurs primarily by adenine salvage using APRT b. requires ATP uptake from the blood c. depends upon the action of nucleoside phosphorylase d. is accomplished entirely by the action of adenylate kinase e. involves hypoxanthine salvage using HGPRT

Correct answer = D. The major tissues in which glucose transport requires insulin are muscle and adipose tissue. The metabolism of the liver responds to insulin, but hepatic glucose transport is determined by blood glucose concentration and does not require insulin. Brain, RBC, and lens of the eye have insulin-insensitive uptake of glucose.

In which one of the following tissues is glucose transport into the cell insulin-sensitive? A. Brain. B. Lens of the eye. C. Red blood cells. D. Adipose tissue. E. Liver.

Vitamin A disorders

Inadequate intake or absorption can manifest as night blindness, visual impairment, xerophthalmia(dry eye), and Bitot's spots(keratin in conjunctiva) Deficiencies also results in growth impediment, poor wound healing, dry skin, follicular hyperkeratosis, alopecia, and bronchitis/pnuemonia. Copius amounts result in liver toxicity, and joint pain Isotretinoin in the womb can lead to cleft palate and heart abnormalities

When LDL is taken up by a cell, it will have which of the following impacts to intracellular cholesterol?

Increase in intracellular cholesterol; Decrease on de novo cholesterol synthesis

Correct answer = C. Free fatty acids bound to albumin are increased as a result of an increased activity of hormone-sensitive lipase in adipose tissue. Hepatic ketogenesis is stimulated by elevated levels of glucagon. The formation of acetyl CoA is increased. β-Oxidation of fatty acids in liver provides the acetyl CoA for ketogenesis.

Increased formation of ketone bodies during fasting is a result of: A. decreased levels of circulating glucagon. B. decreased formation of acetyl CoA in the liver. C. increased levels of free fatty acids in blood. D. inhibition of β-oxidation of fatty acids in the liver. E. decreased activity of hormone-sensitive lipase in adipose tissue.

The answer is E: Hypoxanthine and xanthine. As shown below, the target of allopurinol, the enzyme xanthine oxidase, catalyzes two reactions. The fi rst is the conversion of hypoxanthine (which is produced during the degradation of adenine) to xanthine and the second is the conversion of xanthine (which is produced during the degradation of guanine) to uric acid. Thus, in the presence of allopurinol, hypoxanthine accumulates from the degradation of adenine and xanthine accumulates via the guanine degradative pathway. Both of these compounds are more soluble than urate, thus alleviating the major problem in gout.

Individuals with gout are given allopurinol for long-term management of the disease. In such individuals, which of the following bases would accumulate in the urine? (A) Urate and xanthine (B) Guanine and adenine (C) Hypoxanthine and guanine (D) Xanthine and guanine (E) Hypoxanthine and xanthine

E.

Induction of CYP2E1 would result in which of the following? a. A decreased clearance of ethanol from the blood b. A decrease in the rate of acetaldehyde production c. A low possibility of the generation of free radicals d. Protection from hepatic damage e. An increase of one's alcohol tolerance level

The answer is E. An increase in the concentration of CYP2E1 (the MEOS system) would result in an increase of ethanol metabolism and clearance from the blood (thus, A is incorrect). An increased rate of acetaldehyde production would result (thus, B is incorrect). The increase in CYP2E1 would cause an increase in the probability of producing free radicals (thus, C is incorrect). Answer D is incorrect because hepatic damage would be more likely to occur because there is an increase of free radical production. Answer E is correct because the increased clearance rate of ethanol from the blood results in a higher alcohol tolerance level.

Induction of CYP2E1 would result in which of the following? A. A decreased clearance of ethanol from the blood B. A decrease in the rate of acetaldehyde production C. A low possibility of the generation of free radicals D. Protection from hepatic damage E. An increase of one's alcohol tolerance level

The answer is C. In the fed state, insulin will be released because of the increase in blood glucose levels. Insulin will act on muscle cells to increase glucose uptake in the muscle. Insulin will also stimulate the liver to synthesize both glycogen and fatty acids, which leads to enhanced triglyceride synthesis and very low-density lipoprotein (VLDL) production to deliver the fatty acids to other tissues of the body. Insulin will stimulate glucose uptake in fat cells, but does not stimulate fatty acid synthesis in the fat cells (i.e., unique to the liver), but will lead to enhanced triglyceride synthesis in the fat cells.

Insulin release in the fed state will lead to which of the following metabolic changes?

During digestion, what is the role of colipase?

Interact with dietary fats to allow pancreatic lipase to hydrolize triacylglycerols

The answer is c. (Murray, pp 474-480. Scriver, pp 1623-1650.) Certain amino acids and lipids are dietary necessities because humans cannot synthesize them. The energy usually obtained from carbohydrates can be obtained from lipids and the conversion of some amino acids to intermediates of the citric acid cycle. These alternative substrates can thus provide fuel for oxidation and energy plus reduce equivalents for biosynthesis. Iodine is important for thyroid hormone synthesis, whereas calcium is essential for muscle contraction and bone metabolism.

Intestinal bowel resections are necessary for autoimmune inflammatory diseases like Crohn disease and for congenital anomalies like malrotation or volvulus (twisted intestine with impaired blood supply). Once the absorptive intestinal mucosa falls below a certain length, oral or parenteral alimentation must be instituted to maintain nutrition. In such solutions, which of the following nutrients is most dispensable? a. Protein b. Iodine c. Carbohydrates d. Lipids e. Calcium

The answer is d. (Murray, pp 190-196. Scriver, pp 2705-2716.) Infants placed on chronic low-fat formula diets often develop skin problems, impaired lipid transport, and eventually poor growth. This can be overcome by including linoleic acid to make up 1-2% of the total caloric requirement. Essential fatty acids are required because humans have only ∆4,∆5,∆6, and ∆9 fatty acid desaturase. Only plants have desaturase greater than ∆9. Consequently, certain fatty acids such as arachidonic acid cannot be made "from scratch" (de novo) in humans and other mammals. However, linoleic acid, which plants make, can be converted to arachidonic acid. Arachidonate and eicosapentaenoate are 20-carbon prostanoic acids that are the starting point of the synthesis of prostaglandins, thromboxanes, and leukotrienes.

It has been noted that infants placed on extremely low-fat diets for a variety of reasons often develop skin problems and other symptoms. This is most often due to which of the following? a. Lactose intolerance b. Glycogen storage diseases c. Antibody abnormalities d. Deficiency of fatty acid desaturase greater than ∆9 e. Deficiency of chylomicron and VLDL production

The answer is D. Kwashiorkor is a disease that results from eating a calorie-adequate diet that lacks protein. None of the other answers is correct.

Kwashiorkor can result from which of the following? A. Consuming a calorie-deficient diet that is also deficient in protein. B. Consuming a calorie-adequate diet that is deficient in carbohydrates. C. Consuming a calorie-adequate diet that is deficient in fatty acids. D. Consuming a calorie-adequate diet that is deficient in proteins. E. Consuming a calorie-deficient diet that is primarily proteins.

Lactose intolerance

Lactase, normally in small intestines, hydrolyzes disaccharide lactoses into glucose and galactose, genetic defect leads to primary intolerance, infants of all races have lactase but the adults of many races do not, this reduction in enzyme leads to secondary lactose intolerance Within 2 hours of a few ounces of milk these patients experience diarrhea and intestinal gas caused by lactose fermentation

Fatty Acid Synthase (FAS)

Large multi-enzyme complex. Composed of 2 identical dimers (260 kDa each). Two dimers arranged in head to tail conformation Each has 7 enzyme activities and an acyl carrier protein (ACP).

Lead poisoning chelating agents

Lead inhibits ALA dehydratase and ferrochelatase which are involved in biosynthesis of heme i.e. Blood Patients with lead poisoning may present with abd pain, sideroblastic anemia, irritability, HA, nervous system development impairment and encephalopathy Tx includes Ca-EDTA with dimercaprol, lead has higher affinity that Ca for EDTA thus replaces it an is excreted in the urine In children Tx is succimer a chelating agent(dimercaptosuccinic acid)

The answer is C. The enzyme defect in Lesch-Nyhan syndrome involves hypoxanthine-guanine phosphoribosyltransferase (HGPRT). This enzyme converts the free base to a nucleotide—specifically, guanine to GMP and hypoxanthine to IMP. Adenine phosphoribosyltransferase (APRT) converts adenine to AMP (thus, A is incorrect). Adenosine kinase converts adenosine to AMP (thus, B is incorrect). There are no enzymes to convert guanosine to GMP or thymine to TMP (thus, D and E are incorrect). Pyrimidine nucleoside phosphorylase will convert thymine to thymidine but not to the nucleotide level. Thymidine kinase converts thymidine to TMP (thus, F is incorrect).

Lesch-Nyhan syndrome is due to an inability to catalyze which of the following reactions? A. Adenine to AMP B. Adenosine to AMP C. Guanine to GMP D. Guanosine to GMP E. Thymine to TMP F. Thymidine to TMP

The answer is d. (Murray, pp 242-263. Scriver, pp 1667-1724.) Amino acid degradation ultimately leads to the formation of ammonium ion (NH4+), which is toxic in significant amounts. In the liver of humans, as in most terrestrial vertebrates, NH4+ is produced and converted into urea for excretion. For many amino acids, the conversion of α-amino groups into ammonium ion and then into urea is carried out by two groups of enzymes. Transaminases (aminotransferases) transfer α-amino groups to α-ketoglutarate to form glutamate, which is then oxidatively deaminated by glutamate dehydrogenase to release free ammonium ions that can be converted to urea.

Liver aminotransferases, which are also called transaminases, catalyze the transfer of α-amino groups from many different amino acids to α-ketoglutarate. The intermediate produced is deaminated back to αketoglutarate with the formation of ammonium ion. The structure of α-ketoglutarate is shown below. Which of the following is the intermediate produced? a. Aspartate b. Alanine c. Oxaloacetate d. Glutamate e. Pyruvate

Liver versus muscle glycogen phosphorylase

Liver mutation causes Hers disease(GSD IV), regulated by free glucose but not AMP Muscle mutation causes McArdle syndrome(GSD V), regulated by AMP Both allosterically affectd by G6P and ATP, and PhK activating site A third form is in brain astrocytes lacks PhK activating site but is regualted by AMP, brain has small amounts of glycogen

Carnitine palmitoyltransferase II (CPT-II)

Located in the inner mt-membrane Transfers Fatty Acyl from FA-carnitine to CoA Forms FA-CoA FA-CoA is now in the mt-Matrix

Fatty Acyl CoA Synthetase

Located on the outer mito-membrane Reaction catalyzed on cytoplasmic side of membrane Activates LCFAs using ATP as an energy source Forms a thioester bond between FAs and CoA to form FACoA

Hypoxia

Low O2 concentration in the body tissues Local due to vascular occlusion and in the whole body at high altitudes or when cardiac output is decreased Compensation is done through anaerobic respiration leading to build up of lactate thus lactic acidosis

D.

Macrocytic anemia and spina bifida are symptoms of which deficiency? A. Pantothenic acid B. Vitamin B2 (riboflavin) C. Vitamin B12 D. Folate (folic acid) E. Vitamin C

The correct answer is (E). 1 and 2. Both of these are inhibitors of gln-PRPP amidotransferase and their effect is synergistic. 3. Since the kinetics of gln-PRPP amidotransferase are sygmoidal with respect to PRPP, and the concentrations are in the steep part of the curve, [PRPP] can significantly influence the rate. 4. GTP provides the energy for the branch from IMP to AMP.

Major controls of de novo AMP synthesis include: 1. allosteric inhibition by GMP 2. allosteric inhibition by AMP 3. availability of PRPP 4. stimulation by GTP a. 1, 2 and 3 b. 1 and 3 c. 2 and 4 d. 4 only e. All four

Acid-base balance of the kidneys

Major role of the kidneys is to regulate blood pH due to their ability to remove H+ from blood in form of NH4+ and to reabsorb bicarbonate ions HCO3- Low pH triggers removal of H+ and absorption of HCO3- High pH less removal of H+ and less absorption of HCO3-

Following a meal, the ratio of insulin to glucagon will be > 1. Which of the following compounds will be elevated to inhibit carnitine palmitoyl transferase I?

Malonyl CoA

The answer is C: Ribonucleotide reductase. Deoxyguanosine would be first acted on by purine nucleoside phosphorylase, which would produce guanine and deoxyribose-1-phosphate. The guanine would be converted to GMP by HGPRT, and the GMP phosphorylated to GDP. The GDP would be reduced by ribonucleotide reductase to dGDP, which is then phosphorylated again to produce dGTP. Since guanine is a purine, pyrimidine nucleoside phosphorylase is not required in this pathway. There is no deoxyguanosine kinase (the only purine nucleosides that can be phosphorylated by adenosine kinase are adenosine and deoxyadenosine). APRT only works for the adenine base, not guanine. The 5′-nucleotidase is not required as there are no dephosphorylation events in the pathway outlined.

Many anticancer drugs are given to patients in their nucleoside form, rather than the nucleotide form. Which enzyme below will be required in the conversion of deoxyguanosine to dGTP? (A) Pyrimidine nucleoside phosphorylase (B) Deoxyguanosine kinase (C) Ribonucleotide reductase (D) Adenine phosphoribosyltransferase (E) 5′-nucleotidase

Temperature effect on lipid content

Membranes are very responsive to environment Low temps cause increase in lipids containing unsaturated FA to lower Tm and allow for more fluid membranes at lower temps, otherwise the membranes would become too rigid

Lactic acidosis

Metabolic acidosis brought on by build up of lactate in blood and tissue Sx are rapid breathing, vomiting and abd pain Lactate accumulation points to hypoxia due to it being the end of glycolysis Caused by some medications, or an IEOM, mutation in mitoDNA leading to mitochondrial encephalopathy, lactic acidosis, and stroke(MELAS) as well as deficiencies in pyruvate DH and pyruvate carboxylase Most common is alpa subunit of PDH mutation that uses Vitamin B1 and in some cases B1 can be used to treat the mutation Pyruvate carboxylase deficiency can lead to hypoglycemia due to gluconeogenic pathway disruption Alcoholics are prone to lactic acidosis due to the NAD+ being used to metabolize EtOH in the liver leading to increased pyruvate to lactate conversion to meet NAD+ demands

Inhibition of metalloenzymes

Metalloenzymes require metal cofactors which are inhibited by chelating agents such as EDTA

The correct answer is (C). 1. Pyrimidine synthesis doesn't involve the one-carbon pool. 2. The methyl group that appears in dTMP is synthesized during the transfer of the methylene group from the one-carbon pool. 3. This conversion involves an oxidation and an amination but no additional carbons. 4. Purine synthesis requires two components of the one-carbon pool. In the absence of this enzyme, dietary folate could not be reduced to the tetrahydro form necessary for carbon carriage.

Methotrexate is an inhibitor of dihydrofolate reductase. Administration of methotrexate would inhibit: 1. de novo synthesis of UMP 2. conversion of dUMP to dTMP 3. conversion of IMP to GMP 4. de novo synthesis of IMP a. 1, 2 and 3 b. 1 and 3 c. 2 and 4 d. 4 only e. All four

D.

Microsomal heme oxygenase system catalyzes the first step in the degradation of heme, converting it to A. Protoporphyrin III B. Porphin C. Bilirubin D. Biliverdin E. Bilirubin diglucuronide

The answer is a. (Murray, pp 122-129, 163-172. Scriver, pp 2297-2326.)The sources of NADPH for synthesis of fatty acids are the pentose phosphate pathway and cytosolic malate formed during the transfer of acetyl groups to the cytosol as citrate. The enzyme citrate lyase splits citrate into acetyl CoA and oxaloacetate. The oxaloacetate is reduced to malate by NADH. NADP-linked malate enzyme catalyzes the oxidative decarboxylation of malate to pyruvate and carbon dioxide. Thus, the diffusion of excess citrate from the mitochondria to the cytoplasm of cells not only provides acetyl CoA for synthesis of fatty acids but NADPH as well. One NADPH is produced for each acetyl CoA produced. However, most of the NADPHs needed for synthesis of fatty acids are derived from the pentose phosphate pathway. For this reason, adipose tissue has an extremely active pentose phosphate pathway.

Most of the reducing equivalents utilized for synthesis of fatty acids can be generated from which of the following? a. The pentose phosphate pathway b. Glycolysis c. The citric acid cycle d. Mitochondrial malate dehydrogenase e. Citrate lyase

The answer is d. (Murray, pp 481-497. Scriver, pp 3897-3964.) In order to be converted to thrombin during clot formation, prothrombin must bind Ca2+, which allows it to anchor to platelet membranes produced by injury. Prothrombin's affinity for Ca2+ is dependent on the presence of 10 γ-carboxyglutamate residues found in the first 35 amino acid residues of its amino terminal region. The vitamin K-dependent γ-carboxylation of prothrombin is a posttranslational modification that occurs as nascent prothrombin is synthesized on liver rough endoplasmic reticulum and passes into the lumen of the reticulum. The anticoagulants warfarin and dicumarol are structural analogues that block the γ-carboxylation of prothrombin by substituting for vitamin K. Hence, the prothrombin produced has a weak affinity for Ca2+ and cannot properly bind to platelet membranes in order to be converted to thrombin. Exposure of the fetus to warfarin during maternal therapy can produce a syndrome involving small, "fleur-de lys" nose and skeletal defects.

Mothers taking warfarin for anticoagulation during pregnancy may have children with fetal warfarin syndrome involving very short nose and skeletal changes. Studies of the actions of the anticoagulants dicumarol and warfarin (the latter also a hemorrhagic rat poison)—have revealed which of the following? a. Vitamin C is necessary for the synthesis of fibrinogen b. Vitamin C activates fibrinogen c. Vitamin K is a clotting factor d. Vitamin K is essential for γ-carboxylation of glutamate e. The action of vitamin E is antagonized by these compounds

The answer is A: Tryptophan. Most drugs used to treat depression do so by elevating serotonin levels, and serotonin is derived from tryptophan (see the figure below). Tyrosine is the precursor for catecholamines, while glutamate is the precursor of GABA. Histidine is the precursor for histamine, while glycine itself acts as a neurotransmitter in the brain.

Mr Smith had been prescribed a drug to treat his depression. One of the effects of the drug is to maintain elevated levels of a particular neurotransmitter that has been derived from which of the following amino acids? (A) Tryptophan (B) Tyrosine (C) Glutamate (D) Histidine (E) Glycine

The answer is A. Mrs. Jones's diet lacks grain products, fruits, and vegetables, all of which are good sources of vitamin C. Her diet is adequate in protein, as eggs, milk, cheese, and cream contain significant levels of protein. Her calcium levels should be fine because of the milk, cream, and cheese in her diet. Vitamin B12 is derived from foods of animal origin such as eggs, milk, and cheese. As the patient's weight has been stable for a year, her diet contains sufficient calories to allow her to maintain this weight, which is in the normal range for a patient who is 5 ft 4 in tall, as her BMI is 21.5.

Mrs. Jones is a sedentary 83-year-old woman who is 5 ft 4 in tall and weighs 125 lb. She has been at this weight for about a year. She says that a typical diet for her includes a breakfast of toast (white bread, no butter), a boiled egg, and coffee with cream. For lunch, she often has a cheese sandwich (white bread) and a glass of whole milk. For supper, she prefers cream of chicken soup and a slice of frosted cake. Mrs. Jones' diet is most likely to be inadequate in which one of the following? A. Vitamin C B. Protein C. Calcium D. Vitamin B12 E. Calories

F16BPhosphatase deficiency

Mutation in the rate limiting enzyme Leads to hypoglycemia, lactic acidosis, ketosis, apnea, and hyperventilation triggered by fasting or ingestion of fructose, glycerol and sorbitol in infants and young children

The answer is E. Myelin basic protein is a basic protein, indicating that it must contain a significant number of lysine and arginine residues. MBP is found on the intracellular side of the myelin membrane, and its role is to compact the membrane by binding to negative charges on both sides of it, thereby reducing the "width" of the membrane. Both Schwann cells and oligodendrocytes synthesize myelin (thus, A is incorrect). MBP is not a transmembrane protein (proteolipid protein in the CNS and Po in the PNS are, so B is incorrect), and because MBP is found intracellularly, answers C and D cannot be correct.

Myelin basic protein can best be described by which of the following? A. It is synthesized in Schwann cells, but not in oligodendrocytes. B. It is a transmembrane protein found only in peripheral myelin. C. It attaches the two extracellular leaflets together in central myelin. D. It contains basic amino acid residues that bind the negatively charged extracellular sides of the myelin membrane together. E. It contains lysine and arginine residues that binds the negatively charged intracellular sides of the myelin membrane together.

Elevated levels of NADH produced from the oxidation of alcohol can disrupt the TCA cycle. Which of the following best describes the interaction of NADH with isocitrate dehydrogenase?

NADH is an allosteric inhibitor

Sources of NADH and FADH2

NADH- TCA cycle, oxidative decarboxylation of pyruvate and oxidation of ketone bodies FADH2-succinate DH, glycerol phosphate shuttle and fatty acyl CoA DH

P:O ratio

NADH-3 ATP FADH2- 2 ATP

Rotenone

Natural pesticide and fish poison, rapid photodecomposition Inhibits NADH dehydrogenase(complex I) of ETC, prevents transfer of electrons from Fe-S to ubiquinone Inhibition can be overcome by vitamin K3 allowing electrons to bypass the site of inhibition Chronic poisoning is linked to Parkinson disease

The answer is e. (Murray, pp 481-497. Scriver, pp 3897-3964.) Spina bifida, or myelomeningocele, is a defect of the lower neural tube that produces an exposed spinal cord in the thoracic or sacral regions. Exposure of the spinal cord usually causes nerve damage that results in paralysis of the lower limbs and urinary bladder. Anencephaly is a defect of the anterior neural tube that results in lethal brain anomalies and skull defects. Folic acid is necessary for the development of the neural tube in the first few weeks of embryonic life, and the children of women with nutritional deficiencies have higher rates of neural tube defects. Because neural tube closure occurs at a time when many women are not aware that they are pregnant, it is essential that all women of childbearing age take a folic acid supplement of approximately 0.4 mg per day. Frank folic acid deficiency can also cause megaloblastic anemia because of a decreased synthesis of the purines and pyrimidines needed for cells to make DNA and divide. Deficiencies of thiamine in chronic alcoholics are related to WernickeKorsakoff syndrome, which is characterized by loss of memory, lackadaisical behavior, and a continuous rhythmic movement of the eyeballs. Thiamine dietary deficiency from excess of polished rice can cause beriberi. Niacin deficiency leads to pellagra, a disorder that produces skin rash (dermatitis), weight loss, and neurologic changes including depression and dementia. Riboflavin deficiency leads to mouth ulcers (stomatitis), cheilosis (dry, scaly lips), scaly skin (seborrhea), and photophobia. Because biotin is widely distributed in foods and is synthesized by intestinal bacteria, biotin deficiency is rare. However, the heat-labile molecule avidin, found in raw egg whites, binds biotin tightly and blocks its absorption, causing dermatitis, dehydration, and lethargy. Lactic acidosis results as a buildup of lactate due to the lack of functional pyruvate carboxylase when biotin is missing. Vitamin C deficiency leads to scurvy, which causes bleeding gums and bone disease. Vitamin B12 can be deficient due to a lack of intrinsic factor, which is a glycoprotein secreted by gastric parietal cells. A lack of intrinsic factor or a dietary deficiency of cobalamin can cause pernicious anemia and neuropsychiatric symptoms. The only known treatment for intrinsic factor deficiency (vitamin B12 deficiency) is intramuscular injection of cyanocobalamin throughout the patient's life.

Neural tube defects such as anencephaly and spina bifida have higher frequencies in certain populations like those of Celtic origin and in certain regions like South Texas. This suggestion of environmental cause produced research showing that deficiency of which of the following vitamins is associated with the occurrence of neural tube defects (anencephaly and spina bifida)? a. Ascorbic acid (vitamin C) b. Thiamine (vitamin B1) c. Riboflavin (vitamin B2) d. Niacin (vitamin B3) e. Folic acid

Describe the Warburg Effect.

Normal cells respire via oxidative phosphorylation when oxygen is plentiful. Tumor cells, however, respire via increased glycolysis and lactic acid fermentation even when oxygen is present. Normal, proliferating cells will also use this method of respiration.

Roles of nucleosides and nucleotides

Nucleotides are important cosubstrates in enzymatic reactions, as well as parts of cofactors such as CoA, FAD, FMN, UDP-Glc, and NADPH/NADH cAMP and cGMP serve as regulators in pathways Nucleosides are important in vitamin structure such as B12

C.

Of the following risk factors, which one has been shown by various research studies to account for 50% of a person's risk for alcoholism? a. Gender b. Ethnicity c. Genetics d. Parental influence

Omega-3 & omega-6 FA

Omega-3 FA are essential FA whose final double bonds are located 3 carbon atoms away from the end of the hydrocarbon tail(linolenic acid) Omega-6 has a double bond 6 carbons away from the tail(linoleic acid and arachidonic acid)

Essential fatty acids

Omega-3 and 6 fatty acids cannot be synthesized by humans so we must attain them from foodstuffs Two precursors are known as linolenic acid and alpha-linolenic acid and are termed essential FA The PUFA arachadonic acid can be synthesized from alpha-linolenic acid and serves as the precursor to several eicosanoid products Linolenic acid can also be used to make EPA and DHA

Glucokinase as a glucose sensor and blood glucose buffer

One gene is regulated by insulin which controls hepatic expression however the pancreatic beta-cells always have expression of GK Beta-insulin Alpha-glucagon GK, ATP sensitive K+ channel and Ca2+ channel together serve as glucose sensor to determine insulin release

The answer is c. (Murray, pp 481-497. Scriver, pp 3897-3964.) The vitamin riboflavin (vitamin B2) is a precursor of two cofactors involved in electron transport systems, riboflavin 5′-phosphate, also known as flavin mononucleotide (FMN), and flavin adenine dinucleotide (FAD). Strictly speaking, these compounds are not nucleotides, as they contain the sugar alcohol ribitol, not ribose. The cofactors are strongly bound to their apoenzymes and function as dehydrogenation catalysts. Pyruvate dehydrogenase is a multienzyme complex and contains the enzyme dihydrolipoyl dehydrogenase, which has as its prosthetic group two molecules of FAD per molecule of enzyme. In the overall reaction, the reduced FAD is reoxidized by NAD+. Mutations at several loci encoding the components of pyruvate dehydrogenase cause the clinical phenotype of Leigh syndrome (256000) with seizures, low tone, neurodegeneration, and lactic acidosis. Succinate dehydrogenase also contains tightly bound FAD, one molecule per molecule of enzyme. Glutamate, lactate, malate, and glyceraldehyde3-phosphate dehydrogenases all use nicotinamide dinucleotide cofactors and do not contain FAD as a prosthetic group.

One of many roles for vitamins is their use as a reactive agent at the active sites of enzymes. Since these are catalytic reactions, the vitamin is not consumed in the reaction and is required in small amounts. Knowledge of this common mechanism should discourage use of megavitamin supplements that are promoted as cures for autism, cancer, and colds, but an unfortunate 30% of the American public admit to taking such supplements. Which of the following enzymes uses a vitamin-derived cofactor that is reoxidized by but different from NAD+? a. Lactate dehydrogenase b. Glutamate dehydrogenase c. Pyruvate dehydrogenase d. Malate dehydrogenase e. Glyceraldehyde-3-phosphate dehydrogenase

The answer is a. (Murray, pp 481-497. Scriver, pp 3897-3964.) The vitamin folic acid is provided commercially and pharmaceutically as the stable 5-formyltetrahydrofolate known as folinic acid or its synthetic analogue leucovorin. Addition of folate to foods (bread) and encouragement of preconceptional vitamins with folate was prompted by its ability to lower the incidence of neural tube defects by two- to threefold. Folic acid is biologically active as the interconvertible forms tetrahydrofolate (THF), methylTHF, and other methylated forms (methylene THF, N5, N10-methylene THF) that are important for one-carbon (methyl) transfers and interconversions (glycine-serine, formate-formylmethinone, formate-CO2 homocysteine-methionine, uracil-thymine). THF is required in two steps of purine synthesis and thus required in the de novo synthesis of ATP and GTP. Although de novo synthesis of the pyrimidine ring does not require tetrahydrofolate, the methylation of deoxyuridine monophosphate (dUMP) to form thymine from uracil does. In this thymidylate synthetase reaction, methylene THF donates a methyl group and is converted to dihydrofolate which requires action of dihydrofolate reductase to regenerate THF. Methotrexate inhibits dihydrofolate reductase, depletes THF pools, and thus would elevate substrates of enzymes dependent on this cofactor like dUMP. THF is also a cofactor for methionine synthase that coverts homocysteine to methionine, an enzyme deficient in one form of homocystinuria (236200).

One of the first chemotherapeutic agents was methotrexate, a compound that was effective in killing rapidly dividing cells like those of leukemias. This compound would be expected to elevate concentrations of which of the following compounds? a. Homocysteine and dUMP b. Thymine and choline c. Serine and methionine d. Glycine and methionine e. Homocysteine and thymine

The answer is D. Vitamin B6 participates in transamination and decarboxylation reactions (and indirectly in deamination reactions). The one common feature in the synthesis of serotonin, GABA, norepinephrine, and histamine is decarboxylation of an amino acid, which requires vitamin B6. The other reactions are not required in the biosynthesis of these neurotransmitters.

One of the presenting symptoms of vitamin B6 deficiency is dementia. This may result from an inability to synthesize serotonin, norepinephrine, histamine, and gamma-aminobutyric acid (GABA) from their respective amino acid precursors. This is because vitamin B6 is required for which type of reaction? A. Hydroxylation B. Transamination C. Deamination D. Decarboxylation E. Oxidation

Voltage-gated ion channels

Open/close in response to change in membrane potential Depolarization occurs in response to an influx of positively charged ions and triggers the opening of the voltage gated channels allowing ions to pass down their concentration gradient Found in excitable cells i.e. Sodium channel in neurons

D.

Orotate is synthesized by the following enzyme. A. Aspartate transcarbamoylase B. Orotate phosphoribosyl transferase C. Carbamoyl phosphate synthetase D. Dihydroorotate dehydrogenase E. Orotidylic acid dicarboxylase

D

Orotate is synthesized by the following enzyme: A. Aspartate transcarbamoylase B. Carbamoyl phosphate synthetase C. Dihydro-orotase D. Dihydroorotate dehydrogenase E. Orotate phosphoribosyl transferase F. Orotidylic acid dicarboxylase

D. Orotic acid is an intermediate in the synthesis of the pyrimidines.

Orotic acid would be an intermediate. A. Catabolism of guanine B. Catabolism of uracil a. A only b. B only c. Both A and B d. Neither A or B

The answer is C. His protein intake of 150 kcal is about 37 g protein (150 kcal / 4 kcal/g = 37 g) below the Recommended Dietary Allowance (RDA) of 0.8 g protein per kilogram body weight (thus, A is incorrect, as Otto weighs approximately 88 kg). His carbohydrate intake of 150 kcal is below the glucose requirements of his brain and red blood cells (about 150 g/day; see Chapter 2) (thus, B is incorrect). Therefore, he will be breaking down muscle protein to synthesize glucose for the brain and other glucose-dependent tissues and adipose tissue mass to supply fatty acids for muscle and tissues able to oxidize fatty acids. Because he will be breaking down muscle protein to amino acids and converting the nitrogen from both these amino acids and his dietary amino acids to urea, his nitrogen excretion will be greater than his intake and he will be in negative nitrogen balance (thus, D is incorrect). It is unlikely that he will develop hypoglycemia while he is able to supply gluconeogenic precursors.

Otto Shape, an overweight medical student (see Chapter 1), discovered that he could not exercise enough during his summer clerkship rotations to lose 2 to 3 lb/wk. He decided to lose weight by eating only 300 kcal/day of a dietary supplement that provided half the calories as carbohydrate and half as protein. In addition, he consumed a multivitamin supplement. During the first 3 days on this diet, which statement best represents the state of Otto's metabolism? A. His protein intake met the RDA for protein. B. His carbohydrate intake met the fuel needs of his brain. C. Both his adipose mass and his muscle mass decreased. D. He remained in nitrogen balance. E. He developed severe hypoglycemia.

Disulfide bonds

Oxidation of sulfur on cysteine residues in lumen of rER creates covalent disulfide bonds, usually in secreted proteins This does not happen in the cytosol due to high concentration of glutathione Keratin is high in disulfide bonds, forming hair, manipulation can cause straight or kinked hair

3-Hydoxyacyl CoA Dehydrogenase (β-Hydoxyacyl CoA Dehydrogenase)

Oxidizes the carbon to form ketoacyl CoA and NADH (produces 2.5 ATP via ETC)

Acyl CoA Dehydrogenase (ACAD)

Oxidizes the β carbon to produce FADH₂ and trans-enoyl-CoA. FADH₂ enters CoQ of ETC to generate 2 ATPs Four Types of ACADs: - Short chain Acyl CoA dehydrogenase (SCAD) - Medium chain Acyl CoA dehydrogenase (MCAD) - Long chain Acyl CoA dehydrogenase (LCAD) - Very long chain Acyl CoA dehydrogenase (VLCAD) Inherited defects in MCAD-most common

Phenylketonuria

PKU caused by a defect in PAH most common inborn error of metabolism(first included in infant screening) Toxic metabolites: phenylpyruvate, phenyllactate, and phenylacetate Leads to disrupted neurotransmission, blocked AA transport to brain, disrupts myelin formation, impairs brain function Treat with limited Phe, or by adding Tyr to formula Secondary PKU is a deficiency in THB which is a cofactor of PAH

De Novo Synthesis of Purine

PRPP Synthetase Glutamine: phoshoribosyl pyrophosphate amidotransferase Adenylosuccinate synthetase and Adenylosuccinase OR IMP dehydrogenase and GMP Synthetase

Phosphatidylserine as marker for apoptosis

PS found in inner leaflet PS is found on outer leaflet during apoptosis, which labels the cell for degradation by phagocytes

The answer is b. (Murray, pp 481-497. Scriver, pp 3897-3964.) Pantothenic acid is phosphorylated and complexed with the amino acid cysteine to form 4-phosphopantetheine, the precursor for coenzyme A (CoA) and the acyl carrier protein (ACP) that participates in fatty acid synthesis. The thiol group of 4-phosphopantetheine is a carrier of acyl groups in CoA (A stands for acetylation or acetyl group) and ACP (fatty acyl groups). CoA is one of the major molecules in metabolism, carrying a pantetheine group bound to adenosine ribonucleotide-3′-phosphate via a 5′-diphosphate (pyrophosphate). Acetyl groups are linked to the reactive terminal sulfhydryl group to produce acetyl-CoA, which has a high acetyl transfer potential. CoA carries and transfers acetyl groups in much the same way as ATP transfers activated phosphoryl groups. CoA is involved in fatty acid synthesis, fatty acid β-oxidation, and the citric acid cycle; it is not involved in glycolysis or gluconeogenesis, where acetyl transfer does not occur. Gluconeogenesis generates glucose by converting pyruvate to oxaloacetate (via pyruvate carboxylase) to phosphoenopyruvate (via phosphoenopyruvate carboxykinase) to fructose-1,6-bisphosphate (through reversal of glycolytic enzymes) to fructose-6-phosphate (via fructose-1, 6-bisphosphatase) to glucose-6-phosphate (through reversal of phosphohexose isomerase) to glucose (through glucose-6-phosphatase). Special enzymes are required at steps where reversal of glycolysis is not energetically feasible.

Pantothenic acid is important for which of the following steps or pathways? a. Pyruvate carboxylase b. Fatty acid synthesis c. Pyruvate carboxykinase d. Gluconeogenesis e. Glycolysis

The answer is B: Reduction of heme synthesis. The boy is suffering from lead poisoning, which he obtained from eating the flaking paint chips. Lead inhibits the δ-aminolevulinic acid dehydratase step of heme synthesis, leading to reduced heme levels (see the figure on page 133). In addition, the ferrochelatase step (in which iron is inserted into the newly synthesized heme ring) is also inhibited by lead. The reduced heme levels reduce the amount of functional hemoglobin synthesized, leading to the microcytic anemia observed in the child. Lead does not interfere with iron transport or inhibit part of the phosphatidyl inositol cycle (lithium is the metal that does that). DNA synthesis is not impaired by lead, nor does lead inhibit gene expression of the globin chains. Cytochrome synthesis is also decreased and may contribute to the lethargy observed in the child

Parents bring their 6-year-old son to the pediatrician due to the parents being concerned about "mental retardation." Blood work demonstrated a microcytic anemia and basophilic stippling. During the patient history, it became apparent that the boy often stayed with his grandparents, who owned a 150-year-old apartment. The boy admitted to eating paint chips from the radiators in the apartment. The boy's anemia is most likely the result of which one of the following? (A) Inhibition of iron transport (B) Reduction of heme synthesis (C) Inhibition of the phosphatidyl inositol cycle (D) Blockage of reticulocyte DNA synthesis (E) Inhibition of β-globin gene expression

Ubiquinone radical

Partially reduced ubiquinone radical is an intermediate in the transfer of electrons from complex I to ubiquinone as well as in the transfer from reduced ubiquinone to complex III The electron can be transferred to O2 to form superoxide(ROS along with H2O2 and OH radical) and can damage cells Superoxide can be converted via superoxide dismutase into H2O2 which is detoxified by glutathione reductase into water and O2

NADPH and glyceraldehyde 3-phosphate are essential for the synthesis of fatty acids and triacylglycerols. Which of the following pathways can supply both of these products?

Pentose Pathway

HMG CoA reductase is highly regulated within the cell. Which of the following will reduce the activity of HMGCoA reductase?

Phosphorylation by a protein kinase

Ketoacidosis during starvation and diabetes

Physiological ketosis- mild to moderate in ketone bodies, fasting in pregnancy and in babies, prolonged excercise and ketogenic diet Pathological ketoacidosis- occurs when glucagon/insulin ratio is increased, favoring FA breakdown Increased acyl CoA in hepatic mitochondria Increased gluconeogenesis, reduced OAA Increased ketone bodies Acetoacetate and beta-hydroxybuturate are strong acids, lower pH, causing acidosis Increased excretion of acetoacetate and beta-hydroxybuturate via urine Acetone exhaled in breath, fruity odor in uncontrolled diabetics

Enzyme replacement therapy

Pompe disease treatment includes enzyme replacement with recombinant human acid alpha-glucosidase, IV infusion in infants Patient cannot have cardiac hypertrophy and it is expensive, but reduces respiratory distress and increases survival rate

Hemolytic anemia

Premature destruction of erythrocytes Inherited defects in the membrane or its metabolism or hemoglobinopathies Nutritional deficiencies in iron, folate, B12 contribute as well as viral, bacterial and protozoal infections Markers include elevated lactate DH and unconjugated bilirubin Deficiencies in glycolytic enzymes(PGI, TPI, PK) can be inherited leading to decreased storage for RBCs due to the dependencies on glycolysis Aberrant pumps lead to Na concentration increase and swelling then lysing

Pentose Phosphate Pathway

Produces ribose 5'-phosphate and NADPH Erythrocytes, liver, testes, mammary glands and the adrenal cortex which use NADPH to maintain a reducing environment(glutathione) for reduction of FA and steroids Liver is primary site of purine and pyrimidine synthesis and utilizes R5P and Asp, Gln, Gly as starting materials

Which of the following processes provides substrate for gluconeogenesis?

Protein catabolism

Exopeptidase

Proteolytic enzyme Attacks at C- (carboxypeptidase) or N-terminus (aminopeptidase) ends

Endopeptidase

Proteolytic enzyme Attacks within the protein at a specific site (digests internal peptide bonds).

C.

Purine nucleotide biosynthesis can be inhibited by which of the following? a. Guanosine triphosphate (GTP) b. Uridine monophosphate (UMP) c. Adenosine monophosphate (AMP) d. Adenosine triphosphate (ATP) e. Inosine diphosphate (IDP)

The answer is B. The reaction pathway in which methionine goes to cysteine and alpha-ketobutyrate requires pyridoxal phosphate at two steps: the cystathionine beta-synthase reaction and the cystathionase reaction. Phenylalanine to tyrosine requires BH4; propionyl-CoA to succinyl-CoA requires B12; pyruvate to acetyl-CoA requires thiamine pyrophosphate, lipoic acid, NAD, FAD, and coenzyme A; and glucose to glycogen does not require a cofactor.

Pyridoxal phosphate is required for which of the following reaction pathways or individual reactions? A. Phenylalanine → tyrosine B. Methionine → cysteine + alpha-ketobutyrate C. Propionyl-CoA → succinyl-CoA D. Pyruvate → acetyl-CoA E. Glucose → glycogen

The answer is C. Glycogen phosphorylase requires pyridoxal phosphate to break the alpha-1,4-linkages in glycogen. None of the other pathways listed contains an enzyme that requires this cofactor.

Pyridoxal phosphate, which is required for transaminations, is also required for which of the following pathways? A. Glycolysis B. Gluconeogenesis C. Glycogenolysis D. The TCA cycle E. Fatty acid oxidation

Gluconeogenesis

Pyruvate carboxylase (coenzyme: biotin) Malate dehydrgenase PEP carboxykinase Fructose 1,6-bisphosphatase Glucose-6-phosphatase

Rh factor and hemolytic disease of newborn

RBCs express Rh factor, must be crossed Rh+ express D antigen, Rh- do not Erythroblastocic fetalis-incompatibility between blood of mother and fetus Mom is Rh- and fetus is Rh+, mom produces Ab which cross placenta and attack the fetus, risk increases in other pregnancies

ABO blood system

RBCs have antigens on the surface Blood transfusion-RBCs from donor to recipient Plasma may contain Ab to one more ABO and RH Ags, blood needs to be crossmatched Incompatibility can lead to hemolysis, renal failure and shock

RBCs and glycolysis

RBCs lack mitochondria, glycolysis is the only ATP mechanism, defect in glycolysis inactivates the ATP-dependent ion pumps which are essential for viability Destruction leads to hemolytic anemia

Maple syrup urine disease

Rare autosomal disease Decreased branched-chain alpha-keto acid DH complex activity Branched-chain ketoaciduria: hallmark of maple syrup smell in infants Metabolites are toxic to brain, contributing to mental retardation Treat with low BCAA diet Thiamine supplements can restore BCKD activity

A.

Recent evidence suggests that high levels of homocysteine may damage the arteries, may make blood more likely to clot, and may make blood vessels less flexible. Some reports suggest that the high homocysteine level is due to low levels of pyridoxal phosphate. Pyridoxal phosphate is found as a coenzyme in which of the following enzymes? A. Cystathionine synthase B. Cystathioninase C. Carbamoyl phosphate synthetase D. Phenylalanine hydroxylase E. Methylmalonyl CoA mutase

Malate Dehydrogenase

Reduces OAA into malate or Oxidizes malate into OAA.

Blood glucose

Regulated with insulin and glucagon (normal is 70-100) Below 60 is hypoglycemia leading to sweats, hunger, and trembling Below 40 convulsion, coma and brain damage and death High fasting levels over 126 and postprandial over 199 are indicative of DM

Correct answer = D. Low insulin levels favor the liver producing ketone bodies, using acetyl coenzyme A it generated by β-oxidation of fatty acids provided by the adipose. Low insulin also causes activation of hormone-sensitive lipase, decreased glycogen synthesis, and increased gluconeogenesis and glycogenolysis.

Relative or absolute lack of insulin in humans would result in which one of the following reactions in the liver? A. Increased glycogen synthesis. B. Decreased gluconeogenesis from lactate. C. Decreased glycogenolysis. D. Increased formation of 3-hydroxybutyrate. E. Decreased action of hormone-sensitive lipase

Nucleosidases

Removes ribose group - converts them into pyrimidines and purines

Transaminases

Require coenzyme: pyridoxyl-5'-phosphate (PLP) - Derivative of Vitamin B₆ Clinical relevance: - Alanine transaminase (ALT) - Aspartate transaminase (AST)

Ribonucleases and Deoxyribonucleases

Respectively, convert RNA and DNA to short oligomers

Ligand-gated ion channels

Respond to ligands(neurotransmitter of hormone) Transport ions rapidly down their concentration gradients I.e. glutatmate receptor, antagonist of receptor is used to treat Alzheimer's(Mimantine/Namenda)

Correct answer = B. Retinyl esters are incorporated into chylomicrons. Retinoic acid cannot be reduced to retinol. Retinal, the aldehyde form of retinol, is the chromophore for rhodopsin. Retinal is photoisomerized during the visual cycle. Retinoic acid, not retinol, is the most important retinoid.

Retinol: A. can be enzymically formed from retinoic acid. B. in its ester form is transported from the intestine to the liver in chylo microns. C. is the light-absorbing portion of rhodopsin. D. is phosphorylated and dephosphorylated during the visual cycle. E. mediates most of the actions of the retinoids.

B.

Rhodopsin deficiency, night blindness, retarded growth, skin disorders, and increased infection risk are symptoms of which deficiency? A. Vitamin B3 (niacin) B. Vitamin A C. Vitamin B1 (thiamine) D. Vitamin B2 (riboflavin) E. Pantothenic acid

Degredation of Nucleotides

Ribonucleases and Deoxyribonucleases Phosphodiesterases Nucleotidases Nucleosidases

The answer is B: 5-hydroxytryptophan. The child has nonclassical phenylketonuria (PKU). Classical PKU is due to a defect in phenylalanine hydroxylase, leading to accumulation of phenylalanine derivatives. These interfere with amino acid transport into the brain and can lead to cognitive disorders if not treated, usually, by a low- phenylalanine diet. However, in nonclassical PKU, the required cofactor for the phenylalanine hydroxylase reaction, tetrahydrobiopterin, is deficient. This will lead to similar biochemical symptoms (elevation of phenylalanine derivatives), but, in addition, the catecholamines (dopamine, epinephrine, and norepinephrine) and serotonin cannot be synthesized as those pathways require tetrahydrobiopterin. Giving 5-hydroxytryptophan bypasses the block in serotonin biosynthesis, and would have to be a supplement for these children along with dihydroxyphenylalanine (DOPA), which is the hydroxylated precursor for catecholamine biosynthesis. Providing tyrosine will not overcome the block in neurotransmitter biosynthesis. Providing phenylalanine just makes the problem worse. Neither melanin nor alanine will bypass the metabolic block of this disease.

Routine newborn screening identified a child with elevated levels of phenylpyruvate and phenyllactate in the blood. Despite treating the child with a restricted diet, evidence of developmental delay became apparent. Supplementation with which of the following would be beneficial to the child? (A) Tyrosine (B) 5-hydroxytryptophan (C) Melanin (D) Phenylalanine (E) Alanine

The answer is E: Thiamine. The child has maple syrup urine disease, a defect in the branched-chain α-keto acid dehydrogenase step that utilizes all three branchedchain α-keto acids as substrates. The reaction catalyzed by this enzyme is an oxidative decarboxylation reaction, which requires the same five cofactors as do pyruvate and α-ketoglutarate dehydrogenase; thiamine, NAD+, FAD, lipoic acid, and coenzyme A. A subset of patients with this disorder has a mutation in the E1 subunit of the enzyme, which has reduced the affinity of the enzyme for vitamin B1. Increasing the concentration of B1 can therefore overcome the effects of the mutation and allow the enzyme to exhibit sufficient activity to reduce the buildup of the toxic metabolites. While niacin and riboflavin are required for the enzyme, the mutation in the enzyme is such that the affinity of these cofactors for the enzyme has not been altered. B12 and B6 are not required for this reaction.

Routine newborn screening identified a child with elevated levels of α-ketoacids of the branched-chain amino acids. A certain subset of such children will respond well to which of the following vitamin supplementation? (A) Niacin (B) Riboflavin (C) B12 (D) B6 (E) Thiamine

Enoyl CoA Hydratase

Saturates the alkene with water to form β hydroxy acyl CoA

The answer is C: ATP and NADPH. The β-cells of the pancreas monitor both ATP and NADPH levels in order for insulin release to occur. The NADPH levels are increased through enhanced pyruvate cycling (see the figure below), which occurs when pyruvate levels increase (which is correlated with an increase in glucose levels within the β-cell). Increased glucose also leads to an increase in ATP, which leads to changes in ion fluxes across the membrane, resulting in insulin release. Glucose-6-phosphate, carbon dioxide, and NADH are not necessary for insulin release in response to glucose.

Sequelae of insulin resistance in type 2 diabetes mellitus and metabolic syndrome is reduced secretion of insulin in response to increases in blood glucose. Insulin release from the pancreas appears to be dependent upon increase in concentration of which pair of metabolites? (A) ATP and CO2 (B) ATP and NADH (C) ATP and NADPH (D) Glucose-6-phosphate and CO2 (E) Glucose-6-phosphate and NADH

Albinism and tyrosinase

Severe lack of melanin Tyrosinase defect Partial or complete lag of pigmentation in skin, hair and eyes

The answer is A. Both carbamoyl phosphate synthetase (CPS) I and II use carbon dioxide as the carbon source in the production of carbamoyl phosphate. CPSI is located in the mitochondria, whereas CPSII is in the cytoplasm (thus, B is incorrect). CPSI can fix ammonia; CPSII requires glutamine as the nitrogen source (thus, C is incorrect). N-acetylglutamate activates CPSI; CPSII is activated by PRPP (thus, D is incorrect). UMP inhibits CPSII, but has no effect on CPSI (thus, E is also incorrect).

Similarities between carbamoyl phosphate synthetase I and carbamoyl phosphate synthetase II include which one of the following? A. Carbon source B. Intracellular location C. Nitrogen source D. Regulation by N-acetyl glutamate E. Regulation by UMP

Ferredoxin

Small proteins serving as carriers of e- in mitochondrial cytochrome P-450 systems Reduction and oxidation of iron allow transfer between ferredoxin family members Adrenodoxin(FDX1) is specifically used in synthesis of steroid hormones as well as bile acids and vitamin D FDX2 is used in heme-a synthesis as well iron-sulfur proteins Ferredoxins play a role in bacterial metabolism and chloroplast photosynthesis

All of the following statements regarding the mechanism of action of the steroid hormones are true EXCEPT which one? A. Steroid hormones are lipid-soluble, therefore can readily traverse the cell membranes. B. In general, the steroid hormones act by affecting the rate of protein synthesis. C. Steroid hormones combine with cytoplasmic receptors and stimulate the production of cAMP. D. In general, steroid hormones take a rather long time (1-8 hours) to produce an effect.

Steroid hormones can bind receptors in the cytoplasm, but they generally do not use cAMP signaling.

Acetyl CoA in other metabolic pathways

Substrate for TCA, building block of FA and isoprenoids Junction of catabolic and anabolic pathways Fasting or diabetes leads to increase in Acetyl CoA due to FA breakdown but cannot be oxidized because OAA is used up in gluconeogenesis It then goes into ketone bodies for alternative energy to brain and muscles

Enzymes that require Zn as Cofactor

Superoxide dismutase (antioxidant) Collagenase Alcohol dehydrogenase Alkaline phosphatase Transcription factors (proteins that contain Zn-finger domains) Carbonic anhydrase

Succinyl CoA in heme synthesis

TCA cycle oxidizes Acetyl CoA along with creating precursors for other biological molecules i.e. Succinyl CoA combines with glycine to form alpha-aminolevulinic acid as the first step in heme synthesis Catalyzed by ALAA synthase, rate limiting in heme synthesis, and requires PLP(Vit B6)

What is the specific amino acid change that occurs within the ras protein found in your textbook review question 1 on page 366?

The DNA code for glycine is changed to insert a valine.

The answer is D. The Recommended Daily Allowance (RDA) of a nutrient is determined from the Estimated Average Requirement (EAR) ! 2 standard deviations (SD) of the mean, and should meet the needs for 97% to 98% of the healthy population. It is, therefore, a reasonable goal for the intake of a healthy individual. The EAR is the amount that prevents development of established signs of deficiency in 50% of the healthy population. Although data with laboratory animals have been used to establish deficiency symptoms, RDAs are based on data collected on nutrient ingestion by humans.

The RDA is best described by which one of the following? A. The average amount of a nutrient required each day to maintain normal function in 50% of the US population. B. The average amount of a nutrient ingested daily by 50% of the US population. C. The minimum amount of a nutrient ingested daily that prevents deficiency symptoms. D. A reasonable dietary goal for the intake of a nutrient by a healthy individual. E. It is based principally on data obtained with laboratory animals.

Glucose cannot be synthesized from the β-oxidation of even chain fatty acids because:

The acetyl CoA produced from β-oxidation is completely oxidized in the TCA cycle

The answer is D: The position of two amino acids is reversed in Humalog as compared to Humulin R. The sequence of Humulin R is the same as native insulin, but Humalog has switched the positions of amino acids 28 and 29 of the B chain and is designated as lys- pro insulin, for the two amino acids that have switched positions (regular insulin has the sequence pro-lys at these positions, whereas lys-pro insulin has lys-pro at these positions). The C-peptide is removed from both forms of insulin (insulin would have no biological activity if the C-peptide were not removed), and both contain the same disulfide bonds. There are no his-tags present on Humulin. This minor difference in insulin structure allows Humulin to be absorbed much faster than Humalog, yet still retain its normal affinity for the insulin receptor.

The biochemical difference between Humulin R and Humalog is which of the following? (A) The C-peptide remains in Humalog and is removed from Humulin R (B) Disulfide bond formation is prevented in Humalog and is present in Humulin R (C) The amino acid sequence is completely reversed in Humalog as compared to Humulin R (D) The position of two amino acids is reversed in Humalog as compared to Humulin R (E) Humulin R contains genetically engineered histidine residues so it can complex with nickel, which is not present in Humalog

Correct answer = B. The cyclic heme molecule is oxidatively cleaved to form biliverdin. The catabolism occurs in the cells of the reticulo endothelial system, particularly the spleen, and results in the liberation of carbon monoxide. Protoporphyrinogen is an intermediate in the synthesis, not degradation, of heme. Cytochromes and other non-hemoglobin hemeproteins are also precursors of bilirubin.

The catabolism of hemoglobin: A. occurs in red blood cells. B. involves the oxidative cleavage of the porphyrin ring. C. results in the liberation of carbon dioxide. D. results in the formation of protoporphyrinogen. E. is the sole source of bilirubin.

The answer is a. (Murray, pp 249-263.) Asparagine is converted to aspartate by the enzyme asparaginase. A transaminase subsequently converts aspartate to oxaloacetate. The amino group from aspartate is used to convert pyruvate to alanine. Likewise, glutamine is converted to glutamate and then α-ketoglutarate by glutaminase and a transaminase.

The citric acid cycle intermediate oxaloacetate is formed by which amino acid? a. Asparagine b. Glutamine c. Proline d. Serine e. Theronine

Which reaction (A-I) produces the common methylating agent, SAM?

The correct answer is A. SAM participates in the synthesis of many compounds that contain methyl groups. It is used in reactions that add methyl groups to either oxygen or nitrogen. SAM is synthesized from methionine (knowing its full name, S-adenosylmethionine may clue you in) by adding ATP to methionine, demonstrated in reaction A.

A febrile 54-year-old alcoholic comes to the emergency room complaining of an inability to walk because his foot hurts. The night before he consumed a rather sumptuous meal and drank a large amount of alcohol. Physical examination is notable for a tender erythematous area with heat over the metatarsophalangeal joint. His breath has a fruity sweet odor, and he displays deep shallow breathing. He has been taking medication for swelling of the joints. His clinical values reveal: The high ß-hydroxybutyrate to acetoacetate ratio of 8 is due to: A. high NAD+/NADH ratio in the cytosol. B. low NAD+/NADH ratio in the mitochondria. C. high NAD+/NADH ratio in the mitochondria. D. low NAD+/NADH ratio in the cytosol. E. regeneration of NAD+ by pyruvate conversion to lactate.

The correct answer is B. The low ratio of NAD+ to NADH in mitochondria leads to the conversion of acetoacetate into betahydroxybutyrate in alcoholic ketoacidosis.

A 35-year-old man has acute myelogenous leukemia. He is currently being treated with multiple antileukemic agents. One week into therapy, he develops oliguric renal failure with an increase in serum blood urea nitrogen, creatinine, and uric acid. A urinalysis reveals an acid pH, and numerous crystals are present on sediment examination. Which of the following best explains the mechanism for the renal failure? A. Drug nephrotoxicity B. Increased degradation of purine nucleotides C. Increased degradation of pyrimidine nucleotides D. Leukemic infiltration of the kidneys

The correct answer is B. Treatment of disseminated malignancies like acute myelogenous leukemia causes the release of purine nucleotides from the nuclei of the killed cancer cells. Uric acid is the end-product of degradation of purine nucleotides. An acid urine pH causes uric acid to form crystals in the tubules of the nephron leading to renal failure. This complication is prevented by allopurinol, which blocks xanthine oxidase and prevents the conversion of xanthine to uric acid.

A febrile 54-year-old alcoholic comes to the emergency room complaining of an inability to walk because his foot hurts. The night before he consumed a rather sumptuous meal and drank a large amount of alcohol. Physical examination is notable for a tender erythematous area with heat over the metatarsophalangeal joint. His breath has a fruity sweet odor, and he displays deep shallow breathing. He has been taking medication for swelling of the joints. His clinical values reveal: Which of the following findings best describes the pathophysiologic process underlying these findings? A. Increased activity of HGPRTase B. Increased activity of xanthine oxidase C. Decreased renal excretion of uric acid D. Decreased PRPP amidotransferase activity E. Decreased activity of glucose-6-phosphate dehydrogenase activity

The correct answer is C. Based on the lab values, this is the only possible answer. If A was increased, uric acid would be lower. Uric Acid would be higher if B was increased. D is incorrect, because creatinine is elevated. If E was true, blood sugar would be much lower.

A febrile 50-year-old man with a 10-year history of alcohol abuse complains of pain in his left big toe. Physical examination reveals erythema and heat over the metatarsophalangeal joint. A complete blood cell count shows an absolute neutrophilic leukocytosis with greater than 10% band neutrophils in a 100 cell count. A synovial tap is performed and shows needle-shaped crystals phagocytosed by neutrophils. What is the mechanism for the joint inflammation? A. Decreased activity of hypoxanthine-guanine phosphoribosyltransferase (HGPRT) B. Decreased activity of xanthine oxidase C. Decreased renal excretion of uric acid D. Increased degradation of pyrimidines E. Increased activity of phosphoribosyl-pyrophosphate synthetase

The correct answer is C. The patient has acute gouty arthritis involving the metatarsophalangeal joint of the left big toe. The presence of negatively birefringent crystals in synovial fluid is diagnostic of gout. Most cases of gout result from underexcretion of uric acid in the proximal tubules of the kidneys. Gout is commonly seen in patients with alcohol abuse because of the increase in synthesis of lactic acid and β-hydroxybutyric acid, which compete with uric acid for excretion in the proximal tubules. In the metabolism of alcohol, NADH, acetate, and acetyl CoA are metabolic end-products. The increase in NADH increases the conversion of pyruvate to lactate by lactate dehydrogenase. Acetyl CoA is utilized by the liver to synthesize ketone bodies. The excess in NADH causes acetoacetate to convert to β-hydroxybutyric acid.

A 2-year-old boy of normal weight and height is brought to a clinic because of excessive fatigue. Blood work indicates an anemia, with microcytic hypochromic red cells. The boy lives in a 100-year-old apartment building and has been caught ingesting paint chips. His parents indicate that the child eats a healthy diet and takes a Flintstones vitamin supplement every day. His anemia is most likely attributable to a deficiency in which of the following? A. Iron B. Vitamin B12. C. Folate D. Heme E. Vitamin B6.

The correct answer is D. Lead poisoning interrupts heme synthesis at the ALA-dehydratase and ferrochelatase steps. Marks' 4th edition, Chap 44, ques 2, page 965

Which reaction (A-I) is adenosylcobalamin dependent?

The correct answer is I. Vitamin B12 (also called cobalamin) is involved in two reactions in the body: 1, the transfer of a methyl group from N5-methyl-FH4 to homocysteine to form methionine, and 2, shown above, the rearrangement of L-methylmalonyl-CoA to form succinyl CoA, reaction I.

The answer is E. The other end products are acetoacetate, acetyl-CoA, fumarate, oxaloacetate, alpha-ketoglutarate, and pyruvate.

The degradation of amino acids can be classified into families, which are named after the end product of the degradative pathway. Which of the following is such an end product? A. Citrate B. Glyceraldehyde 3-phosphate C. Fructose 6-phosphate D. Malate E. Succinyl-CoA

C.

The development of hepatic fibrosis after ethanol consumption is related to the stimulation of the: a. Hepatocytes b. Kupffer cells c. Stellate (Ito) cells d. Natural Killer cells

B

The end product of purine catabolism in humans is: A. Allantoin B. Uric acid C. Urea D. Ammonia E. Aspartate

A.

The fate of acetate, the product of ethanol metabolism, is which of the following? a. It is taken up by other tissues and activated to acetyl-CoA b. It is toxic to the tissues of the body and can lead to hepatic necrosis c. It is excreted in the bile d. It is converted into NADH by ADH e. It enters the TCA cycle directly to be oxidized

The answer is A. Acetate is converted to acetyl-CoA by other tissues so that it can enter the TCA cycle to gene rate ATP. Answer B is incorrect because acetaldehyde, not acetate, is toxic to cells. Answer C is incorrect because acetate is excreted by the lung and kidney, and not in the bile. Answer D is incorrect because acetate cannot enter the TCA cycle directly. It must be converted to acetylCoA first. Answer E is incorrect because alcohol dehydrogenase converts ethanol into acetaldehyde. It does not convert acetate into NADH.

The fate of acetate, the product of ethanol metabolism, is which of the following? A. It is taken up by other tissues and activated to acetylCoA. B. It is toxic to the tissues of the body and can lead to hepatic necrosis. C. It is excreted in bile. D. It enters the TCA cycle directly to be oxidized. E. It is converted into NADH by ADH.

A. De novo synthesis occurs with PRPP only. B. Very little adenine is salvaged. C. See above. What adenine is salvaged would be by PRT since the phosphorylase is used primarily for pyrimidines. D. Correct. The resulting dADP would be converted to the triphophate by a nucleoside diphosphate kinase. E. That pathway converts dUMP to dTMP.

The formation of dATP for DNA synthesis occurs primarily by: a. de novo synthesis beginning with dPRPP b. salvaging adenine using APRT c. salvaging adenine using a nucleoside phosphorylase and dR 1-P d. converting ADP to dADP using thioredoxin e. converting dIMP to dAMP using 5,10-methylene THF

The answer is D. The fruity odor is due to acetone, which is being exhaled. The acetone is derived from the spontaneous decarboxylation of acetoacetate (one of the ketone bodies) to acetone within the blood and tissues

The fruity odor noticed by the ER physician is due to which one of the following? (A) Oxidation of acetoacetate (B) Reduction of acetoacetate (C) Conversion of acetoacetate to acetoacetyl-CoA (D) Decarboxylation of acetoacetate (E) Carboxylation of acetoacetate

The answer is C. The pathway followed is glutamine to glutamate, to glutamate semialdehyde, to ornithine, and then, after condensation with carbamoyl phosphate, to citrulline. Aspartate, succinyl-CoA, serine, and fumarate are not part of this pathway.

The gut uses glutamine as an energy source, but it can also secrete citrulline, synthesized from the carbons of glutamine. Which of the following compounds is an obligatory intermediate in this conversion (consider only the carbon atoms of glutamine while answering this question)? A. Aspartate B. Succinyl-CoA C. Glutamate D. Serine E. Fumarate

B?

The key reaction of the salvage pathway of purine nucleotide biosynthesis is catalyzed by: A. Cyclohydrolase B. Phosphoribosyltransferase C. Ribose 5'-phosphate synthetase D. Formyltransferase E. Adenylosuccinate synthetase

Arsenite and lipoic acid

The lipoic acid subunit(E2) is modified by arsenite(arsenic derivative), it links with the two SH groups of lipoic acid, irreversibly. This also affects AKG DH and BCAK DH Arsenic builds up slowly and is detectable in the hair

A. This is the control for purine nucleotide synthesis. B. This is true in bacteria but not in man. C. This is the control of CPS I which leads to urea synthesis but NAG is not an activator of CPS II. D. Although this is true for some pathways, eg. degradation of purines, it is not true for this one. E. Correct. UTP is competitive with ATP in this reaction.

The major control of de novo pyrimidine nucleotide synthesis in man is: a. feedback inhibition of glutamine-PRPP amidotransferase b. feedback inhibition of aspartate transcarbamylase c. availability of N-acetyl glutamate d. substrate availability e. competitive inhibition of carbamoyl phosphate synthetase II

The answer is D. Adipose triacylglycerol is the largest energy store in the body and is the predominant fuel in longterm aerobic exercise. During exercise, muscle glycogen is used for bursts of speed but not for long-term energy requirements. Liver glycogen is used to maintain blood glucose levels for use by the nervous system and as a supplement for use by the muscle when rapid speed is required; however, it is not designed to be a long-term energy source. The brain does not contain significant levels of glycogen, and lactic acid produced by the red blood cells is used as a gluconeogenic precursor by liver, but not as a fuel for muscle.

The major metabolic fuel for participating in a prolonged aerobic exercise event is which of the following? A. Liver glycogen B. Muscle glycogen C. Brain glycogen D. Adipose triacylglycerol E. Red blood cell-produced lactate

The answer is A. CPSI is the major regulated step, being activated by N-acetylglutamate, the synthesis of which is stimulated by arginine. None of the other urea-cycle enzymes is regulated allosterically.

The major regulated step of the urea cycle is which one of the following? A. Carbamoyl phosphate synthetase I B. Ornithine transcarbamoylase C. Argininosuccinate synthetase D. Argininosuccinate lyase E. Arginase

The answer is B: Ability to produce insulin. By defi nition, a type 1 diabetic cannot produce insulin. A type 2 diabetic produces insulin, but has become resistant to the effects of insulin. Weight does not differentiate between type 1 and 2 diabetics (although most type 2 diabetics are overweight). Blood glucose levels are elevated in both types of diabetes and cannot differentiate between them. Neither LDL levels nor triglyceride levels will differentiate between these two major forms of diabetes.

The major, defining difference between a type 1 diabetic and a type 2 diabetic is which of the following? (A) Weight (B) Ability to produce insulin (C) LDL levels (D) Blood glucose levels (E) Serum triglyceride levels

What is the cause of the change to metabolism in tumor cells? A. Mutations in biosynthetic machinery B. Mutations in tumor suppressor genes and oncogenes C. Changes to metabolic pathway enzymes D. Increased demands of dividing cells

The mutations in the oncogenes/tumor suppressor genes directly reprogram cell metabolism to increase nutrient uptake and biosynthesis. While rapidly dividing cells would have increased demands, this is not the cause of the metabolism changes.

The answer is D: B6. All three neurotransmitters (GABA, dopamine, and histamine) are derived via decarboxylation of an amino acid precursor. Such amino acid decarboxylation reactions require pyridoxal phosphate (derived from B6). GABA is derived from the decarboxylation of glutamate, dopamine from the decarboxylation of dihydroxyphenylalanine, and histamine from the decarboxylation of histidine. NAD, thiamine (B1), ribofl avin (B2), or cobalamin (B12) are not required for any of these reactions to occur.

The neurotransmitters GABA, dopamine, and histamine are all derived from an amino acid precursor. The generation of these neurotransmitters from the appropriate amino acids requires which one of the following cofactors? (A) NAD+ (B) B1 (C) B2 (D) B6 (E) B12

The answer is D. The nitrogens in urea are derived from carbamoyl phosphate and aspartate directly during one turn of the cycle. The nitrogen in the side chain of ornithine is never incorporated into urea because it stays with ornithine (thus, A, B, and C are incorrect). Glutamine does not donate a nitrogen directly during the urea cycle, so E and F are also incorrect.

The nitrogens in urea are derived directly from which of the following compounds? A. Ornithine and carbamoyl phosphate B. Ornithine and aspartate C. Ornithine and glutamate D. Carbamoyl phosphate and aspartate E. Carbamoyl phosphate and glutamine F. Aspartate and glutamine

Depriving cells of GMP and dGTP

The oxidation of IMP to XMP allows for therapeutic intervention IMP DH is the target for drugs which inhibit the replication of DNA in B and T cells by depriving them of GMP and dGTP Good target for graft rejection

The answer is B: Cortisol stimulation of amino acid release from the muscle. The weight loss seen in type 1 diabetics (untreated) results from the need of the liver for gluconeogenic precursors, many of which are derived from amino acids obtained from muscle protein breakdown. Cortisol release will signal the muscle to release amino acids for use by the liver. Glucagon signals triglyceride degradation, not production. Insulin does signal an inhibition of fatty acid oxidation, but that is not occurring in an individual who does not make insulin (type 1 diabetic). Since the muscle is oxidizing fatty acids for energy, there is no activation of the AMP-activated protein kinase, as the energy levels are not low. Muscle acetyl-CoA carboxylase 2 (which produces malonyl-CoA, which would inhibit fatty acid oxidation via inhibiting carnitine palmitoyl transferase 1) is not activated under these conditions, due to the lack of insulin.

The polyphagia observed in the untreated type 1 diabetic, who has lost 6 lb in the last 2 weeks, is due to which of the following? (A) Glucagon stimulation of triacylglycerol production (B) Cortisol stimulation of amino acid release from the muscle (C) Insulin-induced inhibition of fatty acid oxidation (D) AMP kinase-induced activation of GLUT4 transporters (E) Activation of muscle acetyl-CoA carboxylase-2

The answer is C: Water. Serine donates a carbon to THF to form N5, N10-methylene THF. This is oxidized to form N5, N10-methenyl THF, which is then hydrolyzed with water to form N10-formyl THF (see the fi gure below). As such, glycine, FAD, B12, and B6 are not required for these conversions to take place.

The primary route of carbon entry into the tetrahydrofolate (THF) pool is via the serine hydroxymethyl transferase reaction. Which of the following is required to convert that initial form of the THF into the form that can donate carbons to de novo purine synthesis? (A) Glycine (B) FAD (C) Water (D) B12 (E) B6

The answer is C. Stretching aids in stimulating blood flow to the muscles, which enhances oxidative muscle metabolism (by allowing for better oxygen delivery). Stretching, per se, does not stimulate epinephrine release (thus, A is incorrect), nor does it activate glycolysis in either the liver or muscle.

The process of stretching before exercise has which of the following biochemical benefits? A. Stimulates the release of epinephrine B. Activates glycolysis in the liver C. Increases blood flow to the muscles D. Activates glycolysis in the muscles E. Stimulates glycogenolysis in the liver

Correct answer = D. Because thymidine is essentially found only in DNA, its incorporation would most accurately reflect the rate of DNA synthesis. Uridine is found only in RNA and could be used to measure the rate of RNA synthesis. Phosphate, adenine, and guanine are present in both DNA and RNA, and could not be used to specifically measure synthesis of either one.

The rate of DNA synthesis in a culture of cells could be most accurately determined by measuring the incorporation of which of the following radiolabeled compounds? A. Adenine. B. Guanine. C. Phosphate. D. Thymidine.

The answer is B. If dGTP were to accumulate in cells, the dGTP would bind to the substrate specificity site of ribonucleotide reductase and direct the synthesis of dADP. This would lead to elevations of dATP levels, which would inhibit the activity of ribonucleotide reductase. The inhibition of ribonucleotide reductase leads to a cessation of cell proliferation, as the supply of deoxyribonucleotides for DNA synthesis become limiting. Answer A is incorrect because ATP would need to bind to the substrate specificity site to direct the synthesis of dCDP. That would not occur under these conditions of elevated dGTP levels. Answer C is incorrect because the enzyme works only on diphosphates; AMP would never be a substrate for this enzyme. Answer D is incorrect because the thioredoxin is always regenerated and does not become rate limiting for the reductase reaction. Answer E is incorrect because dGTP does not bind to the activity site of the reductase; only ATP (activator) or dATP (inhibitor) is capable of binding to the activity site.

The regulation of ribonucleotide reductase is quite complex. Assuming that an enzyme deficiency leads to highly elevated levels of dGTP, what effect would you predict on the reduction of ribonucleotides to deoxyribonucleotides under these conditions? A. Elevated levels of dCDP will be produced. B. The formation of dADP will be favored. C. AMP will begin to be reduced. D. Reduced thioredoxin will become rate-limiting, thereby reducing the activity of ribonucleotide reductase. E. Deoxy-GTP will bind to the overall activity site and inhibit the functioning of the enzyme.

The answer is A. The resting metabolic rate (RMR) is the calories being expended by a recently awakened resting person who has fasted for 12 to 18 hours and whose body temperature is at 20°C. It is equivalent to the energy expenditure of our major organs and resting skeletal muscle. Women generally have a lower RMR per kilogram body weight because more of their body weight is usually metabolically less active adipose tissue. Children have a higher RMR per kilogram body weight because more of their body weight is metabolically active organs such as brain. The RMR increases in a cold environment because more energy is being expended to generate heat. The RMR is not equivalent to our daily energy expenditure (DEE), which includes RMR, physical activity, and diet-induced thermogenesis.

The resting metabolic rate is best explained by which one of the following statements? A. It is equivalent to the caloric requirement of our major organs and resting muscle. B. It is generally higher per kilogram body weight in women than in men. C. It is generally lower per kilogram body weight in children than adults. D. It is decreased in a cold environment. E. It is approximately equivalent to the daily energy expenditure.

The answer is D. Cortisol is released during fasting and times of stress and signals muscle cells to initiate ubiquitin-mediated protein degradation. The other hormones listed do not have this effect on muscle protein metabolism. Insulin stimulates protein synthesis; glucagon has no effect on muscle because muscle has no glucagon receptors. Epinephrine initiates glycogen degradation but not protein degradation, and glucose is not a signaling molecule for muscle as it can be for the pancreas.

The signal that indicates to muscle that protein degradation needs to be initiated is which of the following? A. Insulin B. Glucagon C. Epinephrine D. Cortisol E. Glucose

The answer is D. Glutamine is derived from glutamate, which is formed from alpha-ketoglutarate. Only isoleucine and valine can give rise to glutamine because leucine is strictly ketogenic. These amino acids give rise to succinyl-CoA, which goes around the TCA cycle to form citrate (after condensing with acetyl-CoA), which then forms isocitrate (the correct answer), and the isocitrate is converted to alpha-ketoglutarate. Urea, pyruvate, phosphoenolpyruvate, and lactate are not required intermediates in the conversion of BCAA carbons to glutamine carbons.

The skeletal muscles convert branched-chain amino acid (BCAA) carbons to glutamine for export to the rest of the body. An obligatory intermediate, which carries carbons originally from the BCAAs, in the conversion of BCAAs to glutamine, is which of the following? A. Urea B. Pyruvate C. Lactate D. Isocitrate E. Phosphoenolpyruvate

The answer is A. Myelin contains very high levels of cholesterol and cerebrosides, particularly galactosylcerebrosides.

The two lipids found in highest concentration in myelin are which of the following? A. Cholesterol and cerebrosides such as galactosylceramide B. Cholesterol and phosphatidylcholine C. Galactosylceramide sulfatide and sphingomyelin D. Plasmalogens and sphingomyelin E. Triacylglycerols and lecithin

Metabolic defects in FA oxidation

These disorders are in enzymes or proteins involved in transport or breakdown of FA Carnitine shuttle, acyl CoA DH, TFP, and last 3 steps of beta degradation Symptoms include mental retardation, neuropathy, adrenal insufficiency, and hypoglycemia Autosomal recessive Screened in newborns

Beriberi and Wernicke-Korsakoff syndrome

Thiamine(B1) deficiency, Sx include weight loss, SOB, difficulty walking, confusion, speech difficulty, lack of coordination, pain, and involuntary eye movements Both are Dx through thiamine serum levels, uncommon in the western countries due to vitamin fortified foods Dry beriberi-affects cardiovascular function Tx through thiamine supplements and other water soluble vitamins Common in alcoholics due to poor nutrtion and EtOH inhibits thiamine absoprtion Chronic alcoholic thiamine deficiency is termed W-K syndrome Thiamine deficiency leads to increased pyruvate and AKG due to impairment of PDC and AKG DH which requires thiamine PP as cofactor

A. Adenylate kinase does this. B. This is a methylation by thymidylate synthetase. C. Correct. Thioredoxin fluctuates between the sulfhydryl and disulfide forms. D. This is allopurinol. E. A variety of enzymes are involved here, but not thioredoxin.

Thioredoxin is involved in the: a. conversion of AMP to ATP b. conversion of dUMP to dTMP c. conversion of a ribonucleotide to a deoxyribonucleotide d. inhibition of xanthine oxidase as a treatment for gout e. degradation of nucleoprotein

C.

This X-linked disorder is characterized by hepatic encephalopathy and protein intolerance. A. Lesch-Nyhan syndrome B. Jaundice C. Ornithine transcarbamoylase deficiency D. Adenosine deaminase deficiency E. β-Aminoisobutyric aciduria

A

This enzyme assembles the carbon chain backbone of pyrimidines A. Aspartate transcarbamoylase B. Carbamoyl phosphate synthetase C. Dihydro-orotase D. Dihydroorotate dehydrogenase E. Orotate phosphoribosyl transferase F. Orotidylic acid dicarboxylase

C.

This enzyme assembles the carbon chain backbone of pyrimidines A. Carbamoyl phosphate synthetase B. Orotidylic acid dicarboxylase C. Aspartate transcarbamoylase D. Orotate phosphoribosyl transferase E. Dihydroorotate dehydrogenase

Thyroglobulin and thyroid hormones

Thyroglobulin leads to T3 and T4 which are iodized tyrosine molecules Hyperthyroidism(Grave's disease) is treated with carbimazole and propylthiouracil -blocks iodination of thyroglobulin, decreasing production of T4 and T3

What is the primary role of LCAT (Lecithin-cholesterol acyltransferase)?

To esterify cholesterol transported by HDL

High energy phosphoric acid anhydride bonds and thioester bonds

Triphosphate nucleotides such as GTP, CTP, TTP and UTP store energy in the phosphoric acid anhydride bonds as well as in the thioester bonds(formed between thiol of CoA and carboxylic acids Energy released is equivalent to ATP

Troponin in MI

Troponin plays a role in muscle contraction and relaxation based on Ca2+ levels IC It is comprised of three subunits, the tropomyosin(T), inhibitory(I) and the calcium binding(C) cTn-I is in cardiac muscle, sTn-I is in skeletal muscle

The answer is d. (Murray, pp 481-497. Scriver, pp 3897-3964.) The component that can be produced from tryptophan is nicotinamide, which is joined with adenosine diphosphate to form the important cofactor and ADP-ribose donor, nicotinamide adenine dinucleotide (NAD). Nicotinamide and nicotinic acid were discovered as the essential nutrient niacin that could be used to treat pellagra. Niacin is not strictly a vitamin because it can be derived from tryptophan, but its dietary deficiency contributes to pellagra along with deficiencies of riboflavin (vitamin B2) and pyridoxine (vitamin B6) that are involved in the biosynthesis of niacin from tryptophan. NAD+ is a cofactor required by all dehydrogenases and NADPH, produced by the pentose phosphate shunt is utilized in reductive synthesis of compounds such as fatty acids. Since photosensitivity is common in DNA repair disorders like xeroderma pigmentosum (278730), it is possible that deficient ADP-ribosylation of DNA repair topoisomerases relates to the photosensitivity of pellagra. Pantothenic acid and coenzyme A are involved in acetylation and acyltransfer reactions important in fatty acid metabolism, biotin and carboxybiotin in carboxylation reactions like those deficient in multiple carboxylase deficiency (253270). Intrinsic factor is a protein secreted by the gastric mucosa that is important for binding and absorption of cobalamin (vitamin B12). Pyridoxal phosphate also is involved in amino acid metabolism (transamination), muscle glycogen breakdown (glycogen phosphorylase), and steroid hormone action (removes hormone-receptor complexes from DNA, terminating their action.)

Tryptophan deficiency in diet or disease can cause pellagra, a condition with skin rash on sunlight exposure (photosensitivity), diarrhea, and death. Tryptophan can be a precursor for one part of a compound that is an enzyme cofactor and donor of ADP-ribose that is added to certain histones and DNA repair enzymes (topoisomerases). Which of the following components can be derived from tryptophan, along with its active compound? a. Biotin, carboxybiotin b. Intrinsic factor, cobalamin c. Pantothenic acid, coenzyme A d. Nicotinamide, nicotinamide adenine dinucleotide e. Pyridoxine, pyridoxal phosphate

The answer is D: Osmotic imbalance due to increased glucose levels in the urine. In an untreated type 1 diabetic, glucose levels in the blood exceed the renal threshold for reabsorption of the glucose from the urine, so blood glucose levels rise in the urine. This creates an osmotic imbalance, which forces more water into the urine, leading to polyuria (frequent urination). This is not due to urea production (and, since a type 1 diabetic does not produce insulin, insulin cannot be stimulating urea production). It is not due to an increase of ketones in the blood. And, since insulin is not present, it cannot be due to insulin stimulation of glucose resorption in the kidney.

Type 1 diabetics, prior to diagnosis, display polydipsia, polyuria, and polyphagia. The polyuria is due to which of the following? (A) Insulin stimulation of urea production (B) Osmotic imbalance due to elevated ketones in the blood (C) Osmotic imbalance due to reduced glucose levels in the urine (D) Osmotic imbalance due to increased glucose levels in the urine (E) Insulin stimulation of glucose resorption in the kidney

The answer is d. (Murray, pp 242-248. Scriver, pp 1909-1964.) Carbamoyl phosphate synthase I catalyzes the first reaction of urea biosynthesis and is the rate-limiting step. Formation of carbamoyl phosphate requires 2 ATP, one as a phosphate donor and the other to provide the driving force for synthesis of the amide bond and the mixed acid anhydride bond. Argininosuccinic acid synthase also requires 1 ATP.

Urea synthesis requires five enzymes. Which of the following enzymes requires two ATPs? a. Arginase b. Argininosuccinase c. Argininosuccinic acid synthase d. Carbamoyl phosphate synthase I e. Ornithine carboxylase

Acyl CoA Desaturase

Utilizes NADPH (or NADH) and O₂ to catalyze desaturation of fatty acid in SER

Correct answer = D. Vitamin K is essential for clot formation, decreases coagulation time, and is present in low concentrations in milk. It is one of four fat-soluble vitamins.

Vitamin K: A. plays an essential role in preventing thrombosis. B. increases the coagulation time in newborn infants with hemorrhagic disease. C. is present in high concentration in cow or breast milk. D. is synthesized by intestinal bacteria. E. is a water-soluble vitamin.

Hyperhomocysteinemia and homocystinuria

Vitamin deficiencies (B6, B12, folic acid), cystathionine beta-synthase defect Plays a role in artherosclerotic heart disease, and stroke as well as eye lens dislocation, osteoporosis and mental retardation Vitamin supplements can normalize homocysteine levels

Cystathionine β-synthase, methylmalonyl CoA mutase, branched-chain α-keto acid dehydrogenase, tyrosinase, phenylanine hydroxylase. [Note:Deficiency of dihydropteridine reductase or any of the enzymes needed for BH4 synthesis can also result in hyperphenylalaninemia.

What enzyme is deficient in: homocystinuria, methylmalonic acidemia, MSUD, oculocutaneous albinism, PKU?

A.

What is not true of cirrhosis, the liver disease? a. Mostly affects obese individuals b. Results from a fatty and inflamed liver c. Develops in 15-20% of cases of alcoholism d. Fat cells are replaced by scar tissue

Blood ammonia level would expected to be elevated in ornithine transcarbamylase deficiency but not in UMP synthase deficiency.

What laboratory test would help in distinguishing an orotic aciduria caused by ornithine transcarbamylase deficiency from that caused by UMP synthase deficiency?

B.

Which base derivative can serve as a precursor for the synthesis of two of the other base derivatives shown? a. Cytidine triphosphate (CTP) b. Uridine monophosphate (UMP) c. Deoxythymidine monophosphate (dTMP) d. Adenosine triphosphate (ATP) e. Deoxyadenosine monophosphate (dAMP)

B.

Which is the most common cause of microcytic hypochromic anemia in American women aged 20 to 50 years? a.Autoimmune hemolysis b.Iron deficiency c. Vitamin B12 deficiency d. Thalassemia minor e. Viral infection

A.

Which is the rate-controlling step of pyrimidine synthesis that exhibits allosteric inhibition by cytidine triphosphate (CTP)? a. Aspartate transcarbamoylase b. HGPRT c. Thymidylate synthase d. Ribose-phosphate pyrophosphokinase e. Xanthine oxidase

The correct answer = D. Kwash iorkor is caused by inadequate protein intake in the presence of fair to good energy (calorie) intake. Typical findings in a patient with Kwashiorkor include abdominal and peripheral edema (note the swollen belly and legs of the child) caused largely by a decreased serum albumin concentration. Anorexia is almost always present. Weight for height can be normal. Treatment includes a diet adequate in calories and protein.

Which of the findings in the child would support a diagnosis of kwashiorkor? A. Shows increased serum albumin. B. Shows a good appetite. C. Appears plump due to increased adipose tissue. D. Displays abdominal and peripheral edema. E. Has markedly decreased weight for height.

D

Which of the following amino acid(s) is(are) required for the de novo synthesis of the pyrimidine ring by mammalian cells? A. Aspartate B. Glutamine C. Glycine D. Aspartate and glutamine are required E. Aspartate, glutamine, and glycine are required

The answer is b. (Murray, pp 242-248.) Most amino acids except lysine, threonine, proline, and hydroxyproline can undergo transamination to convert α-amino acids and α-keto acids. Transamination reactions can contribute amino groups to the urea cycle for urea biosynthesis or initiate catabolism of the amino acid. Six amino acids (listed as answer options) are degraded to pyruvate, joining another six (tyrosine, phenylalanine, lysine, hydroxylysine, tryptophan, methionine) that are in part converted to acetylCoA. Cysteine can be made from cystine (essentially a cysteine dimer with disulfide linkage) by cystine reductase. The inborn error cystinosis (219800) results from mutations in a lysosomal membrane transporter, causing cystine accumulation in lysosomes with crystals in the lens of the eye and renal proximal tubules. Children with cystinosis present in early childhood with short stature, photophobia, and progressive renal disease. The synthetic compound cysteamine can complex with cystine and allow egress from lysosomes, slowing the progression of renal failure.

Which of the following amino acids (1) is the product of reduction from another amino acid, (2) can be transaminated to pyruvate, and (3) accumulates in an inborn error affecting the eyes and kidney? a. Alanine b. Cysteine c. Serine d. Glycine e. Hydroxyproline

E.

Which of the following are possible mechanisms that underlie the liver damage as seen in cirrhosis? a. Free radicals b. Build-up pf acetaldehyde concentration c. Inflammation d. Fibrosis e. All of the above

The answer is d. (Murray, pp 173-179. Scriver, pp 2297-2326.) The key enzymatic step of fatty acid synthesis is the carboxylation of acetyl-CoA to form malonyl-CoA. The carboxyl of biotin is covalently attached to an ε-amino acid group of a lysine residue of acetyl-CoA carboxylase. The reaction occurs in two stages. In the first step, a carboxybiotin is formed: HCO3− + biotin-enzyme + ATP → CO2-biotin-enzyme + ADP + Pi In the second step, the CO2 is transferred to acetyl-CoA to produce malonylCoA: CO2-biotin-enzyme + acetyl CoA → malonyl-CoA + biotin-enzyme None of the other cofactors listed are involved in this reaction.

Which of the following cofactors must be utilized during the conversion of acetyl-CoA to malonyl-CoA? a. Thiamine pyrophosphate b. Acyl carrier protein (ACP) c. NAD1 d. Biotin e. FAD

C.

Which of the following compounds is a required substrate for purine biosynthesis? a. 5-methyl thymidine b. Ara C c. Ribose 5 phosphate d. 5-phosphoribosylpyrophosphate (PRPP) e. 5-fluorouracil

The answer is d. (Murray, pp 293-302. Scriver, pp 2513-2570.) 5′-phosphoribosyl-1-pyrophosphate (PRPP) donates the ribose phosphate unit of nucleotides and is absolutely required for the beginning of the synthesis of purines. In fact, the enzymes regulating the synthesis of PRPP and the subsequent synthesis of phosphoribosylamine from PRPP are all end product-inhibited by inosine monophosphate (IMP), adenosine monophosphate (AMP), and guanosine monophosphate (GMP), the products of this reaction pathway.

Which of the following compounds is a required substrate for purine biosynthesis? a. 5-methyl thymidine b. Ara C c. Ribose phosphate d. 5-phosphoribosylpyrophosphate (PRPP) e. 5-FU

The answer is b. (Murray, pp 286-302. Scriver, pp 2513-2570.) The degradation of purines to urate can lead to gout when an elevated level of urate is present in serum, causing the precipitation of sodium urate crystals in joints. The excessive production of urate in many patients seems to be connected to a partial deficiency of hypoxanthine-guanine phosphoribosyl transferase (HGPRT). Allopurinol, an analogue of hypoxanthine, is a drug used to correct gout. It accomplishes this by inhibiting the production of urate from hypoxanthine and in doing so undergoes suicide inhibition of xanthine oxidase. Ribose phosphate and PRPP are required for purine synthesis. 5fluorouracil (5-FU) and cytosine arabinoside (Ara C) are cancer chemotherapy agents, the former being an analogue of thymine that inhibits thymidylate synthetase and the latter an inhibitor of RNA synthesis.

Which of the following compounds is an analogue of hypoxanthine? a. Ara C b. Allopurinol c. Ribose phosphate d. 5-phosphoribosylpyrophosphate (PRPP) e. 5-FU

B.

Which of the following compounds is an analogue of hypoxathine? a. Ara C b. Allopurinol c. Ribose phosphate d. 5-phosphoribosylpyrophosphate (PRPP) e. 5-FU

The answer is e. (Murray, pp 286-292. Scriver, pp 2513-2570.) Adenosine diphosphate (adenylate) is joined to nicotinamide to form nicotinamide adenine dinucleotide (NAD), further phosphorylated at the 3' position of ribose to form NADP. Niacin can be synthesized from tryptophan and is not strictly a vitamin. Deficiency of tryptophan and niacin can occur from diet or with diseases that increase requirements, causing pellagra (photosensitivity, dermatitis, psychosis). Ribonucleosides consist of a base at the 1' position of ribose, while ribonuceotides have 5′-phosphate groups. Guanosine (G) and adenosine (A) contain purine bases, cytosine (C), uridine (U), and thymidine (T) pyrimidine bases. Ribonucleotides (adenylate, guanidylate, cytidylate, uridylate) have a phosphate ester on the 5′-hydroxyl of ribose (note that the ribo- prefix is usually omitted). Deoxyribonucleotides have a phosphate ester on the 5′-hydroxyl of deoxyribose (deoxyadenylate, deoxyguanidylate, deoxycytidylate, and thymidylate).The ribonucleosides A, G, C, and U can be incorporated into RNA, while the deoxyribonucleosides A, G, C, and T join with deoxyribose (lacking a hydroxyl at the 2' carbon) and are incorporated into DNA. Uridine occurs only as the ribonucleoside, thymidine as the deoxyribonucleotide (actually as thymidylate deoxyribonucleotide synthesized from uridylate by thymidylate synthetase. Hypoxanthine and inosine are precursors of purine A, G synthesis.

Which of the following compounds is joined with nicotinamide to form NAD and NADP, components that are deficient in niacin deficiency? a. Cytosine monophosphate b. Inosine diphosphate c. Thymidine monophosphate d. Hypoxanthine monophosphate e. Adenosine diphosphate

The answer is a.(Murray, pp 481-497. Scriver, pp 3897-3964, 121-138, 287-320.) Vitamin K is essential for the posttranscriptional modification of prothrombin by γ-carboxylation of glutamate residues. A functional deficiency exists in patients treated with analogues of vitamin K such as the Coumadin derivatives. The analogues act as anticoagulants by competing with vitamin K and preventing the production of functional prothrombin. By administration of vitamin K, hemorrhage can be prevented in such patients. Vitamin K is normally obtained from green, leafy vegetables in the diet (not from citrus fruits or red meat). Intestinal bacteria also synthesize the vitamin, but even broad-spectrum antibiotic therapy does not completely sterilize the intestine. A deficiency of vitamin K can cause hemorrhagic disease in newborn infants because their intestines do not have the bacteria that produce vitamin K and because vitamin K does not cross the placenta. The neonatal deficiency occurs in term or premature infants.

Which of the following conditions most rapidly produces a functional deficiency of vitamin K? a. Coumadin therapy to prevent thrombosis in patients prone to clot formation b. Broad-spectrum antibiotic therapy c. Lack of red meat in the diet d. Lack of citrus fruits in the diet e. Premature birth

E.

Which of the following consequences of chronic alcohol consumption is irreversible? a. Inhibition of fatty acid oxidation b. Activation of triacylglycerol synthesis c. Ketoacidosis d. Lactic acidosis e. Liver cirrhosis

The answer is a. (Murray, pp 293-302. Scriver, pp 2513-2570.) During purine ring biosynthesis, the amino acid glycine is completely incorporated to provide C4, C5, and N7. Glutamine contributes N3 and N9, aspartate provides N1, and derivatives of tetrahydrofolate furnish C2 and C8. Carbon dioxide is the source of C6. In pyrimidine ring synthesis, C2 and N3 are derived from carbamoyl phosphate, while N1, C4, C5, and C6 come from aspartate.

Which of the following contributes nitrogen atoms to both purine and pyrimidine rings? a. Aspartate b. Carbamoyl phosphate c. Carbon dioxide d. Glutamine e. Tetrahydrofolate

The answer is c. (Murray, pp 481-497. Scriver, pp 3897-3964.) The absorption of vitamin B12 from the intestine is very complex, involving a binding protein called intrinsic factor with other transcobalamin proteins that also carry it in blood. Intrinsic factor is secreted by the gastric mucosa, and pernicious anemia results from gastric atrophy at older ages or more rarely, from mutations affecting the intrinsic factor itself that present with cobalamin deficiency in childhood (e.g., 261000). Inability to absorb vitamin B12 from the gastrointestinal tract causes more severe deficiency than nutritional deprivation in vegetarian diets or small bowel disease. Intrinsic factor may also be diminished by autoantibodies in autoimmune diseases like diabetes mellitus or Graves disease (hyperthyroidism). When the symptoms of cobalamin deficiency go beyond megaloblastic anemia to neurologic symptoms like numbness and weakness in the extremities, poor coordination, dementia they are called "pernicious" because of potential irreversibility and death. Clinical signs of pernicious anemia may not appear until 3 to 5 years following the onset of vitamin B12 deficiency and the neurologic signs may occur without obvious anemia. Cheilosis is dryness and scaling of the lips that is characteristic of riboflavin (vitamin B2) deficiency. Scurvy is caused by vitamin C deficiency and is characterized by bleeding gums and bone disease. Rickets is softening and deformation of the bones due to vitamin D deficiency or defects in vitamin D processing. The word beriberi is Singhalese for "I cannot," referring to muscular atrophy and paralysis caused by the inflammation of multiple nerves (polyneuritis). Beriberi is caused by thiamine (vitamin B1) deficiency and is common in Asians who subsist on a diet of polished white rice.

Which of the following disorders is associated with a deficiency of vitamin B12? a. Cheilosis b. Beriberi c. Pernicious anemia d. Scurvy e. Rickets

The answer is C. Pepsinogen, under acidic conditions, autocatalyzes its conversion to pepsin in the stomach. Both enteropeptidase and aminopeptidases are synthesized in active form by the intestine (thus, A and D are incorrect). Enteropeptidase activates trypsinogen (thus, B is incorrect), which then activates proelastase (thus, E is incorrect).

Which of the following enzymes is activated through an autocatalytic process? A. Enteropeptidase B. Trypsinogen C. Pepsinogen D. Aminopeptidase E. Proelastase

The answer is a. (Murray, pp 173-179. Scriver, pp 2327-2356.) AcetylCoA carboxylase catalyzes the first step of lipogenesis in which acetyl-CoA is linked to malonyl-CoA. This enzyme is activated by citrate. Acetyl-CoA does not readily cross the mitochondrial membrane. Instead, citrate translocates to the cytosol where it is cleaved to acetyl-CoA and oxaloacetate by ATP-citrate lyase. Citrate increases in the fed state and indicates an abundant supply of acetyl-CoA for lipogenesis.

Which of the following enzymes is most important in regulating lipogenesis? a. Acetyl-CoA carboxylase b. Acetyl transacetylase c. Enoyl reductase d. Hydratase e. 3-ketoacyl reductase

The answer is a. (Murray, pp 481-497. Scriver, pp 3935-3964.) Biotin functions to transfer carbon dioxide to substrates, adding a carboxyl group. Pyruvate carboxylase and enzymes of the holocarboxylase complex that degrade organic acids (propionate, metabolites of leucine) require biotin to transfer an activated carbonyl group. Biotin may be depleted by a deficiency of the enzyme biotinidase (253260), rendering the mentioned carboxylases less active and producing accumulation of the mentioned organic acids with severe acidosis. Nutritional deficiency of biotin is virtually unknown, but can be induced with raw egg white which contains avidin, a biotin-binding protein. Biotin deficiency causes skin rashes and hair loss, symptoms also seen in biotinidase enzyme deficiency. Thiamine is required for the reactions catalyzed by pyruvate dehydrogenase, transketolases, and α-ketoglutarate dehydrogenase. Kinases such as those in glycolysis require ATP as a cofactor.

Which of the following enzymes requires a vitamin that is rarely deficient except in those eating fad diets with excess raw egg white? a. Pyruvate carboxylase b. Pyruvate dehydrogenase c. Phosphoenolpyruvate carboxykinase d. Glucokinase e. Fructokinase

The answer is e. (Murray, pp 481-497. Scriver, pp 3897-3964.) Vitamin A is a fat-soluble vitamin that can be deficient in combination with thiamine and riboflavin deficiencies in dry climates with food shortages, with other fat-soluble vitamins (D, E, K) in disorders associated with intestinal malabsorption, and in hypothyroidism where there is a defective conversion of carotene to vitamin A. Carotenes and carotenoids in plants (yellow corn, carrots, sweet potatoes, leafy vegetables, green peas) are converted to retinaldehyde in the intestinal mucosa (then to retinol), while retinol is found in animal tissues like egg yolks, fish oils, butter, liver, kidney. The first symptoms of vitamin A deficiency are dryness of the eyes (xerophthalmia) with decreased vision in dim light (night blindness), followed by photophobia, corneal irritation, ulceration, and destruction of the eye. Dry skin and rashes also occur. The importance of eggs in people with restricted diets was vividly portrayed in James Clavell's novel, King Rat, where they were prized for prevention of blindness. Pyridoxine is present in vegetables, cereals, and fruits, niacin in liver, poultry, and eggs; tetrahydrofolate in milk, uncooked fruits, and vegetables; and riboflavin in milk, eggs, meats, and fruits.

Which of the following foods should be emphasized for individuals with dry eyes and decreased vision in dim light? a. Human and cow (not goat) milk, uncooked fruits, vegetables b. Milk, eggs, meats, fruits c. Vegetables, cereals, fruits d. Liver, poultry, eggs e. Egg yolks, fish oils, leafy vegetables

The answer is c. (Murray, pp 481-497. Scriver, pp 3897-3964.) Pyridoxine (vitamin B6) deficiency usually occurs concurrently with deficiency of other B vitamins or in association with drug therapy in individuals who are slow-metabolizers for the antituberculosis drug isoniazid and others like penicillamine or sulfa antibiotics (243400). Pyridoxine is present in many foods, particularly vegetables, cereals, and fruits. Niacin (precursor to nicotinamide adenine dinucleotide) is abundant in liver, poultry, and eggs; tetrahydrofolate in human and cow (not goat) milk, uncooked fruits, and vegetables; riboflavin (vitamin B2—precursor to flavin adenine mononucleotide) in milk, eggs, meats, and fruits; retinoic acid (vitamin A) in animal tissues like egg yolks, fish oils with other carotenoids in leafy vegetables. The coenzyme pyridoxal phosphate is a versatile compound that aids in amino acid transaminations, deaminations, decarboxylations, and transulfurations. It is also important for operation of glycogen phosphorylase. A common feature of these reactions is formation of a Schiff-base intermediate with a specific lysine group at the active site of the appropriate enzymes.

Which of the following foods should be emphasized for individuals with peripheral neuritis, insomnia, mouth and skin irritation, and diarrhea? a. Human and cow (not goat) milk, uncooked fruits, vegetables b. Milk, eggs, meats, fruits c. Vegetables, cereals, fruits d. Liver, poultry, eggs e. Egg yolks, fish oils, leafy vegetables

B.

Which of the following intermediates of hemoglobin degradation is excreted with bile into the small intestine? A. Urobilin B. Bilirubin glucuronide C. Bilirubin D. Biliverdin E. Heme

The answer is d. (Murray, pp 173-179; Scriver, pp 2327-2356.) Acetyl-CoA is carboxylated to form malonyl CoA through the addition of carbon dioxide by acetyl-CoA carboxylase. The acetyl and malonyl-CoA groups are added to sulfydryl groups of fatty acid synthase multienzyme complex (one on each subunit) through transacylation reactions. Condensation forms acetoacetyl-S-enzyme on one subunit and a free sulfydryl group of the other subunit—a sequence of enzyme reactions then converst the acetoacetyl-S-enzyme to acyl (acetyl) enzyme. A second round of two-carbon addition begins as another malonyl CoA residue displaces the acyl-S-enzyme to the other sulfhydryl group, then condenses to extend the acyl group by two carbons. Fatty acid synthesis then proceeds by successive addition of malonyl CoA residues with condensation, causing the acyl chain to grow by two carbons with each cycle.

Which of the following is an important intermediate in the biosynthesis of fatty acids? a. Carnitine b. Cholesterol c. Glucose d. Malonyl-CoA

C.

Which of the following is involved in purine synthesis, and movement of pyruvic acid into the citric acid cycle? A. Vitamin B1 (thiamine) B. Vitamin B3 (niacin) C. Biotin D. Vitamin B2 (riboflavin) E. Pantothenic acid

The answer is c. (Murray, pp 173-179. Scriver, pp 2297-2326.) Two major enzyme complexes are involved in the synthesis of fatty acids. The first is acetyl-CoA carboxylase, which synthesizes malonyl CoA by the steps shown below for the synthesis of palmitate: 7 acetyl-CoA + 7 HCO3− + 7 ATP → 7 malonyl-CoA + 7 ADP + 7 Pi Using the malonyl-CoA, palmitate is then synthesized by seven cycles of the fatty acid synthetase complex, whose stoichiometry is summarized below: acetyl-CoA + 7 malonyl-CoA + 14 NADPH → palmitate + 7 CO2 + 14 NAD+ + 8 CoA + 6 H2O As can be seen from the equations above, the necessary amount of malonyl-CoA is synthesized. Palmitate is subsequently synthesized from malonyl-CoA and one initial acetyl-CoA. Thus, acetyl-CoA, NADPH, ATP, and HCO3− are all necessary in this process. In contrast, FADH2 is not utilized in fatty acid synthesis, but is one of the products of fatty acid oxidation. Vitamin B12 is required for conversion of propionic acid to methylmalonic acid, a step in the β-oxidation of odd-numbered fatty acid chains.

Which of the following is not used in the synthesis of fatty acids? a. ATP b. Cobalamin (vitamin B12) c. FADH2 d. HCO3− e. NADPH

The answer is b. (Murray, pp 611-613. Scriver, pp 2205-2216.) The antioxidant activity of glutathione is dependent on maintenance of its reduced state. The enzyme glutathione reductase transfers electrons from NADPH via FAD to oxidized glutathione. Oxidized glutathione is composed of two glutathione molecules held together by a disulfide bridge. Reduced glutathione is a tripeptide with a free sulfhydryl group. It is the presence of the free sulfhydryl group that is of importance to the antioxidant activity of glutathione. In red blood cells, the function of cysteine residues of hemoglobin and other proteins is maintained by the reducing power of glutathione.

Which of the following is the important reactive group of glutathione in its role as an antioxidant? a. Serine b. Sulfhydryl c. Tyrosine d. Acetyl-CoA e. Carboxyl

The answer is b. (Murray, pp 205-230. Scriver, pp 2705-2716.) The uptake of exogenous cholesterol by cells results in a marked suppression of endogenous cholesterol synthesis. Low-density human lipoprotein not only contains the greatest ratio of bound cholesterol to protein but also has the greatest potency in suppressing endogenous cholesterogenesis. LDLs normally suppress cholesterol synthesis by binding to a specific membrane receptor that mediates inhibition of hydroxymethylglutaryl (HMG) coenzyme A reductase. In familial hypercholesterolemia (143890), the LDL receptor is dysfunctional, with the result that cholesterol synthesis is less responsive to plasma cholesterol levels. Suppression of HMG CoA reductase is attained using inhibitors (statins) that mimic the structure of mevalonic acid, the natural feedback inhibitor of the enzyme.

Which of the following is the major source of extracellular cholesterol for human tissues? a. Very-low-density lipoproteins (VLDLs) b. Low-density lipoproteins (LDLs) c. High-density lipoproteins (HDLs) d. Albumin e. γ-Globulin

The answer is a. (Murray, pp 75, 293-302. Scriver, pp 2513-2570.) Aspartate transcarbamoylase catalyzes the first reaction unique to pyrimidine biosynthesis. This enzyme is inhibited by CTP but activated by ATP. Both ATP and CTP bind at a different site from either substrate. Aspartate transcarbamoylase consists of multiple catalytic and regulatory subunits. Each regulatory subunit contains at least two CTP binding sites.

Which of the following is the rate-controlling step of pyrimidine synthesis that exhibits allosteric inhibition by cytidine triphosphate (CTP)? a. Aspartate transcarbamoylase b. Hypoxanthine-guanine phosphoribosyl transferase (HGPRT) c. Thymidylate synthase d. Ribose-phosphate pyrophosphokinase e. Xanthine oxidase

B.

Which of the following is true of alcohol metabolism? a. Women and men metabolize alcohol the same b. Women have a lower activity of alcohol dehydrogenase c. The recommended daily intake for women is 3 drink servings per day d. The recommended daily intake for men is 4 drink servings per day

The answer is e. (Murray, pp 145-152, 293-302. Scriver, pp 2513-2536.) The activated form of glucose utilized for the synthesis of glycogen and galactose is UDP-glucose, which is formed from the reaction of glucose-1-phosphate and UTP. The conversion of galactose to glucose is at the UDP-sugar level, and is deficient in galactosemia (230400). UDP derivatives of glucose and galactose and of sugar amines (glucosamine, N-acetylmannosamine or neuraminic/sialic acids) are key precursors for synthesis of derivative polysaccharide chains including cerebrosides and gangliosides. A group of neurolipidoses with developmental regression and neurodegeneration result from deficiencies in enzymes that degrade complex polysaccharides (glycosphingolipids)

Which of the following nucleotides is associated with activated sugars in galactose, glycogen, and glycoprotein? a. Adenosine diphosphate b. Guanosine diphosphate c. Thymidine diphosphate d. Cytosine diphosphate e. Uridine diphosphate

The answer is b.(Murray, pp 130-144, 163-172, 180-189. Scriver, pp 4517-4554.) The pentose phosphate pathway does not generate any ATP but instead forms NADPH and ribose phosphate. Glycolysis produces a net two ATP molecules per glucose. The citric acid cycle produces a net 12 ATP per turn of the cycle. Fatty acid oxidation of palmitate results in a total of 129 ATP. Electron transport in the respiratory chain results in 5 ATP for each of the first 7 acetyl-CoA produced by the oxidation of palmitate for a total of 35 ATP. Each of the 8 acetyl-CoA molecules produced from palmitate results in 12 ATP from the citric acid cycle for 96 total ATP. This gives a total of 131 ATP per palmitate oxidized, minus 2 ATP for the initial activation of palmitate for a grand total of 129 ATP per palmitate.

Which of the following processes generates the most ATP? a. Citric acid cycle b. Fatty acid oxidation c. Glycolysis d. Pentose phosphate pathway e. Glycogenolysis

The answer is a. (Murray, pp 481-497. Scriver, pp 3897-3964.) Vitamin A is essential for the normal differentiation of epithelial tissue as well as normal reproduction. Yellow and dark green vegetables as well as fruits are good sources of carotenes, which serve as precursors of vitamin A. However, egg yolk, butter, cream, and liver and kidneys are good sources of preformed vitamin A. Vitamin A is necessary for vision, not hearing. The visual pigment rhodopsin is formed from the protein opsin and 11-cis-retinal. During the photobleaching of rhodopsin, all-trans-retinalplus opsin is formed from dissociated rhodopsin, causing an impulse that is transmitted by the optic nerve to the brain. 11-cis-retinal is isomerized from trans-retinal, which spontaneously combines with opsin to reform rhodopsin, making it ready for another photochemical cycle. All transretinoic acid (tretinoin) has been found to be effective for topical treatment of psoriasis. Another form of vitamin A is 13-cis-retinoic acid (Accutane), which has been found to be effective in the treatment of severe cases of acne. Accutane causes birth defects of the face and brain if taken during the first trimester of pregnancy. Vitamin A is not synthesized in the skin. Vitamin D (derivatives of calciferol) can be synthesized in the skin under the influence of sunlight from 7-dehydrocholesterol, an intermediate in cholesterol synthesis.

Which of the following statements regarding vitamin A is true? a. Vitamin A promotes maintenance of epithelial tissue b. Vitamin A is necessary for hearing but not for vision c. Vitamin A is synthesized in skin d. All vitamin A derivatives are safe to use during pregnancy e. Vitamin A is a form of calciferol

The answer is b. (Murray, pp 481-497. Scriver, pp 3897-3964.)Nicotinamide adenine dinucleotide (NAD+) is the functional coenzyme derivative of niacin. It is the major electron acceptor in the oxidation of molecules, generating NADH, which is the major electron donor for reduction reactions. Thiamine (also known as vitamin B1) occurs functionally as thiamine pyrophosphate and is a coenzyme for enzymes such as pyruvate dehydrogenase. Riboflavin (vitamin B2) functions in the coenzyme forms of flavin mononucleotide (FMN) or flavin adenine dinucleotide (FAD). When concentrated, both have a yellow color due to the riboflavin they contain. Both function as prosthetic groups of oxidation-reduction enzymes or flavoproteins. Flavoproteins are active in selected oxidation reactions and in electron transport, but they do not have the ubiquitous role of NAD+.

Which of the following vitamins becomes a major electron acceptor, aiding in the oxidation of numerous substrates? a. Vitamin B6 b. Niacin c. Riboflavin d. Thiamine e. Vitamin B1

The answer is b. (Murray, pp 481-497. Scriver, pp 3897-3964.) Pantothenate is the precursor of CoA, which participates in numerous reactions throughout the metabolic scheme. CoA is a central molecule of metabolism involved in acetylation reactions. Thus a deficiency of pantothenic acid would have severe consequences. There is no documented deficiency state for pantothenate, however, because this vitamin is common in foodstuffs.

Which of the following vitamins is the precursor of CoA? a. Riboflavin b. Pantothenate c. Thiamine d. Cobamide e. Pyridoxamine

The answer is a. (Murray, pp 481-497. Scriver, pp 3897-3964.) Riboflavin deficiency involves the insidious onset of photophobia, a burning sensation in the eyes, sore mouth (stomatitis) and tongue (glossitis), oily skin with rash (seborrheic dermatitis), and weight loss, confusion, dizziness, headache, and weakness. Retinol deficiency would cause night blindness and dry eyes that could be part of the described disorder, niacin deficiency rash (pellagra) with neurologic symptoms, thiamine deficiency heart failure and neurologic symptoms if acute (beri beri) or more chronic neuritis, pyridoxine deficiency infantile convulsions or peripheral neuritis (numbness and tingling, more common in slow metabolizers of drugs like isoniazid).

Which of the following vitamins would most likely be deficient in an adult who avoids bright light, has sore eyes, mouth, and tongue, feels tired and confused? a. Riboflavin b. Retinol c. Niacin d. Thiamine e. Pyridoxine

The answer is e. (Murray, pp 481-497. Scriver, pp 3897-3964.) Ascorbic acid (vitamin C) is found in fresh fruits and vegetables. Deficiency of ascorbic acid produces scurvy, the "sailor's disease." Ascorbic acid is necessary for the hydroxylation of proline to hydroxyproline in collagen, a process required in the formation and maintenance of connective tissue. The failure of mesenchymal cells to form collagen causes the skeletal, dental, and connective tissue deterioration seen in scurvy. Thiamine, niacin, cobalamin, and pantothenic acid can all be obtained from fish or meat products. The nomenclature of vitamins began by classifying fat-soluble vitamins as A (followed by subsequent letters of the alphabet such as D, E, and K) and water-soluble vitamins as B. Components of the B vitamin fraction were then given subscripts, e.g., thiamine (B1), riboflavin (B2), niacin [nicotinic acid (B3)], panthothenic acid (B5), pyridoxine (B6), and cobalamin (B12). The water-soluble vitamins C, biotin, and folic acid do not follow the B nomenclature.

Which of the following vitamins would most likely become deficient in a person who develops a completely carnivorous lifestyle? a. Thiamine b. Niacin c. Cobalamin d. Pantothenic acid e. Vitamin C

The correct answer is (B). Since the only control on uric acid synthesis is the availablility of substrates, anything which contributes to an increase in substrate concentration could lead to hyperuricemia. PRPP overcomes the normal feedback inhibition of synthesis. Both C and D would lead to more substrates. Since HG-PRT is an active consumer of PRPP and also the main salvage mechanism for both the guanine and adenine nucleotides, a deficiency would lead to high synthesis of nucleotides and, therefore, uric acid. An inhibitor of xanthine oxidase would specifically lower uric acid levels.

Which of the following would NOT be expected to contribute to hyperuricemia (gout)? a. Unusually high levels of PRPP b. Inhibition of xanthine oxidase c. Unusually high turnover of nucleic acids d. High activity of adenosine deaminase e. Deficiency of HGPRT

B.

Which of the following would be expected to occur after acute alcohol ingestion? a. The activation of fatty acid oxidation b. Lactic acidosis c. The inhibition of ketogenesis d. An increase in the NAD+/NADH ratio e. An increase in gluconeogenesis

The answer is B. There is an increase in the NADH/ NAD+ ratio because NADH is produced by the conversion of ethanol to acetate (thus, D is incorrect). The increased ratio of NADH/NAD+ favors the conversion of gluconeogenic precursors (such as lactate and oxaloacetate) to their reduced counterparts (lactate and malate, respectively), in order to generate NAD+ for ethanol metabolism. This reduces the concentration of gluconeogenic precursors, slows down gluconeogenesis (thus, E is incorrect), and can lead to lactic acidosis. Answer A is incorrect because the increase of NADH inhibits fatty acid oxidation. Answer C is incorrect because ketogenesis increases as a result of the increase of NADH. NADH inhibits key enzymes of the TCA cycle, thereby diverting acetyl-CoA from the TCA cycle and toward ketone body synthesis.

Which of the following would be expected to occur after acute alcohol ingestion? A. The activation of fatty acid oxidation B. Lactic acidosis C. The inhibition of ketogenesis D. An increase in the NAD+/NADH ratio E. An increase in gluconeogenesis

The answer is d. (Murray, pp 163-172, 293-302. Scriver, pp 2513-2570.) Carbamoyl phosphate (CAP) synthase I is found in mitochondrial matrix and is the first step in urea synthesis, condensing CO2 and NH4+. Hyperammonemia occurs when CAP is deficient. CAP synthase II forms CAP as the first step in pyrimidine synthesis. Its complete deficiency would probably be a lethal mutation. When its activity is decreased, purine catabolism to uric acid is decreased, decreasing the possibility of hyperuricemia. In contrast, gout, Lesch-Nyhan syndrome, high xanthine oxidase activity, and von Gierke's disease [glycogen storage disease type la (232200)] all lead to increased urate production and excretion.

Which of the following would make hyperuricemia very unlikely in a patient? a. Lesch-Nyhan syndrome b. Gout c. Xanthine oxidase hyperactivity d. Carbamoyl phosphate synthase deficiency e. Purine overproduction secondary to von Gierke's disease

D.

Which of the following would rule out hyperuricemia in a patient? a. Lesch-Nyhan symdrome b. Gout c. Xanthine oxidase hyperactivity d. Carbamoyl phosphate synthase deficiency e. Purine overproduction secondary to Von Gierke's disease

The answer is D. High levels of amino acids in the blood stimulate the pancreas to release glucagon (thus, A, C, E, and G are incorrect). Insulin is also released, but the glucagon/insulin ratio is such that the liver still uses the carbons of amino acids to synthesize glucose (thus, A, E, and F are incorrect). However, the insulin levels are high enough to stimulate BCAA uptake into the muscle for oxidation (thus, E, F, G, and H are incorrect).

Which of the profiles indicated in the following would occur within 2 hours after eating a meal that was very high in protein and low in carbohydrates?

The answer is D. Cytochrome P450 enzymes oxidize their substrates, transferring the electrons to molecular oxygen to form water and a hydroxylated product. The enzymes require NADPH (thus, B is incorrect) and are located in the endoplasmic reticulum membrane (thus, A is incorrect). Oxygen does not induce all cytochrome P450 members (although it is a substrate for all these isozymes; thus, C is incorrect), and although these enzymes proceed through a free radical mechanism, the final products are not radicals (thus, E is incorrect).

Which one of the following characteristics of cytochrome P450 enzymes is correct? A. They are all found in the Golgi apparatus and are referred to as microsomal enzymes. B. They all contain a flavin-containing reductase unit that uses NADH and not NADPH as a source of electrons. C. They are all inducible by oxygen, which binds to the iron of the cytochrome. D. They all oxidize the substrate on which they act. E. They all generate a free radical compound as a final product of the reaction.

The answer is E. Liver cirrhosis is irreversible. It is an end-stage process of liver fibrosis. Answers A, B, C, and D are all consequences of liver disease, but they are all reversible. Therefore, E is the only answer that is correct.

Which one of the following consequences of chronic alcohol consumption is irreversible? A. Inhibition of fatty acid oxidation B. Activation of triacylglycerol synthesis C. Ketoacidosis D. Lactic acidosis E. Liver cirrhosis

A.

Which one of the following contributes nitrogen atoms to both purine and pyrimidine rings? a. Aspartate b. Carbamoyl phosphate c. Carbon dioxide d. Glutamine e. Tetrahydrofolate

The answer is C. Glutamate dehydrogenase fixes ammonia into alpha-ketoglutarate, generating glutamate, in a reversible reaction that also requires NAD(P)H. Alaninepyruvate aminotransferase catalyzes the transfer of nitrogen from alanine to an alpha-keto acid acceptor but does not use ammonia as a substrate. Glutaminase converts glutamine to glutamate and ammonia, but the reaction is not reversible. Arginase splits arginine into urea and ornithine, and argininosuccinate synthetase forms argininosuccinate from citrulline and aspirate. The only other two enzymes that can fix ammonia into an organic compound are carbamoyl phosphate synthetase I and glutamine synthetase.

Which one of the following enzymes can fix ammonia into an organic molecule? A. Alanine-pyruvate aminotransferase B. Glutaminase C. Glutamate dehydrogenase D. Arginase E. Argininosuccinate synthetase

Correct answer = A. Methotrexate interferes with folate metabolism by acting as a competitive inhibitor of the enzyme dihydrofolate reductase. This starves cells for tetrahydrofolate, and makes them unable to synthesize purines and dTMP. IMP dehydrogenase is inhibited by mycophenolic acid. Ribonucleotide reductase is inhibited by hydroxyurea. Thymidylate synthase is inhibited by 5-fluorouracil. Xanthine oxidase is inhibited by allopurinol; probenecid increases renal excretion of urate, but does not inhibit its production.

Which one of the following enzymes of nucleotide metabolism is correctly paired with its pharmacological inhibitor? A. Dihydrofolate reductase—methotrexate B. IMP dehydrogenase—hydroxyurea C. Ribonucleotide reductase—5-fluorouracil D. Thymidylate synthase—allopurinol E. Xanthine oxidase—probenecid

Correct answer = D. 3-Hydroxybutyrate (or βhydroxybutyrate) —a ketone body—synthesis is enhanced in the liver by low insulin levels, which favor activation of hormone-sensitive lipase and release of fatty acids from adipose tissue. Glycogen synthesis is decreased, whereas gluconeogenesis and glycogenolysis are increased.

Which one of the following is characteristic of low insulin levels? A. Increased glycogen synthesis. B. Decreased gluconeogenesis from lactate. C. Decreased glycogenolysis. D. Increased formation of 3-hydroxybutyrate. E. Decreased action of hormone-sensitive lipase.

The answer is c. (Murray, pp 481-497. Scriver, pp 3897-3964.) Selected minerals are important cofactors for enzyme reactions; cofactors are distinguished from coenzymes because cofactors do not function in group transfer and do not undergo chemical reactions (other than changes in valence due to oxidation/reduction). Cofactors are usually metallic ions rather than organic molecules, including cobalt, copper, iron, molybdenum, selenium, and zinc. Examples include copper in cytochrome oxidase, iron in all the cytochromes, magnesium for all enzymes utilizing ATP, and zinc in lactate dehydrogenase. Zinc deficiency causes a clinical syndrome called acrodermatitis enterohepatica (201100) with growth failure, diarrhea, loss of hair, eyelashes, and eyebrows, and skin rashes with redness and scaling on the extremities (acrodermatitis). Arsenic, lead, and antimony cause disease when present in excess while vanadium (along with silicon, nickel, and tin) is known to be essential from experimental nutritiion studies but its role is not defined. Fluoride (preventing dental caries) and lithium (a therapy for depression) have effects on humans but are not known to be essential nutrients.

Which one of the following is definitely associated with a human disease when deficient? a. Arsenic b. Lead c. Zinc d. Antimony e. Vanadium

Correct answer = C. TAG-rich chylomicrons are synthesized in (and released from) the intestine following ingestion of a meal. Glucagon is depressed in the absorptive period. Acetoacetate, free fatty acids, and lactate are not elevated.

Which one of the following is elevated in plasma during the absorptive (fed) period as compared with the postabsorptive (fasted) state? A. Glucagon. B. Acetoacetate. C. Chylomicrons. D. Free fatty acids. E. Lactate.

Correct answer = A. Elevated blood glucose occurs in type 1 diabetes as a result of a lack of insulin. In type 2 diabetes, hyperglycemia is due to a defect in β-cell function and insulin resistance. The amino acid sequence of insulin is not changed in diabetes. Both forms of the disease show complex genetics. Ketoacidosis is more common in type 1 disease.

Which one of the following is most often found in untreated patients with type 1 and type 2 diabetes? A. Hyperglycemia. B. Extremely low levels of insulin synthesis and secretion. C. Synthesis of an insulin with an abnormal amino acid sequence. D. A simple pattern of genetic inheritance. E. Ketoacidosis.

Correct answer = D. The carbon skeletons of glucogenic amino acids are used by the liver for gluconeogenesis. Liver glycogen is nearly depleted by 12 hours after a meal, and muscle glycogen cannot give rise to free glucose because muscle lacks glucose 6-phosphatase. Acetoacetate is metabolized to acetyl CoA, which is not glucogenic. Lactate can arise from anaerobic glycolysis in muscle and red blood cells, but is less important than amino acids as a source of glucose.

Which one of the following is the most important source of blood glucose during the last hours of a 48-hour fast? A. Muscle glycogen. B. Acetoacetate. C. Liver glycogen. D. Amino acids. E. Lactate.

Correct answer = C. The cAMP cascade initiated by glucagon causes the liver to degrade glycogen, releasing glucose to the blood. High levels of blood glucose decrease the release of glucagon from the α cells of the pancreas. Glucagon levels increase following ingestion of a protein-rich meal. In addition to glucagon, epi nephrine and cortisol are also important in increasing glucose production in hypoglycemia. Glucagon increases the formation of ketone bodies by the liver.

Which one of the following statements about glucagon is correct? A. High levels of blood glucose increase the release of glucagon from the α cells of the pancreas. B. Glucagon levels decrease following ingestion of a protein-rich meal. C. Glucagon increases the intracellular levels of cAMP in liver cells, causing an increase in glycogenolysis. D. Glucagon is the only hormone important in combating hypoglycemia. E. Glucagon depresses the formation of ketone bodies by the liver.

Correct answer = D. The amino nitrogen of dietary protein is excreted as urea. The two nitrogens enter the urea cycle as ammonia and aspartate. Urea is produced by the hydrolysis of arginine. The cleavage of argininosuccinate does not require ATP. The urea cycle occurs partly in the mitochondria.

Which one of the following statements about the urea cycle is correct? A. The two nitrogen atoms that are incorporated into urea enter the cycle as ammonia and alanine. B. Urea is produced directly by the hydrolysis of ornithine. C. ATP is required for the reaction in which argininosuccinate is cleaved to form arginine. D. Urinary urea is increased by a diet rich in protein. E. The urea cycle occurs exclusively in the cytosol.

Correct answer = B. Phenyllactate, phenylacetate, and phenylpyruvate, which are not normally produced in significant amounts in the presence of functional phenylalanine hydroxylase, are elevated in PKU, and appear in the urine. In patients with PKU, tyrosine cannot be synthesized from phenylalanine and, hence, becomes essential and must be supplied in the diet. Treatment must begin during the first 7-10 days of life to prevent mental retardation. Discontinuance of the phenylalanine-restricted diet before 8 years of age is associated with poor performance on IQ tests. Adult PKU patients show deterioration of attention and speed of mental processing after discontinuation of the diet. Elevated levels of phenylalanine are teratogenic. Lifelong restriction of dietary phenylalanine is, therefore, recommended.

Which one of the following statements concerning a 1-week-old male infant with undetected classic phenylketonuria is correct? A. Tyrosine is a nonessential amino acid for the infant. B. High levels of phenylpyruvate appear in his urine. C. Therapy must begin within the first year of life. D. A diet devoid of phenylalanine should be initiated immediately. E. When the infant reaches adulthood, it is recommended that diet therapy be discontinued

Correct answer = D. Methionine is the precursor of cysteine. An increase in the availability of gluconeogenic amino acids from the catabolism of body protein is associated with increased ammonia and results in increased urea production. The essential amino acids leucine and lysine are ketogenic. Ornithine and citrulline are amino acids that are intermediates in the urea cycle, but are not found in tissue proteins because there are no codons for them. Phenylalanine is essential regardless of the level of tyrosine.

Which one of the following statements concerning amino acids is correct? A. An increase in gluconeogenesis from amino acids results in a decrease in urea formation. B. All essential amino acids are glycogenic. C. Ornithine and citrulline are found in tissue proteins. D. Cysteine is an essential amino acid in individuals consuming a diet severely limited in methionine. E. In the presence of adequate dietary sources of tyrosine, phenylalanine is not an essential amino acid.

Correct answer = D. Trans fatty acids raise plasma cholesterol levels. Corn oil and soybean oil are examples of fats rich in polyunsaturated fatty acids. Triacylglycerols obtained from plants generally contain more unsaturated fatty acids than those from animals. Olive oil, the staple of the Mediterranean diet, is rich in monounsaturated fats. Coconut and palm oils are unusual plant oils in that they are rich in saturated fats.

Which one of the following statements concerning dietary lipid is correct? A. Corn oil and soybean oil are examples of fats rich in saturated fatty acids. B. Triacylglycerols obtained from plants generally contain less unsaturated fatty acids than those from animals. C. Olive oil is rich in saturated fats. D. Fatty acids containing double bonds in the trans configuration, unlike the naturally occurring cis isomers, raise plasma cholesterol levels. E. Coconut and palm oils are rich in polyunsaturated fats.

Correct answer = B. The increased insulin and decreased glucagon levels characteristic of the fed state promote the synthesis of fructose 2,6bisphosphate. Most covalently modified enzymes are in the dephosphorylated state and are active. Acetyl CoA is not elevated in the fed state. The transport of glucose in the liver is not insulin sensitive. Synthesis of glucokinase is enhanced in the fed state.

Which one of the following statements concerning the fed state is correct? A. Most enzymes that are regulated by covalent modification are in the phosphorylated state. B. Hepatic fructose 2,6-bisphosphate is elevated. C. The oxidation of acetyl CoA is increased. D. Insulin stimulates the transport of glucose into hepatocytes. E. The synthesis of glucokinase is repressed.

Correct answer = C. Vitamin B12 requires intrinsic factor for its absorption. A deficiency of vitamin B12 is most often caused by a lack of intrinsic factor. However, high does of the vitamin, given orally, are sufficiently absorbed to serve as treatment for pernicious anemia. The cofactor forms are methylcobalamin and deoxyadenosylcobalamin. Vitamin B6, not vitamin B12, is involved in the transfer of amino groups. B12 is found in food derived from animal sources.

Which one of the following statements concerning vitamin B12 is correct? A. The cofactor form is vitamin B12 itself. B. It is involved in the transfer of amino groups. C. It requires a specific glycoprotein for its absorption. D. It is present in plant products. E. Its deficiency is most often caused by a lack of the vitamin in the diet.

Correct answer = A. Renal failure results in the decreased ability to form the active form of the vitamin, which must be supplied. The vitamin is not required in individuals exposed to sunlight. 1,25-Dihydroxycholecalciferol is the active form of the vitamin. Vitamin D and parathyroid hormone both increase serum calcium. A deficiency of vitamin D decreases the secretion of calcitonin.

Which one of the following statements concerning vitamin D is correct? A.Chronic renal failure requires the oral administration of 1,25-dihydroxycholecalciferol. B.It is required in the diet of individuals exposed to sunlight. C.25-Hydroxycholecalciferol is the active form of the vitamin. D.Vitamin D opposes the effect of parathyroid hormone. E.A deficiency in vitamin D results in an increased secretion of calcitonin.

D.

Which statement best describes xanthine? a. It is direct precursor of guanine b. It covalently binds to allopurinol c. It is a substrate rather than a product of the enzyme xanthine oxidase d. It is oxidized to form uric acid e. It is oxidized to form hypoxanthine

The answer is B: Triglycerides with medium-chain fatty acids. Triglycerides containing medium-chain fatty acids are absorbed directly by the intestinal epithelial cells, and sent into the circulation to the liver, bypassing the need to be incorporated into chylomicrons. Thus, conditions which might lead to fat malabsorption, such as pancreatic insuffi ciency, or decreased bile acid secretion, do not affect the absorption of these triglycerides. Long-chain triglycerides do require the actions of pancreatic lipase and bile salts for absorption. Short-chain fatty acids are not utilized until they reach the large intestine, where colonic bacteria use them primarily for energy. Thus, their nutrient value to humans is very low.

You are treating a patient with a fat malabsorption problem, and you suggest that the patient switch his or her diet to one that contains which of the following? (A) Triglycerides with long-chain fatty acids (B) Triglycerides with medium-chain fatty acids (C) Triglycerides with short-chain fatty acids (D) Triglycerides with a mixture of long-chain and short-chain fatty acids (E) Triglycerides with a mixture of long-chain and medium-chain fatty acids

The answer is C: Cystathionine b-synthase. Cystathionine β-synthase has a requirement of pyridoxal phosphate, and in about 50% of the cases of defective synthase enzymes, increasing the concentration of B6 can overcome the effects of the mutation on the enzyme. While a defect in methionine synthase will lead to elevated homocysteine (see the fi gure below; cystathionine β-synthase is enzyme 3 and methionine synthase is enzyme 1), this enzyme requires B12, not B6. A defect in N5, N10 methylene tetrahydrofolate reductase will also lead to elevated homocysteine, but that enzyme has a requirement for NADH, not vitamin B6. A defect in cystathionase (another B6 requiring enzyme) will block the degradation of cystathionine, which will accumulate, but will not lead to signifi cantly elevated homocysteine. S-adenosyl homocysteine hydrolase is the enzyme that converts S-adenosyl homocysteine to homocysteine and adenosine; lack of its activity will lead to a reduction, not an increase, in homocysteine levels.

You have an elderly patient with a history of heart attacks (MIs) and strokes (CVAs). Blood work indicates an elevated homocysteine level, which is reduced by the patient taking pharmacological doses of pyridoxamine. An enzyme that would benefit from such treatment in lowering homocysteine levels is which of the following? (A) Methionine synthase (B) N5, N10 methylene tetrahydrofolate reductase (C) Cystathionine β-synthase (D) Cystathionase (E) S-adenosyl homocysteine hydrolase

D.

You may have heard the advice to not drink on an empty stomach because: a. If you drink on an empty stomach the alcohol will sour in the acidic environment of the stomach causing you to vomit. b. Food slows the stomach's emptying rate thus slowing the absorption of alcohol into the bloodstream. c. Food stimulates secretions such as gastric acid which dilutes the alcohol and slows its absorption into the bloodstream. d. b and c

The answer is B: C-peptide levels. It is diffi cult, at times, to differentiate type 1 diabetes mellitus and ketoacidosis from type 2 diabetes mellitus hyperosmolar state and lactic acidosis unless testing for acetate and/or β-hydroxybutyrate (ketone bodies) are specifically ordered when acidosis is noticed. At this point in the patient's disease process, ketone bodies should be normal. The patient is already on insulin, so insulin levels would not be helpful. Measurement of blood glucose levels, whether fasting or not, and determination of Hb1AC levels cannot differentiate type 1 from type 2 diabetes (since the patient is on insulin). Since insulin is secreted as a macromolecule and does not become active until C-peptide is cleaved from the macromolecule, a C-peptide level would be helpful in this differentiation. Type 1 diabetes mellitus (no endogenous insulin produced) should give a very low or nonexistent level of C-peptide, whereas type 2 diabetes mellitus (insulin resistance) should give a normal or high level of C-peptide. Exogenous (commercial) insulin lacks the C-peptide, so the injected insulin will not interfere with this measurement.

You see a 56-year-old female patient in follow-up after discharge from the hospital. She was treated for ketoacidosis and hyperglycemia and now is on basal and rapid acting insulins. You wonder if she really has type 1 diabetes mellitus and was in ketoacidosis or has type 2 diabetes mellitus and had a hyperosmolar state with lactic acidosis. Which of the following lab tests would help you determine whether this patient has type 1 or type 2 diabetes mellitus? (A) Insuline levels (B) C-peptide levels (C) Fasting blood glucose levels (D) Random blood glucose levels (E) Hemoglobin A1C levels

The answer is B: Self injection of insulin. This patient could inject exogenous insulin to simulate an insulinoma. The symptoms and lab findings would be identical unless a C-peptide analysis was done. Injecting insulin between meals leads to hypoglycemia as the insulin stimulates glucose transport from the blood into the peripheral tissues, in the absence of dietary glucose. The figure below compares the effects of hypoglycemia (what is occurring in this case) versus hyperglycemia (as in an untreated diabetic) on a patient. Injecting glucagon would cause release of glucose from glycogenolysis (and gluconeogenesis), resulting in a higher blood glucose level. Amylin is a compound which blocks the action of glucagon, so an amylin blocker would be the same as injecting glucagon (blocking amylin activity would increase glucagon activity, since amylin is no longer active). Carbohydrate loading is an attempt to raise glycogen stores for more glucose availability during prolonged exercise and would not lead to hypoglycemic episodes. Metformin blocks liver gluconeogenesis during the fasting state, so more fatty acids are utilized. It also reduces insulin resistance. It does not stimulate insulin release and does not produce hypoglycemia

Your 20-year-old male patient has had multiple episodes of lightheadedness, sweating, fatigue, tremor, and intense hunger. He had one seizure. During two of these episodes, his blood glucose was 40 mg/dL. This patient was desperately trying to get a discharge from the military, and you suspected he was inducing his symptoms by doing which of the following? (A) Self injection of glucagon (B) Self injection of insulin (C) "Carb loading" before exercise (D) Taking metformin before exercise (E) Taking an amylin blocker

The answer is A: Adenine. This person has gout. Gout is caused by uric acid crystallization into a joint and an intense inflammatory reaction to those crystals. The X-ray demonstrated soft-tissue swelling over the first metatarsophalangeal joint and typical gouty erosion. Uric acid is an insoluble breakdown product of purines (adenine, hypoxanthine, or guanine). Pyrimidines (thymine, uracil, and cytosine) breakdown to different water-soluble products that do not crystallize. Ribose-5-phosphate is also degraded to very water-soluble products.

Your 56-year-old male patient presents with intense redness, heat, and pain over his right great toe at the metatarsophalangeal joint. Fluid from this joint shows bifringent crystals. An X-ray of the foot is shown below. This disease is caused by the degradation of an excessive amount of which of the following? (A) Adenine (B) Thymine (C) Uracil (D) Cytosine (E) Ribose-5-phosphate

The answer is A: A macrocytic anemia. Methotrexate acts by inhibiting dihydrofolate reductase such that THF cannot be formed (either from folate or dihydrofolate), and a functional folate defi ciency results (see the fi gure below). The folate defi ciency then results in a macrocytic anemia due to the lack of DNA synthesis. Red cell precursors increase in mass but cannot divide due to the lack of precursors for DNA replication. As a result, larger than normal cells are released into the circulation, although the overall red cell number decreases, resulting in an anemia. Both thalassemia and spherocytosis lead to microcytic anemia. Vitamin B12 levels would not be affected, and the normal occult blood tests and colonoscopy indicate that there is no bleeding leading to the anemia.

Your 60-year-old female patient has psoriasis and has been treated with methotrexate for several years. She has no other medical problems and her preventive screenings, including fecal occult blood tests and colonoscopy, have all been normal. She has developed an anemia. Which of the following would you expect to find when working up her anemia? (A) A macrocytic anemia (B) A microcytic anemia (C) Thalassemia (D) Spherocytes (E) A low vitamin B12 level

The answer is D: It inhibits glucagon secretion. Pramlintide is an amylin agonist used to lower postprandial blood glucose. Amylin is a peptide hormone secreted by the beta cells of the pancreas (with insulin), and inhibits glucagon secretion when blood glucose levels are elevated after a meal (thus aiding insulin action). Glucagon stimulates release of glucose from glycogen and further raises blood glucose. Insulin stimulates glycogenesis and storage of glucose which lowers blood glucose. Inhibiting insulin secretion would worsen the problem of high blood glucose levels. Decreasing glucose-6-phosphate or stimulating hexokinase or glycogen phosphorylase would increase glycogenolysis and raise blood glucose, which is opposite what one wants to accomplish in a diabetic patient.

Your diabetic patient has recently been placed on pramlintide (Symlin) to help control his diabetes. Which of the following best describes the mechanism of action of this medication? (A) It decreases glucose-6-phosphate (B) It increases hexokinase (C) It stimulates glycogen phosphorylase (D) It inhibits glucagon secretion (E) It inhibits insulin secretion

The answer is E: dUMP. Hydroxyurea, in addition to inducing γ-chain synthesis of hemoglobin, is also an inhibitor of ribonucleotide reductase. If ribonucleotide reductase is inhibited, the cells' ability to generate deoxyribonucleotides will be impaired, and DNA synthesis will be hindered. Since blood cells are regenerated at a rapid rate, they are one of the first cells affected by an inhibition of DNA synthesis, and the result is a decrease of blood cells in the patient. Of the answers listed, the synthesis of only dUMP requires the activity of ribonucleotide reductase. Hydroxyurea does not interfere with the synthesis of N5-methyltetrahydrofolate, 5′-phosphoribosyl 1′-amine, PRPP, or adenosylcobalamin.

Your patient has sickle cell disease and is being treated with hydroxyurea. After 2 weeks on the drug, you find greatly reduced levels of most blood cell types, and the patient is removed from the drug to allow his blood cell counts to stabilize. One potential reason for this side effect of hydroxyurea treatment is its ability to alter the synthesis of which of the following metabolites? (A) N5-methyltetrahydrofolate (B) 5′ phosphoribosyl 1′ amine (C) PRPP (D) Adenosylcobalamin (E) dUMP

The answer is B: PPAR-γ . Thiazolidinediones (TZDs), of which pioglitazone is a member bind to peroxisome proliferator activated receptor-γ (PPAR-γ) in the adipocyte and activate the synthesis and release of adiponectin, which acts on target cells to reduce blood glucose levels (by upregulating GLUT4 content of the membranes) and to reduce circulating triglyceride levels (through phosphorylation and inhibition of acetyl-CoA carboxylase 2, which relieves the inhibition of carnitine palmitoyl transferase I). While adiponectin levels rise, which leads to a stimulation of the AMP-activated protein kinase, neither of those effects is due to a direct interaction with the TZD. LKB1, an upstream kinase responsible for activating the AMP-activated protein kinase, and leptin are not involved in the response to TZDs

Your patient with metabolic syndrome is in for a checkup. His HbA1C is 9.0 and his fasting triglycerides are 325 mg/dL. You prescribe pioglitazone (Actos) to better treat his diabetes, but nothing else specific for the high lipids. A month later, the fasting triglyceride levels have dropped to 155 mg/dL due to a direct activation of which of the following? (A) AMP-activated protein kinase (B) PPAR-γ (C) Leptin (D) Adiponectin (E) LKB1

The answer is B: Potato. Amounts of simple carbohydrates in a meal are the most reliable indicators of postprandial rise in blood glucose. As the HbA1C nears normal or target values, fasting blood glucose values are usually normal and postprandial glucose values have a much more important effect on the HbA1C. Proteins, fats, and complex carbohydrates are absorbed more slowly than simple carbohydrates. This can be viewed as the glycemic index (the ability of a food to rapidly raise blood glucose). A higher number for the glycemic index means a more rapid and higher rise of blood glucose. Potatoes have the highest glycemic index of the mentioned foods and are composed of simple carbohydrates. Broccoli has more complex carbohydrates and a much lower glycemic index. Meat is mostly proteins and fats. Milk contains proteins and so has a lower glycemic index than potatoes (whole milk also has fats and an even lower glycemic index). Diet drinks contain no carbohydrates or calories. A table of the glycemic index of common foods is listed in Table 19-1, with the values adjusted to a white bread level of 100.

Your patient with type 2 diabetes mellitus is usually in good control with an HbA1C of 7.1 and fasting blood glucose values between 90 and 100 mg/dL. His problem is with his 1-h postprandial glucose levels at lunch and dinner. A recall of his usual diet reveals some type of meat, potato, broccoli, milk, and diet drink at these meals. Which of these foods is most likely responsible for his postprandial high blood glucose? (A) Meat (B) Potato (C) Broccoli (D) Milk (E) Diet drink

The answer is D: Activation of SOCS3. Large adipocytes release leptin as a satiety signal. The leptin travels to the hypothalamus and binds to receptors which signal the release of anorexigenic neuropeptides, which signal "stop eating." However, leptin also induces the expression of suppressor of cytokine signaling (SOCS3), which blocks the action of leptin. With constant leptin release, SOCS3 levels are raised, and leptin can no longer induce the release of anorexigenic signals, leading to overeating. The leptin pathway (a JAK/STAT receptor and signaling mechanism) does not involve SMAD4 (a component of the TGF-β signaling pathway). The leptin resistance is not due to downregulation of the leptin receptors, or the activation of the insulin receptor. The downregulation of the release of anorexigenic factors is a consequence of leptin resistance, not a cause.

Your patient, with a BMI of 36 and a waist circumference of 44 in., has a fasting blood glucose level of 145 mg/dL. It is likely that your patient is now leptin resistant. This has occurred due to which of the following? (A) Activation of SMAD4 (B) Downregulation of the leptin receptor (C) Activation of the insulin receptor (D) Activation of SOCS3 (E) Downregulation of anorexigenic factors

The answer is C: Activation of pyruvate carboxylase. The patient most likely has metabolic syndrome, and the liver has become resistant to the action of insulin. Because of this, gluconeogenesis is enhanced in the liver. Due to the large size of the adipocytes, free fatty acid levels in the portal circulation are high (visceral adipocytes release compounds into the portal vein), and the liver is oxidizing fatty acids for energy, generating acetyl-CoA. The acetyl-CoA is an activator for pyruvate carboxylase, and an inhibitor of pyruvate dehydrogenase. The activation of pyruvate carboxylase contributes to the activation of gluconeogenesis, and high blood glucose levels. The intestinal epithelial cells release glucose, obtained from the diet, down its concentration gradient, so enhanced release would only occur when blood glucose levels were very low. If GLUT4 transporters were stimulated in muscle, blood glucose levels would drop, as the muscle would be capable of removing glucose from the circulation. The insulin-resistance that is occurring also affects the muscle, such that insulin can no longer stimulate glucose transport into that tissue. Liver utilizes both GLUT1 and GLUT4 transporters, but due to the insulin resistance, the liver is exporting glucose rather than using it. Activation of protein kinase B occurs when insulin binds to its receptor; since the liver is resistant to insulin action, the level of protein kinase B activation is reduced.

Your patient, with a BMI of 36 and a waist circumference of 44 in., has a fasting blood glucose level of 145 mg/dL. One reason for the elevated blood glucose is which of the following? (A) Enhanced release of glucose from the intestinal epithelial cells (B) Stimulation of GLUT4 transporters in muscle (C) Activation of pyruvate carboxylase (D) Inhibition of liver GLUT4 transporters (E) Activation of protein kinase B

The answer is E: Fatty acids. Since the patient is exhibiting signs of metabolic syndrome, a key component of which is insulin resistance, the muscle has difficulty in transporting glucose from the circulation into the tissue. This means that the muscle will use fatty acids as its primary energy source. When abundant fatty acids are available, the muscle will utilize the fatty acids preferentially and will not use amino acids as an energy source. The muscle (other than the heart) does not utilize lactate for energy, and glycerol can only be metabolized in the liver, as that is the only tissue which contains glycerol kinase. The use of fatty acids by the muscles, instead of glucose, also contributes to the hyperglycemia observed in these patients.

Your patient, with a BMI of 36 and a waist circumference of 44 in., has a fasting blood glucose level of 145 mg/dL. The primary energy source being used by the muscle is which of the following (in the untreated state)? (A) Glucose (B) Amino acids (C) Lactate (D) Glycerol (E) Fatty acids

The answer is B: Humulin R is complexed with zinc, which slows its absorption. Humulin R (regular acting) is a hexamer complexed with zinc. After injection, the concentration of the insulin has to be reduced (through diffusion) for monomers and dimers of insulin to leave the zinc complex. This dramatically slows the time of insulin appearing in the circulation. Humalog, with a slightly different amino sequence, is not complexed with zinc and is absorbed much more rapidly from the injection site than Humulin R. Both types of insulin are taken subcutaneously, not orally, even when using an insulin pump. Pumps will not alter absorption, just delivery and time of delivery of the insulin. Humalog is not complexed with manganese.

Your type 1 diabetic patient was managing their disease using a combination of Humulin R and Humalog. The Humalog is more rapid acting than the Humulin R due to which of the following? (A) Humalog is taken orally, rather than subcutaneously (B) Humulin R is complexed with zinc, which slows its absorption (C) Humalog is complexed with manganese, which accelerates its absorption (D) Humulin R is taken orally, which slows its absorption (E) Humalog is taken through an insulin pump mechanism

The answer is B: Inhibition of the electron transfer chain. Metformin partially inhibits complex I of the electron transport chain. This leads to reduced ATP production, which, as energy is required, rapidly increases AMP levels due to the adenylate kinase reaction. The increase in AMP levels leads to the activation of the AMP-activated protein kinase (AMPK), which is the primary messenger for metformin's effects. Metformin does not activate adenylate cyclase, nor does LKB1 (it has been postulated that LBK1 is constantly phosphorylating the AMP-activated protein kinase, but a phosphatase is always inactivating the AMPK. When AMP levels rise, however, AMP inhibits the phosphatase, leading to fully active AMPK). Metformin also has no direct effect on the rate-limiting step of purine production, amidophosphoribosyltransferase, or of adenylate kinase.

Your type 2 diabetic patient has been taking metformin for the past 6 months and has reduced fasting blood glucose levels from 185 to 112 mg/dL. This occurs due to which of the following effects of metformin? (A) Activation of adenylate cyclase (B) Inhibition of the electron transfer chain (C) Activation of LKB1 (D) Stimulation of amidophosphoribosyl transferase (E) Stimulation of adenylate kinase

When glucagon binds to its receptor on a liver cell, which of the following sequence of events occurs to activate protein kinase A?

activation of G protein, activation of adenylate cyclase, production of cAMP

When glucagon binds to receptors on mammalian liver cells, it activates Protein Kinase A. This kinase will phosphorylate regulatory enzymes and have which of the following impacts on metabolism?

activation of glycogen phosphorylase and activation of glycogenolysis

Which of the following hormones is correctly described?

adiponectin: produced in adipose tissue, stimulates fatty acid oxidation

Which of the following pathways would primarily support blood glucose levels between meals but would be fairly inactive after >36 hours of fasting?

glycogenolysis

Elevated levels of acetyl CoA can be converted into ketone bodies and can potentially be used as fuel by all of the following tissues except:

liver

Which of the following leads to the formation of atherosclerotic plaques?

uptake of oxidized LDL particles by macrophages

Carnitine palmitoyltransferase I (CPT-I)

{also call carnitine acyltransferase} Located in intermembrane space Transfers Fatty Acyl from FA-CoA to Carnitine Forms FA-Carnitine Rate-limiting enzyme in FA degradation Inhibited by Malonyl CoA

Correct answer = B. The activity of δ-aminolev ulinic acid synthase controls the rate of porphyrin synthesis. The hepatic form of the enzyme is increased in patients treated with certain drugs, and requires pyridoxal phosphate as a coenzyme. Another enzyme in the pathway (δaminolevulinic acid dehydrase) is extremely sensitive to the presence of heavy metals.

δ-Aminolevulinic acid synthase activity: A. in liver is frequently decreased in individuals treated with drugs, such as the barbiturate phenobarbital. B. catalyzes a rate-limiting reaction in porphyrin biosynthesis. C. requires the coenzyme biotin. D. is strongly inhibited by heavy metal ions such as lead. E. occurs in the cytosol.


Ensembles d'études connexes

[History] Chapter 2: How did Singapore become a British trading post?

View Set

Ch 10 Promoting a Healthy Pregnancy

View Set

B-Law CH 42 terms and things to know from notes

View Set

Chapter 8 Quiz (Political Parties)

View Set

Marketing Ch.11, Chapter 10 quiz MKT

View Set